Sei sulla pagina 1di 346

CHAPTER 1

INTRODUCTION TO CELLS
2009 Garland Science Publishing
3rd Edition

Unity and Diversity of Cells


1-1 The smallest unit of life is a(n)
(a) DNA molecule.
(b) cell.
(c) organelle.
(d) virus.
(e) protein.
1-2 For each of the following sentences, fill in the blanks with the best word or phrase selected
from the list below. Not all words or phrases will be used; each word or phrase should be used
only once.
Cells can be very diverse: superficially, they come in various sizes, ranging from
bacterial cells such as Lactobacillus, which is a few __________________ in length, to
larger cells such as a frogs egg, which has a diameter of about one
__________________. Despite the diversity, cells resemble each other to an astonishing
degree in their chemistry. For example, the same twenty __________________ are used
to make proteins. Similarly, the genetic information of all cells is stored in their
__________________. Although __________________ contain the same type of
molecules as cells, their inability to reproduce themselves by their own efforts means
that they are not considered living matter.
amino acids
DNA
fatty acids
plasma membranes

micrometer(s)
millimeter(s)
plants

viruses
yeast
meter

1-3 Which of the following statements about the basic chemistry of cells is TRUE?
(a) All cells contain exactly the same proteins.
(b) All proteins are constructed from the same 22 amino acids.
(c) The genetic instructions in all cells are stored in DNA.
(d) All organisms contain the same genes.
(e) All of the above

1-4

Which of the following statements is TRUE?


(a)
Mutations are always harmful to an organism.
(b)
Mutation is essential for evolution to occur.
(c)
Mutation is the only source of genetic differences between parents and offspring
in plants and animals.
(d)
Mutation always leads to evolution.
(e)
Mutations always lead to evolutionary dead ends.

Cells Under the Microscope


1-5

What unit of length would you generally use to give the measurements of a typical
human cell?
(a)
Centimeters
(b)
Nanometers
(c)
Millimeters
(d)
Micrometers

1-6

A.
B.

1-7

State whether you would use a phase-contrast light microscope, a fluorescence


microscope, an electron microscope, or none of the above to do the following things:
A.
look at unstained living animal cells.
B.
look at ribosomes.
C.
look at an electron.
D.
look at a living cell expressing green fluorescent protein.
E.
do confocal microscopy.

What sets the limit on the size of structure that can be seen in a light microscope?
Why are tissues usually cut into thin sections and stained before examination
under a light microscope?

The Procaryotic Cell


1-8

Which of the following statements concerning procaryotes are TRUE?


(a)
They have no nucleus and hence no DNA.
(b)
They have no Golgi apparatus.
(c)
They can form simple multicellular organisms.
(d)
They include bacteria, yeast, and protozoans.
(e)
They are all able to live on inorganic energy sources.

The Eucaryotic Cell


1-9

The most reliable feature distinguishing a eucaryotic cell from a procaryotic cell is the
(a)
presence of a plasma membrane.
(b)
presence of a nucleus.
(c)
eucaryotic cells larger size.
(d)
presence of DNA.
2

1-10

Correct each of the following so that it becomes a TRUE statement about mitochondria.
A.
Mitochondria take in carbon dioxide and release oxygen.
B.
ADP is synthesized from ATP in mitochondria.
C.
Mitochondria are enclosed by two membranes, the outer one of which is highly
folded.
D.
Mitochondria are thought to be derived from photosynthetic bacteria.
E.
Mitochondria are found in aerobic procaryotes.

1-11

For each of the following sentences, fill in the blanks with the best word or phrase
selected from the list below. Not all words or phrases will be used; each word or phrase
should be used only once.
Eucaryotic cells are bigger and more elaborate than procaryotic cells. By
definition, all eucaryotic cells have a __________________, usually the
most prominent organelle in the eucaryotic cell. Another organelle found
in essentially all eucaryotic cells is the __________________, which
generates the chemical energy for the cell. On the other hand, a(n)
__________________ can be found only in the cells of plants and algae,
and performs photosynthesis. If we were to strip away the plasma
membrane from a eucaryotic cell and remove all of its membrane-enclosed
organelles, we would be left with the __________________, which
contains many long, fine filaments of protein that are responsible for cell
shape and structure and thereby form the cells __________________.
chloroplast
chromosome
cytoskeleton

1-12

cytosol
endoplasmic reticulum
mitochondrion

nucleus
ribosomes

On the schematic drawing of an animal cell in Figure Q1-12 match the labels given in the
list below to the numbered label lines.
A.
Plasma membrane
B.
Nuclear envelope
C.
Cytosol
D.
Golgi apparatus
E.
Endoplasmic reticulum

Figure Q1-12
3

1-13

In an aerobic bacterium, where do you think most of the proteins responsible for cellular
respiration are located?

1-14

Which one of the following statements is TRUE for mitochondria only, and not for both
mitochondria and chloroplasts?
(a)
They are enclosed by a double membrane.
(b)
They are thought to be derived from procaryotes.
(c)
They cannot grow and reproduce when isolated from the cell.
(d)
They reproduce by dividing in two.
(e)
They are found in all aerobic eucaryotic cells.

1-15

You fertilize egg cells from a healthy plant with pollen (which contains the male germ
cells) that has been treated with DNA-damaging agents. You find that some of the
offspring have defective chloroplasts, and that this characteristic can be passed on to
future generations. This surprises you at first because you happen to know that the male
germ cell in the pollen grain contributes no chloroplasts to the fertilized egg cell and thus
to the offspring. What can you deduce from these results?

1-16

Which of the following organelles is surrounded by two layers of membrane?


(a)
Endoplasmic reticulum
(b)
Nucleus
(c)
Lysosome
(d)
Peroxisome
(e)
Vacuole

1-17

In a eucaryotic cell specialized for secretion, which internal organelles would you expect
to be particularly abundant?

1-18

From the list below, select the THREE cellular structures or compartments that are found
in all cells.
(a)
Nucleus
(b)
Ribosomes
(c)
Cytosol
(d)
Mitochondria
(e)
Chloroplasts
(f)
Plasma membrane
(g)
Endoplasmic reticulum
(h)
Lysosomes

1-19

You have grown a culture of human cells and discover that it is heavily contaminated
with bacteria. Which of the following procedures will most likely eliminate the bacteria
without killing the human cells?
(a)
Treating the culture with a drug that causes microtubules to fall apart.
(b)
Diluting a small portion of the contaminated culture with 1000 times as much
fresh nutrient broth and regrowing the cells.
(c)
Treating the culture with a drug that damages DNA.
(d)
Treating the culture with a drug that dissolves cell walls.
(e)
Treating the culture with a detergent that destroys cell membranes.

1-20

Circle the appropriate cell type in which the listed structure or molecule can be found.
Note that the structure or molecule can be found in more than one type of cell.
Structure or Molecule
A.
DNA

Cell Type
plant

animal

B.

nucleus

animal

plant

bacterial

C.

plasma membrane

animal

plant

bacterial

D.

chloroplast

animal

plant

bacterial

E.

cell wall

animal

plant

bacterial

F.

lysosome

animal

plant

bacterial

G.

mitochondrion

animal

plant

bacterial

H.

Golgi apparatus

animal

plant

bacterial

bacterial

1-21

The protozoan Didinium feeds on other organisms by engulfing them. Why are bacteria,
in general, unable to feed on other cells in this way?

1-22

The specialized cell types in the body of a multicellular organism are different from each
other chiefly because
(a)
each cell type contains different genes.
(b)
different genes are switched on in different cell types.
(c)
some cell types have lost some of the genes that were present in the fertilized egg.
(d)
the fertilized egg divides by cell divisions that do not give rise to genetically
identical cells.
(e)
the different cell types contain fundamentally different organelles.

1-23

List the following items in order of size from the smallest to the largest.
A.
Protein molecule
B.
Human fat cell
C.
Carbon atom
D.
Ribosome
E.
Yeast cell
F.
Mitochondrion

Model Organisms
1-24

Given what you know about the differences between procaryotic cells and eucaryotic
cells, rate the following things as good or bad processes to study in the model
organism, E. coli.
A.
formation of the endoplasmic reticulum
B.
DNA replication
C.
how the actin cytoskeleton contributes to cell shape
D.
how cells decode their genetic instructions to make proteins
E.
how mitochondria get distributed to cells during cell division

1-25

A.
B.

In what way does a fungal cell structurally resemble a plant cell, rather than an
animal cell?
Which principal organelle does a plant cell contain that a fungal cell does not?

1-26

You wish to explore how mutations in specific genes affecting sugar metabolism might
alter tooth development. Which organism is likely to provide the best model system for
your studies and why?
(a)
Humans
(b)
Mice
(c)
E. coli
(d)
Arabidopsis
(e)
Gorillas

1-27

Circle the simplest model organism best used to study the following processes:
Process
Model Organism
A.

programmed cell death

E. coli

yeast

C. elegans

B.

chloroplast function

C. elegans

Arabidopsis

Drosophila

C.

immunology

mouse

yeast

Arabidopsis

D.

development of a
multicellular tissue

Drosophila

E. coli

yeast

1-28

When the genomes of distantly related organisms, such as a fly and a mouse, are
compared, they are found to contain some genes that encode proteins with almost
identical amino acid sequences. Explain how this happens.

1-29

Genes that have homologues in a variety of species have been discovered through the
analysis of genome sequences. In fact, it is not uncommon for a homologous gene to
encode a protein that looks similar in amino acid sequence in organisms as diverse as
budding yeast, archeons, plants, and humans. Even more remarkably, many of these
proteins can functionally substitute for their homologues in other organisms. Explain
what it is about the origins of cells that makes it possible for proteins expressed by
homologous genes to be functionally interchangeable in different organisms.

1-30

Your friend has just returned from a deep sea mission and claims to have found a new
single-celled life form. He believes this new life form may not have descended from the
common ancestor that all types of life on Earth share. However, hes never taken Cell
Biology, so he asks you determine whether his claim is true. In order to verify or dispute
your friends claim, you realize that you must first make a list of characteristics common
to all procaryotes and eucaryotes, so that you can check whether this new life form is
similar or different from all other types of life on Earth. Name two basic characteristics
that you could check to distinguish all procaryotes and eucaryotes from newly derived
life forms.

How We Know: Lifes Common Mechanisms


1-31

One effective strategy for investigating how a particular cellular process works is to
identify the gene required for the process to be carried out normally. A common
technique used to identify these genes is to isolate organisms that are defective in
particular cellular functions by randomly inducing mutations in individual genes. When
a gene is mutant, the protein encoded by this gene no longer functions, and the organism
shows a defect in the cellular function of interest; these organisms are considered
mutants. Thus, scientists look for mutant organisms whose cells cannot carry out the
process of interest and then determine the identity of the gene whose function has been
altered. Sometimes scientists are interested in studying processes that are essential to the
cell (required for the cell to live). Explain what temperature-sensitive mutants are and
why they are helpful for the study of essential processes, especially in single-celled
organisms such as yeast and bacteria.

Answers
1-1

(b)

1-2

Cells can be very diverse: superficially, they come in various sizes, ranging from
bacterial cells such as Lactobacillus, which is a few micrometers in length, to larger
cells such as a frogs egg, which has a diameter of about one millimeter. Despite the
diversity, cells resemble each other to an astonishing degree in their chemistry. For
example, the same twenty amino acids are used to make proteins. Similarly, the genetic
information of all cells is stored in their DNA. Although viruses contain the same type
of molecules as cells, their inability to reproduce themselves by their own efforts means
that they are not considered living matter.

1-3

(c)

1-4

(b)

1-5

(d)

1-6

A.
B.

The wavelength of visible light.


Most tissues are not transparent enough to be examined directly in a light
microscope. Transparency is increased by slicing them into thin sections, and
staining shows the different cellular structures in contrasting colors.

1-7

A.
B.
C.
D.
E.

phase-contrast light microscope


electron microscope
none of the above
fluorescence microscope
fluorescence microscope

1-8

(b) and (c)

1-9

(b)

1-10

A.
B.
C.
D.
E.

Mitochondria take in oxygen and release carbon dioxide.


ATP is synthesized from ADP in mitochondria.
Mitochondria are enclosed by two membranes, the inner one of which is highly
folded.
Mitochondria are thought to be derived from aerobic bacteria.
Mitochondria are found in aerobic eucaryotes.

1-11

Eucaryotic cells are bigger and more elaborate than procaryotic cells. By definition, all
eucaryotic cells have a nucleus, usually the most prominent organelle in the eucaryotic
cell. Another organelle found in essentially all eucaryotic cells is the mitochondrion,
which generates the chemical energy for the cell. On the other hand, a(n) chloroplast
can be found only in the cells of plants and algae, and performs photosynthesis. If we
were to strip away the plasma membrane from a eucaryotic cell and remove all of its
membrane-enclosed organelles, we would be left with the cytosol, which contains many
long, fine filaments of protein that are responsible for cell shape and structure and
thereby form the cells cytoskeleton.

1-12

A.
B.
C.
D.
E.

1-13

In the plasma membrane. According to the theory of mitochondrial origin outlined in this
chapter, the plasma membrane of the engulfed bacterium would become the inner
mitochondrial membrane, where most of the proteins involved in cellular respiration are
located.

1-14

(e)

1-15

Your results show that not all of the information required for making a chloroplast is
encoded in the chloroplasts own DNA; some, at least, must be encoded in the DNA
carried in the nucleus. The reasoning is as follows. Genetic information is only carried in
DNA, thus the defect in the chloroplasts must be due to a mutation in DNA. But all of the
chloroplasts in the offspring (and thus all of the chloroplast DNA) must derive from those
in the female egg cell, since chloroplasts only arise from other chloroplasts. Hence, all of
the chloroplasts contain undamaged DNA from the female parents chloroplasts. In all
the cells of the offspring, however, half of the nuclear DNA will have come from the
male germ cell nucleus, which combined with the female egg nucleus at fertilization.
Since this DNA has been treated with DNA-damaging agents, it must be the source of the
heritable chloroplast defect. Thus, some of the information required for making a
chloroplast is encoded by the nuclear DNA.

1-16

(b)

1-17

the endoplasmic reticulum and the Golgi apparatus

1-18

(b), (c), and (f)

1-19

(d)

Plasma membrane3
Nuclear envelope5
Cytosol1;
Golgi apparatus2
Endoplasmic reticulum4

Bacteria have cell walls, whereas mammalian cells do not.

1-20

A.
B.
C.
D.
E.
F.
G.
H.

animal
animal
animal

animal
animal
animal

plant
plant
plant
plant
plant
plant
plant
plant

bacterial
bacterial
bacterial
bacterial

1-21

Didinium engulfs prey by changing its shape, and for this it uses its cytoskeleton.
Bacteria have no cytoskeleton, and cannot easily change their shape because they are
generally surrounded by a tough cell wall.

1-22

(b)

1-23

1C, 2A, 3D, 4F, 5E, 6B

1-24

A.
B.
C.
D.
E.

bad
good
bad
good
bad

1-25

A.
B.

Like plant cells, fungal cells have cell walls.


Chloroplasts

1-26

(b)

Mice are likely to provide the best model system. Mice have teeth and have long
been used as a model organism. Mice reproduce relatively rapidly and the
extensive scientific community that works on mice have developed techniques to
facilitate genetic manipulations.
Humans are not a model system. E. coli (a bacterium) and Arabadopsis (a plant)
do not have teeth. Gorillas, although they have teeth, would not be a good model
organism for many reasons. First, there is not an extensive scientific community
working on the molecular and biochemical mechanisms of cellular behaviors in
gorillas. Second, gorillas are expensive to house and, thus, perform experiments
on. Third, gorillas do not reproduce rapidly, a characteristic desirable in model
organisms. Finally, techniques for facile genetic manipulations on gorillas have
not been extensively developed.

1-27

A.
B.
C.
D.

C. elegans
Arabidopsis
mouse
Drosophila

10

1-28

All living organisms are descended from a common ancestor. This means that their
individual genes are also descended from common ancestral genes. Genes in different
species that trace their descent back to a common ancestral gene in this way (that is,
homologous genes) become different from one another through mutation and natural
selection. However, the protein products of many genes are highly optimized for specific
functions, involving precisely adjusted interactions of the protein molecule with other
molecules in the cell. Almost any mutation altering the amino acid sequence of such a
protein will be harmful and will be eliminated by natural selection. As a result, the amino
acid sequence of the protein may remain almost unchanged over long periods of
evolutionary time.

1-29

All living beings on Earth (and thus, all cells) are thought to be derived from a common
ancestor. Solutions to many of the essential challenges that face a cell (such as the
synthesis of proteins, lipids, and DNA) appear to have been achieved in this ancient
common ancestor. The ancestral cell therefore possessed sets of proteins to carry out
these essential functions. Many of the essential challenges facing modern-day cells are
the same as those facing the ancestral cell, and the ancient solutions are often still
effective. Thus, it is not uncommon for organisms to use proteins and biochemical
pathways inherited from their ancestors. While these proteins often show some speciesspecific diversification, they still retain the basic biochemical characteristics of the
ancestral protein. For example, homologous proteins often retain their ability to interact
with a specific protein target, even in diverse cell types. Because the basic biochemical
characteristics are retained, homologous proteins are capable of functionally substituting
for one another.

1-30

Any two reasonable answers are OK. For example:


1.
2.
3.

The genomic information is encoded in nucleic acids.


A particular set of twenty amino acids is used to make protein.
Phospholipids are used to create cell membranes.

Nucleic acids, proteins made of amino acids, and phospholipids are all complex
molecules produced and utilized by all known living cells on Earth. These compounds
are not easily created in the absence of life. If one were to discover a new life form that
did not contain these compounds, which are central to life as we know it, it would be
likely that this new life form comes from an ancestor that used very different strategies to
survive.

11

1-31

Temperature-sensitive mutants are organisms that contain a genetic mutation to make


them sensitive to temperature. A temperature-sensitive mutant usually has a mutation in
a gene that results in the production of a protein that does not function properly at a
certain temperature (the restrictive temperature). At the permissive temperature, the
mutant cells can live and reproduce normally. However, at the restrictive temperature,
the cells will display the fatal defect.
If a scientist is interested in an essential process, the scientist may set out to isolate
mutant organisms defective in that essential process. In order to study an organism, it is
important to be able to propagate it. However, a single-celled organism whose cells are
defective in an essential process will die (and be unable to be propagated). Temperaturesensitive mutations permit the organisms to be propagated at the permissive temperature
(where the proteins function normally) and allow the scientist to study the consequences
of a lack of the essential gene function at the restrictive temperature (where the protein is
defective).

12

CHAPTER 2
CHEMICAL COMPONENTS OF CELLS
2009 Garland Science Publishing
3rd Edition

Chemical Bonds
2-1 If the isotope 32S has 16 protons and 16 neutrons, how many protons and how many neutrons
will the isotope 35S has?
2-2 A.
B.
C.

If 0.5 mole of glucose weighs 90 g, what is the molecular weight of glucose?


What is the concentration, in grams per liter (g/l), of a 0.25 M solution of
glucose?
How many molecules are there in 1 mole of glucose?

2-3 Which of the following elements is LEAST abundant in living organisms?


(a) Sulfur
(b) Carbon
(c) Oxygen
(d) Nitrogen
(e) Hydrogen
2-4 Your friend learns about Avogadros number and thinks it is so huge that there may not
even be a mole of living cells on Earth. You have recently heard that there are about 50 trillion
(50 1012) human cells in each adult human body, so you bet your friend $5 that there is more
than a mole of cells on Earth. Once you learn that each human contains more bacterial cells
(in the digestive system) than human cells, you are sure that you have won the bet. The human
population is now more than 6 billion (6 106). What calculation can you show your friend
to convince him you are right?
2-5 Atoms form covalent bonds with each other by
(a) sharing protons.
(b) sharing electrons.
(c) transferring electrons from one atom to the other.
(d) sharing neutrons.
(e) attraction of positive and negative charges.
2-6 A carbon atom contains six protons and six neutrons.
A. What are its atomic number and atomic weight?
B. How many electrons does it have?
C. What is its valence? How does this affect carbons chemical behavior?
D. Carbon with an atomic weight of 14 is radioactive. How does it differ in structure from
nonradioactive carbon? How does this difference affect its chemical behavior?

13

2-7

An ionic bond between two atoms is formed as a result of the


(a)
sharing of electrons.
(b)
loss of a neutron from one atom.
(c)
loss of electrons from both atoms.
(d)
loss of a proton from one atom.
(e)
transfer of electrons from one atom to the other.

2-8

Which of the following pairs of elements are likely to form ionic bonds? Use Figure Q2-8
if necessary.

Figure Q2-8
(a)
(b)
(c)
(d)
(e)

Hydrogen and hydrogen


Magnesium and chlorine
Carbon and oxygen
Sulfur and hydrogen
Carbon and chlorine

14

2-9

For each of the following sentences, fill in the blanks with the best word or phrase
selected from the list below. Not all words or phrases will be used; each word or phrase
should be used only once.
Whereas ionic bonds form a(n) __________________, covalent bonds
between atoms form a(n) __________________. These covalent bonds
have a characteristic bond __________________ and become stronger and
more rigid when two electrons are shared in a(n) __________________.
Equal sharing of electrons yields a(n) __________________ covalent
bond. If one atom participating in the bond has a stronger affinity for the
electron, this produces a partial negative charge on one atom and a partial
positive charge on the other. These __________________ covalent bonds
should not be confused with the weaker __________________ bonds that
are critical for the three-dimensional structure of biological molecules and
for interactions between these molecules.
charge
covalent
double bond
ionic

length
molecule
noncovalent
nonpolar

polar
salt
single bond
weight

2-10

A.
B.

In what scientific units is the strength of a chemical bond usually expressed?


If 0.5 kilocalories of energy is required to break 6 1023 bonds of a particular
type, what is the strength of this bond?

2-11

Approximately how many hydrogen bonds does it take to give an aggregate binding
strength nearly equal to a single covalent bond?
(a)
1
(b)
3
(c)
50
(d)
500
(e)
5000

2-12

After looking at Figure Q2-8 above, which of the following pairs of atoms do you expect
to be able to form double bonds with each other?
(a)
Mg and Ca
(b)
C and Cl
(c)
S and O
(d)
C and H
(e)
He and O

15

2-13

A.
B.
C.

Sketch three different ways three water molecules could be held together by
hydrogen bonding.
On a sketch of a single water molecule, indicate the distribution of positive and
negative charge (using the symbols + and ).
How many hydrogen bonds can a hydrogen atom in a water molecule form? How
many hydrogen bonds can the oxygen atom in a water molecule form?

2-14

Which of the following statements about hydrogen bonds are TRUE?


(a)
They are weak covalent bonds that are easily disrupted by heat.
(b)
They are weak bonds formed between hydrocarbons in water.
(c)
They are weak bonds formed between nonpolar groups.
(d)
They are weak bonds only formed in the presence of water.
(e)
They are weak bonds involved in maintaining the conformation of
macromolecules.

2-15

Based on what you know about the properties of water, which of the following statements
about methanol (CH3OH) are TRUE?
(a)
Methanol molecules form more hydrogen bonds than water molecules do.
(b)
The boiling point of methanol is higher than that of water.
(c)
Salts such as NaCl are less soluble in methanol than in water.
(d)
Methanol is a more cohesive liquid than water.
(e)
Methanol has a higher surface tension than water.

2-16

A.
B.
C.

What is the pH of pure water?


What concentration of hydronium ions does a solution of pH 8 contain?
Complete the following reaction:
CH3COOH + H2O

D.

Will the reaction in C occur more readily (be driven to the right) if the pH of the
solution is high?

16

2-17

The amino acid histidine is often found in enzymes. Depending on the pH of its
environment, sometimes histidine is neutral and other times it acquires a proton to
become positively charged. Consider an enzyme with a histidine side chain that is known
to play an important role in the function of the enzyme. It is not clear whether this
histidine is required in its protonated or unprotonated state. To answer this question you
measure enzyme activity over a range of pH, with the results shown in Figure Q2-17.
Which form of histidine is necessary for the active enzyme?

Figure Q2-17

Molecules in Cells
2-18

Match the chemical groups shown in the first list with their names selected from the
second list.
List 1
A. OH
B. C = O
C. COOH
D. CH3
E. NH2

List 2
1. Amino
2. Aldehyde
3. Phosphate
4. Carboxyl
5. Carbonyl (ketone)
6. Methyl
7. Amido
8. Ester
9. Hydroxyl

17

2-19

Match the macromolecules shown in the first list with their small molecule building
blocks from the second list.
List 1
A. Polysaccharides
B. DNA
C. RNA
D. Proteins
E. Lipids

List 2
1. Amino acids
2. Deoxyribonucleotides
3. Aldehydes
4. Pyrophosphates
5. Ribonucleotides
6. Fatty acids
7. Sugars
8. Steroids

2-20

Which of the following are examples of isomers?


14C and 12C
(a)
(b)
Alanine and glycine
(c)
Adenine and guanine
(d)
Glycogen and cellulose
(e)
Glucose and galactose

2-21

Which of the following is FALSE of condensation reactions?


(a)
Produce many biological polymers from monomers.
(b)
Consume H2O molecules.
(c)
Aid in storage of energy reserves.
(d)
Are the opposite of hydrolysis reactions.
(e)
Are usually catalyzed in cells by enzymes.

2-22

A.
B.
C.

How many carbon atoms does the molecule represented in Figure Q2-22 have?
How many hydrogen atoms?
What type of molecule is it?

Figure Q2-22

18

2-23

On the phospholipid molecule in Figure Q2-23 label each numbered line with a correct
term selected from the list below.
A.
B.
C.
D.
E.
F.
G.
H.
I.
J.

Phosphate
Nonpolar head group
Glycerol
Polar head group
Saturated fatty acid
Acetic acid
Sugar
Hydrophobic region
Hydrophilic region
Nonsaturated fatty acid

Figure Q2-23
2-24

Phospholipids can form bilayer membranes because they are


(a)
hydrophobic.
(b)
lipids.
(c)
amphipathic.
(d)
hydrophilic.
(e)
amphoteric.

2-25

A.

Write out the sequence of amino acids in the following peptide using the full
names of the amino acids.
ProValThrGlyLysCysGlu

B.
C.

Write the same sequence using the single letter code for amino acids.
According to the conventional way of writing the sequence of a peptide or a
protein, which is the C-terminal amino acid and which is the N-terminal amino
acid in the above peptide?
19

2-26

Which of the following statements about amino acids is TRUE?


(a)
Twenty-two amino acids are commonly found in proteins.
(b)
Most of the amino acids used in protein biosynthesis have charged side chains.
(c)
Amino acids are often linked together to form branched polymers.
(d)
D- and L-amino acids are found in proteins.
(e)
All amino acids contain an NH2 and a COOH group.

2-27

Proteins can be modified by a reaction with acetate that results in the addition of an acetyl
group to lysine side chains as shown in Figure Q2-27. The bond shown in the box in the
acetylated lysine side chain is most like a(n)

Figure Q2-27
(a)
(b)
(c)
(d)
(f)

ester.
peptide bond.
phosphoanhydride bond.
hydrogen bond.
phosphoester bond.

2-28

DNA differs from RNA in


(a)
the number of different bases used.
(b)
the number of phosphates between the sugars in the sugar-phosphate backbone.
(c)
the kind of sugar found in the sugar-phosphate backbone.
(d)
one of the purines used.
(e)
the chemical polarity of the polynucleotide chain.

2-29

Which of the following statements is FALSE about ATP?


(a)
Contains high energy phosphoanhydride bonds.
(b)
Is sometimes called the universal currency in the energy economy of cells.
(c)
Can be incorporated into DNA.
(d)
Can be hydrolyzed to release energy to power hundreds of reactions in cells.
(e)
Comprises a sugar, phosphate groups, and a nitrogenous base.

20

2-30

Which of the following are likely to be disrupted by high concentrations of salt?


(a)
A lipid bilayer
(b)
The peptide bonds in a protein
(c)
A complex of two proteins
(d)
The sugar-phosphate backbone of a nucleic acid
(e)
An oil droplet in water

Macromolecules in Cells
2-31

You are trying to make a synthetic copy of a particular protein but accidentally join the
amino acids together in exactly the reverse order. One of your classmates says the two
proteins must be identical, and bets you $20 that your synthetic protein will have exactly
the same biological activity as the original. After having read this chapter, you have no
hesitation in staking your $20 that it wont. What particular feature of a polypeptide chain
makes you sure your $20 is safe, but that your project will have to be redone.

2-32

A protein chain folds into its stable and unique 3-D structure, or conformation, by making
many noncovalent bonds between different parts of the chain. Such noncovalent bonds
are also critical for interactions with other proteins and cellular molecules. From the list
provided, choose the class(es) of amino acids that are most important for the interactions
detailed below.
A.
Forming ionic bonds with negatively charged DNA
B.
Forming hydrogen bonds to aid solubility in water
C.
Binding to another water-soluble protein
D.
Localizing an integral membrane protein that spans a lipid bilayer
E.
Packing tightly the hydrophobic interior core of a globular protein
acidic
basic

2-33

nonpolar
uncharged polar

DNA is negatively charged at physiological pH. A protein Z binds to DNA through


noncovalent ionic interactions involving lysines. What will be the effect of acetylation of
the lysine side chains (see Figure Q2-27) in protein Z on the strength of this binding?
(a)
It should increase because the acetylated lysine will form a greater number of
ionic interactions with DNA.
(b)
It should decrease because the acetylated lysine no longer has a positive charge.
(c)
It should have no effect because the unmodified lysine would not have formed an
ionic interaction with the DNA.
(d)
It should have no effect because the bond formed between lysine and the acetyl
group still has a positive charge.
(e)
It should decrease unless the DNA can become more negatively charged.

21

2-34

You are studying a microorganism in which a male turns pink in the presence of a
female. The male becomes pink because a protein X secreted by the female binds to
and activates a protein Y on the male that is responsible for the color change. You have
isolated a strain of the microorganism that produces a mutant form of protein X. This
strain behaves normally at temperatures lower than 37C, but at higher temperatures it
cannot turn pink. Could any of the following changes in mutant protein X explain your
results? If so, which ones, and explain why.
(a)
It makes an extra hydrogen bond to protein Y.
(b)
It makes fewer hydrogen bonds to protein Y.
(c)
It makes a covalent bond to protein Y.
(d)
It is completely unfolded at temperatures lower than 37C.
(e)
It is completely unfolded at temperatures higher than 37C.
(f)
It is unable to bind to protein Y at any temperature.

22

Answers
2-1

16 proteins and 19 neutrons

2-2

A.
B.
C.

180 daltons. A mole of a substance has a mass equivalent to its molecular weight
expressed in grams.
45 g/l
6 1023 molecules

2-3

(a)

Sulfur is the least abundant element among the choices given.

2-4

Avogadros number, or 6 1023, is the number of atoms (or units) in a mole. If you
multiply the number of people on Earth by the number of cells in the human body, then
double it to account for the bacteria, you will calculate: (6 109) (50 1012) 2 = 6
1023. Thus, there must be much more than a mole of living cells on Earth, and you win
$5.

2-5

(b)

2-6

A.
B.
C.

D.

2-7

(e)

2-8

(b)

The atomic number of carbon, which is the number of protons, is six. The atomic
weight, which is the number of protons plus neutrons, is 12.
The number of electrons, which equals the number of protons, is six.
The valence is the minimum number of electrons that must be lost or gained to fill
the outer shell of electrons. The first shell can accommodate two electrons and the
second shell, eight. Carbon therefore has a valence of four because it needs to
gain four additional electrons (or would have to give up four electrons) to obtain a
full outermost shell. Carbon is most stable when it shares four additional
electrons with other atoms (including other carbon atoms) by forming four
covalent bonds.
Carbon14 has two additional neutrons in its nucleus. As its electrons determine
the chemical properties of an atom, carbon14 is chemically identical to carbon12.

Magnesium has a valence of two and chlorine has a valence of one. Thus, two
chlorine atoms can each accept an electron donated by magnesium to yield a salt,
designated as MgCl2 that contains twice as many Cl chlorine anions as Mg2+
magnesium cations. All ions in this salt will have full outermost electron shells.

23

2-9

Whereas ionic bonds form a salt, covalent bonds between atoms form a molecule. These
covalent bonds have a characteristic bond length and become stronger and more rigid
when two electrons are shared in a double bond. Equal sharing of electrons yields a
nonpolar covalent bond. If one atom participating in the bond has a stronger affinity for
the electron, this produces a partial negative charge on one atom and a partial positive
charge on the other. These polar covalent bonds should not be confused with the weaker
noncovalent bonds that are critical for the three-dimensional structure of biological
molecules and for interactions between these molecules.

2-10

A.
B.

2-11

(c)

2-12

(c)

Sulfur and oxygen both require two electrons to fill their outer shell and can do so
by sharing four electrons and forming a double bond.

2-13

A.
B.

See Figure A2-13A.


See Figure A2-13B.

kilocalories per mole (or kilojoules per mole)


0.5 kcal/mole

Figure A2-13
C.
2-14

Hydrogen can form one; oxygen two.

Choice (e) is the answer. Hydrogen bonds are critical for maintaining the conformation,
or 3-D structure, of biological macromolecules like proteins and nucleic acids. Choice
(a) is false because hydrogen bonds are not covalent. Choice (b) is false because the
nonpolar-CH groups on hydrocarbons cannot form good hydrogen bonds, in water or out
of it. Choice (c) is essentially another way of stating choice (b) and thus is false. Choice
(d) is false because many molecules besides water can form hydrogen bonds and do so
regardless of whether or not water is present.
24

2-15

Choice (c) is the answer. In methanol one of the hydrogens of a water molecule has been
replaced by a nonpolar methyl group. Methanol will form fewer hydrogen bonds (thus
choice (a) is false) and make fewer ionic interactions than water does. The ability of
water to dissolve salts is a direct consequence of its ability to make ionic interactions.
Salts are therefore less soluble in methanol. Choices (b), (d), and (e) are all false because
the high boiling point, high degree of cohesion, and high surface tension of water are all a
result of the extensive hydrogen bonding between water molecules. As methanol makes
fewer hydrogen bonds, its boiling point will be lower, it will be less cohesive, and it will
have a lower surface tension than water.

2-16

A.
B.
C.
D.

2-17

Assuming the change in enzyme activity is due to the change in the protonation state of
histidine, the enzyme must require histidine in the protonated, charged state. The enzyme
is active only at low, acidic pH, where the proton (or hydronium ion) concentration is
high and thus loss of a proton from histidine will be disfavored so that histidine is likely
to be protonated.

2-18

A9; B5; C4; D6; E1

2-19

A7; B2; C5; D1; E6

2-20

(e)

2-21

Choice (b) is the answer. A condensation reaction releases a water molecule when
forming polymers (like polysaccharide energy reserves) from monomeric units (like
simple sugars), whereas the reverse hydrolysis reaction consumes a water molecule (thus,
choices (a), (c), and (d) are correct). Most synthetic reactions in cells are catalyzed by
enzymes (thus choice (e) is correct).

pH 7
108 M
CH3COO + H3O+
Yes. If the pH is high, then the concentration of hydronium ions will be low.
Therefore the rightward reaction, which produces hydronium ions, will be
favored.

Glucose and galactose are both six-carbon sugars and thus both have the formula
C6H12O6. They are thus isomers of each other. 14C and 12C are examples of
isotopes. Adenine and guanine are bases containing different numbers of nitrogen
and oxygen atoms. Glycogen and cellulose are different polymers of glucose.
Alanine and glycine are amino acids with quite different side chains, a methyl
group and a hydrogen atom, respectively.

25

2-22

A.
B.
C.

20 carbon atoms
31 hydrogen atoms
A fatty acid (Figure A2-22 is an arachidonic acid).

Figure A2-22
2-23

1D; 2A; 3C; 4J; 5I; 6H; 7E

2-24

(c)

2-25

A.
B.
C.

2-26

Choice (e) is true. As their name implies, all amino acids have at lease one amino (NH2)
group and at least one acidic carboxylic (COOH) group. It is through these two groups
that they form peptide bonds. There are 20 common amino acids (choice (a) is false), and
four or five of these have charged side chains (choice (b) is false). Each amino acid
forms only two covalent bonds with other amino acids, one bond at the amino group and
another at the carboxyl group (choice (c) is false); an exception to this is cysteine,
because the side chains of two cysteines can form a covalent sulfhydryl bond to crosslink
different regions of a polypeptide chain. Choice (d) is false because only L-amino acids
are found in proteins.

2-27

(b)

The indicated bond is an amide. Like a peptide bond, it is formed by reaction


between a carboxyl group and an amino group.

2-28

(c)

RNA contains the ribose sugar whereas DNA contains the deoxyribose sugar.
They also differ in one of the pyrimidine bases used; RNA contains the
pyrimidine uracil, while DNA instead contains thymine. All the other features are
the same.

2-29

(c)

ATP is used in energy conversions, contains ribose, and can be incorporated into
RNA. But synthesis of DNA requires the deoxyribose form of the nucleotide,
dATP. All the other statements about ATP are true.

2-30

Choice (c) is correct. Noncovalent ionic interactions such as those that hold two proteins
together are most likely to be disrupted by salt. Lipid bilayers (choice (a)) and a lipid
droplet (choice (e)) are held together by hydrophobic interactions on which salt will
have no effect. Choices (b) and (d)are examples of covalent bonds, which are not
disrupted by salt.

proline-valine-threonine-glycine-lysine-cysteine-glutamic acid (or glutamate)


PVTGKCQ
C-terminal is glutamic acid (or glutamate); N-terminal is proline.

26

2-31

As a peptide bond has a distinct chemical polarity, a polypeptide chain also has a distinct
polarity. (See Figure A2-31.) The reversed protein chains cannot make the same
noncovalent interactions during folding and thus will not adopt the same 3-D structure as
the original protein. The activities of these two proteins will definitely be different, since
the activity of a protein depends on its 3-D structure. It is unlikely that the reverse chain
will fold into any well-defined, and hence, functionally-useful structure at all, because it
has not passed the stringent selective pressures imposed during evolution.

Figure A2-31
2-32

A.
B.
C.
D.
E.

basic
uncharged polar
uncharged polar, basic, and acidic
nonpolar
nonpolar

2-33

Choice (b) is correct. Unmodified lysine side chains are positively charged and hence
attractive to the negatively charged DNA (thus choice (c) is incorrect). Because
acetylation neutralizes the positive charge (thus choice (d) is incorrect), the acetylated
form of protein Z will form fewer ionic bonds with DNA (thus choice (a) is incorrect),
and thus the strength of the interaction will decrease. Choice (e) is incorrect, since
increasing the number of negative charges on DNA would have no effect once the
positive charge on the lysine has been neutralized.

2-34

Choices (b) and (e) are possible explanations. If protein X makes fewer hydrogen bonds
to protein Y, the two proteins will bind less tightly and may come apart at temperatures
above 37C. Thermal motion is one of the forces that can disrupt the weak noncovalent
bonds responsible for holding X and Y together. The male will, therefore, not be able to
turn pink above 37C. Weak noncovalent bonds are also responsible for folding X into
the proper 3-D structure. If protein X is completely unfolded at elevated temperatures it
will not be able to bind to protein Y, so choice (e) could be the correct answer. In
contrast, choice (d) would explain a protein X that is able to bind to protein Y only at
high temperatures, and would result in a strain that would turn pink only at high
temperatures. Choice (a) would produce a protein X that would bind to protein Y more
tightly than the normal protein, and would therefore be likely to bind (and turn pink) at
temperatures above 37C. If a covalent bond was made (choice (c)), it is unlikely that
such a bond would be disrupted by any temperature in which the microorganism could
survive; the microorganism would therefore turn pink at any temperature. Choice (f)
would result in a strain that could not turn pink at any temperature.

27

CHAPTER 3
ENERGY, CATALYSIS, AND BIOSYNTHESIS
2009 Garland Science Publishing
3rd Edition

Catalysis and the Use of


Energy by Cells
3-1 If you weigh yourself on a scale one morning then eat four pounds of food during
the day, will you weigh four pounds more the next morning? Why or why not? Hint:
What happens to the atoms contained in the food and the energy stored in the
chemical bonds of food molecules?
3-2 Living organisms require a continual supply of energy to exist because
they (a) defy the laws of thermodynamics. (b) convert it into heat energy,
which powers biosynthetic reactions. (c) create order out of disorder
inside their cells. (d) cause the entropy in the universe to decrease. (e)
are closed systems isolated from the rest of the universe.

3-3 Life is thermodynamically possible because living things (a) release


heat to the environment. (b) increase the degree of order in the universe.
(c) reproduce themselves. (d) carry out energetically favorable reactions
only. (e) can carry out a chain of reactions that is energetically
unfavorable.
3-4 The energy required by a human cell to grow and reproduce is
provided by (a) the generation of order inside it. (b) its anabolic
metabolism. (c) its catabolic metabolism. (d) generation of heat. (e) its
biosynthetic reactions.

3-5 Which of the following statements about photosynthesis are TRUE? (a)
Photosynthesis is irrelevant to the existence of animals. (b) Photosynthesis converts
light energy into heat energy and chemical bond energy. (c) Photosynthesis
consumes activated carrier molecules. (d) Photosynthesis is the opposite of carbon
fixation in the Earths carbon cycle. (e) Photosynthesis increases global warming.

29

3-6

A.

Complete the equation for respiration:


Sugars + _________ _________ + H2O + heat energy + _________ energy

B.
3-7

What does this process have in common with a fire that burns the polysaccharides
in wood? How does it differ?

For each of the pairs AD in Figure Q3-7, pick the more reduced member of the pair.

Figure Q3-7
3-8

Are the following statements TRUE or FALSE? Explain.


A.
If oxidation occurs in a reaction, it must be accompanied by a reduction.
B.
The hydrogenation of an unsaturated fatty acid to a saturated fatty acid, as in the
conversion of vegetable oil to margarine, is an example of an oxidation reaction.
C.
The oxidation state of an atom influences its diameter.

30

3-9

For each of the following sentences, fill in the blanks with the best word or phrase
selected from the list below. Not all words or phrases will be used; each word or phrase
should be used only once.
By definition, catalysis allows a reaction to occur more
__________________. Chemical reactions occur only when there is a
loss of __________________ energy. Enzymes act more
__________________ than other catalysts. A catalyst reduces the
__________________ energy of a reaction.
activation
chemical bond
completely
favorable

free
kinetic
rapidly
selectively

slowly
unfavorable

3-10

Which of the following reactions are energetically favorable?


(a)
base + sugar + phosphate nucleotide
(b)
amino acid + amino acid peptide
(c)
CO2 + H2O sugar
(d)
sucrose CO2 + H2O
(e)
N2 + H2 ammonia

3-11

Which of the following statements about enzymes is correct?


(a)
Catalysis of an energetically unfavorable reaction by an enzyme will enable that
reaction to occur.
(b)
An enzyme can direct a molecule along a particular reaction pathway.
(c)
An enzyme can catalyze many chemically different reactions.
(d)
An enzyme can bind to many structurally unrelated substrates.
(e)
Enzymes are permanently altered after catalyzing a reaction.

3-12

Which of the following CANNOT be a reason that cells use enzymes rather than heat to
increase the rate of biochemical reactions?
(a)
The temperature increase required to speed up a reaction by an appreciable extent
is often huge.
(b)
Reactions cannot be coupled without enzymes.
(c)
An enzyme catalyzes just one or a very small number of different reactions; heat
would affect all the reactions in a cell.
(d)
Enzymes change the equilibrium of biochemical reactions.
(e)
Enzymes can accelerate reactions to a much greater extent than can heat.

31

3-13

The graph Q3-13 is an energy diagram for an energetically favorable reaction of substrate
S converted to product P. Indicate the following on the graph.
A.
The activation energy for the reaction
B.
The free energy change for the reaction
C.
Draw a new curve on the graph to indicate how an enzyme that converts S to P
will change the energetics of the reaction.

Figure Q3-13
3-14

Energetically favorable reactions are those that


(a)
decrease the entropy of a system.
(b)
increase the free energy of a system.
(c)
have a positive G.
(d)
decrease the free energy of a system.
(e)
create order in a system.

3-15

A.

Which one or more of the following reactions will NOT occur spontaneously
under the standard conditions that specify G?
(a)
(b)
(c)
(d)

B.

ADP + Pi ATP
glucose-1-phosphate glucose-6-phosphate
glucose + fructose sucrose
glucose CO2 + H2O

G = +7.3 kcal/mole
G = 1.7 kcal/mole
G = +5.5 kcal/mole
G = 686 kcal/mole

Which of the reactions in A could be coupled to any of the energetically


unfavorable reactions to enable them to occur?

32

3-16

Energy diagrams for the conversion of a substrate S into a product P are shown in Figure
Q3-16. The top diagram shows the original energy profile for the reaction. For each
description of an altered reaction below, choose a matching altered energy diagram,
labeled 1-6, from the figure.

Figure Q3-16
A.
B.
C.
D.

G is more positive
G is more negative
Reaction is catalyzed
No change in G and no change in activation energy.
33

3-17

Any reaction A B is at equilibrium when


(a)
G = 0.
(b)
G = 0.
(c)
[A] = [B].
(d)
G = G.
(e)
both forward and backward rates reach zero.

3-18

Consider the reaction X Y in a cell at 37C. At equilibrium, the concentrations of X


and Y are 50 M and 5 M, respectively. Using the equations below and your new
knowledge, answer the following questions.
G = 0.616 ln Keq
G = G + 0.616 ln [Y]/[X]
Recall that the natural log of a number z will have a negative value when z < 1, positive
when z > 1, and 0 when z = 1.
A.
What is the value of Keq for this reaction?
B.
Is the standard free energy change of this reaction positive or negative? Is the
reaction X Y an energetically favorable or unfavorable reaction under standard
conditions?
C.
What is the value of the standard free energy? Refer to Table 3-1 in the textbook
or use a calculator.
D.
Imagine circumstances in which the concentration of X is 1000 M and Y is
1 M. Is conversion of X to Y favorable? Will it happen quickly?
E.
Imagine starting conditions in which the reaction X Y is unfavorable, yet the
cell needs to produce more Y. Describe two ways that this may be accomplished.

3-19

The equilibrium constant for the binding of a protein to its ligand can depend on all of the
following EXCEPT the
(a)
number of noncovalent bonds formed between the protein and the ligand.
(b)
concentration of the ligand.
(c)
exact fit of the binding site to the ligand.
(d)
temperature.
(e)
pH.

34

3-20

Protein E can bind to two different proteins, S and I. The binding reactions are described
by the following equations and values:
E + S ES
E + I EI

Keq for ES = 10
Keq for EI = 2

Based on the equilibrium constant values, which one of the following statements is
TRUE?
(a)
E binds I more tightly than S.
(b)
When S is present in excess, no I molecules will bind to E.
(c)
The binding energy of the ES interaction is greater than that of the EI interaction.
(d)
Changing an amino acid on the binding surface of I from a basic amino acid to an
acidic one will likely make the free energy of association with E more negative.
(e)
Binding of S to E will decrease the entropy of these two proteins, and thus it is an
unfavorable reaction.
3-21

When the polymer X-X-X... is broken down into monomers, it is phosphorylyzed


rather than hydrolyzed, in the repeated reaction:
X-X-X... + P X-P + X-X... (reaction 1)
Given the G values of the reactions shown in Table Q3-21, what is the expected ratio
of X-phosphate (X-P) to free phosphate (P) at equilibrium for reaction 1?
(a)
(b)
(c)
(d)
(e)

1:106
1:104
1:1
104:1
106:1

Table Q3-21
X-X-X... + H2O X + X-X...
X + ATP X-P + ADP
ATP + H2O ADP + P

G = 4.5 kcal/mole
G = 2.8 kcal/mole
G = 7.3 kcal/mole

35

3-22

The following reactions take place in a cell located next to a blood vessel.
XY
Y + O2 Z + CO2

G = 10 kcal/mole
G = +0.5 kcal/mole

Normally, the blood vessel brings in oxygen and takes away carbon dioxide, but years of
overindulgence have taken their toll, and it has become completely clogged with
cholesterol, cutting off the blood supply. Which of the following molecules would be
expected to accumulate in large amounts?
(a)
X
(b)
Y
(c)
Z
(d)
Y and Z
(e)
X and Z
3-23

Figure Q3-23 shows graphs of energy diagrams for the unfavorable reaction X Y, the
favorable reaction Y Z, and the composite reaction X Z.
A.
B.

Draw lines near the composite reaction curve to indicate the free energy change
for each of the three reactions: G(XY), G(YZ), and G(XZ).
Is the composite reaction favorable or unfavorable?

Figure Q3-23

36

3-24

A.
B.

You are measuring the effect of temperature on the rate of an enzyme-catalyzed


reaction. If you plot reaction rate against temperature, which of the graphs in
Figure Q3-24 would you expect your plot to resemble?
Explain why temperature has this effect.

Figure Q3-24
3-25

Are the following statements about diffusion TRUE or FALSE?


A.
The diffusion rate for small molecules is much slower in the dense gel of the
cytoplasm than in pure water.
B.
The diffusion rate is faster for large macromolecules than for small molecules.
C.
The rate of diffusion determines the association rate of all binding reactions.
D.
The number of reactions per second that an enzyme catalyzes is independent of
diffusion.
E.
Diffusion causes a molecule to move along a fairly straight path.

3-26

Which of the following features generally tend to increase the number of catalytic events
an enzyme accomplishes per second?
(a)
Increased substrate concentration
(b)
More positive free energy of binding between enzyme and substrate
(c)
Decreased strength of binding between enzyme and product
(d)
Increased diffusion rate of substrate
(e)
Increased salt concentration

37

How We Know: Using Kinetics to Model and Manipulate Metabolic


Pathways
3-27

The product Y of an enzymatic reaction absorbs light at the wavelength 260 nm and the
product Z of another reaction absorbs at 340 nm, unlike the common substrate of the
reactions. A spectrophotometer was used to measure the initial rate of production of Y
and Z by the reactions shown in Figure Q3-27A.
The initial rates were measured for several independent reactions, all containing equal
amounts of enzyme A or enzyme B and differing amounts of substrate X. A graph was
made of the initial reaction rate (v) plotted against the concentration of X ([X]) (Figure
Q3-27). Given the data shown, are the following statements TRUE or FALSE?

Figure Q3-27
A.
B.
C.
D.
E.

Vmax(A) < Vmax(B).


At [X] = 10 M, the amounts of enzymes A and B limit the rate of reaction.
KM(A) > KM(B). (Hint: [S] = KM when v = Vmax/2.)
The turnover number of enzyme A is greater than the turnover number of enzyme
B. (Hint: Turnover number = Vmax/[enzyme].)
If [X] = 1 M and both enzymes are present, most of the substrate will be
converted to Y rather than Z.

38

3-28

Consider a description of an enzymatic reaction pathway that begins with the binding of
substrate S to enzyme E and ends with the release of product P from the enzyme.
E + S ES EP E + P
Under many circumstances,
KM = [E] [S] / [ES]
A.
B.

What proportion of enzyme molecules are bound to substrate when [S] = KM?
Recall that when [S] = KM, the reaction rate is Vmax/2. Does your answer to Part
A make sense in light of this rate information?

Activated Carrier Molecules and Biosynthesis


3-29

Which of the following is NOT a crucial benefit of using enzymes to catalyze biological
reactions?
(a)
Enzymes are highly selective in which reactions they catalyze.
(b)
Enzymes can drive an unfavorable reaction by coupling it to a favorable reaction,
either directly or via activated carrier molecules.
(c)
Enzymes make reactions occur faster than without catalysis.
(d)
Enzymes change the equilibrium of a reaction to make it more favorable.
(e)
The activity of enzymes can be modulated by inhibitors and other small molecules
to respond to the needs of the cell at each moment.

3-30

Consider an analogy between reaction coupling and money. In a simple economy, barter
provides a means of direct exchange of material goods. For example, the owner of a cow
may have excess milk and need eggs, whereas a chicken owner has excess eggs and
needs milk. Provided these two people are in close proximity and can communicate, they
may exchange or barter eggs for milk. But in a more complex economy, money serves as
a mediator for the exchanges of goods or services. For instance, the cow owner with
excess milk may not need other goods until three months from now or may want goods
from someone who does not need milk. In this case, the energy from providing milk to
the economy can be temporarily stored as money, which is a form of energy used for
many transactions in the economy. Using barter and money as analogies, describe two
mechanisms that can serve to drive an unfavorable chemical reaction in the cell.

3-31

A common means of providing energy to an energetically unfavorable reaction in a cell is


by
(a)
generation of a higher temperature by the cell.
(b)
transfer of a phosphate group from the substrate to ADP.
(c)
enzyme catalysis of the reaction.
(d)
coupling of ATP hydrolysis to the reaction.
(e)
coupling of the synthesis of ATP to the reaction.

39

3-32

An anhydride formed between a carboxylic acid and a phosphate (Figure Q3-32A) is


formed as a high-energy intermediate in some reactions in which ATP is used as the
energy source. Arsenate mimics phosphate and can also be incorporated into a similar
high-energy intermediate (Figure Q3-32B). The reaction profiles for the hydrolysis of
these two high-energy intermediates are given in Figure Q3-32C. What is the effect of
substituting arsenate for phosphate in this reaction?

Figure Q3-32
(a)
(b)
(c)
(d)
(e)

It forms a high-energy intermediate of lower energy.


It forms a high-energy intermediate of the same energy.
It decreases the stability of the high-energy intermediate.
It increases the stability of the high-energy intermediate.
It has no effect on the stability of the high-energy intermediate.

40

3-33

You are studying a biochemical pathway that requires ATP as an energy source. To your
dismay, the reactions soon stop, partly because the ATP is rapidly used up and partly
because an excess of ADP builds up and inhibits the enzymes involved. You are about to
give up when the following table from a biochemistry textbook catches your eye.
G

Hydrolysis reaction
creatine + ATP

enzyme A

enzyme B

ATP + H2O
pyrophosphate + H2O
glucose 6-phosphate + H2O

enzyme D

enzyme E

creatine phosphate + ADP

+ 3 kcal/mole

ADP + phosphate

7.3 kcal/mole

2 phosphate

7 kcal/mole

glucose + phosphate

3.3 kcal/mole

Which of the following reagents are most likely to revitalize your reaction?
(a)
A vast excess of ATP
(b)
Glucose 6-phosphate and enzyme E
(c)
Creatine phosphate and enzyme A
(d)
Pyrophosphate
(e)
Pyrophosphate and enzyme D
3-34

Which of the following statements is TRUE?


(a)
The oxidation of food molecules generates NAD+.
(b)
NADH and NADPH are found in mutually exclusive parts of the cell.
(c)
The ratio of NADPH:NADP+ is higher than the ratio of NADH:NAD+ because
each molecule of NADPH is a stronger reducing agent than a molecule of NADH.
(d)
Many enzymes can use NADPH and NADH interchangeably.
(e)
One molecule of NADPH can cause the transfer of two hydrogen atoms.

3-35

Match the activated carrier molecules in List 1 with the groups they transfer, selected
from List 2. Write the appropriate number beside each item in List 1.
List 1
A. ATP
B. Acetyl CoA
C. NADPH
D. Carboxylated biotin
E. S-adenosylmethionine

List 2
1. COO
2. e and H+
3. Glucose
4. PO43
5. CH3
6. Nucleotide
7. COCH3
8. Amino acid
41

3-36

Which of the following processes must be coupled to an energetically favorable reaction


in order to occur?
(a)
Conversion of protein into amino acids
(b)
Polymerization of amino acids into polypeptides
(c)
Conversion of glucose to carbon dioxide and water
(d)
Formation of a bilayer from phospholipids in water
(e)
The hydrolysis of ATP

3-37

The enzymes that catalyze the synthesis of macromolecules do not also catalyze their
breakdown by hydrolysis because
(a)
enzymes can catalyze reactions in only one direction.
(b)
hydrolysis is not an energetically favorable reaction.
(c)
the hydrolytic reaction is not the reverse of the reaction pathway that is used for
biosynthesis.
(d)
enzymes are destroyed immediately after synthesis is completed.
(e)
biosynthesis proceeds more rapidly than hydrolysis.

3-38

The energy required for the addition of a C nucleotide subunit (CMP) to a growing
polynucleotide chain is originally derived from the hydrolysis of ATP. Explain how this
is achieved.

42

Answers
3-1

No, you will not weigh four pounds more the next morning because only a small portion
of the mass of the food will form components of the body. Much of the mass of food is
released as CO2 that is breathed out into the atmosphere or is released into the
environment as waste products. Most of the energy contained in the chemical bonds of
the food molecules is converted to energy to maintain order among molecules in the
body, energy to move and think, and energy for anabolic or biosynthetic reactions to
rearrange the atoms from food into useful chemical structures (biological small molecules
and macromolecules). As part of the process, a great deal of the bond energy is also
converted to heat.

3-2

Choice (c) is the answer. Choice (a) is incorrect as no system, living or otherwise, can
defy the laws of thermodynamics. Choice (b) is incorrect as living organisms do not use
heat to power biochemical reactions. Heat is produced in the course of biochemical
reactions. Choice (d) is incorrect: although living organisms are causing a local decrease
in entropy, they cannot cause a decrease in the entropy of the universe as a whole as that
would be a thermodynamic impossibility. Choice (e) is incorrect as living organisms are
not closed systems.

3-3

(a)

3-4

Choice (c) is the answer. Catabolic reactions are the reactions in which a cell breaks
down food molecules, releasing the energy held within their chemical bonds. Choices
(a), (b), and (e) are energy-requiring processes.

3-5

Choice (b) is true. Photosynthesis harvests light energy from the sun and converts it into
chemical bond energy. Choice (a) is false because food molecules and oxygen produced
by photosynthesis are the sole source of the energy that powers nearly all living nonphotosynthetic organisms. Photosynthesis activates carrier molecules as intermediates in
the fixation of inorganic carbon dioxide into organic sugar molecules (so choices (c)
and (d) are false). By consuming carbon dioxide and producing oxygen photosynthesis
lessens global warming cause by the greenhouse effect (choice (e) is false).

3-6

A.
B.

By releasing heat to their environment, living things increase the entropy of the
environment, thus compensating for the decrease in entropy inside cells. Living
things, therefore, satisfy the second law of thermodynamics. They use special
pathways for all their reactions that allow them to be energetically favorable.

Sugars + O2 CO2 + H2O + heat energy + chemical bond energy


Both respiration and burning are reactions that use oxygen gas to oxidize complex
organic carbon molecules into CO2 + H2O. Burning is an uncontrolled oxidation
in which the energy is all dissipated as heat; respiration is a multi-step, controlled
oxidation that harnesses the energy in high-energy chemical bonds that are useful
for anabolic reactions of cells.

43

3-7

Aii; Bii; Ci; Dii. More reduced means having more electrons; gain of
electrons can result in an increased negative charge or a decreased positive charge and
can be due to an increase in the number of hydrogen atoms in a molecule.

3-8

A.
B.
C.

True. A redox reaction involves the complete or partial transfer of electrons from
one molecule or atom to another. The donor is oxidized and the recipient is
reduced in the reaction.
False. Hydrogenation is a special kind of reduction reaction, involving receipt of
an electron from a donor molecule and acquisition of a proton, usually from
water. Hydrogenation increases the number of C-H bonds in a molecule.
True. The diameter of an atom is influenced by the amount of negative charge, or
electron density, surrounding it. The more reduced an atom becomes, the larger
will be its electron cloud.

3-9

By definition, catalysis allows a reaction to occur more rapidly. Chemical reactions


occur only when there is a loss of free energy. Enzymes act more selectively than other
catalysts. A catalyst reduces the activation energy of a reaction.

3-10

(d)

3-11

(b)

3-12

(d)

Enzymes change only the activation energy of a reaction, not the free energy
difference between reactants and products and thus cannot change the equilibrium
concentrations of reactants and products.

44

3-13

See Figure A3-13.

Figure A3-13
A.
B.
C.

Activation energy is (a minus b).


Change in free energy for the reaction is (b minus c).
An enzyme will make the value of (a) smaller and leave the values for (b) and (c)
unchanged.

3-14

(d)

3-15

A.
B.

3-16

A2; B3; C1; D4. Graph 4 is the same as the graph for the original reaction in
terms of the relative energetic differences between substrates, transition states, and
products; the reaction diagram curve is simply positioned higher on the y-axis.

3-17

Choice (a) is correct. The value of G for the reaction A B is zero when there is no
net tendency for either A B or B A, which is the definition of equilibrium. G is a
constant and is thus always the same regardless of whether the reaction has reached
equilibrium or not. Thus choices (b) and (d) are incorrect. Choice (c) is an incorrect
answer; although a particular reaction might be at equilibrium when the concentration of
substrate equals that of product, this is not true for most reactions. Choice (e) is not a
definition of equilibrium, but of a reaction that is not occurring at all.

(a) and (c). Only reactions with a negative G can occur spontaneously.
Coupling of reaction (d) to either of the reactions (a) or (c) would provide an
overall negative G for the coupled reactions, thus enabling them to occur.

45

3-18

A.
B.

C.
D.

E.

Keq = [Y]/[X] = 5 M/50 M = 0.1.


The standard free energy change, G, is positive because Keq is less than 1.
Under standard conditions (equal concentrations of X and Y), the reaction X Y
is unfavorable.
G = 0.616 ln Keq = 0.616 ln 0.1 = (0.616) (2.3) = 1.4 kcal/mol.
Yes, the conversion is favorable because the value of [Y]/[X] is less than the
equilibrium value. However, the speed of the reaction cannot be determined from
the free energy difference. For example, combustion of this piece of paper is a
highly favorable reaction, yet it will not happen in our lifetime without a catalyst.
The cell may directly couple the unfavorable reaction to a second, energetically
favorable reaction whose negative G has a value larger than the positive G of
the X Y reaction; the coupled reaction will have a G equal to the sum of the
component reactions. Alternatively, more X will be converted to Y if the
concentration of Y drops; this may happen if Y is converted to Z in a second
reaction or if Y is exported from the cell or compartment where the X Y
reaction occurs.

3-19

(b)

The equilibrium constant measures the strength of the interaction between a


protein and its ligand and is independent of the concentration of either the protein
or the ligand. The strength of the protein-ligand interaction increases as the
number of noncovalent bonds between the two increases. The shape of the
binding site affects the ability of the protein side chains to interact with portions
of the substrate molecule. Both temperature and pH can disrupt noncovalent
bonds that not only affect the binding, but are also responsible for keeping the
protein folded and thus functional.

3-20

Choice (c) is true. The binding energy is the standard free energy of the binding reaction,
and thus is proportional to ln Keq. As the binding energy increases, the equilibrium
constant for the association reaction becomes larger. Choices (a) and (b) are false,
because although E binds S more tightly than it does I, some E molecules will still be
bound to I molecules in most circumstances; indeed, if the number of I molecules far
exceeds the number of S molecules, more E molecules will be present in an EI complex
than in an ES complex. Choice (d) is false; although not enough information is given to
be certain, it is likely that binding is normally strengthened by an ionic interaction
between a basic amino acid in I and an acidic amino acid in Ethus, if anything, the
binding energy will be reduced by the amino acid change and the free energy change will
be less (not more) negative. Choice (e) is false, because although the association of two
molecules often does decrease their own entropy, it can increase the entropy of other
molecules in the system. For example, heat release by the binding reaction can increase
the entropy of the system and its surroundings.

46

3-21

(c)

Reaction 1 can be written as the sum of the three reactions given, since the ATP
used in Step 2 is restored in Step 3.
X-X-X... + H2O X + X-X...
X + ATP X-P + ADP
ADP + P ATP + H2O

G = 4.5 kcal/mole
G = 2.8 kcal/mole
G = +7.3 kcal/mole

Since G values are additive, Gtotal = 0, and if G = 0, Keq = 1, meaning that


[products]/[reactants] = 1, and the ratio of X-P to P is 1:1.
3-22

(b)

The constant removal of CO2 and replenishment of O2 by the blood normally


drives the reaction Y Z. Therefore, when CO2 is allowed to accumulate and O2
drops, Y will accumulate. Because the G of the first reaction is very negative,
Y can accumulate to a very high level without causing significant amounts of X to
build up.

3-23

A.

See Figure A3-23 for correct labeling of figure.

Figure A3-23

3-24

B.

Favorable

A.
B.

Graph 1
By increasing thermal motion, increasing the temperature increases the rate of
diffusion of components and the number of collisions of sufficient energy to
overcome the activation energy. An increase in temperature will thus increase the
reaction rate initially. However, enzymes are proteins and are held together by
noncovalent interactions, so at very high temperatures, the enzyme will begin to
denature and the reaction rate will fall.

47

3-25

A.
B.
C.
D.
E.

False. The diffusion rate is almost as fast in the cytoplasm and in water.
False. The diffusion coefficient of a molecule decreases with increasing mass
(shape is also a factor).
False. Although some binding reactions are diffusion-limited, others require an
unusually energetic collision to overcome an energy barrier.
False. The diffusion rate influences how often an enzyme will encounter (and
thus bind) its substrate.
False. Diffusing molecules move in a random walk, in which they change
direction frequently after colliding with other molecules.

3-26

Choices (a), (c), and (d) are correct. The higher the concentration and diffusion rate of
substrate, the more frequently a free enzyme will collide with and bind its substrate. The
less tightly an enzyme binds its product, in general, the faster the product will dissociate
from the enzyme, leaving it free to bind a fresh substrate. Choice (b) is incorrect because
a more positive free energy of binding indicates it is less energetically favorable and thus
occurs less often. Choice (e) is likely to be false because many enzyme-substrate binding
interactions rely on ionic bonds that are weakened by high salt concentrations; in
addition, high salt concentrations can distort protein conformations.

3-27

A.
B.
C.
D.
E.

True
True
False
False
True

3-28

A.
B.

When [S] is substituted for KM in the equation, it becomes clear that [E] = [ES].
Thus, half of the enzyme molecules are free and half are bound to the substrate.
Yes. If half of the enzyme molecules are bound to the substrate, it makes intuitive
sense that the reaction rate is half of the maximum possible rate or half of the rate
observed when all of the enzyme molecules are bound to the substrate.

3-29

(d)

Enzymes cannot change the equilibrium of a reaction.

3-30

Barter is analogous to the direct coupling of a favorable to an unfavorable reaction by a


single enzyme. Money is analogous to the storage of energy from a favorable reaction in
the form of high-energy bonds in an activated carrier molecule. Such activated carrier
molecules can drive a huge variety of other unfavorable reactions in the cell, either by
being hydrolyzed to provide the needed energy for a reaction or by transferring the
activated chemical group to another molecule.

3-31

(d)

48

3-32

Choice (c) is correct. The activation energy of the arsenate compound is extremely low,
as can be seen from the reaction profile, meaning that its high-energy intermediate is very
unstable and will be spontaneously hydrolyzed more rapidly than the phosphate
compound. In fact, this hydrolysis occurs rapidly without enzyme catalysis, even in
cellular conditions. Thus choices (d) and (e) are false. Choices (a) and (b) are false as
more energy is released by the hydrolysis of the arsenoanhydride bond (as inferred by the
greater difference in energy level between reactants and products in Figure Q3-32) so, by
definition, the arsenoanhydride bond is said to have more energy than the
phosphoanhydride bond.

3-33

(c)

An excess of ATP will initially restore the reactions, but as ATP is hydrolyzed,
ADP will build up and inhibit the enzymes again. Pyrophosphate does not look
like ATP and is therefore unlikely to be used by the enzymes as an alternative
energy source. Pyrophosphate + enzyme D will just heat things up. What you
need is a high-energy source of phosphate that can convert ADP back to ATP.
Since the G of the reaction,
ATP + creatine ADP + creatine phosphate,
catalyzed by enzyme A is greater than zero, the addition of creatine phosphate and
enzyme A can be used to form ATP from ADP, regenerating the ATP while also
forming creatine as a waste product.

3-34

Choice (e) is correct. NADPH has two electrons and one proton more than NADP+ (like
NADH compared to NAD+) and donates both electrons. Protons are always present in
solution. So the recipient molecule effectively acquires two hydrogen atoms. Choice (a)
is false because oxidation of food molecules produces NADH, not NAD+. Chioce (b) is
false because NADH and NADPH can be found in the same parts of the cell, but are used
for different functions; this is possible because the enzymes that recognize one do not
recognize the other; thus choice (d) is also false. Choice (c) is false because the parts of
NADPH and NADH that participate in reduction are identical, and thus they both have
essentially the same reducing power.

3-35

A4 (phosphate group); B7 (acetyl group); C2; D1 (carboxyl group); E5


(methyl group).

3-36

(b)

Polymerization of amino acids into polypeptides lead to the formation of peptide


bonds that have higher energy than the free amino acids and also represents an
increase in order. Hence, it can only be brought about via an input of energy. The
other processes are thermodynamically spontaneous.

49

3-37

(c)

Hydrolysis is not the reverse of the reactions catalyzed by biosynthetic enzymes.


For instance, the reactions involved in RNA biosynthesis are:
polynucleotide(n) + NTP polynucleotide(n + 1) + PPi
PPi + H2O 2 Pi.
The reverse reactions are:
2 Pi PPi + H2O
PPi + polynucleotide(n + 1) polynucleotide(n) + NTP,
Not:
polynucleotide(n + 1) + H2O polynucleotide(n) + NMP (nucleoside
monophosphate),
which is the reaction by which RNA is hydrolyzed. (a) and (b) are untrue:
enzymes catalyze both forward and reverse reactions, and hydrolysis is an
energetically favorable reaction. (d) is untrue, since enzymes are unchanged by
participating in catalysis. Whether (e) is true or not for any particular reaction is
irrelevant.

3-38

In order to add the C nucleotide to the polynucleotide chain, it must be in the form of a
CTP (cytidine triphosphate). Conversion of CMP to CTP occurs by the sequential
transfer of two terminal phosphate groups from two molecules of ATP. Thus, hydroylysis
of ATP is coupled to phosphorylation of CMP and then to phosphorylation of CDP.
Subsequently, the reaction that adds CMP to the polynucleotide chain releases
pyrophosphate (PPi), which is hydrolyzed to inorganic phosphate; this favorable reaction
provides the energetic drive for the overall condensation (or polymerization) reaction.

50

CHAPTER 4
PROTEIN STRUCTURE AND FUNCTION
2009 Garland Science Publishing
3rd Edition

The Shape and Structure of Proteins


4-1 For each of the following sentences, fill in the blanks with the best word or phrase
selected from the chapter on Protein Structure and Function.
A protein is similar to a charm bracelet that has a linear linked chain or backbone
with charms hanging off at regular intervals. Similar to the amino acid sequence of
a protein, it is the sequence and identity of the charms that dictate the character of
the bracelet. The part of a protein that is analogous to the bracelet chain is called the
__________________ backbone. The parts of a protein that are analogous to the
charms are called the __________________.

4-2 Which of the following statements is TRUE?


(a)
(b)
(c)
(d)
(e)

Peptide bonds are the only covalent bonds that can link together two amino acids in proteins.
The polypeptide backbone is free to rotate about each peptide bond.
Nonpolar amino acids tend to be found in the interior of proteins.
The sequence of the atoms in the polypeptide backbone varies between different proteins.
A protein chain ends in a free amino group at the C-terminus.

51

4-3

For each of the following sentences, fill in the blanks with the best word or phrase
selected from the list below. Not all words or phrases will be used; each word or phrase
should be used only once.
A newly synthesized protein generally folds up into a
__________________ conformation. All the information required to
determine a proteins conformation is contained in its amino acid
__________________. On heating, a protein molecule will become
__________________ due to breakage of __________________ bonds.
On removal of urea, an unfolded protein can become
__________________. The final folded conformation adopted by a
protein is that of __________________ energy.
composition
covalent
denatured
highest

4-4

irreversible
lowest
noncovalent
renatured

reversible
sequence
stable
unstable

Typical folded proteins have a stability ranging from 7 to 15 kcal/mol at 37C. Stability
is a measure of the equilibrium between the folded (F) and unfolded (U) forms of the
protein, with the unfolded form having a greater free energy. See Figure Q4-4. For a
protein with a stability of 7.1 kcal/mol, calculate the fraction of protein that would be
unfolded at equilibrium at 37C. The equilibrium constant (Keq) is related to the free
energy (G) by the equation: Keq = 10G/1.42

Figure Q4-4
4-5

You wish to produce a human enzyme, protein A, by introducing its gene into bacteria.
The genetically engineered bacteria make large amounts of protein A, but it is in the form
of an insoluble aggregate with no enzymatic activity. Which of the following procedures
might help you to obtain soluble, enzymatically active protein? Explain your reasoning.
A.
Make the bacteria synthesize protein A at a slower rate and in smaller amounts.
B.
Dissolve the protein aggregate in urea, then dilute the solution and gradually
remove the urea.
C.
Treat the insoluble aggregate with a protease.
D.
Make the bacteria overproduce chaperone proteins in addition to protein A.
E.
Heat the protein aggregate to denature all proteins, then cool the mixture.

52

4-6

Which of the following statements about proteins is TRUE?


(a)
The three-dimensional structure of a protein can usually be predicted from
knowledge of its amino acid sequence.
(b)
Two proteins having similar amino acid sequences will often have similar shapes.
(c)
Proteins containing fewer than 100 amino acids cannot fold into stable structures.
(d)
Most proteins contain more than 2000 amino acids.
(e)
The detailed three-dimensional structure of a protein can usually be determined
by electron microscopy.

4-7

The helix and sheet are found in many different proteins because they are formed by
(a)
hydrogen bonding between the amino acid side chains most commonly found in
proteins.
(b)
noncovalent interactions between amino acid side chains and the polypeptide
backbone.
(c)
ionic interactions between charged amino acid side chains.
(d)
hydrogen bonding between atoms of the polypeptide backbone.
(e)
hydrophobic interactions between the many nonpolar amino acids.

4-8

A.

In the schematic diagram of a protein given in Figure Q4-8, label the three protein
strands that are linked together in a sheet with a b.

Figure Q4-8
B.

Is this sheet parallel or antiparallel?

53

4-9

For each polypeptide sequence listed, choose from the options given below to indicate
which secondary structure the sequence is most likely to form upon folding. The
nonpolar amino acids are italicized.
A.
B.
C.

Leu-gly-val-leu-ser-leu-phe-ser-gly-leu-met-trp-phe-phe-trp-ile
Leu-leu-gln-ser-ile-ala-ser-val-leu-gln-ser-leu-leu-cys-ala-ile
Thr-leu-asn-ile-ser-phe-gln-met-glu-leu-asp-val-ser-ile-arg-trp
amphipathic helix
amphipathic sheet

hydrophilic helix
hydrophilic sheet

hydrophobic helix

4-10

A helical structure
(a)
will contain two, three, four, or some other exact number of subunits per each turn
of the helix.
(b)
that is right-handed if viewed from one end will appear to be left-handed if
viewed from its other end.
(c)
can form only by joining together a string of identical protein molecules.
(d)
can form either within a single large molecule or from an assembly of separate
molecules.
(e)
is usually maintained entirely by covalent bonds.

4-11

Drawn below are segments of sheets, which are rigid pleated structures held together
by hydrogen bonds between the peptide backbones of adjacent strands (Figure Q4-11).
The amino acid side chains attached to the C carbons are omitted for clarity.

Figure Q4-11
A.
B.

Indicate whether each structure is parallel or antiparallel.


Draw the hydrogen bonds as dashed lines (- - - - - -).

54

4-12

For each of the following sentences, fill in the blanks with the best word or phrase
selected from the list below. Not all words or phrases will be used; each word or phrase
should be used only once.
The helices and sheets are examples of protein __________________
structure. A protein such as hemoglobin, which is composed of more than
one protein __________________, has __________________ structure.
A proteins amino acid sequence is known as its __________________
structure. A protein __________________ is the modular unit from which
many larger single-chain proteins are constructed. The three-dimensional
conformation of a protein is its __________________ structure.
allosteric
domain
helix

4-13

ligand
primary
quaternary

secondary
subunit
tertiary

You are digesting a protein 625 amino acids long with the enzymes Factor Xa and
thrombin, which are proteases that bind to and cut proteins at particular short sequences
of amino acids. You know the amino acid sequence of the protein and so can draw a map
of where factor Xa and thrombin should cut it (Figure Q4-13). You find, however, that
treatment with each of these proteases for an hour results in only partial digestion of the
protein, as summarized under the figure. List the segments (AE) of the protein that are
most likely to be folded into compact, stable domains.

Figure Q4-13
4-14

Calculate how many different amino acid sequences there are for a polypeptide chain 10
amino acids long.

4-15

All known proteins in cells adopt a single stable conformation because


(a)
any chain of amino acids can fold up into only one stable conformation.
(b)
protein chains that can adopt several different conformations have been weeded
out by natural selection.
(c)
chaperone proteins prevent the protein from adopting a preferred unstable
conformation.
(d)
they are complexed with other molecules that keep them in that one particular
conformation.
(e)
one conformation always has the most positive free energy.
55

4-16

A friend tells you that she has just discovered that the protein responsible for causing
dogs to chase cars is a member of the MAP protein kinase family. In response to your
blank stare, she adds that the yeast protein Fus3p, which is involved in response to a yeast
hormone is also a MAP kinase family member. Although you still have no idea of what
either a MAP kinase or Fus3p is, which one or more of the following statement(s) can
you safely predict to be TRUE? Explain your reasoning.
(a)
The dog protein and Fus3p have mostly similar amino acid sequences.
(b)
The dog protein and Fus3p catalyze the transfer of a phosphate group to another
molecule.
(c)
The dog protein phosphorylates the same type of molecule that Fus3p
phosphorylates.
(d)
The dog protein and Fus3p have identical three-dimensional structures.
(e)
The dog protein is involved in response to hormones.

4-17

A hemoglobin molecule
(a)
is composed of four protein domains.
(b)
is a dimer of polypeptide chains.
(c)
has two binding sites for nitrogen gas.
(d)
is composed of two different types of protein subunits.
(e)
is composed of four identical protein subunits.

4-18

When purified samples of protein Y and a mutant version of protein Y are both washed
through the same gel-filtration column, mutant protein Y runs through the column much
slower than the normal protein. Which one or more of the following changes in the
mutant protein is/are most likely to explain this result?
(a)
The loss of a binding site on the mutant protein surface through which protein Y
normally forms dimers
(b)
A change that results in the mutant protein acquiring an overall positive instead of
a negative charge
(c)
A change that results in mutant protein Y being larger than the normal protein
(d)
A change that results in mutant protein Y having a slightly different shape from
the normal protein
(e)
The loss of a binding crevice in the mutant protein Y for a small molecule ligand

4-19

Examine the three protein monomers in Figure Q4-19. From the arrangement of
complementary binding surfaces, which are indicated by similarly shaped protrusions and
invaginations, decide whether each monomer could assemble into a defined multimer, a
filament, or a sheet.

Figure Q4-19

56

4-20

For each of the following indicate whether the individual folded polypeptide chain forms
a globular (G) or fibrous (F) protein molecule.
A.
Keratin
B.
Lysozyme
C.
Elastin
D.
Collagen
E.
Hemoglobin
F.
Actin

4-21

SS bonds
(a)
are formed by the cross-linking of methionine residues.
(b)
are formed mainly in proteins that are retained within the cytosol.
(c)
stabilize but do not change a proteins final conformation.
(d)
can be broken by oxidation through agents such as mercaptoethanol.
(e)
rarely form in extracellular proteins.

How Proteins Work


4-22

Which of the following statements is FALSE?


(a)
The three dimensional structure of a protein dictates its function by determining
its binding specificity for other molecules.
(b)
Many proteins have more than one binding site.
(c)
Binding between protein and ligand generally involves noncovalent bonds.
(d)
Proteins are designed to bind their ligands as tightly as possible.
(e)
Changes in the amino acid sequence of a protein can decrease binding to a ligand,
even if the altered amino acid does not lie in the binding site for the ligand.

4-23

For each of the following sentences, fill in the blanks with the best word or phrase
selected from the list below. Not all words or phrases will be used; each word or phrase
should be used only once.
The human immune system produces __________________ of different
immunoglobulins, also called __________________, which enable the
immune system to recognize and fight germs by specifically binding one
or a few related __________________. The hypervariable structural
element that forms the ligand binding site is comprised of several
__________________. Purified antibodies are useful for a variety of
experimental purposes, including protein purification using
__________________ chromatography.
affinity
antibodies
antigens
strands

billions
coiled coils
hundreds
ion exchange

57

ligands
loops
size exclusion

4-24

For each functional class of enzymes in List 1, choose the kind of reaction it catalyzes
from List 2. Use each item in List 2 no more than once.
List 1
A. Kinase
B. Phosphatase
C. Polymerase
D. Hydrolase
E. Dehydrogenase
F. ATPase
G. Synthase
H. Protease

4-25

List 2
1. Oxidation of substrate
2. Reduction of substrate
3. Addition of phosphate group
4. Removal of phosphate group
5. Cleavage of substrate using water
6. Condensation reaction in anabolic pathway
7. Repeated condensation reactions with similar
building blocks to form a growing chain
8. Hydrolysis of nucleotide triphosphate
9. Hydrolysis of peptide bonds

For each of the following sentences, fill in the blanks with the best word or phrase
selected from the list below. Not all words or phrases will be used; each word or phrase
should be used only once.
Any substance that will bind to a protein is known as its
__________________. Enzymes bind their __________________ at the
__________________. The enzyme hexokinase is so specific that it reacts
with only one of the two __________________ of glucose. Enzymes
catalyze a chemical reaction by lowering the __________________,
because they provide conditions favorable for the formation of a
__________________ intermediate called the __________________.
Once the reaction is completed, the enzyme releases the
__________________ of the reaction.
activation energy
active site
free energy
high-energy

4-26

inhibitors
isomers
ligand
low-energy

products
substrates
transition state

One way that an enzyme can lower the activation energy required for a reaction is to bind
the substrate(s) and distort its structure so that the substrate more closely resembles the
transition state of the reaction. This mechanism will be facilitated if the shape and
chemical properties of the enzymes active site are more complementary to the transition
state than to the undistorted substrate, in other words, if the enzyme were to have higher
affinity for the transition state than for the substrate. Knowing this, your friend looked in
an organic chemistry textbook to identify a stable chemical that closely resembles the
transition state of a reaction that converts X into Y. She generated an antibody against
this transition state analog and mixed the antibody with chemical X. What do you think
might happen?

58

4-27

Which of the following statements is FALSE?


(a)
Polypeptide chains are never covalently linked to lipids or sugars.
(b)
Most vitamins bind noncovalently to the active sites of particular enzymes and
provide essential catalytic assistance.
(c)
Some catalytic functions would be difficult or impossible using only the
chemistry offered by the 20 amino acid side chains.
(d)
Small molecules tightly bound to enzymes can form transient bonds during
catalysis.
(e)
Proteins that interact with photons and gas molecules often do so by virtue of
tightly bound small molecules.

How Proteins Are Controlled


4-28

The biosynthetic pathway for the two amino acids E and H is shown schematically in
Figure Q4-28. You are able to show that E inhibits enzyme V, and H inhibits enzyme X.
Enzyme T is most likely to be subject to feedback inhibition by __________________
alone.

Figure Q4-28
(a)
(b)
(c)
(d)
(e)
4-29

A
B
C
E
H

In general, a ligand that binds to only one conformation of an allosteric protein will
stabilize the bound conformation. Oxygen binds to the oxy conformation of
hemoglobin, while carbon dioxide and the small organic molecule BPG bind to the
deoxy conformation, each at different sites. Which of the following statements is
FALSE?
(a)
A high concentration of carbon dioxide will stimulate binding of BPG to
hemoglobin.
(b)
A high concentration of oxygen will stimulate dissociation of BPG and carbon
dioxide from hemoglobin.
(c)
A high concentration of carbon dioxide will cause oxygen to dissociate.
(d)
In the presence of BPG, a lower concentration of carbon dioxide is required to
cause dissociation of oxygen.
(e)
A variant of hemoglobin that has a lower affinity for BPG will bind to oxygen
less tightly than normal hemoglobin in the presence of the same level of BPG.
59

4-30

The movement of a ciliated protozoan is controlled by a protein called RacerX. When


this protein binds to another protein found at the base of the cilia, it stimulates the cila to
beat faster and the protozoan to swim faster. This ciliar protein, Speed, can be
phosphorylated and can only bind to RacerX in its phosphorylated form. You have
identified the threonine residue at which Speed is phosphorylated and changed it to an
alanine residue. How would you expect the mutant protozoan to behave?
(a)
Swims fast occasionally
(b)
Always swims fast
(c)
Never swims fast
(d)
Switches rapidly back and forth between fast and slow swimming
(e)
Cannot move at all

4-31

Which of the following mutant protozoa would swim fast all of the time?
(a)
One lacking the protein kinase that phosphorylates the Speed protein
(b)
One lacking the Speed protein
(c)
One lacking the RacerX protein
(d)
One in which the protein phosphatase that dephosphorylates the Speed protein is
produced in much greater amounts than normal
(e)
One lacking the protein phosphatase

4-32

Some of the enzymes that oxidize sugars to yield useable cellular energy (for example,
ATP) are regulated by phosphorylation. For these enzymes, would you expect the
inactive form to be the phosphorylated form or the dephosphorylated form? Explain your
answer.

4-33

GTP-binding proteins
(a)
form a transient covalent bond with guanine nucleotides.
(b)
are generally activated by factors that increase their rate of GTP hydrolysis.
(c)
immediately release the GDP produced by GTP hydrolysis.
(d)
reset themselves by phosphorylating bound GDP.
(e)
do not readily exchange bound GDP for GTP unless stimulated to do so.

4-34

The Ras protein is a GTPase that functions in many growth-factor signaling pathways. In
its active form, with GTP bound, it transmits a downstream signal that leads to cell
proliferation; in its inactive form, with GDP bound, the signal is not transmitted.
Mutations in the gene for Ras are found in many cancers. Of the choices below, which
alteration of Ras activity is most likely to contribute to the uncontrolled growth of cancer
cells?
(a)
A change that prevents Ras from being made.
(b)
A change that increases the affinity of Ras for GDP.
(c)
A change that decreases the affinity of Ras for GTP.
(d)
A change that decreases the affinity of Ras for its downstream targets.
(e)
A change that decreases the rate of hydrolysis of GTP by Ras.

60

4-35

A molecule of the motor protein Winnebago, supplied with ATP, is moving along a
microtubule in the direction shown in Figure Q4-35.

Figure Q4-35
What will happen if you suddenly remove all ATP from the system by adding an enzyme
that hydrolyzes ATP?
(a)
No change: Winnebago will continue to move from point A to point B.
(b)
Winnebago will wander back and forth along the microtubule.
(c)
Winnebago will move backwards (towards point A instead of point B).
(d)
Winnebago will stall on the microtubule.
(e)
Winnebago will continue to move from point A to point B, but at a slower rate.
4-36

Assembling the individual enzymes required for a multistep process into a protein
machine is likely to increase the efficiency with which the entire process is carried out in
all of the following ways EXCEPT by
(a)
increasing the rate at which the individual enzymes encounter their substrates.
(b)
ordering the reactions sequentially.
(c)
increasing the Vmax of the individual enzymes.
(d)
coordinating the regulation of the individual enzymes.
(e)
coordinating the movement of the enzymes.

4-37

Explain how new proteomic approaches may advance the goal of being able to predict the
three-dimensional structure of a protein from its amino acid sequence. Why is this an
important goal?

61

Answers
4-1

A protein is similar to a charm bracelet that has a linear linked chain or backbone with charms
hanging off at regular intervals. Similar to the amino acid sequence of a protein, it is the
sequence and identity of the charms that dictate the character of the bracelet. The part of a
protein that is analogous to the bracelet chain is called the polypeptide backbone. The parts
of a protein that are analogous to the charms are called the side chains.

4-2

Choice (c) is the answer. Choice (a) is untrue, as some proteins also contain covalent
disulfide bonds (SS bonds) linking two amino acids. Choice (b) is untrue, as the
peptide bond is rigid. Choice (d) is untrue, as the sequence of atoms in the polypeptide
backbone itself is always the same from protein to protein; it is the order of the amino
acid side chains that differs. Choice (e) is untrue, as a protein chain has a carboxyl group
at its C-terminus.

4-3

A newly synthesized protein generally folds up into a stable conformation. All the
information required to determine a proteins conformation is contained in its amino acid
sequence. On heating, a protein molecule will become denatured due to breakage of
noncovalent bonds. On removal of urea, an unfolded protein can become renatured.
The final folded conformation adopted by a protein is that of lowest energy.

4-4

The G of the unfolding reaction is equal to the stability of the protein, 7.1 kcal/mol. At
equilibrium, the ratio of unfolded to folded protein is
Keq = [U]/[F] = 10G/1.42 = 107.1/1.42 = 105.
Thus, one molecule in 100,000 is unfolded.

62

4-5

Choices A, B, and Dare all worth trying. Some proteins require molecular chaperones in
order to fold properly within the environment of the cell. In the absence of chaperones, a
partly folded polypeptide chain has exposed amino acids that can form noncovalent
bonds with other regions of the protein itself and with other proteins, thus causing
nonspecific aggregation of proteins.
A.
Since the protein you are expressing in bacteria is being made in large quantities,
it is possible that there are not enough chaperone molecules in the bacterium to
fold the protein. Expressing the protein slowly and at lower levels might increase
the amount of properly folded protein.
B.
Removing the urea slowly and gradually often allows the protein to refold.
Presumably, under less crowded conditions, the protein should be able to refold
into its proper conformation.
C.
Treating the aggregate with a protease, which cleaves peptide bonds, will probably
solubilize the protein by trimming it into pieces that do not interact as strongly with
one another; however, chopping up the protein will also destroy its enzymatic activity.
D.
Overexpressing chaperone proteins might increase the amount of properly folded
protein. Urea should solubilize the protein and completely unfold it.
E.
Heating can lead to the partial denaturation and aggregation of proteins to form a
solid gelatinous mass, as when cooking an egg white, and rarely helps solubilize
proteins.

4-6

(b)

4-7

(d)

4-8

A.

Since a proteins three-dimensional structure is determined by its amino acid


sequence, proteins with similar amino acid sequences will often have very similar
shapes.

See Figure A4-8.

Figure A4-8
B.

Antiparallel
63

4-9

A.

B.

C.

4-10

(d)

4-11

A.
B.

Hydrophobic helix. Nearly all of the amino acid side chains in this sequence
are nonpolar or hydrophobic, which favors the only hydrophobic option given in
the list.
Amphipathic helix. In an ideal helix, there are 3.6 residues per complete
turn. Thus, an amphipathic helix with one hydrophobic side and one hydrophilic
side will have, minimally, nonpolar side chains (N) repeating every third then
next fourth amino acid: NxxNxxxNxxNxxxN. Polar side chains (P) will exhibit
the same pattern, but shifted relative to the nonpolar side chains: for example,
xxPxxxPxxPxxxPxxP.
Amphipathic sheet. Due to the zig-zag like structure of a sheet, a sequence
with alternating nonpolar and polar side chains may form an amphipathic sheet
that is hydrophobic on one side and hydrophilic on the other.

(A) is parallel and (B) is antiparallel


See Figure A4-11.

Figure A4-11
4-12

The helices and sheets are examples of protein secondary structure. A protein such
as hemoglobin, which is composed of more than one protein subunit, has quaternary
structure. A proteins amino acid sequence is known as its primary structure. A protein
domain is the modular unit from which many larger single-chain proteins are
constructed. The three-dimensional conformation of a protein is its tertiary structure.

64

4-13

Segments B and D. To cut the protein chain, Factor Xa and thrombin must bind to their
preferred cutting sites. If these sites are folded into the interior of a stable protein domain,
it will be much more difficult for the proteases to gain access to them than if they are part
of a relatively unstructured part of the chain. Hence, sites that are folded inside of a
protein domain are protected from cleavage by a protease. From the sizes of the
fragments produced by digestion of the protein with Factor Xa, we can conclude that the
enzyme does not cut at the sites in regions B or D, although it does cut in region E. From
the sizes of the fragments produced by thrombin, we can conclude that this enzyme cuts
at the sites in A, C, and E. Therefore, the segments of the protein that are most likely to
be folded into compact stable domains are B and D.

4-14

The quantity 2010 = approximately 1013.

4-15

(b)

4-16

Choices (a) and (b) are the answers. Members of the same protein family have similar
protein sequences, similar three-dimensional structures, and roughly similar chemical
activities (for example, all of the serine proteases catalyze the cleavage of a peptide
bond). So if the dog protein is a MAP protein kinase, it is similar in sequence to other
MAP kinases (choice (a)) and most likely has kinase activity (the transfer of a phosphate
group from ATP to another molecule) (choice (b)). However, the actual substrates and
the physiological function of proteins in the same family can differ quite markedly, so it
is unlikely that the dog protein phosphorylates the same type of molecule that Fus3p does
or is involved in the same type of response that Fus3p mediates.

4-17

(d)

4-18

Choice (a) is the answer. Dimers formed by a normal protein will run through the gelfiltration column faster than a mutant protein Y monomer. Choices (b) and (e) are
unlikely, as gel-filtration columns separate proteins on the basis of size, not charge or
affinity for small molecules. Choice (c) is unlikely, since if the mutant protein were
larger than normal it would be less able to enter the porous beads and would run through
the column at a faster speed than the normal protein. Choice (d) is unlikely, since a small
change in shape without a change in size would be unlikely to have a major effect on the
behavior of a protein in gel-filtration column.

4-19

A.
B.
C.

4-20

AF; BG; CF; DF; EG; FG

4-21

Choice (c) is the answer. Choice (a) is incorrect since SS bonds are formed between
cysteines. Choice (b) is incorrect, as they are formed mainly in extracellular proteins.
Choice (d) is incorrect; they are broken by mercaptoethanol, but by reduction not
oxidation. Choice (e) is incorrect for the reason stated in choice (b).

Defined multimer with four subunits, called a tetramer.


Sheet
Filament

65

4-22

(d)

False. Most proteins need to release their ligand at some point. For example,
hemoglobin would be useless as a carrier of oxygen if it never released the
oxygen to the tissues that need it. In addition, enzymes could not be catalysts if
they did not release the product after its formation from substrate.

4-23

The human immune system produces billions of different immunoglobulins, also called
antibodies, which enable the immune system to recognize and fight germs by
specifically binding one or a few related antigens. The hypervariable structural element
that forms the ligand binding site is comprised of several loops. Purified antibodies are
useful for a variety of experimental purposes, including protein purification using affinity
chromatography.

4-24

A3; B4; C7; D5; E1; F8; G6; H9

4-25

Any substance that will bind to a protein is known as its ligand. Enzymes bind their
substrates (or inhibitors) at the active site. The enzyme hexokinase is so specific that it
reacts with only one of the two isomers of glucose. Enzymes catalyze a chemical
reaction by lowering the activation energy, because they provide conditions favorable
for the formation of a high-energy intermediate called the transition state. Once the
reaction is completed, the enzyme releases the products of the reaction.

4-26

If your friend was lucky, she made a catalytic antibody that catalyzed the conversion of
X into Y. Such catalytic antibodies have been isolated and shown to catalyze a variety of
reactions, but with lower efficiency than genuine enzymes.

4-27

(a)

4-28

(c)

If E alone inhibited T, then it would be possible to shut down the pathway even if
H were in low abundance. Likewise, if H alone inhibited T, it would be possible
to shut down the pathway even if E were in low abundance. An enzyme is
generally not inhibited by its substrate. Levels of C will build up only if both E
and H are abundant and have inhibited V and X. It is more likely that C alone
rather than B alone will inhibit T, since B will accumulate only after C has done
so.

4-29

(e)

Since carbon dioxide and BPG stabilize the form of hemoglobin that oxygen
cannot bind to, either BPG or carbon dioxide will stimulate the dissociation of
oxygen; likewise, a high concentration of oxygen will stimulate the dissociation
of carbon dioxide and BPG. Since BPG and carbon dioxide both bind to and
stabilize the same form of hemoglobin, these two ligands should help each other
bind. Since BPG binding stimulates the dissociation of oxygen, a variant of
hemoglobin that does not bind BPG well should bind more tightly to oxygen.

66

4-30

(c)

The alanine side chain has no hydroxyl (OH) group and therefore cannot be
phosphorylated. Because the altered Speed protein cannot be phosphorylated, it
can never bind RacerX. When RacerX is not bound to Speed, the cilia beat more
slowly and thus the mutant protozoan would never swim fast.

4-31

Choice (e) is the answer. The lack of the protein phosphatase would mean that the Speed
protein could remain phosphorylated all the time, causing the organism to swim fast. A
mutant missing RacerX (choice (c)) would not be able to swim fast at all and neither
would one missing the protein kinase that phosphorylates Speed (choice (a)) or one
lacking the Speed protein (choice (b)). One that overproduced the protein phosphatase
(choice (d)) would keep the Speed protein permanently dephosphorylated and thus would
also be unable to swim fast.

4-32

In general the inactive form is the phosphorylated form. The main purpose of glycolysis
and the citric acid cycle is to generate ATP; thus the enzymes are inactive when ATP is
high and active when ATP is low. It makes sense that cells would not want to have to
phosphorylate their enzymes to turn them on when ATP levels are already low, since
phosphorylation requires ATP.

4-33

Choice (e) is the answer. GTP-binding proteins generally hydrolyze GTP and then retain
the bound GDP until stimulated to exchange GDP for GTP by some other protein. The
conformational change driven by hydrolysis, therefore, is due to the loss of a single
phosphate group, not the whole guanine nucleotide (choice (c)). G proteins do not form
covalent intermediates with either GTP or GDP (choice (a)). Since the GTP-bound form
is usually the active form, a factor that stimulates hydrolysis will inhibit the protein
(choice (b)). G proteins are reset by nucleotide exchange, not by rephosphorylation of
bound GDP (choice (d)).

4-34

(e)

This choice will increase the amount of Ras that is in the GTP-bound state and
thus increase the strength of the proliferative signal. All of the other changes will
decrease the strength of the proliferative signal sent through the pathway by Ras.

4-35

(d)

Motor proteins that are capable of unidirectional movement require ATP (or GTP)
hydrolysis to drive them in one direction. This is because the conformational
change is coupled to ATP hydrolysis, such that movement in the reverse direction
requires ATP synthesis. Hence, if ATP is depleted, the protein will completely
stop moving, being unable to move forward for lack of ATP and unable to move
backward because ATP synthesis is thermodynamically unfavorable.

67

4-36

(c)

If the product of one enzyme is the substrate for another, assembling the enzymes
into a machine will bring the enzymes closer to their substrates because the
product will have to diffuse only a short distance to the next enzyme. If the
enzymes are properly arranged spatially, one can easily imagine how the machine
could also facilitate the ordering of reactions. Gathering the enzymes into a
complex also makes it easier to regulate and move all of the enzymes together. A
machine is only as good as each of its parts, however, and if an enzyme has a low
turnover number, complexing it with other proteins is unlikely to increase the
enzymes ability to convert substrate into product.

4-37

It is hoped that large-scale analyses of many protein structures simultaneously will


quickly complete the list of all protein folding patterns and all protein domains that
evolved in living organisms. The identification of the thousands of different protein
foldsthe structural units that form the basis of all proteinsmay allow elucidation of
the rules that determine the conformation adopted by each amino acid sequence. Then,
when a scientist identifies a protein responsible for a particular disease or discovers a
protein in a new organism, it will be immediately possible to hypothesize its structure and
thus its function.

68

CHAPTER 5
DNA AND CHROMOSOMES
2009 Garland Science Publishing
3rd Edition

How We Know: Genes are Made of DNA


5-1 Using terms from the list below, fill in the blanks in the following brief description of the
experiment with Streptococcus pneumoniae that identified which biological molecule
carries heritable genetic information. Some terms may be used more than once.
Cell-free extracts from S strain cells of S. pneumoniae were fractionated to
__________________ DNA, RNA, protein, and other cell components. Each fraction
was then mixed with __________________ cells of S. pneumoniae. Its ability to
change these into cells with __________________ properties resembling the
__________________ cells was tested by injecting the mixture into mice. Only the
fraction containing __________________ was able to __________________ the
__________________ cells to __________________ (or __________________ )
cells that could kill mice.
carbohydrate
DNA
identify
label

lipid
nonpathogenic
pathogenic
purify

The Structure and Function of DNA


5-2 In a DNA double helix
(a)
(b)
(c)
(d)
(e)

the two DNA strands are identical.


purines pair with purines.
thymine pairs with cytosine.
the two DNA strands run antiparallel.
the nucleotides are ribonucleotides.

69

R strain
RNA
S strain
transform

5-3

On the diagram of a small portion of a DNA molecule in Figure Q5-3, match the labels
below to the numbered label lines.

Figure Q5-3
(a)
(b)
(c)
(d)
(e)
(f)
5-4

Base
Sugar
Phosphate
Hydrogen bond
5 end
3 end

The structures of the four bases in DNA are given in Figure Q5-4.

Figure Q5-4
A.
B.

Which are purines and which are pyrimidines?


Which bases pair with each other in double-stranded DNA?
70

5-5

Using the structures in Figure Q5-4 as a guide, sketch the hydrogen bonds that form when
the appropriate bases form base pairs in DNA. Hint: The bases in the figure are all drawn
with the NH that attaches to the sugar at the bottom of the structure.

5-6

A ribbon model of the DNA double helix showing the major and minor grooves is
reproduced in Figure Q5-6.

Figure Q5-6
A.
B.
5-7

Draw on the figure to indicate the length of a single full helical turn.
How many base pairs per turn does a DNA helix have?

Given the sequence of one strand of a DNA helix:


5-GCATTCGTGGGTAG-3
give the sequence of the complementary strand and label the 5 and 3 ends.

5-8

Which of the following sequences can fully base-pair with itself?


A.
5-AAGCCGAA-3
B.
5-AAGCCGTT-3
C.
5-AAGCGCAA-3
D.
5-AAGCGCTT-3
E.
5-AATTGGCC-3

5-9

The DNA from two different species can often be distinguished by a difference in the
(a)
ratio of A + T to G + C.
(b)
ratio of A + G to C + T.
(c)
ratio of sugar to phosphate.
(d)
presence of bases other than A, G, C, and T.
(e)
number of strands in the helix.

71

5-10

When double-stranded DNA is heated, the two strands separate into single strands in a
process called melting or denaturation. The temperature at which half of the duplex
DNA molecules are intact and half have melted is defined as the Tm.
A.
Do you think Tm is a constant or is dependent on other small molecules in the
solution? Do you think high salt concentrations increase, decrease, or have no effect
on Tm?
B.
Under standard conditions, the expected melting temperature in degrees Celsius can be
calculated using the equation Tm = 59.9 + 0.41 [%(G + C)] [675/length of duplex].
Does the Tm increase or decrease if there are more G + C (and thus fewer A + T) base
pairs? Does the Tm increase or decrease as the length of DNA increases? Why?
C.
Calculate the predicted Tm for a stretch of double helix that is 100 nucleotides
long and contains 50% G + C content.

5-11

Consider the structure of the DNA double helix.


A.
You and a friend want to split a double-stranded DNA molecule so you each have
half. Is it better to cut the length of DNA in half so each person has a shorter
length, or to separate the strands and each take one strand? Explain.
B.
In the original 1953 publication describing the discovery of the structure of DNA,
Watson and Crick wrote, It has not escaped our notice that the specific pairings
we have postulated immediately suggest a possible copying mechanism for the
genetic material. What did they mean?

5-12

A.

B.

In principle what would be the minimum number of consecutive nucleotides necessary


to correspond to a single amino acid to produce a workable genetic code? Assume that
each amino acid is encoded by the same number of nucleotides. Explain your
reasoning.
On average how often would the nucleotide sequence CGATTG occur in a DNA
strand 4000 bases long? Explain your reasoning.

72

The Structure of Eucaryotic Chromosomes


5-13

For each of the following sentences, fill in the blanks with the best word or phrase
selected from the list below. Not all words or phrases will be used; each word or phrase
should be used only once.
In eucaryotic __________________, DNA is complexed with proteins to
form __________________. The paternal and maternal copies of human
Chromosome 1 are __________________, whereas the paternal copy of
Chromosome 1 and the maternal copy of Chromosome 3 are
__________________. Cytogeneticists can determine large-scale
chromosomal abnormalities by looking at a patients __________________.
Fluorescent molecules can be used to paint a chromosome by virtue of DNA
__________________, and thereby to identify each chromosome using
microscopy.
bands
chromatin
chromosomes
condensation

5-14

A.
B.
C.

5-15

extended
homologous
hybridization
karyotype

kinetochore
nonhomologous

Define a gene.
Consider two different species of yeast that have similar genome size. Is it likely
that they contain a similar number of genes? A similar number of chromosomes?
Figure 5-15 in the textbook shows the G + C content and genes found along a
single chromosome. Is there any relationship between the G + C content and the
locations of genes?

The human genome is comprised of 23 pairs of chromosomes found in nearly every cell
in the body. Answer the quantitative questions below by choosing one of the numbers in
the following list:
23
46
A.
B.
C.

69
92

>200
>109

How many centromeres are in each cell? What is the main function of the
centromere?
How many telomeres per cell? What is their main function?
How many replication origins per cell? What is their main function?

73

5-16

For each of the following sentences, fill in the blanks with the best word or phrase
selected from the list below. Not all words or phrases will be used; each word or phrase
should be used only once.
Each chromosome is a single molecule of __________________ whose
extraordinarily long length can be compacted by as much as
__________________-fold during __________________ and tenfold
more during __________________. This is accomplished by binding to
__________________ that help package the DNA in an orderly manner so
it can fit in the small space delimited by the __________________. The
structure of the DNA-protein complex, called __________________, is
highly __________________ over time.
10,000
100
1000
cell cycle
cell wall
chromatin

5-17

chromosome
different
DNA
dynamic
interphase
lipids

mitosis
nuclear envelope
nucleolus
proteins
similar
static

For each of the following sentences, choose one of the options enclosed in square
brackets to make a correct statement about nucleosomes.
A.
Nucleosomes are present in [procaryotic/eucaryotic] chromosomes, but not in
[procaryotic/eucaryotic] chromosomes.
B.
A nucleosome contains two molecules each of histones [H1 and H2A/H2A and
H2B] as well as histones H3 and H4.
C.
A nucleosome core particle contains a core of histone with DNA wrapped around
it approximately [twice/three times/four times].
D.
Nucleosomes are aided in their formation by the high proportion of
[acidic/basic/polar] amino acids in histone proteins.
E.
Nucleosome formation compacts the DNA into approximately [one-third/onehundredth/one-thousandth] of its original length.

74

5-18

For each of the following sentences, fill in the blanks with the best word or phrase
selected from the list below. Not all words or phrases will be used; each word or phrase
should be used only once.
Interphase chromosomes contain both darkly staining
__________________ and more lightly staining __________________.
Genes that are being transcribed are thought to be packaged in a
__________________ condensed type of euchromatin. Nucleosome core
particles are separated from each other by stretches of
__________________ DNA. A string of nucleosomes coils up with the
help of __________________ to form the more compact structure of the
__________________. The __________________ model describes the
structure of the 30 nm fiber. The 30 nm chromatin fiber is further
compacted by the formation of __________________ that emanate from a
central __________________.
30 nm fiber
active chromatin
axis
beads-on-a-string
euchromatin

heterochromatin
histone H1
histone H3
histone H4
less

linker
loops
more
synaptic complex
zigzag

5-19

In which of the following instances can the state of chromatin packing differ? Explain
your reasoning.
A.
Between different cells of the same organism.
B.
In different stages of the cell cycle.
C.
In different parts of the same chromosome.
D.
In different members of a pair of homologous chromosomes.

5-20

Evidence suggests that the replication of DNA packaged into heterochromatin occurs
later than the replication of other chromosomal DNA. What is the simplest possible
explanation for this phenomenon?

5-21

If the mottled coloring of calico cats is due to X-chromosome inactivation, which of the
following statements will be TRUE?
(a)
Calico cats can be male or female.
(b)
Female calico cats will be the same color as their mother.
(c)
The mottled color is due to X chromosomes repeatedly switching back and forth
between active and inactive states during development.
(d)
Calico cats with identical patterns will be rare.
(e)
Coat color is determined by a single gene.

75

5-22

A.

B.

5-23

Chromatin remodeling complexes act to regulate gene expression and other DNA
processes that involve access to DNA. A characteristic component of chromatin
remodeling complexes is a powerful ATPase. Why is ATPase activity needed?
Imagine that you have developed a method to isolate all of the histones bound to a
single human chromosome. You then examine histones from the inactive X
chromosome in an individual female and compare them to histones from her
active X chromosome. Do you think these two sets of histones will be the same?
Explain.

Your friend is working in a lab to study how cells adapt to growth on different carbon
sources. He grew half of his cells in the presence of glucose and the other half in the
presence of galactose. Then he harvested the cells and isolated their DNA using a gentle
procedure that leaves nucleosomes and some higher order chromatin structures intact. He
treated the DNA briefly with a low concentration of M-nuclease, a special enzyme that
easily degrades protein-free stretches of DNA. After removing all the proteins, he
separated the resulting DNA on the basis of length. Finally, he used a procedure to
visualize only those DNA fragments from a region near a particular gene called Sweetie
or another gene called Salty. The separated DNA fragments are shown in Figure Q5-23.
Each vertical column, called a lane, is from a different sample. DNA spots near the top
of the figure represent DNA molecules that are longer than those near the bottom. Darker
spots contain more DNA than fainter spots. The lanes are as follows:
1.
2.
3.
4.
5.

marker containing known DNA fragments of indicated lengths.


grew cells in glucose, visualized DNA near Sweetie gene.
grew cells in galactose, visualized DNA near Sweetie gene.
grew cells in glucose, visualized DNA near Salty gene.
grew cells in galactose, visualized DNA near Salty gene.

Figure Q5-23
76

A.
B.
C.

D.
E.

The lowest spot (as observed in lanes 2, 4, and 5) has a length of about 150
nucleotides. Can you propose what it is and how it arose?
What are the spots with longer lengths? Why is there a ladder of spots?
Notice the faint spots and extensive smearing in lane 3, suggesting the DNA could
be cut almost anywhere near the Sweetie gene after growth of the cells in
galactose. This was not observed in the other lanes. What probably happened to
the DNA to change the pattern between lanes 2 and 3?
What kinds of enzymes might have been involved in changing the chromatin
structure from lane 2 into lane 3?
Do you think that gene expression of Sweetie is higher, lower, or the same in
galactose compared to glucose? What about Salty?

77

Answers
5-1

Cell-free extracts from S strain cells of S. pneumoniae were fractionated to purify DNA,
RNA, protein, and other cell components. Each fraction was then mixed with R strain
cells of S. pneumoniae. Its ability to change these into cells with pathogenic properties
resembling the S strain cells was tested by injecting the mixture into mice. Only the
fraction containing DNA was able to transform the R strain cells to pathogenic (or S
strain) cells that could kill mice.

5-2

(d)

5-3

(a)6; (b)3; (c)2; (d)5; (e)1; (f)4

5-4

A.
B.

5-5

See Figure A5-5

Adenine and guanine are purines; cytosine and thymine are pyrimidines.
Cytosine pairs with guanine and adenine with thymine.

Figure A5-5

78

5-6

A.

See Figure A5-6.

Figure A5-6
B.

There are approximately 10 base pairs per turn when the DNA has the standard
conformation.

5-7

5 CTACCCACGAATGC 3.

5-8

D.

5-9

Choice (a) is the answer. Since the sequence of nucleotides in the DNAs from different
species varies considerably, the ratio A-T to G-C base pairs will vary as well. Yet since
A must always pair with T and G with C, the ratio of A+G to C+T must always be 1
(choice (b) is therefore incorrect). Choice (c) is incorrect, since the DNA backbone
always contains a 1:1 ratio of sugar to phosphate. Choice (d) is incorrect, as all living
organisms contain only A, G, C, and T in their DNA (but modifications of these bases
occur in some organisms and under some circumstances, after their incorporation into the
nucleotide chain). Choice (e) is incorrect because all genomic DNA is double stranded
(except in some viruses).

5-AAGCGCTT-3
3-TTCGCGAA-5
None of the other sequences will base-pair with themselves. This double-stranded
DNA molecule has the same sequence whether read forward or backward, which
is why it is described as palindromic. Note that the reading direction is always
defined as 5 to 3. An example of a palindromic sentence is Step on no pets.

79

5-10

A.

B.

C.

5-11

A.

B.

5-12

A.

B.

5-13

Tm depends on the identity and concentration of other molecules in the solution.


High salt concentrations are more effective at shielding the two negatively
charged phosphate-sugar backbones in the double helix from each other, so the
two strands repel each other less strongly. Thus, a high salt concentration
stabilizes the duplex and increases the melting temperature.
The Tm increases as the proportion of G + C bases increases and as the length
increases. The thermal energy required for melting depends on how many
hydrogen bonds between the strands must be broken. Each G-C base pair
contributes three hydrogen bonds, whereas an A-T base pair contributes only two.
Inserting values into the equation in part (B) gives Tm = 59.9 + (0.41 50)
(675/100) = 73.65C, which is about twice the normal temperature of the human
body and nearly too hot to touch.
It is better to separate the stands and each take a single strand, because all of the
information found in the original molecule is preserved in a full-length single
strand but not in a half-length double-stranded molecule.
Watson and Crick meant that the complementary base pairing of the strands
allows a single strand to contain all of the information necessary to direct the
synthesis of a new complementary strand.
As there are 20 amino acids used in proteins, each amino acid would have to be
encoded by a minimum of three nucleotides. For example, a code of two
consecutive nucleotides could specify a maximum of 16 (42) different amino
acids, excluding stop and start signals. A code of three consecutive nucleotides
has 64(43) different members and thus can easily accommodate the 20 amino
acids plus a signal to stop protein synthesis.
As 46 (= 4096) different sequences of six nucleotides can occur in DNA, any
given sequence of six nucleotides would occur on average once in a DNA strand
4000 bases long.

In eucaryotic chromosomes, DNA is complexed with proteins to form chromatin. The


paternal and maternal copies of human Chromosome 1 are homologous, whereas the
paternal copy of Chromosome 1 and the maternal copy of Chromosome 3 are
nonhomologous. Cytogeneticists can determine large-scale chromosomal abnormalities
by looking at a patients karyotype. Fluorescent molecules can be used to paint a
chromosome by virtue of DNA hybridization, and thereby to identify each chromosome
using microscopy.

80

5-14

A.

B.

C.

5-15

A.

B.

C.

A gene is a segment of DNA that stores the information required to specify the
particular sequence found in a protein (or, in some cases, the sequence of a
structural or catalytic RNA).
A similar genome size indicates relatively little about the number of genes and
virtually nothing about the number of chromosomes. For example, the commonly
studied yeasts Saccharomyces cerevisiae (Sc) and Schizosaccharomyces pombe
(Sp) are separated by roughly 400 million years of evolution, and both have a
genome of 14 million base pairs. Yet Sc has 6500 genes packaged into 16
chromosomes and Sp has 4800 genes in 3 chromosomes.
Regions of the chromosome with a high density of genes tend to have about 50%
G + C, whereas those with few genes tend to have a lower G + C content. This is
generally true in most organisms.
There are 46 centromeres per cell, one on each chromosome. The centromeres
play a key role in the distribution of chromosomes to daughter cells during
mitosis.
There are 92 telomeres per cell, two on each chromosome. Telomeres serve to
protect the ends of chromosomes and to enable complete replication of the DNA
of each chromosome all the way to its tips.
There are far more than 200 replication origins in a human cell, probably around
10,000. These DNA sequences direct the initiation of DNA synthesis needed to
replicate chromosomes.

5-16

Each chromosome is a single molecule of DNA whose extraordinarily long length can be
compacted by as much as 1000-fold during interphase and tenfold more during mitosis.
This is accomplished by binding to proteins that help package the DNA in an orderly
manner so it can fit in the small space delimited by the nuclear envelope. The structure
of the DNA-protein complex, called chromatin, is highly dynamic over time.

5-17

A.
B.
C.
D.
E.

Nucleosomes are present in eucaryotic chromosomes, but not in procaryotic


chromosomes.
A nucleosome contains two molecules each of histones H2A and H2B, as well as
histones H3 and H4.
A nucleosome core particle contains a core of histone with DNA wrapped around
it approximately twice.
Nucleosomes are aided in their formation by the high proportion of basic amino
acids in histone proteins.
Nucleosome formation compacts DNA into approximately one-third of its
original length.

81

5-18

Interphase chromosomes contain both darkly staining heterochromatin and more lightly
staining euchromatin. Genes that are being transcribed are thought to be packaged in a
less condensed type of euchromatin. Nucleosome core particles are separated from each
other by stretches of linker DNA. A string of nucleosomes coils up with the help of
histone H1 to form the more compact structure of the 30 nm fiber. The zigzag model
describes the structure of the 30 nm fiber. The 30 nm chromatin fiber is further
compacted by the formation of loops that emanate from a central axis.

5-19

The state of chromosome packing can differ in all of the instances given. Chromatin
differs most dramatically during the different stages of the cell cycle. But even within the
same interphase chromosome, chromatin can be less tightly packed into euchromatin
(especially in regions where genes are actively being expressed) or more densely packed
into heterochromatin (particularly near centromeres and telomeres). In the cells of female
mammals, one of the two homologous X chromosomes is entirely heterochromatic and
transcriptionally inactive. Because different cell types in the same organism are
expressing different genes, not all cells will have the same state of chromatin packing.

5-20

The DNA double helix in heterochromatin may be so tightly packed and condensed that it
is inaccessible to the proteins that bind replication origins, including the DNA replication
machinery. It may take extra time to remodel the chromatin to make it more accessible to
the proteins required to initiate and carry out DNA replication.

5-21

Choice (d) is the answer. Since the pattern of X-chromosome inactivation is established
randomly over several days of embryonic development, at a stage where the embryo has
quite a few cells, it is unlikely that two cats will inactivate the same X chromosome in
exactly the same set of cells. The other statements are false. Once inactivated, an X is
not normally reactivated during the life of the individual (choice (c) is false). If the
sectored coloring is due to X-chromosome inactivation, then the coat-color gene
responsible for the sectoring must be on the X chromosome (although there may be
several additional, different coat color genes located on other chromosomes). The fact
that sectoring results from X-chromosome inactivation cannot (by itself) distinguish
between the possibilities that coat color is determined by a single gene or by multiple
genes (choice (e) is false). Since males have only one X chromosome that they receive
from their mother and that never becomes inactivated, males will not be mottled (choice
(a) is false). Female calico cats receive an X chromosome from the father and another
from the mother, who may have different forms of the coat-color gene, and so female cats
are not necessarily the same color as their mother (choice (b) is false).

82

5-22

A.

B.

5-23

A.

B.

C.

D.

E.

For DNA to be accessiblefor transcription or some other processnoncovalent


bonds must be broken in order to change interactions between DNA and its
packaging proteins. This energetically unfavorable feat can be accomplished by
harnessing the energy released by hydrolysis of ATP. A nucleosome is held
together by a large number of weak, noncovalent interactions. For example,
many ionic bonds are made between the approximately 300 negatively charged
phosphate groups in the double-stranded DNA backbone and the positively
charged amino acid side chains of the eight proteins in the histone octamer. The
standard free energy required to dissociate a single nucleosome into its
components (histone octamer and DNA) is estimated to be around 14 kcal/mol,
roughly twice the amount of standard free energy liberated by the hydrolysis of
ATP to ADP and phosphate.
Based on the information presented in Chapter 5, it seems reasonable to propose
that the nucleosomes on the heterochromatic inactive X chromosome might
contain histone H3 tails with distinctive modifications. Indeed, in line with the
histone code hypothesis, at the time an X chromosome is first becoming inactive
the associated histone H3 becomes uniquely modified.
The lowest spot represents DNA of a length similar to that of the segment of
DNA found in a nucleosome core particle. Partial digestion with an enzyme like
M-nuclease causes breaks in the DNA backbone primarily within the linker DNA
or other DNA segments not bound tightly to histones. Thus, this band is probably
comprised of the DNA bound tightly to a single histone octamer and it arose by
cutting the linker DNA outside a single nucleosome core particle.
The ladder of bands with longer lengths probably corresponds to stretches of
DNA associated with increasing numbers of nucleosomes (1, 2, 3, 4, 5, and so
on). In support of this proposal, adjacent bands differ in size by roughly 200
nucleotides, which is the length of DNA found in a nucleosome core particle plus
neighboring linker DNA. This interpretation must mean that the M-nuclease
digestion did not go to completion; because if all non-nucleosomal DNA were
digested completely, the samples would contain only the 150 base pair fragment.
Based on the ability of M-nuclease to cut anywhere near Sweetie after growth in
galactose, it appears that the DNA is no longer protected from digestion by
binding to histones. Perhaps the wrapping of DNA within the nucleosomes has
been loosened considerably as in Figure 5-29. This change in the nucleosomes
must be specific to the Sweetie gene as it is not seen at the Salty gene or
throughout the genome.
The main candidates for enzymes that catalyzed the nucleosome alterations near
Sweetie are chromatin remodeling complexes and enzymes that covalently
modify histone tails with methyl, acetyl, or phosphate groups.
As the chromatin appears to have been loosened near Sweetie, it seems likely that
Sweetie gene expression is increased when cells are grown in galactose rather
than glucose, whereas Salty gene expression is likely to be the same under the two
conditions. Perhaps the Sweetie gene contains instructions for a protein that is
required for cells to metabolize galactose but not glucose.

83

CHAPTER 6
DNA REPLICATION, REPAIR, AND RECOMBINATION
2009 Garland Science Publishing
3rd Edition

DNA Replication
6-1 DNA replication is considered semiconservative because
(a) after many rounds of DNA replication, the original DNA double helix is still intact.
(b) each daughter DNA molecule consists of two new strands copied from the parent
DNA molecule.
(c) each daughter DNA molecule consists of one strand from the parent DNA molecule
and one new strand.
(d) new DNA strands must be copied from a DNA template.
(e) an RNA primer must be used to initiate synthesis of the DNA strand.

6-2 If the genome of the bacterium E. coli requires about 20 minutes to replicate itself, how
can the genome of the fruit fly Drosophila be replicated in only three minutes?
(a) The Drosophila genome is smaller than the E. coli genome.
(b) Eucaryotic DNA polymerase synthesizes DNA at a much faster rate than procaryotic
DNA polymerase.
(c) The nuclear membrane keeps the Drosophila DNA concentrated in one place in the
cell, which increases the rate of polymerization.
(d) Drosophila DNA contains more origins of replication than E. coli DNA.
(e) Eucaryotes have more than one kind of DNA polymerase.
6-3 Answer the following questions about DNA replication.
A. On a DNA strand that is being synthesized, which end is growingthe 3 end, the
5 end, or both ends? Explain your answer.
B. On a DNA strand that is being used as a template, where is the copying occurring
relative to the replication origin3 of the origin, 5 , or both?
6-4 If DNA strands were paired in a parallel rather than antiparallel fashion, how would the
replication of the DNA differ from that of normal double-stranded DNA?
(a) Replication would not be semiconservative.
(b) Replication origins would not be required.
(c) The replication fork would not be asymmetrical.
(d) The polymerase used would not be self-correcting.
(e) Both new strands would be synthesized discontinuously.

85

6-5

On Figure Q6-5 of a replication bubble:

Figure Q6-5
A.
B.
C.
D.
E.
F.

Indicate where the origin of replication was located (use O).


Label the leading-strand template and the lagging-strand template of the righthand fork [R] as X and Y, respectively.
Indicate by arrows the direction in which the newly made DNA strands (indicated
by dark lines) were synthesized.
Number the Okazaki fragments on each strand 1, 2, and 3 in the order in which
they were synthesized.
Indicate where the most recent DNA synthesis has occurred (use S).
Indicate the direction of movement of the replication forks with arrows.

6-6

The lagging strand is synthesized discontinuously at the replication fork because


(a)
the lagging strand template is discontinuous.
(b)
DNA polymerase always falls off the template DNA every ten nucleotides or so.
(c)
DNA polymerase can polymerize nucleotides only in the 5-to-3 direction.
(d)
DNA polymerase removes the last few nucleotides synthesized whenever it stops.
(e)
None of the above

6-7

Is the following statement TRUE or FALSE?


When bidirectional replication forks from adjacent origins meet, a leading
strand always runs into a lagging strand.
Explain your answer with a diagram illustrating sequential snapshots of the meeting of
two adjacent replication forks.

6-8

Which one of the following statements about the newly synthesized strand of a human
chromosome is correct?
(a)
It was synthesized from a single origin solely by continuous DNA synthesis.
(b)
It was synthesized from a single origin by a mixture of continuous and
discontinuous DNA synthesis.
(c)
It was synthesized from multiple origins solely by discontinuous DNA synthesis.
(d)
It was synthesized from multiple origins by a mixture of continuous and
discontinuous DNA synthesis.
(e)
It was synthesized from multiple origins by either continuous or discontinuous
DNA synthesis, depending on which specific daughter chromosome is being
examined.
86

6-9

You have discovered an Exo mutant form of DNA polymerase in which the 3-to-5
exonuclease function has been destroyed but the ability to join nucleotides together is
unchanged. Which of the following properties do you expect the mutant polymerase to
have?
(a)
It will polymerize in both the 5-to-3 direction and the 3-to-5 direction.
(b)
It will polymerize more slowly than the normal Exo+ polymerase.
(c)
To replicate the same amount of DNA, it will hydrolyze fewer
deoxyribonucleotides than will the normal Exo+ polymerase.
(d)
It will fall off the template more frequently than the normal Exo+ polymerase.
(e)
It will introduce fewer mutations into new strands than the normal Exo+
polymerase.

6-10

Replication of DNA requires a primer to initiate DNA synthesis because


(a)
DNA polymerase can add its first nucleotide only to an RNA chain.
(b)
DNA polymerase can add a nucleotide only to a base-paired nucleotide with a
free 3 end.
(c)
DNA polymerase can polymerize nucleotides only in the 5-to-3 direction.
(d)
DNA polymerase can polymerize DNA only in short fragments.
(e)
DNA polymerase has a 3-to-5 exonuclease activity.

6-11

Indicate whether each of the following statements is correct or incorrect. Explain your
answers.
(a)
Primase is less accurate than DNA polymerase at copying a DNA template.
(b)
The RNA primer remains as a permanent part of the new DNA molecule.
(c)
Replication of the leading strand does not require primase.
(d)
Longer primers are required to synthesize longer DNA fragments.
(e)
Primase can join ribonucleotides to create an RNA strand, using a single-stranded
DNA as a template, without the need for its own primer.

6-12

A.

B.

You are studying a strain of bacteria that carries a temperature-sensitive mutation


in one of the genes required for DNA replication. The bacteria grow normally at
the lower temperature, but when the temperature is raised they die. When you
analyze the remains of the bacterial cells grown at the higher temperature you find
evidence of partly replicated DNA. When the strands of this DNA are separated
by heating, numerous single-stranded DNA molecules around 1000 nucleotides
long are found. Which of the proteins listed below are most likely to be impaired
in these mutant bacteria? Explain your answer.
Next to the proteins listed below, write the number (1, 2, 3, and so on) that
corresponds to the order in which the proteins function during the synthesis of a
new stretch of DNA.
DNA ligase
DNA polymerase
Helicase
Initiator proteins

Primase
Repair polymerase
RNA nuclease
Single-stranded binding protein

87

6-13

Which of the following proteins or protein complexes are most abundant near the
replication fork? Why?
(a)
Single-strand binding protein
(b)
Sliding clamp
(c)
DNA polymerase
(d)
Helicase
(e)
Primase

6-14

A molecule of bacterial DNA introduced into a yeast cell is imported into the nucleus but
fails to replicate. Where do you think the block to replication arises? Choose the protein
or protein complex below that is most likely responsible for the failure to replicate
bacterial DNA. Give an explanation for your answer.
(a)
Primase
(b)
Helicase
(c)
DNA polymerase
(d)
Sliding clamp protein
(e)
Initiator proteins

6-15

Indicate whether the following statements about plasmids are TRUE or FALSE.
(a)
Replication of plasmid DNA is independent of a replication origin.
(b)
Maintenance of plasmid over the course of several cell divisions requires
telomerase activity.
(c)
DNA replication in plasmids is bidirectional.
(d)
Plasmids are experimentally useful for introducing specific DNA sequences into
yeasts and bacteria.
(e)
Plasmids were used to identify the replication origins from human DNA.

6-16

Most cells in the body of an adult human lack the telomerase enzyme because its gene is
turned off and thus not expressed. An important step in the conversion of a normal cell
into a cancer cell, which circumvents normal growth control, is the resumption of
telomerase expression. Explain why telomerase might be necessary for the ability of
cancer cells to divide over and over again.

DNA Repair
6-17

A pregnant mouse is exposed to high levels of a chemical. Many of the mice in her litter
are deformed, but when they are interbred with each other, all their offspring are normal.
Which TWO of the following statements could explain these results?
(a)
In the deformed mice, somatic cells but not germ cells were mutated.
(b)
The original mouses germ cells were mutated.
(c)
In the deformed mice, germ cells but not somatic cells were mutated.
(d)
The toxic chemical affects development but is not mutagenic.
(e)
The original mouse was defective in DNA repair.

88

6-18

During DNA replication in a bacterium, a C is accidentally incorporated instead of an A


into one newly synthesized DNA strand. Imagine this error was not corrected and has no
effect on the ability of the progeny to grow and reproduce.
A.
After this original bacterium divides once, what proportion of its progeny would
you expect to contain the mutation?
B.
What proportion of its progeny would you expect to contain the mutation after
three more rounds of DNA replication and cell division?

6-19

Mismatch repair of DNA


(a)
is carried out solely by the replicating DNA polymerase.
(b)
involves cleavage of the DNA backbone to excise a stretch of single-stranded
DNA containing a mispaired base.
(c)
preferentially repairs the leading strand to match the lagging strand.
(d)
makes replication 100,000 times more accurate.
(e)
can occur only on newly replicated DNA.

6-20

Which of the following DNA repair processes accurately restores genetic information
only immediately after the DNA has been replicated? Explain your answer.
(a)
Repair of deamination
(b)
Repair of depurination
(c)
Mismatch repair
(d)
Repair of pyrimidine dimers

6-21

Several members of the same family were diagnosed with the same kind of cancer when
they were unusually young. Which one of the following is the most likely explanation
for this phenomenon? Possibly, the individuals with the cancer have
(a)
inherited a cancer-causing gene that was mutated in an ancestors somatic cells.
(b)
inherited a mutation in a gene required for DNA synthesis.
(c)
inherited a mutation in a gene required for mismatch repair.
(d)
inherited a mutation in a gene required for the synthesis of purine nucleotides.
(e)
independently accumulated multiple random mutations over a period of years
leading to cancer.

6-22

If uncorrected, deamination of cytosine in DNA is most likely to lead to


(a)
substitution of an AT base pair for a CG base pair.
(b)
deletion of the altered CG base pair from the DNA.
(c)
conversion of the DNA into RNA.
(d)
generation of a thymine dimer.
(e)
None of the above.

6-23

Which of the following compounds is likely to be the most mutagenic?


(a)
One that depurinates DNA.
(b)
One that replaces adenine with guanine during DNA replication.
(c)
One that nicks the sugar-phosphate backbone.
(d)
One that causes thymidine dimers.
(e)
One that crosslinks together the two strands of the double helix.
89

6-24

You have made a collection of mutant fruit flies that are defective in various aspects of
DNA repair. You test each mutant for its hypersensitivity to three DNA-damaging
agents: sunlight, nitrous acid (which causes deamination of cytosine), and formic acid
(which causes depurination). The results are summarized in Figure Q6-24, where a yes
indicates that the mutant is more sensitive than a normal fly and blanks indicate normal
sensitivity.

Figure Q6-24
A.
B.
6-25

Which mutant is most likely to be defective in the DNA repair polymerase?


What aspect of repair is most likely to be affected in the other mutants?

You are examining the DNA sequences that code for the enzyme phosphofructokinase in
skinks and Komodo dragons. You notice that the coding sequence that actually directs
the sequence of amino acids in the enzyme is very similar in the two organisms but that
the surrounding sequences vary quite a bit. What is the most likely explanation for this?
(a)
Coding sequences are repaired more efficiently.
(b)
Coding sequences are replicated more accurately.
(c)
Coding sequences are packaged more tightly in the chromosomes to protect them
from DNA damage.
(d)
Mutations in coding sequences are more likely to be deleterious to the organism
than mutations in noncoding sequences.
(e)
DNA repair enzymes preferentially repair the newly replicated strand to match the
original strand.

90

DNA Recombination
6-26

Homologous recombination is initiated by double-strand breaks (DSBs) in a


chromosome. DSBs arise from DNA damage caused by harmful chemicals or by
radiation (for example, x-rays). During meiosis, the specialized cell division that
produces gametes (sperm and eggs) for sexual reproduction, the cells intentionally cause
DSBs in order to stimulate crossover homologous recombination. If there is not at least
one occurrence of crossing-over within each pair of homologous chromosomes during
meiosis, those non-crossover chromosomes will segregate randomly during division.

Figure Q6-26
A.

B.

C.

6-27

Consider the copy of chromosome 3 that you received from your mother. Is it
identical to the chromosome 3 that she received from her mother (her maternal
chromosome) or identical to the chromosome 3 she received from her father (her
paternal chromosome) or neither? Explain.
Starting with the representation in Figure Q6-26 of the double stranded maternal
and paternal chromosomes found in your mother, draw two possible
chromosomes you may have received from your mother.
What does this indicate about your resemblance to your grandfather and
grandmother?

Identify each statement below as TRUE or FALSE. Explain your answers.


A.
Site-specific recombination can repair sites of damaged DNA.
B.
Mobile genetic elements comprise nearly half of the human genome.
C.
Viruses probably evolved from intracellular mobile genetic elements.
D.
Genes that contain instructions for making motor proteins are called mobile
genetic elements.
E.
Homologous recombination results in the accumulation of mobile genetic
elements.

91

6-28

Which of the following DNA sequences are commonly carried on mobile genetic
elements? You may choose more than one option.
(a)
Transposase gene
(b)
Holliday junction
(c)
Recognition site for transposase
(d)
Antibiotic resistance gene
(e)
Replication origin

6-29

Retrotransposons
(a)
are found only in eucaryotes.
(b)
can move by either the cut-and-paste mechanism or by a mechanism requiring an
RNA intermediate.
(c)
include the transposable elements LINE-1, Alu, and Tn10.
(d)
can move only if they encode a reverse transcriptase.
(e)
were multiplied to high copy numbers in a common ancestor of all mammals.

6-30

Describe the likely consequence of introducing high levels of reverse transcriptase into a
human embryo.

6-31

Some retrotransposons and retroviruses integrate preferentially into regions of the


chromosome that are (a) packaged in euchromatin and (b) located outside the coding
regions of genes that contain information for making a protein. Why might these mobile
genetic elements have evolved this strategy?

6-32

All viruses
(a)
have single-stranded genomes.
(b)
lyse the cells they infect.
(c)
encode all of the enzymes needed to replicate themselves.
(d)
contain both nucleic acid and protein.
(e)
have the same size genomes.

6-33

The enzymes reverse transcriptase and DNA polymerase both synthesize DNA. Which
of the following statements about them are TRUE?
(a)
Reverse transcriptase uses only an RNA template. DNA polymerase uses only a
DNA template.
(b)
Reverse transcriptase can use either an RNA or a DNA template. DNA
polymerase uses only a DNA template.
(c)
DNA polymerase is used only by cells. Reverse transcriptase is used only by
viruses.
(d)
DNA polymerase uses deoxynucleotides. Reverse transcriptase uses
ribonucleotides.
(e)
Reverse transcriptase and DNA polymerase both contain a 3-to-5 exonuclease
activity.

92

6-34

Once a retrovirus has integrated into the genome of a host cell, why would it not be
possible to eradicate the virus by treating the infected cell with reverse transcriptase
inhibitors?

6-35

Why do retroviruses need to package reverse transcriptase molecules into their virus
particles even though they carry the gene for reverse transcriptase in their genomes?

93

Answers
6-1

Choice (c) is the answer. Choices (a) and (b) are false. Although choices (d) and (e) are
correct statements, they are not the reasons that DNA replication is called
semiconservative.

6-2

Choice (d) is the answer. Bacteria have one origin of replication and Drosophila has
many. Choice (a) is incorrect because the Drosophila genome is bigger than the E. coli
genome. Choice (b) is incorrect, as eucaryotic polymerases are not faster than procaryotic
polymerases. Choices (c) and (e) are technically correct statements, but are not relevant
to the question.

6-3

A.
B.

The 3 end. DNA polymerase can add nucleotides only to the 3-OH end of a
nucleic acid chain.
Both, due to the bidirectional nature of chromosomal replication.

6-4

(c)

The antiparallel nature of the strands of normal DNA, combined with the 5-to-3
activity of the DNA polymerase, precludes the continuous synthesis of both
strands and thus requires that the replication fork be asymmetrical. If the strands
were parallel, both new strands could be synthesized continuously.

6-5

See Figure A6-5.

Figure A6-5
6-6

(a)
(b)
(c)
(d)
(e)

False
False
True
False
False

94

6-7

True. See Figure A6-7 for an illustration of the meeting of two adjacent replication forks.

Figure A6-7
6-8

(d)

Each newly synthesized strand in a daughter duplex was synthesized by a mixture


of continuous and discontinuous DNA synthesis from multiple origins. Consider
a single replication origin: The fork moving in one direction synthesizes a
daughter strand continuously as part of leading-strand synthesis; the fork moving
in the opposite direction synthesizes a portion of the same daughter strand
discontinuously as part of lagging-strand synthesis.

6-9

Choice (c) is the answer. An Exo polymerase will be unable to proofread and thus will
hydrolyze fewer nucleotides than one that can proofread, because it cannot remove an
incorrect nucleotide and try again to add the correct nucleotide (thus choice (e) is false).
Choice (a) is unlikely because it postulates an entirely new function for the defective
polymerase. Choice (b) is incorrect because if the rate of polymerization changes, it
would become faster not slower, as a consequence of the Exo polymerase lacking a
proofreading function that might cause it to stop frequently to check its products and
remedy errors. Choice (d) is unlikely, because the ability to stay on the template is due to
the association of polymerase with additional proteins.

6-10

Choice (b) is the answer. Choices (a) and (d) are false statements. Choices (c) and (e) are
true but are not the reason DNA polymerase requires a primer.

95

6-11

(a)
(b)

(c)

(d)
(e)

6-12

A.

B.

Correct. Primase lacks a 3-to-5 exonuclease activity and thus cannot proofread
the nucleotide chain it makes.
Incorrect. A nuclease removes the RNA primer after it provides its 3-OH to
prime the synthesis of DNA. Then enzymes involved in repair synthesis and
ligation join the newly synthesized stretches of DNA to make a continuous strand
containing only DNA.
Incorrect. The leading strand requires primase to initiate DNA synthesis,
although fewer RNA primers will be needed on the leading strand than the
lagging strand.
Incorrect. The length of the primer does not correlate with the length of the DNA
strand that is synthesized.
Correct. Like other RNA polymerases, primase does not require a primer to
initiate synthesis of the nucleotide chain.
The 1000-nucleotide fragments that accumulate in the mutant are likely to be
Okazaki fragments. Thus, the mutant is likely to be defective in the function of
RNA nuclease, repair polymerase, or DNA ligase. These proteins are required to
remove the RNA primer, fill in the gap, and stitch together the Okazaki
fragments, respectively. We could test which enzyme was defective by
determining if the fragments contained a short stretch of RNA at the 5 end
(nuclease defective) or if the fragments annealed to the template leaving gaps
(repair polymerase defective).
DNA ligase8
DNA polymerase5
Helicase2
Initiator proteins1
Primase4
Repair polymerase7
RNA nuclease6
Single-stranded binding protein3

6-13

(a)

Single-strand binding protein is required to coat all single-stranded regions of


DNA that form at a replication fork; this requires many molecules of single-strand
binding protein at each fork. Only one or two molecules of each of the other
proteins or protein complexes are required at each replication fork.

6-14

Choice (e) is the answer. DNA from all organisms is chemically identical except for the
sequence of nucleotides. The proteins listed in choices (a) through (d) can act on any
DNA regardless of its sequence. In contrast, the initiator proteins recognize specific DNA
sequences at the origins of replication. These sequences differ between bacteria and
yeast.

96

6-15

(a)
(b)
(c)
(d)
(e)

6-16

In the absence of telomerase, the lifespan of a cell and its progeny cells is limited. With
each round of DNA replication, the length of telomeric DNA will shrink, until finally all
the telomeric DNA will disappear. Without telomeres capping the chromosome ends, the
ends might be treated like breaks arising from DNA damage or crucial genetic
information might be lost. Cells whose DNA lacks telomeres will stop dividing or die.
However, if telomerase is provided to cells, they may be able to divide indefinitely
because their telomeres will remain a constant length despite repeated rounds of DNA
replication.

6-17

Choice (a) or (d) is correct. Choice (b) cannot account for these results since a mutation
in the original mouses germ cells would have no effect on the fetuses she was already
carrying. Neither can choice (c), as mutations in the germ cells of the fetuses while in
utero would have had no effect on their development, but might have led to mutant mice
among their offspring. If the original mouse were defective in DNA repair (choice (e)),
this would increase the number of mutations caused by the chemical but cannot explain
the observed effects on her offspring and their offspring.

6-18

A.

B.

6-19

False
False
True
True
False

Half or 50%. DNA replication in the original bacterium will create two new
DNA molecules, one of which will now carry a mismatched C-T base pair. So one
daughter cell of that cell division will carry a completely normal DNA molecule;
the other cell will have the molecule with the mutation mispaired to a correct
nucleotide.
A quarter or 25%. At the next round of DNA replication and cell division, the
bacterium carrying the mismatched C-T will produce and pass on one normal
DNA molecule from the undamaged strand containing the T and one mutant DNA
molecule with a fully mutant C-G base pair. So at this stage, one out of the four
progeny of the original bacterium is mutant. Subsequent cell divisions of these
mutant bacteria will give rise only to mutant bacteria, while the other bacteria will
give rise to normal bacteria. The proportion of progeny containing the mutation
will, therefore, remain at 25%.

Choice (b) is the answer. Choice (a) is incorrect, as mismatch repair requires specialized
repair proteins that act after the DNA is replicated or at other times during the life of a
cell (thus choice (e) is incorrect). Choice (c) is incorrect because mismatch repair repairs
the newly replicated leading strand or the lagging strand to match its template strand.
Choice (d) is incorrect, since mismatch repair makes replication approximately 100 (not
100,000) times more accurate.

97

6-20

(c)

6-21

Choice (c) is the answer. In fact, affected individuals in some families with a history of
early-onset colon cancer have been found to carry mutations in mismatch repair genes.
Mutations arising in somatic cells are not inherited (choice (a) is incorrect). A defect in
DNA synthesis or nucleotide biosynthesis would likely be lethal (choices (b) and (d) are
incorrect). Choice (e) describes the way that most cancers arise in people late in life, but
it is extremely unlikely that several individuals in the same family spontaneously
acquired similar random mutations leading to the early onset of the same kind of cancer.

6-22

(a)

6-23

Choice (e) is the answer. Altogether, the DNA repair pathways described in Chapter 6
can easily repair the DNA damage described in options (a)(d), these types of damage
occur on one strand of the double helix, and thus can be repaired using the intact genetic
information encoded on the complementary strand. When the two strands are crosslinked
and thus both strands are damaged, entirely different and probably less accurate pathways
of repair are required.

6-24

A.

B.

Mismatches occur most often as a result of replication errors, in which case the
erroneous base is found only on the newly synthesized strand. In most eucaryotic
cells, the preferential repair of the erroneous base instead of its pairing partner is
thought to require recognition of nicks in the sugar-phosphate backbone of the
newly synthesized DNA strand. These nicks are sealed soon after replication, so
mismatch repair must occur in the short interval between passage of the
replication fork and sealing of the sugar-phosphate backbone in order to
accurately restore the original sequence. When mismatch repair occurs at other
times, it has an equal probability of restoring the original information or setting
the mutation. For the other kinds of DNA damage (U-G mismatch, abasic site,
pyrimidine dimer) it is always evident to determine which strand is damaged and
which strand contains reliable original information.

Mr. Self-Destruct is more likely than the other mutants to be defective in the
DNA repair polymerase because Mr. Self-Destruct is defective in repair of all
three kinds of DNA damage. The repair pathways for all three kinds of damage
are similar in the later steps, including a requirement for the DNA repair
polymerase.
The other mutants are specific for a particular type of damage. Thus the
mutations are likely to be in genes required for the first stage of repair, the
recognition and excision of the damaged bases. Dracula and Mole are likely to be
defective in the recognition or excision of thymidine dimers; Faust is likely to be
defective in the recognition or excision of U-G mismatched base pairs; and
Marguerite is likely to be defective in the recognition or excision of abasic sites.

98

6-25

Choice(d) is the answer. Mutationswhether they arise by mistakes in replication or by


damage to the DNA that remains unrepairedtend to hit the DNA fairly randomly
(choices (a) and (b) are false). However, if a mutation occurs in a protein-coding
sequence rather than in the surrounding DNA, it is more likely to cause a deleterious
change that kills or impairs the organism and thereby decreases the likelihood that the
mutation will be passed on to future generations. Since skinks and Komodo dragons
share a common lizard ancestor, differences in their genomes have arisen during their
divergence from this ancestor. Mutations in noncoding sequences are more likely to have
no effect on the functioning of the organism and thus frequently get passed along to
progeny. Choice (c) is incorrect, as genes that are being expressed tend to be more
loosely packaged than the noncoding DNA. Choice (e) is true, but has no bearing on the
phenomenon described.

6-26

A.
B.

Neither. The copy of chromosome 3 you received from your mother is a hybrid of
the ones she received from her mother and her father.
See Figure A6-26. The right answers include any chromosome in which a portion
matches the information from the paternal chromosome and the remainder
matches the information from the maternal chromosome.

Figure A6-26

6-27

C.

Due to extensive crossing over, you resemble both your grandmother and your
grandfather. If there were no crossing over, then you might have a much stronger
resemblance to one than the other.

A.

False. Homologous recombination, not site-specific recombination, is sometimes


used to repair sites of damaged DNA. Site-specific recombination is used mostly
in the mobilization of mobile genetic elements.
True. Forty-five percent of the human genome is comprised of mobile genetic elements.
True. A good guess for how viruses evolved is that some mobile genetic elements
acquired genes encoding coat proteins and other proteins required for packaging
and cellular escape of the nucleic acids of mobile genetic elements.
False. Cellular motor proteins are completely unrelated to mobile genetic elements.
False. Site-specific recombination, not homologous recombination, is the primary
mechanism for the accumulation of mobile genetic elements. Homologous
recombination does sometimes aid in the repair of DNA damage caused by the
excision of a mobile genetic element from the chromosome, but it does not aid in
the insertion of the element into a new location.

B.
C.

D.
E.

99

6-28

Choices (a), (c), and (d) are correct. A Holliday junction is not a sequence, but a
structural intermediate in homologous recombination (choice (b) is false). Mobile
genetic elements often have no replication origin, since their movement and proliferation
occurs by a type of repair DNA synthesis and their replication during cell division occurs
by passage of a replication fork that originates elsewhere in the chromosome (choice (e)
is false).

6-29

Choice (a) is the answer. Retrotransposons are found only in eucaryotes.


Retrotransposons (by definition) move only through an RNA intermediate (choice (b) is
false), and include LINE-1 and Alu sequences (but not the bacterial transposon Tn10, so
choice (c) is false). Retrotransposons do not necessarily need to provide their own
reverse transcriptase so long as there is an alternative source of reverse transcriptase in
the cell (choice (d) is false). The LINE-1 and Alu retrotransposons that comprise about a
third of the human genome are not identical in sequence or location to the
retrotransposons in other mammals like mice (choice (e) is false).

6-30

The embryo would probably have severe developmental abnormalities or die. The huge
numbers of retrotransposons littering the human genome are largely immobile due to the
accumulation of disabling mutations. However, it is likely that at least a few of the
millions of copies of the transposons still contain the sites necessary for
retrotransposition, although they do not encode a functional reverse transcriptase. High
levels of reverse transcriptase will probably cause many retrotransposition events. The
resultant insertion of retrotransposons is likely to disable genes required for development
or survival.

6-31

The most evolutionarily successful mobile genetic elements are those that are best at
reproducing themselves. In order to increase the number of copies of a particular
element, the element must meet two criteria: (1) it must not kill its host and (2) it must
maximize its ability to continue reproducing. If an element inserts into the coding region
of a gene, it might disable the gene and thereby confer a selective disadvantage in the
reproduction or survival of its host. Thus, elements that devised a way to avoid insertion
into coding regions probably were better able to increase their copy number throughout
the human population. If an element inserts into a heterochromatic region of a
chromosome, its genes may not be expressed and therefore it may become immobile.
Elements that devised a way to direct insertion into euchromatin would be more likely to
maintain mobility and thereby increase their copy number over time.

6-32

Choice (d) is the answer. All viruses contain both protein and nucleic acid. Viruses can
have either double- or single-stranded genomes (choice (a)). Not all viruses lyse the cells
they infect (choice (b)); for example, some bud out of the cell without killing it. Viruses
can have as few as three genes or more than a hundred (choice (e)). No virus is able to
replicate in the absence of a host cell (choice (c)).

100

6-33

Choice (b) is the answer. Reverse transcriptase can use an RNA or DNA template (thus
choice (a) is false). Choice (c) is false because DNA polymerase is often used by DNA
viruses. Choice (d) is false because both enzymes polymerize deoxynucleotides. Choice
(e) is false because reverse transcriptase does not have proofreading exonuclease activity.

6-34

Once the virus has integrated into the genome, it has no further need for reverse
transcriptase. Therefore, an inhibitor of reverse transcriptase may be able to block
infection of other cells by viruses that bud off the infected cell, but it will not be able to
eradicate the integrated virus. If the virus integrates into the genome of a cell with the
potential to divide, it will be faithfully propagated along with all the genomic DNA to all
progeny of that cell.

6-35

Retroviruses carry their own reverse transcriptase with them, as they must produce a
double-stranded DNA copy of their genome before their genes can be transcribed and
expressed.

101

CHAPTER 7
FROM DNA TO PROTEIN: HOW
CELLS READ THE GENOME
2009 Garland Science Publishing
3rd Edition

From DNA to RNA


7-1 RNA in cells differs from DNA in that
(a) it contains the base uracil, which pairs with cytosine.
(b) it is single-stranded and cannot form base pairs.
(c) it is single-stranded and can fold up into a variety of structures.
(d) the nucleotides are linked together in a different way.
(e) the sugar ribose contains fewer oxygen atoms than does deoxyribose.
7-2 Transcription is similar to DNA replication in that
(a) it requires a molecule of DNA helicase to unwind the DNA.
(b) it uses the same enzyme as that used to synthesize RNA primers during DNA
replication.
(c) the newly synthesized RNA remains paired to the template DNA.
(d) nucleotide polymerization occurs only in the 5 -to-3 direction.
(e) an RNA transcript is synthesized discontinuously and the pieces then joined together.
7-3 For each of the following sentences, fill in the blanks with the best word or phrase selected
from the list below. Not all words or phrases will be used; each word or phrase should be used
only once.
In order for a cells genetic material to be utilized, the information is first copied from the
DNA into the nucleotide sequence of RNA in a process called __________________.
Various kinds of RNAs are produced, each with different functions.
__________________ molecules code for proteins, __________________ molecules act
as adaptors for protein synthesis, __________________ molecules are integral
components of the ribosome, while __________________ molecules are important for
splicing of RNA transcripts.
Incorporation
mRNA
pRNA
translation

rRNA
snRNA
transcription

103

transmembrane
tRNA
proteins

7-4

Match the following structures with their names:

Figure Q7-4
7-5

Imagine that an RNA polymerase is transcribing a segment of DNA that contains the
sequence:
5- AGTCTAGGCACTGA -3
3- TCAGATCCGTGACT -5
A.
B.

If the polymerase is transcribing from this segment of DNA from left to right,
which strand (top or bottom) is the template?
What will be the sequence of that RNA (be sure to label the 5 and 3 ends of your
RNA molecule)?

7-6

The sigma subunit of bacterial RNA polymerase


(a)
contains the catalytic activity of the polymerase.
(b)
remains part of the polymerase throughout transcription.
(c)
recognizes promoter sites in the DNA.
(d)
recognizes transcription termination sites in the DNA.

7-7

Which of the following might decrease the transcription of only one specific gene in a
bacterial cell?
(a)
A decrease in the amount of sigma factor
(b)
A decrease in the amount of RNA polymerase
(c)
A mutation that introduced a stop codon into the DNA preceding the coding
sequence of the gene
(d)
A mutation that introduced extensive sequence changes into the DNA preceding
the transcription start site of the gene
(e)
A mutation that moved the transcription termination signal of the gene farther
away from the transcription start site

104

7-8

From the list below, pick THREE reasons why the primase that is used to make the RNA
primer for DNA replication would not be suitable for gene transcription?
(a)
Primase initiates RNA synthesis on a single-stranded DNA template.
(b)
Primase can initiate RNA synthesis without the need for a base-paired primer.
(c)
Primase synthesizes only RNAs of around 5 to 20 nucleotides in length.
(d)
The RNA synthesized by primase remains base-paired to the DNA template.
(e)
Primase uses nucleotide triphosphates.

7-9

Indicate where the following processes take place by adding numbered labeling lines to
the schematic diagram of a eucaryotic cell in Figure Q7-9.

Figure Q7-9
1.
2.
3.
4.
5.

Transcription
Translation
RNA splicing
Polyadenylation
RNA capping

7-10

Total nucleic acids are extracted from a culture of yeast cells and are then mixed with
resin beads to which the polynucleotide 5-TTTTTTTTTTTTTTTTTTTTTTTTT-3 has
been covalently attached. After a short incubation, the beads are then extracted from the
mixture. When you analyze the cellular nucleic acids that have stuck to the beads, which
of the following will be most abundant?
(a)
DNA
(b)
tRNA
(c)
rRNA
(d)
mRNA
(e)
Primary transcript RNA

7-11

Name three modifications that can be made to an RNA molecule in eucaryotic cells
before the RNA molecule becomes a mature mRNA.
105

7-12

The length of a particular gene in human DNA, measured from the start site for
transcription to the end of the protein-coding region, is 10,000 nucleotides, whereas the
length of the mRNA produced from this gene is 4000 nucleotides. What is the most likely
reason for this discrepancy?

7-13

A fragment of human DNA containing the gene for a protein hormone with its regulatory
regions removed is introduced into bacteria; although it is transcribed at a high level into
RNA, no protein is made. When this RNA is extracted from the bacteria, mixed with
human mRNA encoding the same hormone, and then examined in the electron
microscope, you see the following structure (Figure Q7-13). Label each of the statements
below as either consistent or inconsistent with your results and explain your
reasoning.

Figure Q7-13
A.
B.
C.
D.

The human DNA was inserted in the bacterial DNA next to a bacterial promoter
and in its normal orientation.
The human DNA was inserted in the bacterial DNA next to a bacterial promoter
but in an orientation opposite to normal.
The human DNA contained an intron.
The human DNA acquired a deletion while in the bacterium.

7-14

Why is the old dogma one geneone protein not always true for eucaryotic genes?

7-15

Is this statement TRUE or FALSE? Explain your answer.


Since introns do not contain protein coding information, they do not have to be
removed precisely (meaning, a nucleotide here and there should not matter) from
the primary transcript during RNA splicing.

106

7-16

You have discovered a gene (see Figure Q7-16 part A) that is alternatively spliced to produce
several forms of mRNA in various cell types, three of which are shown in part B of Figure
Q7-16. (Note that splicing is indicated by lines connecting the exons that are included in the
mRNA). Your experiments have found that protein translation begins in exon 1. For all
forms of the mRNA, the encoded protein sequence is the same in the regions of the mRNA
that correspond to exons 1 and 10. Exons 2 and 3 are alternative exons used in different
mRNA, as are exons 7 and 8. Which of the following statements about exons 2 and 3 is the
most accurate? Explain your answer.

Figure Q7-16
(a)
(b)
(c)

Exons 2 and 3 must have the same number of nucleotides.


Exons 2 and 3 must contain an integral number of codons (that is, the number of
nucleotides divided by 3 must be an integer).
Exons 2 and 3 must contain a number of nucleotides that when divided by 3,
leaves the same remainder (that is, 0, 1, or 2).

From RNA to Protein


7-17

Which of the following statements about the genetic code are correct?
(a)
All codons specify more than one amino acid.
(b)
The genetic code is redundant.
(c)
All amino acids are specified by more than one codon.
(d)
The genetic code is different in procaryotes and eucaryotes.
(e)
All codons specify an amino acid.

107

NOTE: The following codon table is to be used for Problems Q7-18 7-23, Q7-29, and 7-33.

7-18

7-19

The following DNA sequence includes the beginning of a sequence coding for a protein.
What would be the result of a mutation that changed the C marked by an asterisk to an A?
5- AGGCTATGAATGGACACTGCGAGCCC....
*
Which amino acid would you expect a tRNA with the anticodon 5-CUU-3 to carry?
(Refer to Codon table provided above Q7-18.)
(a)
(b)
(c)
(d)
(e)

7-20

Which of the following pairs of codons might you expect to be read by the same tRNA as
a result of wobble? (Refer to Codon table provided above Q7-18.)
(a)
(b)
(c)
(d)
(e)

7-21

Lysine
Glutamate
Glutamine
Leucine
Phenylalanine

CUU and UUU


GAU and GAA
CAC and CAU
AAU and AGU
CCU and GCU

Below is a segment of RNA from the middle of an mRNA. If you were told that this
segment of RNA was part of the coding region of an mRNA for a large protein, give the
amino acid sequence for the protein that is encoded by this segment of mRNA. (Refer to
Codon table provided above Q7-18.)
5- UAGUCUAGGCACUGA -3

108

7-22

(Refer to Codon table provided above Q7-18.) One strand of a section of DNA isolated
from the bacterium E. coli reads:
5- GTAGCCTACCCATAGG -3
A.

B.
C.
7-23

Suppose that an mRNA is transcribed from this DNA using the complementary
strand as a template. What will be the sequence of the mRNA in this region
(make sure you label the 5 and 3 ends of the mRNA)?
How many different peptides could potentially be made from this sequence of
RNA, assuming translation initiates upstream of this sequence?
What are these peptides? (Give your answer using the one letter amino acid code.)

A strain of yeast translates mRNA into protein with a high level of inaccuracy. Individual
molecules of a particular protein isolated from this yeast have the following variations in
the first 11 amino acids compared with the sequence of the same protein isolated from
normal yeast cells (Figure Q7-23). What is the most likely cause of this variation in
protein sequence? Explain your answer. (Refer to Codon table provided above Q7-18.)

Figure Q7-23
(a)
(b)
(c)
(d)
(e)

7-24

A mutation in the DNA coding for the protein


A mutation in the anticodon of the isoleucine tRNA (tRNAIle)
A mutation in the isoleucyl-tRNA synthetase that decreases its ability to
distinguish between different amino acids
A mutation in the isoleucyl-tRNA synthetase that decreases its ability to
distinguish between different tRNA molecules
A mutation in a component of the ribosome that allows binding of incorrect tRNA
molecules to the A-site

Which of the following statements is TRUE?


(a)
Ribosomes are large RNA structures composed solely of rRNA.
(b)
Ribosomes are synthesized entirely in the cytoplasm.
(c)
rRNA contains the catalytic activity that joins amino acids together.
(d)
A ribosome consists of two equally sized subunits.
(e)
A ribosome binds one tRNA at a time.

109

7-25

Figure Q7-25A shows the stage in translation when an incoming aminoacyl-tRNA has
bound to the A-site on the ribosome. Using the components shown in Figure Q7-25A as a
guide, show on Figure Q7-25B and Q7-25C what happens in the next two stages to
complete the addition of the new amino acid to the growing polypeptide chain.

Figure Q7-25
7-26

A poison added to an in vitro translation mixture containing mRNA molecules with the
sequence 5-AUGAAAAAAAAAAAAUAA-3 has the following effect: the only product
made is a Met-Lys dipeptide that remains attached to the ribosome. What is the most
likely way in which the poison acts to inhibit protein synthesis?
(a)
It inhibits binding of the small subunit of the ribosome to mRNA.
(b)
It inhibits peptidyl transferase activity.
(c)
It inhibits movement of the small subunit relative to the large subunit.
(d)
It inhibits release factor.
(e)
It mimics release factor.

110

7-27

In eucaryotes, but not procaryotes, ribosomes find the start site of translation by
(a)
binding directly to a ribosome-binding site preceding the initiation codon.
(b)
scanning along the mRNA from the 5 end.
(c)
recognizing an AUG codon as the start of translation.
(d)
binding an initiator tRNA.

7-28

Figure Q7-28 shows an mRNA molecule.

Figure Q7-28
A.

B.
7-29

Match the labels given in the list below with the label lines in Figure Q7-28.
(a)
ribosome-binding site
(b)
initiator codon
(c)
stop codon
(d)
untranslated 3 region
(e)
untranslated 5 region
(f)
protein-coding region
Is the mRNA shown procaryotic or eucaryotic? Explain your answer.

A tRNA for the amino acid lysine is mutated such that the sequence of the anticodon is
5-UAU-3 (instead of 5-UUU-3). Which of the following aberrations in protein
synthesis might this tRNA cause? (Refer to Codon table provided above Q7-18.)
(a)
(b)
(c)
(d)
(e)

Read through of stop codons


Substitution of lysine for isoleucine
Substitution of lysine for tyrosine
Substitution of lysine for phenylalanine
Substitution of lysine for the amino-terminal methionine

111

7-30

You have discovered a protein that inhibits translation. When you add this inhibitor to a
mixture capable of translating human mRNA and centrifuge the mixture to separate
polyribosomes and single ribosomes, you obtain the results shown in Figure Q7-30.
Which of the following interpretations are consistent with these observations?

Figure Q7-30
(a)
(b)
(c)
(d)

7-31

The protein binds to the small ribosomal subunit and increases the rate of
initiation of translation.
The protein binds to sequences in the 5 region of the mRNA and inhibits the rate
of initiation of translation.
The protein binds to the large ribosomal subunit and slows down elongation of the
polypeptide chain.
The protein binds to sequences in the 3 region of the mRNA and prevents
termination of translation.

The concentration of a particular protein X in a normal human cell rises gradually from a
low point, immediately after cell division, to a high point, just before cell division, and
then drops sharply. The level of its mRNA in the cell remains fairly constant throughout
this time. Protein X is required for cell growth and survival, but the drop in its level just
before cell division is essential for division to proceed. You have isolated a line of human
cells that grow in size in culture but cannot divide, and on analyzing these mutants, you
find that levels of X mRNA in the mutant cells are normal. Which of the following
mutations in the gene for X could explain these results?
(a)
The introduction of a stop codon that truncates protein X at the fourth amino acid.
(b)
A change of the first ATG codon to CCA.
(c)
The deletion of a sequence that encodes sites at which ubiquitin can be attached to
the protein.
(d)
A change at a splice site that prevents splicing of the RNA.

112

7-32

For each of the following sentences, fill in the blanks with the best word or phrase
selected from the list below. Not all words or phrases will be used; each word or phrase
should be used only once.
Once an mRNA is produced, its message can be decoded on ribosomes.
The ribosome is composed of two subunits: the __________________
subunit, which catalyzes the formation of the peptide bonds that link the
amino acids together into a polypeptide chain, and the
__________________ subunit, which matches the tRNAs to the codons of
the mRNA. During the chain elongation process of translating an mRNA
into protein, the growing polypeptide chain attached to a tRNA is bound to
the __________________ -site of the ribosome. An incoming aminoacyltRNA carrying the next amino acid in the chain will bind to the
__________________ -site by forming base pairs with the exposed codon
in the mRNA. The __________________ enzyme catalyzes the formation
of a new peptide bond between the growing polypeptide chain and the
newly arriving amino acid. The end of a protein-coding message is
signaled by the presence of a stop codon, which binds the
__________________ called release factor. Eventually, most proteins
will be degraded by a large complex of proteolytic enzymes called the
__________________.
A
central
DNA
E
large

medium
P
peptidyl transferase
polymerase
protein

113

proteosome
RNA
small
T
ubiquitin

7-33

After treating cells with a mutagen, you isolate two mutants. One carries alanine and the
other carries methionine at a site in the protein that normally contains valine. After
treating these two mutants again with mutagen, you isolate mutants from each that now
carry threonine at the site of the original valine. Assuming that all mutations caused by
the mutagen involve single nucleotide changes, deduce the codons that are used for
valine, alanine, methionine, and threonine at the affected site. See Figure Q7-33. (Refer
to Codon table provided above Q7-18.)

Figure Q7-33

RNA and the Origins of Life


7-34

According to current thinking, the minimum requirement for life to have originated on
Earth was the formation of a
(a)
molecule that could provide a template for the production of a complementary
molecule.
(b)
double-stranded DNA helix.
(c)
molecule that could direct protein synthesis.
(d)
molecule that could catalyze its own replication.

7-35

Which of the following reactions are known to be carried out by a ribozyme?


(a)
DNA synthesis
(b)
Transcription
(c)
RNA splicing
(d)
Protein hydrolysis
(e)
Polysaccharide hydrolysis

7-36

You are studying a disease that is caused by a virus, but when you purify the virus
particles and analyze them you find they contain no trace of DNA. Which of the
following molecules are likely to contain the genetic information of the virus?
(a)
Protein
(b)
RNA
(c)
Lipids
(d)
Carbohydrates

7-37

Give a reason why DNA makes a better material than RNA for storage of genetic
information and explain your answer.

114

How We Know: Cracking the Genetic Code


7-38

An extraterrestrial organism (ET) is discovered whose basic cell biology seems pretty
much the same as terrestrial organisms except that it uses a different genetic code to
translate RNA into protein. You set out to break the code by translation experiments
using RNAs of known sequence and cell-free extracts of ET cells to supply the necessary
protein-synthesizing machinery. In experiments using the RNAs below, the following
results were obtained when the 20 possible amino acids were added either singly or in
different combinations of two or three:
RNA 1:
RNA 2:

5GCGCGCGCGCGCGCGCGCGCGCGCGCGC3
5GCCGCCGCCGCCGCCGCCGCCGCCGCCGCC3

Using RNA 1, a polypeptide was produced only if alanine and valine were added to the
reaction mixture. Using RNA 2, a polypeptide was produced only if leucine and serine
and cysteine were added to the reaction mixture. Assuming that protein synthesis can
start anywhere on the template, that the ET genetic code is nonoverlapping and linear,
and that each codon is the same length (like the terrestrial triplet code), how many
nucleotides does an ET codon contain?
(a)
2
(b)
3
(c)
4
(d)
5
(e)
6
7-39

NASA has discovered an alien life form. You are called in to help them deduce the
genetic code for this alien. Surprisingly, this alien life form shares many similarities with
life on Earth: this alien uses DNA as its genetic material, makes RNA from DNA, and
reads the information from RNA to make protein using ribosomes and tRNAs. Even
more amazing, this alien uses the same 20 amino acids, like the organisms found on
Earth, and also codes for each amino acid by a triplet codon. However, the scientists at
NASA have found that the genetic code used by the alien life form is different than that
used by life on Earth. The experiment that allowed the NASA scientists to draw this
conclusion involved creating a cell-free protein synthesis system from alien cells and
adding an mRNA made entirely of uracil (poly U). This led to the finding that poly U
directs the synthesis of a peptide containing only glycine. NASA scientists have
synthesized a poly AU mRNA and find that it codes for a polypeptide of alternating
serine and proline residues. From these experiments, can you determine which codons
code for serine and proline? Explain why or why not.
Bonus question: Can you propose a mechanism for how the aliens physiology is altered
so that it uses a different genetic code from life on Earth, despite all the similarities?

115

Answers
7-1

Choice (c) is the correct answer. Choice (a) is untrue, since although RNA contains
uracil, uracil pairs with adenine, not cytosine. Choice (b) is false because RNA can form
base pairs with a complementary RNA or DNA sequence. Choice(d) is false. Choice (e)
is false because ribose contains one more oxygen atom than deoxyribose.

7-2

Choice (d) is the correct answer. RNA polymerase unwinds only a few base pairs of the
double helix at a time and does not need a helicase to do so, which is why choice (a) is
incorrect. The enzyme used to make primers during DNA synthesis is indeed an RNA
polymerase, but it is a special enzyme, primase, and not the enzyme that is used for
transcription, which is why choice (b) is incorrect. Choice (c) is false. Choice (e) is
incorrect because an RNA transcript is made by a single polymerase molecule that
proceeds from the start site to the termination site without falling off.

7-3

In order for a cells genetic material to be utilized, the information is first copied from the
DNA into the nucleotide sequence of RNA in a process called transcription. Various
kinds of RNAs are produced, each with different functions. mRNA molecules code for
proteins, tRNA molecules act as adaptors for protein synthesis, rRNA molecules are
integral components of the ribosome, while snRNA molecules are important for splicing
of RNA transcripts.

7-4

A4; B1; C2; D3

7-5

A.
B.

7-6

(c)

7-7

Choice (d) is the correct answer. Such changes would probably destroy the function of
the promoter, making RNA polymerase unable to bind to it. Decreasing the amount of
sigma factor or RNA polymerase (choices (a) or (b)) would affect the transcription of
most of the genes in the cell, not just one specific gene. Introducing a stop codon before
the coding sequence (choice (c)) would have no effect on transcription of the gene, since
the transcription machinery does not recognize translational stops. Moving the
termination signal farther away (choice (e)) would merely make the transcript longer.

7-8

Choices (a), (c), and (d) are the correct reasons. Choices (b) and (e) are true for both
primase and RNA polymerase.

The bottom strand


5- AGUCUAGGCACUGA -3

116

7-9

See Figure A7-9.

Figure A7-9
7-10

(d)

mRNA is the only type of RNA that is polyadenylated, and this poly(A) tail
would be able to base-pair with the strands of poly(T) on the beads and thus stick
to them. DNA would not be found in the sample, as the poly(A) tail is not
encoded in the DNA and long runs of T are rare in DNA.

7-11

1.
2.
3.

A poly(A) tail must be added.


A 5 cap must be added.
Introns must be spliced out.
(Export from nucleus is also an acceptable answer.)

7-12

The gene contains one or more introns.

7-13

B, C, and D are consistent with the results; A is inconsistent.


B must be true for the RNA produced in the bacterium to be complementary to, and thus
able to pair with, the mRNA from a human cell. If the human DNA had become inserted
in its normal orientation next to the promoter (A), the corresponding portions of RNA
would be identical (or at least very similar) in sequence and thus the two RNAs would
not be complementary and would not pair. The loop formed in the hybrid tells us that one
of the molecules contains sequences that the other is missing. This could come about
either because the bacterial RNA was transcribed from human sequences that acquired a
deletion (B) or because the human gene contains an intron (C).

7-14

The transcripts from some genes can be spliced in more than one way to give mRNAs
containing different sequences, thus encoding different proteins. A single eucaryotic
gene, therefore, may encode more than one protein.

117

7-15

False. Although it is true that the sequences within the introns are mostly dispensable,
the introns must still be removed precisely because an error of one or two nucleotides
would shift the reading frame of the resulting mRNA molecule and change the protein it
encodes.

7-16

Choice (c) is the only answer that must be true for exons 2 and 3. Although choices (a)
and (b) could be true, they dont have to be. Because the protein sequence is the same in
segments of the mRNA corresponding to exons 1 and 10, the choice of either exon 2 or
exon 3 would not alter the reading frame. To maintain the normal reading frame,
whatever it is, the alternative exons must have a number of nucleotides that when divided
by 3 (the number of nucleotides in a codon) give the same remainder.

7-17

Choice (b) is the correct answer. The majority of the amino acids can be specified by
more than one codon. Choice (a) is incorrect because each codon specifies only one
amino acid. Choice (c) is incorrect because tryptophan and methionine are encoded by
only one codon. Choice (d) is incorrect because, with a few minor exceptions, the genetic
code is the same in all organisms. Choice (e) is incorrect because some codons specify
translational stop signals.

7-18

The change creates a stop codon (TGA, or UGA in the mRNA) very near the beginning
of the protein-coding sequence and in the correct reading frame (the beginning of the
coding sequence is indicated by the ATG). Thus, translation would terminate after only
four amino acids had been joined together, and the complete protein would not be made.

7-19

(a)

7-20

Choice (c) is the answer. These two codons differ only in the third position and also
encode the same amino acid, which is the definition of wobble. Although the codons
GAU and GAA (choice (b)) also differs only in the third position, they are unlikely in
normal circumstances to be read by the same tRNA, as they encode different amino acids.

7-21

SLGT is the answer. (Reading frame two is the only reading frame that does not contain
a stop codon.)

7-22

A.
B.

C.

Lys (lysine). As is conventional for nucleotide sequences, the anticodon is given


5 to 3. The complementary base-pairing occurs between antiparallel nucleic acid
sequences, and the codon recognized by this anticodon will therefore be 5-AAG-3.

5-GUAGCCUACCCAUAGG -3
Two. (There are three potential reading frames for each RNA. In this case, they
are:
GUA GCC UAC CCA UAG ...
UAG CCU ACC CAU AGG.....
AGC CUA CCC AUA GG?....
The center one cannot be used in this case, because UAG is a stop codon.)
VAYP
SLPIG
Note: PTHR will not be a peptide because it is preceded by a stop codon.
118

7-23

Choice (c) is the correct answer. A mutation in the isoleucyl-tRNA synthetase that
decreases its ability to distinguish between amino acids would allow an assortment of
amino acids to be attached to the tRNAIle. These assorted aminoacyl-tRNAs would then
base-pair with the isoleucine codon and cause a variety of substitutions at positions
normally occupied by isoleucine. Choice (a) is incorrect because a mutation in the gene
encoding the protein would cause only a single variant protein to be made. Choice (e) is
incorrect because a mutation in the ribosome that allows binding of any amino-acyltRNA to the A site would cause substitutions all over the protein, not only at isoleucine
residues. Choices (b) and (d) are also incorrect. A mutation in the anticodon loop of
tRNAIle (choice (b)) or a mutation in the isoleucine-tRNA synthetase that decreases its
ability to distinguish between different tRNA molecules (choice (d)) would cause
substitution of isoleucine for some other amino acid (which is the opposite of what is
observed).

7-24

Choice (c) is the correct answer. Choice (a) is incorrect because ribosomes contain
proteins as well as rRNA. Choice (b) is incorrect because rRNA is synthesized in the
nucleus, and ribosomes are partly assembled in the nucleus. Choice (d) is incorrect
because a ribosome consists of one small subunit and one large subunit. Choice (e) is
incorrect because a ribosome must be able to bind two tRNAs at any one time.

7-25

See Figure A7-25.

Figure A7-25
119

7-26

Choice (c) is the correct answer. Either choice (a) or (b) would prevent all peptide bond
formation. Choice (d) would have no affect on translation until the stop codon was
reached. Choice (e) would be likely to result in a mixture of polypeptides of various
lengths; a poison mimicking a release factor could conceivably cause only Met-Lys to be
made, but this dipeptide would not remain bound to the ribosome.

7-27

Choice (b) is the correct answer. Choice (a) is true only for procaryotes. Choices (c) and
(d) are true for both procaryotes and eucaryotes.

7-28

A.
B.

(a)3; (b)2; (c)4; (d)6;(e)1; (f)5


The mRNA is procaryotic. It contains coding regions for more than one protein,
as shown by the multiple initiation codons, each preceded by a ribosome-binding
site. It contains an unmodified 5 end, as shown by the three phosphate groups,
and an unmodified 3 end, as shown by the absence of a poly(A) tail.

7-29

(b)

The mutant tRNALys will be able to pair with the codon 5-AUA-3, which codes
for isoleucine.

7-30

Choice (b) is the correct answer. The results in Figure Q730 show a marked decrease in
the number of polyribosomes formed relative to normal. Polyribosomes form because the
initiation of translation is fairly rapid: ribosomes can bind successively to the free 5 end
of an mRNA molecule and start translation before the first ribosome has had a chance to
finish translating the message. Therefore, inhibition of the rate of initiation will tend to
decrease the number of ribosomes in the polyribosome, and in the extreme case there will
be only one ribosome per mRNA. Conversely, increasing the rate of initiation or slowing
the rate of elongation would result in an increased number of ribosomes per
polyribosome (up to a maximum point), making choices (a) and (c) false. Choice (d) is
incorrect, as preventing termination would prevent release of the ribosomes at the end of
the coding sequence and would be expected to freeze the assembled polyribosomes, so
that the ratio of polyribosomes to ribosomes would be much as normal.

7-31

Choice (c) is the correct answer. The drop in level of protein X in the normal cell is most
likely due to protein degradation, since levels of mRNA remain constant. The inability of
the mutant cell to divide could be due to a mutation that inhibits protein degradation. This
would be achieved by removal of sites for attachment of ubiquitin, which targets proteins
for destruction. Choices (a), (b), and (d) would probably not produce the result described,
as without the production of a functional protein X the mutant cells could not grow in
size.

120

7-32

Once an mRNA is produced, its message can be decoded on ribosomes. The ribosome is
composed of two subunits: the large subunit, which catalyzes the formation of the
peptide bonds that link the amino acids together into a polypeptide chain, and the small
subunit, which matches the tRNAs to the codons of the mRNA. During the chain
elongation process of translating an mRNA into protein, the growing polypeptide chain
attached to a tRNA is bound to the P-site of the ribosome. An incoming aminoacyltRNA carrying the next amino acid in the chain will bind to the A-site by forming base
pairs with the exposed codon in the mRNA. The peptidyl transferase enzyme catalyzes
the formation of a new peptide bond between the growing polypeptide chain and the
newly arriving amino acid. The end of a protein-coding message is signaled by the
presence of a stop codon, which binds the protein called release factor. Eventually, most
proteins will be degraded by a large complex of proteolytic enzymes called the
proteosome.

7-33

Given that only single nucleotide changes are involved, the only codons consistent with
the changes are: GUG for valine, GCG for alanine, AUG for methionine, and ACG for
threonine.

7-34

Choice (d) is the correct answer. Choice (a) is incorrect in that although this may have
been a step in self-replication, it would not by itself be sufficient. Choices (b) and (c) are
incorrect, as these stages in the evolution of the cell must have succeeded the formation
of the first self-replicating molecules.

7-35

(c)

7-36

(b)

7-37

Three possible answers are:


1.
The deoxyribose sugar of DNA makes the molecule much less susceptible to
breakage compared to RNA, due to the lack of the hydroxyl group on carbon 2 of
the ribose sugar.
2.
DNA is double stranded and therefore the complementary strand provides a
template from which damage can be repaired accurately.
3.
The use of T in DNA instead of U (as in RNA) protects against the effect of
deamination, a common form of damage. Deamination of T produces an aberrant
base (methyl C), whereas deamination of U generates C, a normal base. The
presence of an abnormal base eases the cells job of recognizing the damaged
strand.

121

7-38

Choice (d) is the correct answer. An organism having codons with an even number of
nucleotides (i.e., 2, 4, or 6) could read 5-GCGCGCGCGC-3 (RNA 1) in either of two
ways, namely GC GC GC GC... or CG CG CG CG... Either one of the two amino
acids alone could have supported protein synthesis, so you would not need them in
combination (thus eliminating choices (a), (c), and (e)). An organism having three bases
per codon could read the sequence 5-GCCGCCGCCGCCGCC-3 (RNA 2) in one of
three ways, namely GCC GCC GCC GCC..., CCG CCG CCG CGG..., or CGC
CGC CGC CGC..., and so again, any one of the three amino acids could have supported
synthesis of a polypeptide, and you would not need to add all three amino acids to
produce a polypeptide chain, thus eliminating choice (b). Only a five-nucleotide code
gives you two different consecutive codons for RNA 1 and three different consecutive
codons for RNA 2.

7-39

No, you cannot definitively determine what the codons that code for serine or valine are
because it could be either UAU or AUA.
Bonus: The alien aminoacyl-tRNA synthetases could adapt a different amino acid to each
tRNA, thus matching an amino acid with a different codon compared to those codons
used by life on Earth.

122

CHAPTER 8
CONTROL OF GENE EXPRESSION
2009 Garland Science Publishing
3rd Edition

An Overview of Gene Expression


8-1 The distinct characteristics of different cell types in a multicellular organism
are produced mainly by the differential regulation of the
(a) replication of specific genes.
(b) transcription of genes transcribed by RNA polymerase II.
(c) transcription of housekeeping genes.
(d) translation of mRNA.
(e) packing of DNA into nucleosomes in some cells and not others.
8-2 In principle, a eucaryotic cell can regulate gene expression at any step in the
pathway from DNA to the active protein. Place the types of control listed below at the
appropriate places on the diagram in Figure Q8-2.

A. Translation control
B. Transcriptional control
C. RNA processing control
D. Protein activity control

123

How Transcriptional Switches Work


8-3

For each of the following sentences, fill in the blanks with the best word or phrase
selected from the list below. Not all words or phrases will be used; each word or phrase
should be used only once.
The genes of a bacterial __________________ are transcribed into a
single mRNA. Many bacterial promoters contain a region known as a(n)
__________________, to which a specific gene regulatory protein binds.
Genes in which transcription is prevented are said to be
__________________. Bacterial genes are regulated by small molecules
such as tryptophan by the interaction of such molecules with
__________________ DNA-binding proteins such as the tryptophan
repressor. Genes that are being __________________ expressed are being
transcribed all the time.
allosteric
constitutively
induced

negatively
operator
operon

positively
promoter
repressed

8-4

A gene regulatory protein called HisP regulates the enzymes for histidine biosynthesis in
the bacterium E. coli. HisP is an allosteric protein whose activity is modulated by
histidine. Upon binding histidine, HisP alters its conformation, dramatically changing its
affinity for the regulatory sequences in the promoters of the genes for the histidine
biosynthetic enzymes.
A.
If HisP functions as a gene repressor, would you expect that HisP would bind
more tightly or less tightly to the regulatory sequences when histidine is
abundant? Explain your answer.
B.
If HisP functions as a gene activator, would you expect that HisP would bind
more tightly or less tightly to the regulatory sequences when histidine levels are
low? Explain your answer.

8-5

In class we talked about how bacterial cells can take up the amino acid tryptophan from
their surroundings, or if the external supply is insufficient, they can synthesize trytophan
by using enzymes in the cell. In some bacteria, the control of glutamine synthesis is
similar to that of tryptophan synthesis, such that the glutamine repressor is used to inhibit
the transcription of the glutamine operon, which contains the genes that code for the
enzymes required for glutamine synthesis. Upon binding to cellular glutamine, the
glutamine repressor binds to a site in the promoter of the operon.
A.
Why is glutamine-dependent binding to the operon a useful property for the
glutamine repressor?
B.
What would you expect to happen to the regulation of the enzymes that synthesize
glutamine in cells that express a mutant form of the glutamine repressor that
cannot bind to DNA?
C.
What would you expect to happen to the regulation of the enzymes that synthesize
glutamine in cells that express a mutant form of the glutamine repressor that binds
to DNA even when no glutamine is bound to it?
124

8-6

In the absence of glucose, E. coli can proliferate using the pentose sugar arabinose. The
ability of E. coli to utilize the sugar arabinose is regulated via the arabinose operon,
depicted in Figure Q8-6. The araA, araB, and araD genes encode enzymes for the
metabolism of arabinose. The araC gene encodes a gene regulatory protein that binds
adjacent to the promoter of the arabinose operon. To understand the regulatory properties
of the AraC protein, you engineer a mutant bacterium in which the araC gene has been
deleted and look at the effect of the presence or absence of the AraC protein on the AraA
enzyme.

Figure Q8-6
A.

B.

If the AraC protein works as a gene repressor, would you expect araA RNA levels
to be high or low in the presence of arabinose in the araC mutant cells? What
about in the araC mutant cells in the absence of arabinose? Explain your
answer.
Your findings from the experiment are summarized in Table 8.6.
Table 8-6
Genotype
araC+ (normal cells)
araC (mutant cells)

araA RNA Levels


in the absence of arabinose in the presence of arabinose
low
high
low
low

Do the results in Table 8-6 indicate that the AraC protein regulates arabinose metabolism
by acting as a gene repressor or a gene activator? Explain your answer.

125

8-7

We have discussed how the lac operon (see Figure Q8-7) is controlled by both the CAP
activator protein and the lac repressor. You create cells that are mutant in the gene
coding for the lac repressor so that now these cells lack the lac repressor under all
conditions. For these mutant cells, state whether the lac operon will be ON or OFF under
the following situations and explain why.

Figure Q8-7
A.
B.
C.
D.
8-8

In the presence of glucose and lactose


In the presence of glucose and the absence of lactose
In the absence of glucose and the absence of lactose
In the absence of glucose and the presence of lactose

You have discovered an operon in a bacterium that is only turned on when sucrose is
present and glucose is absent. You have also isolated three mutants that have changes in
the upstream regulatory sequences of the operon and whose behavior is summarized in
the Table 8-8. You hypothesize that there are two gene regulatory sites in the upstream
regulatory sequence, A and B, which are affected by the mutations. For this question, a
plus (+) indicates a normal site and a minus () indicates a mutant site that no longer
binds its gene regulatory protein.
Table 8-8

normal (A+ B+)


mutant 1
mutant 2
mutant 3
A.
B.
C.

Transcription of the operon in different media


Glucose
Sucrose
Glucose + sucrose
only
only
OFF
ON
OFF
OFF
OFF
OFF
OFF
ON
ON
OFF
OFF
OFF

If mutant 1 has sites AB+, which of these sites is regulated by sucrose and which
by glucose?
Give the state (+ or ) of the A and B sites in mutants 2 and 3.
Which site is bound by a repressor and which by an activator?

126

8-9

Which one of the following is the main reason that a typical eucaryotic gene is able to
respond to a far greater variety of regulatory signals than a typical procaryotic gene or
operon?
(a)
Eucaryotes have three types of RNA polymerase.
(b)
Eucaryotic RNA polymerases require general transcription factors.
(c)
The transcription of a eucaryotic gene can be influenced by proteins that bind far
from the promoter.
(d)
Procaryotic genes are packaged into nucleosomes.
(e)
The protein-coding regions of eucaryotic genes are longer than those of
procaryotic genes.

8-10

Match the following types of RNAs with the main polymerase that transcribes them:
Types of RNAs
A. rRNA genes
B. tRNA genes
C. 5S rRNA genes
D. protein coding genes

8-11

Polymerases
1. RNA polymerase I
2. RNA polymerase II
3. RNA polymerase III

List three ways that the process of eucaryotic transcription differs from the process of
bacterial transcription.

127

8-12

For each of the following sentences, fill in the blanks with the best word or phrase
selected from the list below. Not all words or phrases will be used; each word or phrase
should be used only once.
In eucaryotic cells, general transcription factors are required for the
activity of all promoters transcribed by RNA polymerase II. The assembly
of the general transcription factors begins with the binding of the factor
__________________ to DNA, causing a dramatic local distortion in the
DNA. This factor binds at the DNA sequence called the
__________________ box, which is typically located 25 nucleotides
upstream from the transcription start site. Once RNA polymerase II has
been brought to the promoter DNA, it must be released in order to begin
making transcripts. This release process is facilitated by the addition of
phosphate groups to the tail of RNA polymerase by the factor
__________________. It must be remembered that the general
transcription factors and RNA polymerase are not sufficient to initiate
transcription in the cell and are affected by proteins bound thousands of
nucleotides away from the promoter. Proteins that link the distantly bound
gene regulatory proteins to RNA polymerase and the general transcription
factors include the large complex of proteins called
the__________________. The packing of DNA into chromatin also
affects transcriptional initiation, and histone __________________ is an
enzyme that can render the DNA less accessible to the general
transcription factors.
activator
CAP
deacetylase
enhancer

lac
ligase
mediator
TATA

128

TFIIA
TFIID
TFIIH

8-13

Label the following structures on the figure below.

Figure Q8-13
A.
B.
C.
D.
8-14

Activator protein
RNA polymerase
General transcription factors
Mediator

How are most eucaryotic gene regulatory proteins able to affect transcription when their
binding sites are far from the promoter?
(a)
By binding to their binding site and sliding to the site of RNA polymerase
assembly
(b)
By looping out the intervening DNA between their binding site and the promoter
(c)
By unwinding the DNA between their binding site and the promoter
(d)
By attracting RNA polymerase and modifying it before it can bind to the
promoter

129

8-15

The expression of the BRF1 gene in mice is normally quite low, but mutations in a gene
called BRF2 lead to increased expression of BRF1. You have a hunch that nucleosomes
are involved in the regulation of BRF1 expression and so you investigate the position of
nucleosomes over the TATA box of BRF1 in normal mice and in mice that either lack the
BRF2 protein (BRF2) or part of histone H4 (HHF ) (histone H4 is encoded by the HHF
gene). Your results are summarized below. A normal functional gene is indicated by a
plus (+).
Mouse
BRF2+ HHF+
BRF2 HHF+
BRF2+ HHF
BRF2 HHF

Nucleosome positioning
specific pattern
random
random
random

Relative level of BRF1 mRNA


1
100
1
100

Which of the following conclusions CANNOT be drawn from your data? Explain your
answer.
(a)
BRF2 is required for repression of BRF1.
(b)
BRF2 is required for the specific pattern of nucleosome positions over the BRF1
upstream region.
(c)
The specific pattern of nucleosome positioning over the BRF1 upstream region is
required for BRF1 repression.
(d)
The part of histone H4 missing in HHF mice is not required to form
nucleosomes.
8-16

The yeast Gal4 gene encodes a transcriptional regulator that can bind DNA upstream of
genes required for the metabolism of the sugar galactose and turns on these genes when
necessary. The Gal4 protein contains two protein domains: a DNA-binding domain and
an activation domain. The DNA-binding domain allows it to bind to the appropriate sites
in the promoters of the galactose metabolism genes. The activation domain attracts
histone-modifying enzymes and also binds to a component of the RNA polymerase II
enzyme complex, attracting it to the promoter so the regulated genes can be turned on
when Gal4 is also bound to the DNA. When Gal4 is expressed normally, the genes can
be maximally activated. You decide to try to produce more of the galactose metabolism
genes by overexpressing the Gal4 protein at levels fifty-fold greater than normal. You
conduct experiments to show that you are overexpressing the Gal4 protein and that it is
properly localized in the nucleus of the yeast cells. To your surprise, you find that too
much Gal4 causes the galactose genes to be transcribed only at a low level. What is the
most likely explanation for your findings?

130

The Molecular Mechanisms That Create Specialized Cell Types


8-17

In principle, how many different cell types can an organism having four different types of
gene regulatory proteins and thousands of genes create?
(a)
Up to 4
(b)
Up to 8
(c)
Up to 12
(d)
Up to 16
(e)
Thousands

8-18

A virus produces a protein X that activates only a few of the viruss own genes (V1, V2,
and V3) when it infects cells. The cellular proteins A (a zinc finger protein) and the
cellular protein B (a homeodomain protein) are known to be repressors of the viral genes
V1, V2, and V3. You examine the complete upstream gene regulatory sequences of these
three viral genes and find that:
1.
2.
3.

V1 and V2 contain binding sites for the zinc finger protein A only.
V3 contains a binding site for the homeodomain protein B only.
The only sequence that all three genes have in common is the TATA box.

Label each one of the choices below as LIKELY or UNLIKELY to be an explanation for
your findings. For each choice you label as UNLIKELY, explain why.
A.
B.
C.
D.
E.
8-19

Protein X binds nonspecifically to the DNA upstream of V1, V2, and V3 and
activates transcription.
Protein X binds to a repressor and prevents the repressor from binding upstream
of V1, V2, and V3.
Protein X activates transcription by binding to the TATA box.
Protein X activates transcription by binding to and sequestering protein A and
protein B.
Protein X represses transcription of the genes for proteins A and B.

In mammals, individuals with two X chromosomes are female while individuals with an
X and a Y chromosome are male. It had long been known that a gene located on the Y
chromosome was sufficient to induce the gonads to form testes, which is the main maledetermining factor in development, and people began to search for the product of this
gene, the so-called testes-determining factor (TDF). For several years, the TDF was
incorrectly thought to be a zinc-finger protein encoded by a gene called BoY. Which of
the following is the best evidence that BoY could NOT be the TDF? Explain your answer.
(a)
Some XY individuals that develop into females have mutations in a different
gene, SRY, but are normal at BoY.
(b)
BoY is not expressed in the adult male testes.
(c)
Expression of BoY in adult females does not masculinize them.
(d)
A few of the genes that are known to be expressed only in the testes have binding
sites for the BoY protein in their upstream regulatory sequences, but most do not.
(e)
A gene encoding a protein whose amino acid sequence is very similar to that of
the BoY protein is found on the X chromosome.
131

8-20

From the sequencing of the human genome, we believe that there are approximately
30,000 protein-coding genes in the genome, for which there are an estimated 1500 to
3000 transcription factors. If every gene has a tissue-specific and signal-dependent
transcription pattern, how can such a small number of transcriptional regulatory proteins
generate a much larger set of transcriptional patterns?

How We Know: Gene RegulationThe Story of Eve


8-21

The gene for a hormone necessary for insect development contains binding sites for three
gene regulatory proteins called A, B, and C. Because the binding sites for A and B
overlap, A and B cannot bind simultaneously. You make mutations in the binding sites
for each of the proteins and measure hormone production in cells that contain equal
amounts of the A, B, and C proteins. The results of your studies are summarized in
Figure Q8-22. In each of the following sentences, choose one of the phrases within
square brackets to make the statement consistent with the above results.

Figure Q8-22
A.
B.
C.

Protein A binds to its DNA binding site [more tightly/less tightly] than protein B
binds to its DNA binding site.
Protein A is a [stronger/weaker] activator of transcription than protein B.
Protein C is able to prevent activation by [protein A only/protein B only/both
protein A and protein B].

132

8-22

The Drosophila eve gene has a complex promoter containing multiple binding sites for four
gene regulatory proteins: Bicoid, Hunchback, Giant, and Krppel. Bicoid and Hunchback are
activators of eve transcription while Giant and Krppel repress eve transcription. The patterns
of expression of these regulators are shown in Figure Q8-22A.
As we discussed, the eve promoter contains modules that control expression in various
stripes. You construct a reporter gene that contains the DNA 5 kb upstream of the eve
gene, so that this reporter contains the stripe 3 module, the stripe 2 module, the stripe 7
module, and the TATA box, all fused to the lacZ reporter gene (which encodes the galactosidase enzyme), as shown in Figure Q8-22B. This construct results in expression
of the -galactosidase enzyme in three stripes, which correspond to the normal positions
of stripes 3, 2, and 7.

Figure Q8-22

133

A.
B.

By examining the overlap of sites on the strip 2 module, as depicted in Figure Q822B, what is the biological effect of having some of the gene regulatory protein
binding sites overlap?
You make two mutant versions in which several of the binding sites in the eve
stripe 2 module have been deleted, as detailed below. Refer to Figure Q8-22B for
the positions of the binding sites. (Note, however, that because many of the
binding sites overlap, it is not possible to delete all of one kind of site without
affecting some of the other sites.) Match the appropriate mutant condition with
the most likely pattern of eve expression shown in Figure Q8-22C. Explain your
choices.
i.
ii.

Deletion of the Krppel binding sites in stripe 2.


Deletion of the two bicoid binding sites in the stripe 2 module that are
marked with an asterisk (*) in Figure Q8-22B.

134

Answers
8-1

Choice (b) is the correct answer. The major cause of differences between different cell
types is in the differential expression of protein-coding genes transcribed by polymerase
II, since these genes encode not only the speciality proteins characteristic of different cell
types, but also the gene regulatory proteins required to maintain and control this pattern
of expression. Choice (a) is untrue, since all genes are replicated equally when cells
divide. Choice (c) is untrue, since expression of housekeeping genes do not differ much
from cell to cell, as they mainly encode the proteins that are necessary for all cells to live.
Choice (d) is untrue, as the main stage at which gene expression is regulated is the
initiation of transcription. Choice (e) is untrue, as DNA is packed into nucleosomes in all
eucaryotic cells.

8-2

See Figure A8-2

Figure A8-2
8-3

The genes of a bacterial operon are transcribed into a single mRNA. Many bacterial
promoters contain a region known as a(n) operator, to which a specific gene regulatory
protein binds. Genes in which transcription is prevented are said to be repressed.
Bacterial genes are regulated by small molecules such as tryptophan by the interaction of
such molecules with allosteric DNA-binding proteins such as the tryptophan repressor.
Genes that are being constitutively expressed are being transcribed all the time.

8-4

A.
B.

If HisP functions as a gene repressor, it would bind more tightly to the regulatory
sequences when histidine is abundant, because the histidine biosynthetic genes
should be turned off when the cell has enough histidine.
If HisP functions as a gene activator, it should bind more tightly to the regulatory
sequences when histidine levels are low. When histidine levels are low, the cell
needs to synthesize more histidine.

135

8-5

A.

B.
C.

8-6

A.

B.

8-7

A.
B.

C.

D.

If sufficient glutamine is present in cells, the glutamine repressor will block the
synthesis of enzymes that would make more glutamine. Likewise, if cells are
starved for glutamine, the unoccupied repressor would not bind to the DNA and
the enzymes that synthesize glutamine would be induced. This allows for a direct
connection between the levels of glutamine and the expression of glutamine
synthesizing enzymes.
The glutamine synthesis enzymes would be permanently ON, regardless of the
level of glutamine in the cells.
The glutamine synthesis enzymes would be always OFF, regardless of the level of
glutamine in the cells, since the repressor is always bound to the DNA. These
cells will not be able to grow unless glutamine is added to the medium.
If the AraC protein acted as a gene repressor for the arabinose operon, araA RNA
levels should be high in the presence or absence of arabinose when there is no
AraC protein around. In fact, the araA RNA levels should be high all the time,
regardless of the presence or absence of arabinose, since the AraA gene should be
transcribed under all conditions in the absence of AraC.
The results are consistent with AraC acting as a gene activator for the arabinose
operon. A gene activator must bind to the promoter regions of the arabinose
genes in order to stimulate their transcription. Thus, if the gene for the regulatory
protein is deleted, the arabinose genes cannot be turned on.
Operon OFF. CAP will not bind in the presence of glucose.
Operon OFF. Although normally the lac repressor would bind in the absence of
lactose, the lack of the lac repressor in this case does not matter because the
presence of glucose means that the CAP protein will not bind and activate
transcription.
Operon ON. Normally in the absence of both glucose and lactose, the operon
would be OFF. However, since the cells are lacking the lac repressor, the cells
cannot sense the absence of lactose. Because the CAP protein will bind and
activate transcription, the operon will be ON.
Operon ON. The CAP protein will bind and activate transcription due to the
presence of glucose. It does not matter whether the lac repressor gene is mutant,
because there is lactose available.

8-8

A.
B.
C.

Site A is regulated by sucrose and site B by glucose.


Mutant 2 (A+ B); mutant 3 (A B) or (A B+).
Site A is bound by an activator and site B by a repressor.

8-9

(c)

The fact that, in eucaryotes, gene regulatory proteins can influence the initiation
of transcription even when they are bound far away from the promoter means that
there can be a very large number of gene regulatory sites affecting the same
promoter. Thus, the initiation of transcription can be influenced by a great variety
and number of different signals, each of which may induce the binding of
different gene regulatory proteins to these regulatory regions.

136

8-10

A1; B3; C3; D2

8-11

Any three of these are acceptable.


1.
Bacterial cells contain a single RNA polymerase whereas eucaryotic cells have
three.
2.
Bacterial RNA polymerase can initiate transcription without the help of additional
proteins whereas eucaryotic RNA polymerases need general transcription factors.
3.
In eucaryotic cells, gene regulatory proteins can influence transcriptional
initiation thousands of nucleotides away from the promoter whereas bacterial
regulatory sequences are very close to the promoter.
4.
Eucaryotic transcription is affected by chromatin structure and nucleosomes
whereas bacterial transcription is not.

8-12

In eucaryotic cells, general transcription factors are required for the activity of all
promoters transcribed by RNA polymerase II. The assembly of the general transcription
factors begins with the binding of the factor TFIID to DNA, causing a dramatic local
distortion in the DNA. This factor binds at the DNA sequence called the TATA box,
which is typically located 25 nucleotides upstream from the transcription start site. Once
RNA polymerase II has been brought to the promoter DNA, it must be released in order
to begin making transcripts. This release process is facilitated by the addition of
phosphate groups to the tail of RNA polymerase by the factor TFIIH. It must be
remembered that the general transcription factors and RNA polymerase are not sufficient
to initiate transcription in the cell and are affected by proteins bound thousands of
nucleotides away from the promoter. Proteins that link the distantly bound gene
regulatory proteins to RNA polymerase and the general transcription factors include the
large complex of proteins called the mediator. The packing of DNA into chromatin also
affects transcriptional initiation, and histone deacetylase is an enzyme that can render the
DNA less accessible to the general transcription factors.

8-13

See Figure A8-13.

Figure A8-13
137

8-14

(b)

Of the cases studied thus far, most eucaryotic gene regulatory proteins that act at a
distance are thought to do so by looping out the intervening DNA while at the
same time binding, via the mediator, to proteins that form the initiation complex
at the promoter.

8-15

(c)

All the other conclusions can be drawn from the data. Since the BRF2+ HHF
mutant does not exhibit the specific pattern of nucleosome positioning yet still has
a low level of BRF1 expression, and since the BRF2HHF mutant has high levels
of BRF1 expression (indicating that HHF is not required for BRF1 expression), it
would appear that repression of BRF1 could take place in the absence of
nucleosome positioning. Since nucleosomes are formed in all cases, the missing
portion of histone H4 is not required for their formation.

8-16

In order for Gal4 to work properly, the DNA-bound Gal4 must attract histone-modifying
enzymes and recruit RNA polymerase to the promoter. If there is too much Gal4 in the
cell, the non-DNA-bound Gal4 (or free Gal4) will compete with the DNA-bound Gal4 for
binding to histone modifying enzymes and RNA polymerase. The excess amount of Gal4
forms non-productive complexes with histone modifying enzymes and RNA polymerase,
preventing their recruitment to the promoter and lowering the level of transcription.

8-17

(d)

The type of cell is determined by the particular combination of gene regulatory


proteins active within it. With four different proteins available, there is one
possibility with no proteins at all and one with all four proteins. There are four
possibilities with one protein each, six possible combinations of two different
proteins, and four possible combinations of three different proteins.

8-18

A.

Unlikely. If protein X were to bind nonspecifically to DNA, it would not


specifically regulate a particular subset of genes.
Unlikely. If a single repressor were to bind upstream of, we would expect to have
found a binding site common to all three genes, but there is none.
Unlikely. If a protein X activated gene transcription were to bind to the TATA
box, it would be likely to activate most genes transcribed by RNA polymerase II,
since most genes contain TATA boxes.
Likely
Likely

B.
C.
D.
E.

138

8-19

Choice (a) is the correct answer. XY individuals that develop as females presumably lack
the testis-determining factor (TDF). If BoY is normal in these individuals, it would
strongly suggest that BoY is not the TDF. Although expression of TDF is necessary for
testes development, this does not mean that is must be expressed in adult males once the
gonad has already formed. Similarly, even though TDF expression is sufficient to induce
testis formation, once the structures are formed, TDF may not be able to exert any
additional effect (thus choices (b) and (c) are not considered strong evidence against BoY
being TDF). Choice (d) is not compelling evidence against BoY being the TDF, since
the TDF will not necessarily bind upstream of all of the genes whose expression it
influences; some of the genes it regulates directly probably encode other gene regulatory
proteins that bind to regulatory sites different from the TDF site. The presence of a
protein similar to BoY on the X chromosome (choice (e)) is not necessarily evidence for
or against BoY being the TDF.

8-20

Gene regulatory proteins are generally used in combinations, thereby increasing the
possible regulatory repertoire of gene expression with a limited number of proteins.

8-21

A.
B.
C.

8-22

A.

B.

Protein A binds to its DNA binding site more tightly than protein B binds to its
DNA binding site.
Protein A is a weaker activator of transcription than protein B.
Protein C is able to prevent activation by both protein A and protein B.
Repressor (Krppel and Giant) binding sites do not seem to overlap. Activator
(Bicoid and Hunchback) binding sites also do not seem to overlap. Instead, the
binding sites for repressor proteins seem to overlap with the binding sites for
activator proteins. These overlapping binding sites allow for repressor and
activator proteins to compete for binding to the DNA. It is thought that the
binding of a repressor and an activator is mutually exclusive. The overlap
between the repressor and activator binding sites allows eve expression to be
exquisitely sensitive to the levels of repressors and activators in the cell and
suggests that the repressors function by preventing activator binding. In fact, the
repressors and activators can antagonize each other, allowing the creation of sharp
stripes of transcription from smooth gradients of protein regulatory factors.
i.
Mutant embyo (B). When the Krppel-binding sites are removed, the
effects of the Krppel repressor are eliminated. Stripe 2 expression now
expands slightly in the posterior direction, which is to be expected since
hunchback and bicoid expression extends slightly beyond the posterior
end of stripe 2.
ii.
Mutant embryo (C). When two of the bicoid binding sites are removed,
expression from the promoter is less sensitive to the effects of the bicoid
activator. Thus, stripe 2 appears at its normal position, but the expression
of -galactosidase is decreased.

139

CHAPTER 9
HOW GENES AND GENOMES EVOLVE
2009 Garland Science Publishing
3rd Edition

Generating Genetic Variation


9-1 Which of the following statements are TRUE? (Note that more than one statement may
be true.)
(a) Asexual reproduction involves the formation of germ cells.
(b) A mutation that arises in a mothers somatic cell often causes a disease in her
daughter.
(c) All mutations in a typical asexually reproducing organism are passed onto progeny.
(d) In an evolutionary sense, somatic cells exist only to help propagate germ line cells.
(e) A mutation is passed on to offspring only if it is present in the germ line.

9-2 Transposable elements litter the genomes of primates and a few of them are still capable of
moving to new regions of the genome. If a transposable element jumped into an important gene
in one of your cells when you were a baby and caused a disease, is it likely that your child
would also have the disease? Explain.
9-3 What is the most likely explanation for why the overall mutation rates in bacteria and
humans are roughly similar?
(a) The DNA replication and repair machinery cannot possibly be any more accurate.
(b) Cell division needs to be fast.
(c) Most mutations are silent.
(d) There is a narrow range of mutation rates that offers an optimal balance between
keeping the genome stable and generating sufficient diversity in a population.
(e) It benefits a multicellular organism to have some variability among its cells.

9-4 For each statement below, indicate if it is TRUE or FALSE and explain why.
(a) To address a challenge or develop a new function, evolution essentially builds from
first principles, like an engineer, to find the best possible solution.
(b) Nearly every instance of DNA duplication leads to a new functional gene.
(c) A pseudogene is highly homologous to a functional gene but cannot be expressed due
to mutation(s).
(d) Most genes in vertebrates are unique, and only a few genes are members of multigene
families.
(e) Horizontal transfer is very rare and thus has had little influence on the genomes of
bacteria.

141

9-5

Consider a gene with a particular function. Mutation X and mutation Y each cause
defects in the function of the encoded protein. Yet a gene containing both mutations X
and Y encodes a protein that works even better than the original protein. The odds that a
single mutational event will generate both mutations X and Y are exceedingly small.
Explain a simple way that an organism with a mutant gene containing both mutations X
and Y could arise during evolution.

9-6

For each of the following sentences, fill in the blanks with the best word or phrase
selected from the list below. Not all words or phrases will be used; each word or phrase
should be used only once.
Most variation between individual humans is in the form of
__________________. __________________ may arise by
recombination within introns and can create proteins with novel
combinations of domains. Scientists and government regulators must be
very careful when introducing herbicide-resistant transgenic corn plants
into the environment, because if resistant weeds arise from
__________________ then the herbicides could become useless. Families
of related genes can arise from a single ancestral copy by
__________________ and subsequent __________________.
divergence
exon shuffling
gene duplication
horizontal gene transfer

purifying selection
single nucleotide polymorphisms
synteny
unequal crossing over

142

9-7

Figure Q9-7 shows several possible substrates of exon shuffling. Horizontal lines and
small filled circles represent chromosomes and centromeres, respectively. Exons are
labeled A, B, C, and D. Homologous recombination or shuffling may take place at short,
repeated homologous DNA sequences in introns; because DNA sequences have a
polarity, the repeated sequences can be considered to have a head and a tail and thus are
drawn as arrows. A recombinational crossover is indicated by a big X. Panel (A) shows
that recombination between two direct repeats located on opposite sides of the
centromere yields one circular product that contains a centromere and a second product
that lacks a centromere and will therefore be lost when the cell divides. Panel (B) shows
that recombination between inverted repeats flanking the centromere will keep the
rearranged chromosome intact. Draw the products of recombination when the repeated
sequences are located on different chromosomes, as shown in panels (C) and (D). Will
these products be faithfully transmitted during cell division?

Figure Q9-7
9-8

Which of the following would contribute most to successful exon shuffling?


(a)
Shorter introns
(b)
A haploid genome
(c)
Exons that code for more than one protein domain
(d)
Introns that contain regions of similarity to one another
(e)
Inability of short stretches of amino acids to fold into discrete functional units

143

Reconstructing Lifes Family Tree


9-9

You are working in a human genetics laboratory that studies causes and treatments for
eye cataracts in newborns. This disease is thought to be caused by a deficiency in an
enzyme called galactokinase, but the human gene that encodes this enzyme has not yet
been identified. At a talk by a visiting scientist, you learn about a strain of bakers yeast
that contains a mutation called gal1 in its galactokinase gene. Because this gene is
needed to metabolize galactose, the mutant strain cannot grow in galactose medium.
Knowing that all living things evolved from a common ancestor and that distantly related
organisms often have homologous genes that perform similar functions, you wonder if
the human galactokinase gene can function in yeast. Since you have an optimistic
temperament, you decide to pursue this line of experimentation. You isolate mRNA gene
transcripts from human cells, use reverse transcriptase to make complementary DNA
(cDNA) copies of the mRNA molecules, and ligate the cDNAs into circular plasmid
DNA molecules that can be stably propagated in yeast cells. You then transform the pool
of plasmids into gal1 yeast cells so that each cell receives a single plasmid. What do
you think will happen when you spread the plasmid-containing cells on petri plates that
contain galactose as a carbon source? How can this approach help you find the human
gene encoding galactokinase?

9-10

A.

B.

9-11

When a mutation arises, it can have three possible consequences: beneficial to the
individual, selectively neutral, or detrimental. Order these from most likely to
least likely.
The spread of a mutation in subsequent generations will, of course, depend on its
consequences to individuals that inherit it. Order the three possibilities above
from that which is most likely to spread and become over-represented in
subsequent generations to that which is most likely to become under-represented
or disappear from the population.

Some types of genes are more highly conserved than others. For each of the following
pairs of gene functions, choose the one that is more likely to be highly conserved.
(a)
Genes involved in sexual reproduction vs. genes involved in sugar metabolism
(b)
DNA replication vs. developmental pathways
(c)
Hormone production vs. lipid synthesis

144

9-12

A hypothetical phylogenetic tree is shown in Figure Q9-12. Use this tree to answer the
following questions.

Figure Q9-12
A.
B.
C.
D.

9-13

How many years ago did species M and N diverge from their last common
ancestor?
How much nucleotide divergence is there on average between M and N?
Are M and N more or less closely related to each other than P and S?
In looking for functionally important nucleotide sequences, is it more informative
to compare the genome sequences of M and N or those of M and Q?

For each statement below, indicate if it is TRUE or FALSE and explain why.
(a)
All highly conserved stretches of DNA in the genome are transcribed into RNA.
(b)
To find functionally important regions of the genome, it is more useful to
compare species whose last common ancestor lived 100 million years ago rather
than 5 million years ago.
(c)
Most mutations and genome alterations have neutral consequences.
(d)
Proteins required for growth, metabolism, and cell division are more highly
conserved than those involved in development and in responding to the
environment.
(e)
Introns and transposons tend to slow the evolution of new genes.

145

9-14

A friend of yours has sequenced the genome of her favorite experimental organism, a
kind of yeast. She wants to identify the locations of all the genes in this genome. To aid
her search, she decides to collaborate with another researcher, one who has sequenced the
genome of a distantly related yeast species. Luckily, the absence of introns simplifies the
effort. She and her collaborator use a computer program to align similar stretches of
DNA sequence from the two genomes. The program yields the graphical output shown
in Figure Q9-14, where the horizontal lines represent a portion of the two genomic
sequences and vertical lines indicate where the sequences differ. (No vertical line means
the sequence is identical in the two yeasts.) Label both the functionally conserved
regions and the divergent (nonconserved) sequences. Are all of the functionally
conserved regions likely to be transcribed into RNA? If not, what might be the function
of the nontranscribed conserved regions?

Figure Q9-14
9-15

The genomes of some vertebrates are much smaller than those of others. For example,
the genome of the puffer fish Fugu is much smaller than the human genome, and even
much smaller than those of other fish, primarily due to the small size of its introns.
A.
Describe a mechanism that might drive evolution toward small introns or loss of
introns and could therefore account for the evolutionary loss of introns according
to the introns early hypothesis.
B.
Describe a mechanism that might drive evolution toward more or larger introns
and could thereby account for the evolutionary appearance of introns according to
the introns late hypothesis.

Examining the Human Genome


9-16

For each statement below, indicate if it is TRUE or FALSE and explain why.
(a).
The increased complexity of humans compared to flies and worms is largely due
to the vastly larger number of genes in humans.
(b)
Repeats of the CA dinucleotide are useful for crime investigations and other
forensic applications.
(c)
The majority of single-nucleotide polymorphisms cause no observable functional
differences between individual humans.
(d)
There is little conserved synteny between human and mouse genes.
(e)
The differences between multicellular organisms are largely explained by the
different kinds of genes carried on their chromosomes.

146

9-17

The number of proteins found in humans and other organisms can vastly exceed the
number of genes. This is largely due to
(a)
protein degradation.
(b)
alternative splicing.
(c)
homologous genes.
(d)
synteny.
(e)
mutation.

HOW WE KNOW: COUNTING GENES


9-18

Explain how ESTs are identified and how they aid in finding the genes within an
organisms genome.

147

Answers
9-1

C, D, E. A is false because it is sexual reproduction that requires the production of germline cells. B is false because mutations are carried in the genetic material and the only
genetic material passed along to the offspring of a sexually-reproducing organism comes
from a germ-line cell (not a somatic cell).

9-2

It is not likely that child would have the disease, because it is unlikely that the mutation is
carried in the germ line. Probably the mutation occurred in a cell that gave rise to
somatic cells and not germ cells. Only mutations in germ cells are passed onto progeny.

9-3

Choice (d) is the correct answer. Choices (a), (b), and (e) are probably false. Choice (c)
is true but cannot explain the similar mutation rate.

9-4

(a)

(b)
(c)
(d)
(e)

False. Evolution can work only by tinkering with the tools and materials on hand,
not by starting from scratch to make completely new genes or pathways. New
functions arise from the ancestral functions by a process of gradual mutational
change, and thus may not represent the best possible solution to a problem.
False. Many duplications are subsequently lost or become pseudogenes, and only
a few evolve into new genes.
True. Pseudogenes look very similar to normal genes but cannot produce a fulllength protein due to one or more disabling mutations.
False. A large proportion of the genes in vertebrates (and many other species) are
members of multigene families.
False. By some estimates, 20% of the genomic DNA in some bacterial species
arose by horizontal gene transfer.

9-5

The simplest way to evolve the new gene is by duplication and divergence. If the gene is
duplicated, then the cell or lineage can maintain one functional, intact old copy of the
original gene and can thus tolerate the disabling mutations in the other copy. The second
copy can first be modified by the X or Y mutation that impairs its function; second, at
some later time, the gene with the single mutation can acquire the additional mutation to
yield the doubly mutant X+Y gene with the new or improved function.

9-6

Most variation between individual humans is in the form of single nucleotide


polymorphisms. Exon shuffling may arise by recombination within introns and can
create proteins with novel combinations of domains. Scientists and government
regulators must be very careful when introducing herbicide-resistant transgenic corn
plants into the environment, because if resistant weeds arise from horizontal gene
transfer then the herbicides could become useless. Families of related genes can arise
from a single ancestral copy by gene duplication and subsequent divergence.

148

9-7

See Figure A9-7. The products of panel (C) will be segregated to progeny cells reliably.
In contrast, one product in panel (D) will have two centromeres and the other will lack a
centromere. The chromosome without a centromere will be rapidly lost as cells divide.
The chromosome containing two centromeres probably will be broken during mitosis
(see Chapter 19), and subsequently lost or severely damaged.

Figure A9-7
9-8

Choice (d) is the correct answer. Exon shuffling is facilitated by long introns (thus
choice (a) is incorrect) and by short exons that each code for one protein domain (thus
choice (c) is incorrect). Since exon shuffling can occur via recombination between
introns, introns with regions of similarity to one another will facilitate shuffling. A
haploid genome will probably be less prone to exon shuffling than a diploid genome (thus
choice (b) is incorrect) because having two copies of each gene allows an organism to
keep one copy of the gene as a backup while it shuffles the other copy. Exon shuffling is
possible only because many proteins are modular, composed of short, folded domains
that have discrete functional properties (thus choice (e) is incorrect).

9-9

On galactose medium, the original gal1 yeast cells cannot grow, nor can cells that
received plasmids containing most human cDNA sequences. However, yeast cells that
received a plasmid with the human galactokinase gene will probably be able to grow on
galactose medium and produce many progeny. This kind of selection procedure is very
powerful, because even if only 1 in 100,000 cells has the ability to grow under particular
conditions it will be easy to find it. The other 99,999 cells will die in the petri plate and
will therefore be invisible to the investigator. Indeed, scientists have found that the
human galactokinase gene can function perfectly well in yeast and thus can rescue the
defect of the gal1 mutant. It was initially astonishing that genes from humans can
function properly in yeast, but this phenomenon has now been observed for many genes.

149

9-10

A.

B.

Selectively neutral, detrimental, beneficial. Most nucleotide changes in the


genome, or mutations, will have little to no effect on the fitness of the individual
because many changes are not located in regions that encode a protein or regulate
expression of a gene. Even changes within a coding region may not change the
amino acid encoded or may cause a conservative amino acid change, for example
from one small nonpolar amino acid to another. Most changes that have a
functional consequence will interfere with the regulation of a gene or the behavior
of the encoded protein, usually rendering it useless and occasionally making it
harmful or yielding a new function. Only very rarely will a mutation improve the
performance of the gene or its encoded protein.
Beneficial, selectively neutral, detrimental. Individuals bearing beneficial
mutations will be more likely to have more offspring than others in the
population, and thus the beneficial mutations will become over-represented in the
population in subsequent generations. Individuals bearing detrimental mutations
will be likely to have fewer children and grandchildren, and thus these mutations
will be culled from the population, though perhaps not eliminated.

9-11

(a)
(b)
(c)

Sugar metabolism
DNA replication
Lipid synthesis.
These pathways or phenomena are fundamental to the growth and proliferation of
all cells, including bacteria, and thus are likely to be highly conserved from
species to species.

9-12

A.
B.

M and N diverged 10 million years ago.


There is an average of 2.0% nucleotide substitution in M compared to N (follow
the path connecting the two species, which is twice the distance between each one
and their common ancestor).
Neither more nor less. They show roughly the same degree of relatedness. The
sequence divergence between M and N is about 2.0%, the same as that between P
and S. Both pairs of species diverged 10 million years ago.
It is more informative to compare species that are separated by a greater
evolutionary distance, thus comparing M and Q which diverged 20 million years
ago will be better able to identify sequences likely to be important for function.
Closely related species share many sequences by chance because insufficient time
has been allowed for neutral mutations to accumulate.

C.

D.

150

9-13

(a)
(b)

(c)

(d)

(e)

9-14

False. Many highly conserved stretches of DNA are not transcribed but instead
contain information critical for regulating where and when genes are expressed.
True. Species that diverged recently have many identical stretches of DNA
sequence by chance, whereas sequence similarity between species that diverged
long ago is probably due to functional constraints.
True. Most genomic changes do not alter the amino acid sequence of proteins or
the regulatory properties of genes. Even some mutations that cause minor
alterations have little functional consequence.
True. All organisms need to perform a similar basic set of fundamental functions,
such as those for metabolism, protein synthesis, and DNA replication. Proteins
involved in these functions are shared by descent, and their evolution is
constrained. Different species and cells are likely to require different
developmental paths and to encounter different environmental challenges, so the
proteins involved in these processes will tend to be more variable. For example,
bacteria do not undergo elaborate developmental programs and so lack many of
the regulators of development found in animals.
False. Introns and transposons can act as sites where recombinational crossovers
occur. Transposons can also catalyze genetic rearrangements. Rearrangements
occurring within these sequences are less likely to be detrimental than those
occurring elsewhere in the genome. In general only the short intron sequences
required for splicing are important to intron function; alterations in sequences
outside the splicing sites may have no consequences for intron function and thus
will not be subject to purifying selection.

See Figure A9-14. Not all of the functionally conserved regions will be transcribed into
RNA. Some of the functionally conserved regions are likely to encode RNA and others
are likely to be critical for regulating when the gene is transcribed and when it is turned
off. These nontranscribed regulatory regions may be conserved nearly as much as the
coding regions.

Figure A9-14
9-15

A.
B.

Loss of introns may be caused by spontaneous deletions or selection pressure to


decrease the time or cost of DNA replication.
Acquisition of intron sequences provides a selective advantage in the face of
transposon insertions. According to this idea, introns became sinks for transposon
and virus insertion to protect the rest of the genome. Alternatively, introns may
provide another advantage to the host genome: by providing ample sites for
crossing over, larger introns could facilitate exon shuffling and thus the
generation of new genes with novel functions.

151

9-16

(a)

(b)

(c)
(d)

(e)

False. The number of genes differs only by about a factor of two. It is thought
that the increased complexity of humans is due largely to differences in when and
where the genes are expressed. Differential splicing may also be a major
contributor to the relative complexity of humans.
True. There are CA repeats in many locations throughout the genome. Because
the number repeats at a given location varies greatly between individuals and
families, it can be used as an identifying characteristic to match two samples (e.g.,
blood samples) from the same or related individuals.
True. Nearly all single-nucleotide polymorphisms have no effect on the
appearance or behavior of the individual, but a few cause important differences.
False. Human and mouse chromosomes show extensive synteny, with blocks of
chromosomal DNA exhibiting homologous genes arranged in the same order
between the two species.
False. Multicellular organisms are built from essentially the same toolbox of gene
building blocks, but the parts are put together differently due to regulatory
differences that dictate when and where and how much of each protein is made.
Alternative splicing can also have an important role, as it can generate several
proteins from a single gene in some species, yet the homologous gene in other
species may produce only one protein.

9-17

Choice (b) is the correct answer. Alternative splicing can produce several different
mRNA transcripts from a single gene, and these transcripts can be translated into several
different but related proteins. Choices (c), (d), and (e) do not yield more protein species
than genes. Protein degradation (choice (a)) can produce several proteins from a single
gene, but this mechanism is used sparingly.

9-18

To identify expressed sequence tags, or ESTs, mRNA must first be isolated from cells.
This mRNA is converted into complementary DNA (cDNA) using specialized nucleic
acid polymerases. The nucleotide sequence of a short region of each cDNA is then
determined. Each short sequence (or EST) corresponds to a portion of a gene that was
expressed in the cells from which the mRNA was isolated; each sequence can be used as
a tag to identify or manipulate the gene from which it came. A collection of ESTs can be
fed into a computer to search for matches to the total genome sequence and thereby
identify the sequences and chromosomal locations of many genes.

152

CHAPTER 10
MANIPULATING GENES AND CELLS
2009 Garland Science Publishing
3rd Edition

Isolating Cells and Growing Them In Culture


10-1 For each of the following sentences, fill in the blanks with the best word or phrase
selected from the list below. Not all words or phrases will be used; each word or phrase
should be used only once.
The ability to purify uniform populations of cells from a tissue facilitates their study.
In order to obtain a purified population of cells from a compact tissue,
__________________ enzymes must be used to disrupt the adhesive bonds
between cells. The __________________ cell sorter allows for isolation of a
particular cell type by recognizing cell-surface differences. Once cells are isolated,
their properties can be studied by, for example, biochemical experiments that aim
to purify molecules from cells and reconstitute cellular reactions
__________________ (that is, in a test tube). Although cells may be grown in
culture, most vertebrate cells have a limited number of cell divisions they can
undergo, often due to lack of expression of the __________________ enzyme.
Cells that can divide indefinitely may result from a genetic change; such cells are
considered to be __________________ and can be propagated in culture as a
homogeneous cell line.

agitating
de novo
desiccated
differentiated
electrically excitable

fluorescence-activated
immortalized
in vitro
in vivo
nuclease

polymerase
propagated
proteolytic
telomerase

10-2 Explain why the undifferentiated state of human embryonic stem cells is critical
to their usefulness.

153

How DNA Molecules Are Analyzed


10-3

For each of the following sentences, fill in the blanks with the best word or phrase
selected from the list below. Not all words or phrases will be used; each word or phrase
should be used only once.
A nuclease hydrolyzes the __________________ bonds in a nucleic acid.
Nucleases that cut DNA only at specific short sequences are known as
__________________. DNA composed of sequences from different
sources is known as __________________. __________________ can be
used to separate DNA fragments of different sizes. Millions of copies of a
DNA sequence can be made entirely in vitro using the
__________________ technique.
DNA sequencing
endonucleases
exonucleases
gel electrophoresis
hydrogen
nucleic acid hybridization

10-4

phosphodiester
polymerase chain reaction
recombinant DNA
restriction nucleases
ribonucleases

You have purified DNA from your recently deceased goldfish. Which of the following
restriction nucleases would you use if you wanted to end up with DNA fragments of
average size around 70 kb after complete digestion of the DNA? The recognition
sequence for each enzyme is indicated in the right-hand column. N indicates that any
nucleotide may be in this position and the enzyme will still cleave the site.
(a)
Sau3A I
GATC
(b)
BamH I
GGATCC
(c)
BsaJ I
CCNNGG
(d)
Not I
GCGGCCGC
(e)
Xza I
GAAGGATCCTTC

154

10-5

You have accidentally torn the labels off two tubes, each containing a different plasmid,
and now do not know which plasmid is in which tube. Fortunately, you have restriction
maps for both plasmids, shown in Figure Q10-5. You have the opportunity to test just
one sample from one of your tubes. You have equipment for agarose gel electrophoresis,
a standard set of DNA size markers, and the necessary restriction enzymes.

Figure Q10-5
A.
B.

Outline briefly the experiment you would do to determine which plasmid is in


which tube.
Which restriction enzyme or combination of restriction enzymes would you use in
this experiment?

155

Note: The following codon table is to be used for Problems Q10-6, Q10-24 and Q10-29.

10-6

You have sequenced a short piece of DNA and produced the gel shown below:

Figure Q10-6
A.
B.

What is the sequence of the DNA, starting from the 5 end?


If you knew that this sequence is from the middle of a protein-coding cDNA
clone, what amino acid sequence can be deduced from this sequence?

156

10-7

You have sequenced a fragment of DNA and produced the gel shown in Figure Q10-7.
Near the top of the gel, there is a section where there are bands in all four lanes (indicated
by the arrow). Which of the following mishaps would account for this phenomenon?
Explain your answer.

Figure Q10-7
(a)
(b)
(c)
(d)
(e)

You mistakenly added all four dideoxynucleotides to one of the reactions.


You forgot to add deoxynucleotides to the reactions.
You forgot to add dideoxynucleotides to each of the reactions.
A fraction of the DNA you are sequencing was cut by a restriction nuclease.
Your primer hybridizes to more than one area of the fragment of DNA you are
sequencing.

157

Nucleic Acid Hybridization


10-8

For each of the following sentences, fill in the blanks with the best word or phrase
selected from the list below. Not all words or phrases will be used; each word or phrase
should be used only once.
The technique of __________________ hybridization can be used to
detect a specific RNA expression in a particular region of the brain.
Northern blotting detects a specific sequence in __________________.
Southern blotting detects a specific sequence in __________________. A
short, single-stranded DNA is a(n) __________________. A piece of
DNA used to detect a specific sequence in a nucleic acid by hybridization
is known as a(n) __________________.
DNA
in situ
in vitro
oligonucleotide

polymerase chain reaction


probe
RNA
vector

158

10-9

Assume that defects in a hypothetical gene, X, have been linked to antisocial behavior.
Two copies of a defective gene X predispose a child to bad behavior from childhood,
while a single copy of the gene seems to produce no symptoms until adulthood. Since the
effects of the gene can be counteracted if treatment is started early enough, a program of
voluntary genetic testing is being carried out with delinquent prospective parents. Charles
S. and Caril Ann F. have been arrested on charges of robbery and assault, and Caril Ann
is pregnant with Charless child. You obtain DNA samples from Charles, Caril Ann, and
the fetus. You digest these samples with Not I and use these samples to perform two
Southern Blots, which you probe with two different oligonucleotide probes, A and B, that
hybridize to different parts of the normal gene X, as shown in Figure Q10-9A. Your
results are shown in Figure 10-9B.

Figure Q10-9
A.
B.
C.

Which of the three individuals have defects in gene X?


Which individuals have a single defective gene and which have two defective
copies of the gene?
Indicate the nature (single base-pair mutation or deletion) and location of each
individuals defects on gene X.

10-10 Figure Q10-10 shows a restriction map of a piece of DNA containing your favorite gene.
The arrow indicates the position and orientation of the gene in the DNA. In part B of the
figure are enlargements showing the portions of the DNA whose sequences have been
used to make oligonucleotide probes A, B, C, and D. Which of the oligonucleotides can
be used to detect the gene in each of the following?

Figure 10-10
A.
B.

A Southern blot of genomic DNA cut with Hind III.


A Northern blot.
159

10-11 In situ hybridization can be used to determine the


(a)
sequence of a cloned gene.
(b)
distribution of proteins in tissues.
(c)
position of a cloned fragment of DNA on a plasmid.
(d)
size of a gene.
(e)
distribution of a given type of mRNA in different tissues.
10-12 You are interested in understanding how the brain works, and are using the fruit fly
Drosophila as a model system to study brain development. You perform microarray
analysis to try to determine genes expressed in the fly brain. For your microarray
experiment, you first prepare cDNA from fly brains and label it with a red fluorochrome.
Then, you isolate cDNA from whole flies and label it with a green fluorochrome. Next,
you hybridize these cDNA populations to a microarray containing the Drosophila genes.
From this, you obtain a list of genes that are specifically enriched in the brain (those that
show up as a red spot on the microarray).
You are disappointed because your favorite fly gene, tubby, does not appear on this list,
even though you have repeated the microarray experiment 10 times and did not encounter
any technical difficulties. The reason you thought tubby would appear on this list is that
you believe tubby is important for brain development, since flies mutant in tubby have no
brains. Not to be discouraged, you perform in situ analysis using the tubby DNA as a
probe, and see that it is expressed at high levels in the fly brain of normal flies but not
expressed in animals lacking the tubby gene.
Why do you think tubby did not show up as a gene specifically enriched in the brain in
your microarray experiment?

DNA Cloning
10-13 For each of the following sentences, fill in the blanks with the best word or phrase
selected from the list below. Not all words or phrases will be used; each word or phrase
should be used only once.
Two fragments of DNA can be joined together by __________________.
Restriction enzymes that cut DNA straight across the double helix produce
fragments of DNA with __________________. A fragment of DNA is
inserted into a __________________ in order to be cloned in bacteria. A
__________________ library contains a collection of DNA clones derived
from mRNAs. A __________________ library contains a collection of
DNA clones derived from chromosomal DNA.
blunt ends
cDNA
DNA ligase

DNA polymerase
genomic
probe

160

RNA
staggered ends
vector

10-14 Figure Q10-14 shows the recognition sequences and sites of cleavage for the restriction
enzymes Sal I, Xho I, Pst I, and Sma I and a plasmid with the sites of cleavage for these
enzymes marked.

Figure Q10-14
A.

After which of the following treatments described in choices 1 through 5 can the
plasmid shown in Figure Q10-14 be recircularized simply by treating with DNA
ligase? Assume that after treatment any small pieces of DNA are removed, and it
is the larger portion of plasmid only that you are trying to recircularize.
After digestion with
1.
Sal I alone.
2.
Sal I and Xho I.
3.
Sal I and Pst I.
4.
Sal I and Sma I.
5.
Sma I and Pst I.

B.

In which of the cases 15 can the plasmid be recircularized by adding DNA ligase
after the cut DNA has been treated with DNA polymerase in a mixture containing
the four deoxynucleotides? Again assume that you are trying to recircularize the
larger portion of plasmid.

10-15 Name three features that a cloning vector for use in bacteria must contain. Explain your
answers.
10-16 Which of the following statements about genomic DNA libraries are FALSE?
(a)
The larger the size of the fragments used to make the library, the fewer colonies
you will have to examine to find a clone that hybridizes to your probe.
(b)
The larger the size of the fragments used to make the library, the more difficult it
will be to find your gene of interest once you have identified a clone that
hybridizes to your probe.
(c)
The larger the genome of the organism from which a library is derived, the larger
the fragments inserted into the vector will tend to be.
(d)
The smaller the gene you are seeking, the more likely it is that the gene will be
found on a single clone.
(e)
The shorter the oligonucleotide used to probe the library, the greater the number
of colonies to which the probe will hybridize.
161

10-17 A DNA library has been constructed by purifying chromosomal DNA from mice, cutting
the DNA with the restriction enzyme Not I, and inserting the fragments into the Not I site
of a plasmid vector. What information CANNOT be retrieved from this library?
(a)
Gene regulatory sequences
(b)
Intron sequences
(c)
Sequences of the telomeres (the ends of the chromosomes)
(d)
Amino acid sequences of proteins
10-18 You have the amino acid sequence of a protein and wish to search for the gene encoding
for this protein in a DNA library. Using this protein sequence, you deduce a particular
DNA sequence that can encode for this protein. Why is it unwise to use only this DNA
sequence you have deduced as the probe for isolating the gene encoding your protein of
interest from the DNA library?
10-19 What is the main reason for using a cDNA library rather than a genomic library to isolate
a human gene from which you wish to make large quantities of the human protein in
bacteria?
10-20 Some clones from cDNA libraries can have defects because of the way a cDNA library is
constructed. For each of the dilemmas described in AD, indicate which of the outcomes
listed in 14 you might encounter. Outcomes may be used more than once.
Dilemma
A. The mRNA corresponding to the clone you
are looking for was degraded at its 5 end by
a nuclease.
B. The mRNA corresponding to the clone you
are looking for was degraded at its 3 end by
a nuclease.
C. The 5 end of the reverse transcriptase
product of the gene you are trying to clone
hybridizes to sequences in the middle of the
gene.
D. The gene you are trying to clone has a long
stretch of As in the middle of the coding
sequence.

Outcomes
1. The 5 part of the gene will
be missing.
2. The 3 part of the gene will
be missing.
3. An internal fragment of the
gene will be missing.
4. The gene will be missing
from the library.

10-21 Why is a heat-stable DNA polymerase from a thermophilic bacterium (the Taq
polymerase) used in the polymerase chain reaction rather than a DNA polymerase from
E. coli or humans?

162

10-22 Which of the following is a limitation on the use of PCR to detect and isolate genes?
(a)
The sequence at the beginning and end of the DNA to be amplified must be
known.
(b)
It also produces large numbers of copies of sequences beyond the 5 or 3 end of
the desired sequence.
(c)
It cannot be used to amplify a particular sequence from a mixture of mRNAs.
(d)
It cannot be used to amplify cDNAs.
(e)
It will amplify only sequences present in multiple copies in the DNA sample.
10-23 You have an oligonucleotide probe that hybridizes to part of gene A from a eucaryotic
cell. Indicate whether a cDNA library or a genomic DNA library will be more
appropriate to use for the following applications.
A.
You want to study the promoter of a gene A.
B.
Gene A encodes a tRNA and you wish to isolate a piece of DNA containing the
full-length sequence of the tRNA.
C.
You discover that Gene A is alternatively spliced and you want to see which
predicted alternative splice products are actually produced in a cell.
D.
You want to find both gene A and the genes located near gene A on the
chromosome.
E.
You want to express gene A in bacteria to produce lots of protein A.
10-24 Which of the restriction nucleases listed below can potentially cleave a segment of cDNA
that encodes the peptide KIGDACF? (Refer to Codon table provided above Q10-6.)
Restriction Nuclease
Eco RI
Hind III
Nsi I

Recognition Sequence
GAATTC
AAGCTT
ATGCAT

10-25 You want to amplify the DNA between the two stretches of sequence shown in Figure
Q10-25. Of the listed primers, choose the pair that will allow you to amplify the DNA by
PCR.

Figure Q10-25

163

10-26 Your friend works at the Centers for Disease Control and has discovered a brand new
virus that has been recently introduced into the human population. She has just
developed a new assay that allows her to detect the virus using PCR products made from
the blood of infected patients. The assay utilizes primers in the PCR reaction that
hybridize to sequences in the viral genome.
Your friend is distraught because of the result she obtained (see Figure Q10-26) when she
looked at PCR products made from using the blood of three patients suffering from the
viral disease, from her own blood, and from a leaf from her petunia plant.
You advise your friend not to panic, as you believe she is missing an important control.
Which one of these choices is the best control for clarifying the results of her assay.
Explain your choice.

Figure Q10-26
(a)
(b)
(c)
(d)
(e)

A PCR reaction using blood from a patient that is newly infected but does not yet
show symptoms.
A PCR reaction using blood from a dog.
A PCR reaction from an uninfected person.
Repeating the experiment shes already done with a higher concentration of
primers.
Repeating the experiments shes already done with a new tube of polymerase.

DNA Engineering
10-27 Insulin is a small protein hormone that regulates blood sugar level, and is given by
injection to people who suffer from the disease diabetes. Diabetics once used insulin
purified from pig pancreas to control their diabetes. Give two reasons why the drug
companies who produce insulin wanted to clone the human insulin gene to provide an
alternative source of the hormone.

164

10-28 Scientists produce large quantities of RNA by transcription in vitro rather than by
expression of cloned genes in vivo primarily because
(a)
in vitro transcription removes the need to clone the gene for the RNA into a DNA
vector.
(b)
the viral RNA polymerases used for in vitro transcription are not active in vivo.
(c)
RNA molecules are present in only a few copies per cell and thus only small
amounts of RNA can be prepared in vivo.
(d)
in vitro transcription eliminates the need to purify the RNA produced from the
cells RNA molecules.
10-29 You have been hired to create a cat that will not cause allergic reactions in cat-lovers.
Your co-workers have cloned the gene encoding a protein found in cat saliva, expressed
the protein in bacteria, and shown that it causes violent allergic reactions in people. But
you soon realize that even if you succeed in making a knockout cat lacking this gene,
anyone who buys one will easily be able to make more hypoallergenic cats just by
breeding them. Which of the following will ensure that people will always have to buy
their hypoallergenic cats from you?
(a)
(b)
(c)
(d)
(e)

Injecting the modified ES cells into embryos that have a genetic defect that will
prevent the mature adult from reproducing.
Implanting the injected embryos into a female cat that is sterile due to a genetic
defect.
Selling only the offspring from the first litter of the female cat implanted with the
injected embryos.
Surgically removing the sexual organs of all the knockouts before you sell them.
Selling only male knockouts.

10-30 Your biochemist friend has isolated a protein he thinks is responsible for making you feel
sleepy. Since he knows youre taking Cell Biology, he wants you to help him isolate the
gene encoding this protein. Unfortunately, since your friend could only isolate small
amounts of protein, he was only able to obtain three short stretches of amino acid
sequence from the protein:
(a)
(b)
(c)

H-C-W-K-M
R-S-L-L-S
D-A-Q-W-Y

For each of the three peptides above, you design a set of DNA oligonucleotide probes
that can be used to detect the gene in a library. Which of the three sets of oligonucleotide
probes would be preferable to screen a library? Explain. (Refer to Codon table provided
above Q10-6.)

165

10-31 Your friend has isolated a protein present in the cheek cells of all straight-A seniors at
your University. She says that this protein helps you remember everything you read and
therefore will help cut down on the number of hours needed to study for exams, and calls
the protein geniuszyme. She sequences the protein and designs a probe to isolate the
gene that encodes geniuszyme. To make sure she designed the probe correctly, she
consults with the company that cloned Factor VIII. They have 100% confidence that her
probe will work. She also obtains a high quality liver cDNA library from the company
and uses her probe to try to isolate the gene for geniuszyme. Unfortunately, she is unable
to isolate any clones.
A.
What is the likely explanation for her failure?
B.
Not to be discouraged, your friend has obtained some genomic DNA isolated
from the nuclei of liver cells and has made a genomic library from that DNA. Do
you expect she will succeed in isolating the geniuszyme gene from this library?
Why or why not?

How We Know: Sequencing the Human Genome


10-32 You have been asked to consult for a biotech company that is seeking to understand why
some fungi can live in very extreme environments, such as the high temperatures inside
naturally occurring hot springs. The company has isolated two different fungal species,
F. cattoriae and W. gravinius, both of which can grow at temperatures exceeding 95C.
The company has determined the following things about these fungal species:
Property

F. cattoriae

W. gravinius

Genome size
Repetitive DNA?

1 MB
20% of genome contains
large stretches of CG repeats

3 MB
< 0.1% of genome

By sequencing and examining their genomes, the biotech company hopes to understand
why these species can live in extreme environments. However, the company only has the
resources to sequence one genome, and would like your input as to which species should
be sequenced and whether you believe a shotgun strategy will work in this case. (Be sure
to explain your answer).

166

Answers
10-1

The ability to purify uniform populations of cells from a tissue facilitates their study. In
order to obtain a purified population of cells from a compact tissue, proteolytic enzymes
must be used to disrupt the adhesive bonds between cells. The fluorescence-activated
cell sorter allows for isolation of a particular cell type by recognizing cell-surface
differences. Once cells are isolated, their properties can be studied by, for example,
biochemical experiments that aim to purify molecules from cells and reconstitute cellular
reactions in vitro (that is, in a test tube). Although cells may be grown in culture, most
vertebrate cells have a limited number of cell divisions they can undergo, often due to
lack of expression of the telomerase enzyme. Cells that can divide indefinitely may
result from a genetic change; such cells are considered to be immortalized and can be
propagated in culture as a homogeneous cell line.

10-2

Because embryonic stem cells are undifferentiated, they can retain the ability to give rise
to any tissue in the body. If the differentiation of an embryonic stem cell is guided
appropriately in culture, they could potentially provide a source of cells capable of
replacing or repairing tissues that have been damaged by injury or disease.

10-3

A nuclease hydrolyzes the phosphodiester bonds in a nucleic acid. Nucleases that cut
DNA only at specific short sequences are known as restriction nucleases. DNA
composed of sequences from different sources is known as recombinant DNA. Gel
electrophoresis can be used to separate DNA fragments of different sizes. Millions of
copies of a DNA sequence can be made entirely in vitro by the polymerase chain
reaction technique.

10-4

(d)

A restriction enzyme that has a 4 base-pair recognition sequence cuts on average


once every 44 or 256 base pairs; one that has a 6 base-pair recognition sequence
cuts once every 46 or 4096 base pairs; one that has an 8 base-pair recognition
sequence cuts once every 48 or 65,536 base pairs; one that has a 12 base-pair
recognition sequence cuts once every 412 or 16 million base pairs. So to obtain
fragments of around 70 kb in size, you would cut with an enzyme that recognizes
an 8 base-pair site.

167

10-5

A.

B.

You would first digest your sample with a combination of restriction enzymes
selected so that they give a set of fragment sizes that could have come from only
one of the plasmids. Then you would run the resulting mixture of DNA fragments
on a gel alongside a set of size markers and determine the sizes of each fragment.
By looking at the restriction maps, you should then be able to match your results
to one of the plasmids.
Digestion with any of the following combinations will enable you to distinguish
which plasmid you have: Hind III + Bgl II; Eco RI + Bgl II; Eco RI + Bgl II +
Hind III. The plasmids are the same size, so you cannot distinguish them simply
by making a single cut (with Hind III) and determining the size of the complete
DNA by gel electrophoresis. Nor can you distinguish them by cutting with all four
restriction nucleases, since the set of fragment sizes produced from both plasmids
will be the same. Cutting with Bam HI or Eco RI on their own is not sufficient
because you will get bands of the same size from both plasmid A and plasmid B.
The only difference between the two plasmids is the location of the Bgl II site
relative to the two Bam HI sites, so if you cut with an enzyme that cuts outside the
Bam HI fragment and with Bgl II, you will get different-sized fragments from the
two plasmids.

10-6

A.
B.

TAGACTGACCTG
Arg-Leu-Thr (Only the second reading frame can be used, as reading frame 1
contains a stop codon (TAG), as does reading frame 3 (TGA).)

10-7

Choice (d) is the correct answer. If some of the DNA templates you are sequencing are
cut at one specific site (as would be the case if the DNA were cut by a restriction
enzyme), the polymerase will stop when it comes to the end of the DNA, giving rise to at
least some product of one particular size in all the reaction mixtures. If this were the case,
all four lanes will have a band of this particular size. In addition, you would get normal
sequence from the full-length templates, and normal sequence from those templates in
which the polymerase incorporated a dideoxynucleotide before encountering the end. The
other options are incorrect: if you added all four dideoxynucleotides to one of the
reactions (choice (a)), that lane would have a band at every position because the
polymerase would stop at As, Cs, Gs, and Ts instead of at only one type of nucleotide.
If you forgot to add deoxynucleotides to the reactions (choice (b)), you would not get any
polymerization, and all of your lanes would be blank. If you forgot to add
dideoxynucleotides (choice (c)), the reactions would not stop until the end of the DNA
fragment, and all of the products would be full-length and would all be at the top of the
gel. If your primer hybridized to more than one part of the fragment of DNA you were
sequencing (choice (e)), your gel would look as though two different sequences had been
superimposed onto each other.

10-8

The technique of in situ hybridization can be used to detect a specific RNA expression in
a particular region of the brain. Northern blotting detects a specific sequence in RNA.
Southern blotting detects a specific sequence in DNA. A short, single-stranded DNA is
called a(n) oligonucleotide. A piece of DNA used to detect a specific sequence in a
nucleic acid by hybridization is known as a(n) probe.
168

10-9

A.
B.
C.

10-10 A.
B.

All three are affected.


The two parents have a single defective copy of the gene; the fetus has two
defective copies.
As seen in the two blots in Figure Q10-9B, Caril Ann and Charles each have one
full-length copy of gene X (the bands at the top of the gel), which hybridizes with
both probe A and probe B. The fetus does not. The blot with probe A shows that
Caril Ann and the fetus have a short fragment of gene X that hybridizes with
probe A only, indicating that this copy of gene X has a deletion somewhere other
than in the region recognized by probe A. The second blot (with probe B) shows
that Charles and the fetus have a short fragment that hybridizes with probe B,
indicating that this copy of gene X has a deletion somewhere other than in the
region recognized by probe B. Since the shortened gene found in Charles does not
show up on the probe A blot, this deletion must be in the region of A; similarly,
since the shortened gene found in Caril Ann does not show up on the probe B
blot, her deletion must be in the region of B. The fetus has inherited two defective
copies of gene X, one from each parent.
All four oligonucleotide probes.
Oligonucleotide probe B and oligonucleotide probe D. Both the upper and lower
strands of DNA are present in genomic Southern blots, so all four oligos will
hybridize to either Southern blot. (Oligonucleotides A and B will still be able to
hybridize to genomic DNA cut with Bgl II, since they can still base-pair to the
individual fragments that result from the digest.) Northern blots contain only
RNA, which has the sequence of the upper strand of the DNA. Hence, only
oligonucleotide probe B and oligonucleotide probe D will hybridize to a Northern
blot.

10-11 (e)
10-12 The tubby gene is expressed in all tissues, including the brain. A red spot on a
microarray is indicative of a DIFFERENCE in gene expression between the two RNA
samples being compared. You may expect in this experiment that the tubby gene would
be a yellow spot (a gene expressed at equal levels in both samples). An in situ
experiment looks at the RNA level directly in the tissue of interest, which is why in this
case you see ample levels of tubby RNA.
10-13 Two fragments of DNA can be joined together by DNA ligase. Restriction enzymes that
cut DNA straight across the double helix produce fragments of DNA with blunt ends. A
fragment of DNA is inserted into a vector in order to be cloned in bacteria. A cDNA
library contains a collection of DNA clones derived from mRNAs. A genomic library
contains a collection of DNA clones derived from chromosomal DNA.

169

10-14 A.
B.

1 and 2. When Sal I and Xho I cut DNA, the staggered ends left behind will
match up by base-pairing and thus can be joined by ligase alone.
1, 2, and 4. Sma I cuts and leaves a blunt end. Addition of DNA polymerase and
the four deoxynucleotides will fill in the 5 overhangs generated by digestion with
Sal I and Xho I, leaving blunt ends. Blunt ends can be joined by DNA ligase.
However, 3 overhangs will not be filled in (i.e., those generated by Pst I), as
DNA polymerase moves in a 5 to 3 direction. DNA ligase will not join 3
overhangs to blunt ended DNA, which are the situations presented in choices 3
and 5.

10-15 A cloning vector for use in bacteria must contain:


1.
a bacterial replication origin (to allow the plasmid to be replicated);
2.
at least one unique restriction site (to allow easy insertion of foreign DNA); and
3.
an antibiotic-resistance gene or some other selectable marker gene (to allow
selection for bacteria that have taken up the recombinant plasmids).
10-16 Choice (c) is the correct answer. The size of the fragments that are left after a restriction
digest does not depend on the total size of the genome; it depends on the sequence of the
genome and the frequency with which the restriction enzyme recognition site is found in
the genome. Choices (a) and (b) are true: as a limiting case, you can think of what would
happen if a fragment the size of the entire genome is inserted into the bacterial vector. In
this case, you would have to screen only one colony to find the clone that hybridizes to
your probe, but it will be very difficult to find out where on the insert your gene of
interest lies. Choice (d) is true: the larger the gene you are seeking, the more likely it is
that there will be a restriction fragment in the gene (or that the gene will be broken if the
DNA was fragmented by random shearing), and hence the less likely it is that the entire
gene will be found in one clone. Choice (e) is true: as the size of your oligo probe
decreases, the chance of finding that sequence randomly in the genome increases (just as
the number of restriction sites increases when the size of the recognition site decreases).
10-17 (c)

The very ends of all of the chromosomes are unlikely to be Not I sites, meaning
that the fragments containing the ends of the chromosomes will not be able to
insert into the bacterial vector (since they have not been cut by Not I at both ends)
and will be lost from the library. All sequences present in genomic DNA (which
includes regulatory sequences and introns) should be present in a genomic library.
The coding sequence of the gene (and hence the amino acid sequence of the
encoded protein) is also present in a genomic clone, although it is interrupted by
intron sequences and therefore somewhat difficult (but not impossible) to
determine.

10-18 Because most amino acids can be encoded by more than one codon, a given sequence of
amino acids could be encoded by a number of different nucleotide sequences. Probes
corresponding to all these possible sequences have to be synthesized in order to be sure
of including the one that corresponds to the actual nucleotide sequence of the gene and
thus will hybridize with it.

170

10-19 The gene isolated from a genomic library would still contain introns, and bacteria do not
contain the biochemical machinery for removing introns by RNA splicing. The same
gene isolated from the cDNA library will have already had its introns removed.
10-20 A.
B.

C.

D.

Outcome 1 would occur. If the mRNA is degraded from the 5 end, it would still
be reverse transcribed and would end up in the library as a clone lacking its 5
end.
Outcome 4 would occur. If the mRNA was degraded from the 3 end, it would be
missing its 3 poly(A) tail. In the construction of a cDNA library, only molecules
that still have their poly(A) tail will be reverse transcribed, so mRNAs missing
their 3 end will not be represented in the library.
Outcome 1 would occur. If the 5 end hybridizes to sequences in the middle of the
gene, the hairpin formed when the single-stranded DNA loops back on itself to
form the primer for DNA polymerase will be very large. After this loop is
digested, the remaining double-stranded DNA fragment will be missing the 5 end
of the gene.
Outcome 2 would occur. If the gene has a long stretch of internal As, the poly(T)
primer used in the reverse transcription step could hybridize to the internal
poly(A) stretch rather than to the poly(A) tail, and the resulting cDNA will have
lost its 3 end.

10-21 The PCR technique involves heating the reaction at the beginning of each cycle to
separate the newly synthesized DNA into single strands so that they can act as templates
for the next round of DNA synthesis. Using a heat-stable polymerase avoids having to
add it afresh for each round of DNA replication.
10-22 Choice (a) is the correct answer. In order to construct primers that will bracket the desired
gene, you have to know the sequence at the beginning and end of the DNA to be copied.
Although a few copies of sequences beyond the ends of the desired sequence are made in
the early cycles, these soon become a negligible fraction of the total DNA synthesized,
and thus choice (b) would not be an appropriate usage of PCR and certainly not a
limitation. You can use PCR to amplify a particular RNA sequence (choice (c)) by using
the appropriate primers to first guide the reverse transcriptase reaction that makes a DNA
copy of the RNA and then to guide the synthesis of the complementary DNA strand. PCR
can be used to amplify a sequence from any DNA, including cDNAs (choice (d)). PCR is
extremely sensitive and can detect and amplify a particular sequence even if it is present
only in a single copy in the DNA sample (choice (e)).

171

10-23 A.
B.
C.
D.
E.

Genomic library. (cDNAs are produced from mRNAs; therefore, the promoters
will not be included in a cDNA library.)
Genomic library. (cDNAs are usually constructed using an oligo dT primer;
tRNAs do not have poly(A) tails. If the cDNA library were made using random
primers, it would be unlikely to contain the full-length version of the tRNA.)
cDNA library. (Since cDNAs are produced from mRNAs, isolating cDNAs
would tell you which splice variants are produced in a cell.)
Genomic library. (A genomic DNA fragment can contain the genes next to your
gene of interest; a cDNA will not.)
cDNA library. (Bacteria do not have the ability to remove introns, which may
exist in DNA isolated from a genomic library.)

10-24 The nucleotide sequence that can encode the peptide KIGDACF is below:
K
I
G
D
A
C
F
AAA ATT GGT GAT GCT TGT TTT
G
C
C
C
C
C
C
A
A
A
G
G
The enzyme Nsi I cleaves at ATGCAT.
10-25 The appropriate PCR primers are primer 1 (5-GACCTGTGGAAGC) and primer 8
(5-TCAATCCCGTATG). The first primer will hybridize to the bottom strand and prime
synthesis in the rightward direction. The second primer will hybridize to the top strand
and prime synthesis in the leftward direction. (Remember that strands pair antiparallel.)
The middle two primers in each list (primers 2, 3, 6, and 7) would not hybridize to either
strand. The remaining pair of primers (4 and 5) would hybridize, but would prime
synthesis in the wrong directionthat is, outward, away from the central segment of
DNA. Each of these wrong choices has been made at one time or another in most
laboratories that use PCR. In most cases the confusion arises because the conventions for
writing nucleotide sequences have been ignored. By convention, nucleotide sequences
are written 5 to 3, with the 5 end on the left. For double-stranded DNA, the 5 end of
the top strand is on the left.

172

10-26 (c)

Primers can sometimes hybridize to unintended sequences and produce


unintended products. The appropriate control for your friends experiment would
be DNA from an uninfected person; that way she would be able to determine
whether the bands present in the PCR from her blood truly correspond to product
generated from viral DNA rather than cross-hybridization to DNA sequences in
the human genome, since the bands would be absent from a person uninfected by
the virus in the former case only. Doing PCR from an infected but asymptomatic
person would not be useful, as it would not allow your friend to distinguish
whether she is infected. Although doing PCR from dog blood should not give any
viral bands, any nonspecific products would likely be different from a dog
compared to your friend. The absence of PCR fragments in the petunia lane
suggests that there is no viral contaminant in any of your friends reagents, so
using a new tube of polymerase is not the solution. Increasing the concentration
of primers will exacerbate any nonspecific hybridization.

10-27 Any two of the following would be acceptable.


1.
Cloning the gene allows human insulin to be produced in large quantities from
bacteria or other cells carrying the cloned DNA sequence.
2.
It is easier and less costly to extract the same amount of insulin from a bacterial
culture than from pig pancreas.
3.
Insulin made in a bacterial culture and then purified will be free of any possible
contaminating viruses that pigs (and any other whole animal) tend to harbor.
4.
The pig protein has slight differences from the human protein, which can lead to
side effects on prolonged use. Whenever possible, a human protein would be
preferred for clinical treatment of this sort.
10-28 (d)

In order to produce RNA by transcription in vitro, you must first clone the DNA
of the gene you wish to transcribe in order to get a large amount of pure template.
Many RNAs are produced at high levels in cells. Viral RNA polymerases are able
to transcribe RNA in vivo to especially high levels, since their purpose is to make
high levels of viral proteins.

10-29 Choice (d) is the correct answer. Neutering all the knockout animals that you sell is the
only option of the five listed that will prevent happy pet owners from becoming happy
pet breeders. The situation described in choice (a), will not allow you to make any
knockout cats because the first litter (which will at best have a few mosaics in which one
copy of the gene has been knocked out in the germ cells) will be sterile and you will not
be able to mate them. The genotype of the female cat in which you implant the embryos
has no effect on the genotype of the embryos which is why choice (b) is incorrect. Choice
(c) is incorrect because the first litter will yield mosaic cats that still have one copy of the
allergen-producing gene in their cells, and therefore will not be hypoallergenic. Selling
only male knockouts does no good, because they can be mated to normal females and the
heterozygous offspring can be backcrossed to yield homozygous hypoallergenic cats
(choice (e)).

173

10-30 (a)

H-C-W-K-M. There is the least amount of degeneracy in the nucleotides that


could code for this peptide. (See below)
H
C
W
K
M
CAC TGC TGG AAA ATG
T
T
G
R
S
L
L
S
AGA AGC TTA TTA AGC
G
T
G
G
T
CGA TCA CTA CTA TCA
G
G
G
G
G
C
C
C
C
C
T
T
T
T
T
D
A
Q
W
Y
GAC GCA CAA TGG TAC
T
G
G
T
C
T

10-31 A.
B.

Geniuszyme is not expressed in the liver. Since cDNA is made from mRNA, a
cDNA library is a reflection of the genes expressed in a particular tissue.
Yes, she should be able to isolate the gene, because genomic DNA is essentially
the same in all tissues.

10-32 Even though the genome of F. cattoriae is smaller, the fact that the W. gravinius genome
contains less repetitive DNA makes it more attractive to sequence. Repetitive DNA
makes the assembly of sequenced fragments difficult. Shotgun sequencing would not be
an unrealistic approach for W. gravinius, because the genome of W. gravinius contains
little repetitive DNA and is relatively small. The genome of H. influenzae is 1.83 MB
and was successfully sequenced using the shotgun approach. (For comparison, the
genome of S. cerevisiae is 14 MB.)

174

CHAPTER 11
MEMBRANE STRUCTURE
2009 Garland Science Publishing
3rd Edition

The Lipid Bilayer


11-1 For each of the following sentences, fill in the blanks with the best word or phrase selected from
the list below. Not all words or phrases will be used; each word or phrase should be used only once.
The specialized functions of different membranes are largely determined by the
__________________ they contain. Membrane lipids are __________________ molecules,
composed of a hydrophilic portion and a hydrophobic portion. All cell membranes have the
same __________________ structure, with the __________________ of the phospholipids
facing into the interior of the membrane and the __________________ on the outside. The
most common lipids in most cell membranes are the __________________. The head group of
a glycolipid is composed of __________________.

amphipathic
cholesterol
fatty acid tails
glycolipids
hydrophilic head groups

hydrophobic
lipid bilayer
lipid monolayer
lipids
phosphatidylcholine

phosphatidylserine
phospholipids
proteins
sterols
sugars

11-2 Which of the following membrane lipids does not contain a fatty acid tail?
(a) Phosphatidylcholine (b) A glycolipid (c) Phosphatidylserine (d)
Sphingomyelin (e) Cholesterol

11-3 Which of the following statements regarding lipid membranes is TRUE?


(a) Phospholipids will spontaneously form liposomes in nonpolar solvents.
(b) In eucaryotes, all membrane-enclosed organelles are surrounded by one lipid bilayer.
(c) Membrane lipids diffuse within the plane of the membrane.
(d) Membrane lipids frequently flip-flop between one monolayer and the other.
(e) The preferred form of a lipid bilayer in water is a flat sheet with exposed edges.

175

11-4

A bacterium is suddenly expelled from a warm human intestine into the cold world
outside. Which of the following adjustments might the bacterium make to maintain the
same level of membrane fluidity?
(a)
Increase the length of the hydrocarbon tails in its membrane phospholipids.
(b)
Increase the proportion of unsaturated hydrocarbon tails in its membrane
phospholipids.
(c)
Increase the proportion of hydrocarbon tails with no double bonds in its
membrane phospholipids.
(d)
Decrease the amount of cholesterol in the membrane.
(e)
Decrease the amount of glycolipids in the membrane.

11-5

Which of the following statements regarding the fatty acid tails of phospholipids is
FALSE?
(a)
Phospholipids with unsaturated tails make the bilayer more fluid because the tails
contain fewer hydrogen atoms and thus form fewer hydrogen bonds with each
other.
(b)
Saturated phospholipid tails pack more tightly against each other than do
unsaturated tails.
(c)
Most membrane phospholipids have one fully saturated tail.
(d)
Phospholipid tails in a membrane can interact with each other via van der Waals
interactions.
(e)
Fatty acid tails vary in length.

11-6

New membrane synthesis occurs by


(a)
the spontaneous aggregation of free phospholipids into a new bilayer in the
aqueous environment of the cell.
(b)
incorporation of phospholipids into both faces of a preexisting membrane by
enzymes attached to each face.
(c)
incorporation of phospholipids into one face of a preexisting membrane and their
random redistribution to both faces by flippases.
(d)
incorporation of phospholipids into one face of a preexisting membrane and their
specific redistribution by flippases.

176

11-7

Three phospholipids X, Y, and Z are distributed in the plasma membrane as indicated in


Figure Q11-7. For which of these phospholipids does a flippase probably exist?

Figure Q11-7
(a)
(b)
(c)
(d)
(e)

X only
Z only
X and Y
Y and Z
X and Z

11-8

Where does most new membrane synthesis take place in a eucaryotic cell?
(a)
In the Golgi apparatus
(b)
In the endoplasmic reticulum
(c)
In the plasma membrane
(d)
In the mitochondria
(e)
On ribosomes

11-9

A small membrane vesicle containing a transmembrane protein is shown below. Assume


this membrane vesicle is in the cytoplasm of a cell.

Figure Q11-9
A.
B.

C.

Label the cytosolic and non-cytosolic faces of the membrane vesicle. This
membrane vesicle will undergo fusion with the plasma membrane.
Sketch the plasma membrane after vesicle fusion, indicating the new location of
the vesicle membrane and the transmembrane protein carried by the membrane
vesicle.
On your drawing for B, label the original cytosolic and noncytosolic faces of the
vesicle membranes as it resides in the plasma membrane. Also label the
extracellular space and the cytosol. Indicate the amino and carboxyl terminus of
the inserted transmembrane protein.
177

11-10 Why are glycolipids found on the extracellular, but not the cytoplasmic, surface of the
plasma membrane?
(a)
Flippases transport them from the cytosolic face.
(b)
The enzymes that produce them are present only on the extracellular surface of
the plasma membrane.
(c)
The enzymes that add the sugar groups are confined to the inside of the
endoplasmic reticulum and the Golgi apparatus.
(d)
The oligosaccharides on glycolipids are cleaved off by enzymes found only in the
cytosol.
(e)
They flip spontaneously, after incorporation, due to the hydrophilic sugar head
groups.

Membrane Proteins
11-11 A group of membrane proteins, which can be extracted only from membranes using
detergents, are all found to have a similar amino acid sequence at their carboxyl terminus:
-KKKKKXXC (where K stands for lysine, X stands for any amino acid, and C stands for
cysteine). This sequence is essential for their attachment to the membrane. What is the
most likely way in which the carboxyl-terminal sequence attaches these proteins to the
membrane?
(a)
The cysteine residue is covalently attached to a membrane lipid.
(b)
The peptide spans the membrane as an helix.
(c)
The peptide spans the membrane as part of a sheet.
(d)
The positively charged lysine residues interact with an acidic integral membrane
protein.
11-12 A strain of bacteria secretes a toxin that can lyse human red blood cells. You are able to
partially purify the toxin and find that it is a small protein. Furthermore, the toxin is
capable of rendering liposomes made of pure phospholipids permeable to many different
ions. What type of protein is the bacterial toxin likely to be?
(a)
A flippase
(b)
A -barrel protein
(c)
A protease
(d)
A protein containing a single hydrophobic helix.
(e)
An enzyme that adds carbohydrate groups to lipids.

178

11-13 For each of the following sentences, fill in the blanks with the best word or phrase
selected from the list below. Not all words or phrases will be used; each word or phrase
should be used only once.
There are several ways that membrane proteins can associate with the cell
membrane. Membrane proteins that extend through the lipid bilayer are
called __________________ proteins and have __________________
regions that are exposed to the interior of the bilayer. On the other hand,
membrane-associated proteins do not span the bilayer and instead
associate with the membrane through an -helix that is
__________________. Other proteins are __________________ attached
to lipid molecules that are inserted in the membrane.
__________________ membrane proteins are linked to the membrane
through noncovalent interactions with other membrane-bound proteins.
amphipathic
cortical
covalently
detergent

hydrophilic
hydrophobic
integral
micelle

noncovalently
peripheral
transmembrane
unfolded

11-14 Which of the following statements regarding membrane proteins is FALSE?


(a)
Integral membrane proteins often precipitate (form insoluble aggregates) in
aqueous solutions lacking detergents.
(b)
Some hydrophobic amino acids in membrane proteins are not in contact with the
lipid bilayer.
(c)
Peripheral membrane proteins can be dissociated from membranes using a gentle
detergent.
(d)
Strong detergents can completely unfold both membrane and nonmembrane
proteins.
(e)
In transmembrane proteins that form an aqueous pore through the membrane, the
pore is lined with hydrophobic amino acid side chains.
11-15 Proteins often span the plasma membrane as an - helix. If it takes 20 amino acids to
span a lipid bilayer, which of the three 20-amino acid sequences listed below is the most
likely candidate for such a transmembrane segment? Explain your choice.
(a)
IALIVFGVFAGVIGGILLIS
(b)
KITPVVKHKHDKIDTPLLIR
(c)
DTYYYRRREADDDDDLLISD

179

11-16 Proteins that form a -barrel pore in the membrane have several -strands that span the
membrane. The amino acid side chains facing the inside of the pore would be
hydrophilic whereas the amino acid side chains facing the lipid bilayer would be
hydrophobic. Which of the three 10-amino acid sequences listed below is the most likely
candidate for a transmembrane -strand in a -barrel protein? Explain your choice.
(a)
ADFKLSVELT
(b)
AFLVLDKSET
(c)
AFDKLVSELT
11-17 Which of the following functions does the cell cortex perform?
(a)
It influences the passage of small molecules into and out of the cell.
(b)
It allows cells to change shape and to move.
(c)
It lubricates the cell.
(d)
It restricts the movement of certain proteins in the lipid bilayer.
(e)
It supports and strengthens the membrane.
11-18 You have isolated two mutants of a normally pear-shaped microorganism that have lost
their distinctive shape and are now round. One of the mutants has a defect in a protein
you call A and the other has a defect in a protein you call B. You grind up each type of
mutant cell and normal cells separately and separate the plasma membranes from the
cytoplasm. You then wash the membrane fraction with a low concentration of urea
(which will unfold proteins and disrupt their ability to interact with other proteins) and
centrifuge the mixture. The membranes and their constituent proteins form a pellet while
the proteins liberated from the membranes by the urea wash remain in the supernatant.
When you check each of the factions for the presence of A or B, you obtain the results
given below.

Normal cells
Mutant A
Mutant B

First cell extract


Membrane
Cytosol
A and B
no A or B
A
B
A
B

After urea wash and centrifugation


Membrane
Supernatant
A
B
A
no A or B
A
no A or B

Which of the following statements are consistent with your results?


(a)
Protein A is an integral membrane protein that interacts with B, a peripheral
membrane protein that is part of the cell cortex.
(b)
Protein B is an integral membrane protein that interacts with A, a peripheral
membrane protein that is part of the cell cortex.
(c)
Proteins A and B are both integral membrane proteins.
(d)
The mutation in A affects its ability to interact with B.
(e)
The mutation in A affects its ability to interact with the membrane.

180

11-19 Detergents and phospholipids are both amphipathic molecules. However, when exposed
to water, detergents aggregate into small clusters called micelles while phospholipids
form closed spherical liposomes. Explain what the difference is between the structure of
a detergent compared to the structure of a phospholipid that causes detergents to form
micelles instead of liposomes.
11-20 Diversity among the oligosaccharide chains found in the carbohydrate coating of the cell
surface can be achieved in which of the following ways?
(a)
Varying the types of sugar monomers used
(b)
Varying the types of linkages between sugars
(c)
Varying the number of monomers in the chain
(d)
Varying the number of branches in the chain
(e)
All of the above
11-21 Which of the following statements about the carbohydrate coating of the cell surface is
FALSE?
(a)
It is not usually found on the cytosolic side of the membrane.
(b)
It can play a role in cell-cell adhesion.
(c)
The arrangement of the oligosaccharide side chains are highly ordered, much like
the peptide bonds of a polypeptide chain.
(d)
Specific oligosaccharides can be involved in cell-cell recognition.
(e)
It can protect the cell surface from mechanical and chemical damage.
11-22 Cell membranes are fluid and thus proteins can diffuse laterally within the lipid bilayer.
However, sometimes the cell needs to localize proteins to a particular membrane domain.
Name three mechanisms a cell can use to restrict a protein to a particular place in the cell
membrane.

181

How We Know: Measuring Membrane Flow


11-23 Fluorescence recovery after photobleaching (FRAP) is a technique that allows
visualization of diffusion within the membrane. You set out to perform FRAP using four
different samples of cells. In sample 1, you have fluorescently labeled a phospholipid. In
samples 2, 3, and 4 you have fluorescently labeled membrane proteins X, Y, and Z,
respectively. You photobleach an area of the membrane in each sample and record the
rate of recovery of fluorescence. The data you obtain are shown in the graphs below:

Figure Q11-23
A.
B.

Using the data from these graphs, list the proteins in order of their ability to
diffuse in the membrane, from fastest to slowest.
Given the data on the graphs above, is the following statement a good hypothesis:
protein X and protein Z are always present in the cell as part of the same protein
complex? Explain your answer.

182

11-24 You perform single-particle tracking (SPT) experiments on three proteins, A, B, and C,
and obtain the following pathways:

Figure Q11-24
A.
B.

Which protein displays a pattern of motion similar to a protein that is anchored to


the cytoskeleton?
If you were to find a cell containing a membrane where 10% of its membrane
proteins were anchored to the cytoskeleton, how do you think this would affect
the fluidity of the lipids in the membrane?

183

Answers
11-1

The specialized functions of different membranes are largely determined by the proteins
they contain. Membrane lipids are amphipathic molecules, composed of a hydrophilic
portion and a hydrophobic portion. All cell membranes have the same lipid bilayer
structure, with the fatty acid tails of the phospholipids facing into the interior of the
membrane and the hydrophilic head groups on the outside. The most common lipids in
most cell membranes are the phospholipids. The head group of a glycolipid is composed
of sugars.

11-2

(e)

11-3

(c)

11-4

Choice (b) is the correct answer. At colder temperatures, the membrane will be less fluid.
Hence, in order to maintain the status quo, the bacterium will have to take measures to
increase membrane fluidity. Increasing the length of the hydrocarbon tails (choice (a))
would decrease membrane fluidity, while decreasing the number of glycolipids (choice
(e)) would have little or no effect. Decreasing the proportion of fatty acid tails with no
double bonds (fully saturated) (choice (c)) would decrease membrane fluidity. Bacteria
do not have cholesterol (choice (d)).

11-5

(a)

11-6

(d)

11-7

(c)

11-8

(b)

The remaining answers are false. Phospholipids form bilayers only in polar
solvents. Nuclei and mitochondria are enclosed by two membranes. Membrane
lipids rarely flip-flop between one monolayer and the other. The preferred form
of a lipid bilayer in water is a closed sphere, so that the hydrophobic groups at
the edges of the bilayer can avoid contact with water.

Unsaturated fatty acid tails do have fewer hydrogen atoms and do interact less
well with one another but not for the reason stated. The decrease in interaction is
due to a decrease in van der Waals interactions between the hydrocarbon tails
because they can pack less closely. Hydrocarbon chains cannot form hydrogen
bonds with each other.

As phospholipids are initially inserted into the cytosolic face of the lipid bilayer,
flippases are required to move X and Y to the opposite face, as they cannot
spontaneously flip across the bilayer.

184

11-9

A.

See Figure A11-9A.

Figure A11-9A
B., C. See Figure A11-9B.

Figure A11-9B
11-10 (c)
11-11 (a)

The peptide is extremely hydrophilic and is, therefore, unlikely to be inserted into
the lipid bilayer. It is also too short to span the membrane as an -helix. Although
it is possible that the lysines interact with an acidic membrane protein, if such an
interaction were solely responsible for attaching the protein to the membrane, it
would not require a detergent to remove the protein, since ionic interactions are
disrupted by milder treatments such as salt washes and pH changes.

11-12 Choice (b) is the correct answer. Insertion of a pore-forming protein into the lipid bilayer
will have the effects noted by making the red blood cell unable to regulate its internal ion
composition. The type of protein most likely to form the sort of nonspecific pore
described is a -barrel protein. Enabling membrane lipids to flip from one layer of the
bilayer to the other should not affect permeability (choice (a)). A protease would have no
effect on liposomes made of pure phospholipids (choice (c)). Proteins containing a single
hydrophobic -helix would not form a channel through the membrane (choice (d)).
Addition of carbohydrate groups to lipids should not make the bilayer any more
permeable to ions (choice (e)).

185

11-13 There are several ways that membrane proteins can associate with the cell membrane.
Membrane proteins that extend through the lipid bilayer are called transmembrane
proteins and have hydrophobic regions that are exposed to the interior of the bilayer. On
the other hand, membrane-associated proteins do not span the bilayer and instead
associate with the membrane through an -helix that is amphipathic. Other proteins are
covalently attached to lipid molecules that are inserted in the membrane. Peripheral
membrane proteins are linked to the membrane through noncovalent interactions with
other membrane-bound proteins.
11-14 Choice (e) is the correct answer. In transmembrane proteins that form an aqueous pore
through the membrane, the pore is lined with hydrophilic amino acid side chains. The
other statements are all true. Integral membrane proteins often precipitate in aqueous
solutions because of their stretches of hydrophobic amino acids (choice (a)). Proteins
also contain hydrophobic amino acids in parts of the protein other than the membranespanning region, for example, in the cores of their extracellular or cytoplasmic domains
(choice (b)). Peripheral membrane proteins are attached to the membrane by noncovalent
interactions with other membrane proteins, making their membrane association relatively
weak and thus disruptable by gentle detergents (choice (c)).
11-15 Choice (a) is the correct answer. When an -helix traverses a lipid bilayer, the amino acid
side chains are exposed to the hydrophobic portion of the lipid bilayer and thus should
consist of nonpolar side chains. The sequence given in choice (a) contains nonpolar
amino acids. Choices (b) and (c) are inappropriate, as they both contain several
negatively and positively charged amino acids.
(Note that in addition, choice (b) contains two prolines, which would render this
sequence unable to form an -helical structure and therefore lead to exposure of the
hydrogen-bonding moieties in the polypeptide background to the nonpolar environment
of the lipid bilayer.)
11-16 (a)

In a -sheet, the amino acid side chains project alternately above and below the
plane of the sheet. Therefore, every other amino acid side chain will face the
same side of the strand. If a -sheet were part of a -barrel pore, that would mean
that one side of the sheet would face the lipid bilayer while the other side would
face the hydrophilic pore. This would necessitate an alternation between
hydrophilic and hydrophobic amino acid side chains so that one side of the sheet
would be hydrophobic while the other side was hydrophilic. The amino acids in
choice (a) alternate between amino acids with nonpolar (hydrophobic) side chains
and amino acids with polar (hydrophilic) side chains.

11-17 (b), (d), and (e)

186

11-18 (a), (d) The results from the extracts of normal cells show that protein A is an integral
protein that remains in the membrane through all the treatments, while protein B
is a peripheral protein that can be removed from the membrane by urea. In the cell
extracts from the mutants in A, the protein A still remains in the membrane, but
the B protein does not. This is consistent with the mutation in A affecting its
interaction with B. The same results are obtained when the B protein is mutant,
which is consistent with the idea that A and B interact. The loss of an interaction
between an integral membrane protein and a protein in the cortex would be more
likely to result in a change in cell shape than the loss of an interaction between an
integral membrane protein and a protein on the exterior of the cell.
11-19 A detergent has one hydrophobic tail while a phospholipid has two hydrophobic tails.
This difference is reflected in the geometry of the molecules, with detergent molecules
being shaped more like cones whereas phospholipids are more cylindrical. The cone-like
detergent molecules will aggregate into micelles, with the hydrophilic head group on the
outside and the hydrophobic tail group on the inside. On the other hand, the cylindrical
shape of a phospholipid means that when phospholipids aggregate, the formation of a
bilayer is most energetically favorable, with the hydrophobic tails on the inside of the
two-layered sheet and the hydrophilic heads facing the water molecules. However, it is
not energetically favorable for a phospholipid bilayer to exist as a sheet, because the
exposed free edges would lead to exposure of the hydrophobic tails to water. Therefore,
phospholipid bilayers spontaneously close to form spherical liposomes.
11-20 (e)
11-21 (c)

The sugars in an oligosaccharide side chain attached to the cell surface can be
joined together in many different ways and in varied sequences.

11-22 Any combination of these four answers is acceptable:


1.
The protein can be attached to the cell cortex inside the cell.
2.
The protein can be attached to the extracellular matrix outside the cell.
3.
The protein can be attached to other proteins on the surface of a different cell.
4.
The protein can be restricted in its ability to diffuse by a diffusion barrier, such as
that set up by specialized junctional proteins at a tight junction.
11-23 A.

B.

X diffuses the fastest, followed by Z, with Y barely diffusing at all. (The faster
the recovery of fluorescence in the bleached area, the greater the diffusion
coefficient of the protein and the faster the protein diffuses.)
It is unlikely that X and Z are part of the same protein complex because then the
rate of diffusion of X and Z should be the same.

187

11-24 A.
B.

Protein C displays the least amount of motion and therefore is the one most likely
to be tethered to the cytoskeleton.
The fluidity of the lipid bilayer should not be affected by the anchoring of
membrane proteins. The lipid molecules should still be able to flow around
anchored proteins much like water flows around rocks in a stream. (Note that the
fluidity of the lipid bilayer is affected by the degree of saturation found in the
hydrocarbon tails, the length of the hydrocarbon tails, and in animal cells, the
amount of cholesterol in the membrane.)

188

CHAPTER 12
MEMBRANE TRANSPORT
2009 Garland Science Publishing
3rd Edition

Principles of Membrane Transport


12-1 The most abundant intracellular cation is
(a) Na+
(b) Ca2+
(c) H+
(d) K+
(e) positively charged macromolecules.
12-2 Circle the molecule in each pair that is more likely to diffuse through the lipid
bilayer.

12-3 A hungry yeast cell lands in a vat of grape juice and begins to feast on the
sugars there, producing carbon dioxide and ethanol in the process:

Unfortunately, the grape juice is contaminated with proteases (enzymes that attack
some of the transport proteins in the yeast cell membrane), and the yeast cell dies.
Which of the following could account for the yeast cells demise?
(a)
(b)
(c)
(d)
(e)

Toxic buildup of carbon dioxide inside the cell


Toxic buildup of ethanol inside the cell
Diffusion of ATP out of the cell
Inability to import sugar into the cell
Inability to take water into the cell

189

12-4

For each of the following sentences, fill in the blanks with the best word or phrase
selected from the list below. Not all words or phrases will be used; each word or phrase
should be used only once.
A molecule moves down its concentration gradient by
__________________ transport, but requires __________________
transport to move up its concentration gradient. Carrier proteins and ion
channels function in membrane transport by providing a
__________________ pathway through the membrane for specific polar
solutes or inorganic ions. __________________ are highly selective in
the solutes they transport, binding the solute at a specific site and changing
its conformation in order to transport the solute across the membrane. On
the other hand, __________________ discriminate among solutes mainly
on the basis of size and electrical charge.
active
amino acid
amphipathic

carrier proteins
hydrophilic
hydrophobic

ion channels
noncovalent
passive

Carrier Proteins and Their Functions


12-5

Your friend isolates a new species of yeast that can grow using either ethanol
(CH3CH2OH) or acetate (CH3COO) as the yeasts sole carbon source. He measures the
rate of uptake of these carbon sources by the yeast as a function of concentration of
carbon source and obtains the following graphs:

Figure Q12-5
A.
B.
12-6

Is molecule A or B more likely to utilize a carrier protein to mediate its transport?


Why?
Given what you know about membrane transport, is molecule A more likely to be
ethanol or acetate? Why?

Name the three main types of active transport.

190

12-7

Which of the following descriptions are TRUE of the bacterial protein


bacteriorhodopsin? It is a
(a)
light-driven pump.
(b)
proton pump.
(c)
passive transporter.
(d)
coupled transporter.
(e)
transmembrane protein.

12-8

The Aeroschmidt weed contains an ATP-driven ion pump in its vacuolar membrane that
pumps potentially toxic heavy metal ions such as Zn2+ and Pb2+ into the vacuole. The
pump protein exists in a phosphorylated and an unphosphorylated form and works in a
similar way to the Na+-K+ pump of animal cells. To study its action, you incorporate the
unphosphorylated form of the protein into phospholipid vesicles containing K+ in their
interiors. (You ensure that all of the protein molecules are in the same orientation in the
lipid bilayer.) When you add Zn2+ and ATP to the solution outside such vesicles, you find
that Zn2+ is pumped into the vesicle lumen. You then expose vesicles containing the
pump protein to the solutes as shown in Table 12-8A.
Table 12-8A
Outside
Inside

A
B
C
2+
2+
Zn + ATP Zn
Zn + ATP
K+
2+

D
Zn2+
K+

E
ATP

F
ATP
K+

You then determine the amount of phosphorylated and unphosphorylated ATP-driven ion
pump protein in each sample. Your results are summarized in Table 12-8B, where
indicates an absence of a type of protein and + indicates the presence of a type of
protein.
Table 12-8B
phosphorylated
protein present
unphosphorylated
protein present

A
+

F
++

++

++

++

++

++

What would you expect to happen if you treat vesicles as in lane F, but before
determining the phosphorylation state of the protein, you wash away the outside buffer
and replace it with a buffer containing only Zn2+?
(a)
Nothing will happen. (No Zn2+ will move into the vesicle; no K+ will move out of
the vesicle; the phosphorylation state of the protein will not change.)
(b)
No Zn2+ will move into the vesicle; no K+ will move out of the vesicle; the protein
will become unphosphorylated.
(c)
A small amount of Zn2+ will move into the vesicle; no K+ will move out of the
vesicle; the phosphorylation state of the protein will not change.
(d)
A small amount of Zn2+ will move into the vesicle; no K+ will move out of the
vesicle; the protein will become unphosphorylated.
(e)
A small amount of Zn2+ will move into the vesicle; a small amount of K+ will
move out of the vesicle; the phosphorylation state of the protein will not change.
191

12-9

Explain why Na+ is commonly used to drive the coupled inward transport of nutrients in
animal cells.

12-10 Ouabain inhibits the active uptake of glucose into epithelial cells by
(a)
binding to the glucose-Na+ symport.
(b)
opening K+ channels.
(c)
changing the pH of the cell.
(d)
increasing the intracellular concentration of Na+.
(e)
depleting the cell of ATP.
12-11 Which of the following statements is TRUE?
(a)
Amoebae have carrier proteins that actively pump water molecules from the
cytoplasm to the cell exterior.
(b)
Bacteria and animal cells rely on the Na+K+ pump in the plasma membrane to
prevent lysis due to osmotic imbalances.
(c)
The Na+K+ pump allows animal cells to thrive under conditions of very low
ionic strength.
(d)
The cell wall surrounding plant cells prevents osmosis.
(e)
The Na+K+ pump helps to keep both Na+ and Cl ions out of the cell.
12-12 The Ca2+ pumps in the plasma membrane and endoplasmic reticulum are examples of
(a)
ATP-driven pumps.
(b)
coupled transporters.
(c)
passive carrier proteins.
(d)
ion channels.
(e)
symports.
12-13 Ca2+ pumps in the plasma membrane and endoplasmic reticulum are important for
(a)
maintaining osmotic balance.
(b)
preventing Ca2+ from altering the behavior of molecules in the cytosol.
(c)
providing enzymes in the endoplasmic reticulum with Ca2+ ions that are necessary
for their catalytic activity.
(d)
maintaining a negative membrane potential.
(e)
helping cells import K+.
12-14 Do you agree of disagree with the following statement? Explain your answer.
A symporter would function as an antiporter if its orientation in the
membrane were reversed (that is, if the portion of the protein normally
exposed to the cytosol faced the outside of the cell instead).

192

12-15 You have prepared lipid vesicles (spherical lipid bilayers) that contain Na+K+ pumps as
the sole membrane protein. All of the Na+K+ pumps are oriented in such a way that the
portion of the molecule that normally faces the cytosol is in the inside of the vesicle and
the portion of the molecule that normally faces the extracellular space is on the outside of
the vesicle. Assume that each pump transports one Na+ ion in one direction and one K+
ion in the other direction during each pumping cycle (see Figure Q12-15 for how the
Na+K+ pump normally functions in the plasma membrane).

Figure 12-15
Predict what would happen in each of the following conditions:
A.
The solutions inside and outside the vesicles contain both Na+ and K+ ions but no
ATP.
B.
The solution outside the vesicles contains both Na+ and K+ ions; the solution
inside contains both Na+ and K+ ions and ATP.
C.
The solution outside contains Na+; the solution inside contains Na+ and ATP.

193

12-16 For each of the following sentences, fill in the blanks with the best word or phrase
selected from the list below. Not all words or phrases will be used; each word or phrase
should be used only once.
For an uncharged molecule, the direction of passive transport across a
membrane is determined solely by its __________________ gradient. On
the other hand, for a charged molecule, an additional force called the
__________________ must also be considered. The net driving force for
a charged molecule across a membrane therefore has two components and
is referred to as the __________________ gradient. Active transport
allows the movement of solutes against this gradient. The carrier proteins
called __________________ transporters utilize the movement of one
solute down its gradient to provide the energy to drive the uphill transport
of a second gradient. When this transporter moves both ions in the same
direction across the membrane, it is considered a(n)
__________________; if the ions move in opposite directions, the
transporter is considered a(n) __________________.
antiport
ATP hydrolysis
concentration

coupled
electrochemical
light-driven

membrane potential
symport
uniport

Ion Channels and the Membrane Potential


12-17 Ion channels
(a)
only open in response to a signal of some kind.
(b)
require input of energy in order to function.
(c)
have no limit to the rate at which they can transport ions.
(d)
can transport both negative and positive ions through the same channel.
(e)
allow passage of ions in both directions.
12-18 A gated ion channel
(a)
stays continuously open when stimulated.
(b)
opens more frequently in response to a given stimulus.
(c)
opens more widely the stronger the stimulus.
(d)
remains closed if unstimulated.

194

12-19 You have patch-clamped a single voltage-gated ion channel in a membrane and have
obtained the following recording (see Figure Q12-19A).

[Figure Q12-19A]
For this channel, match the recordings depicted in Figure Q12-19B with the appropriate
choices 1 through 4.
1.
A channel that is closed all the time.
2.
A channel that is open all the time.
3.
A channel where the ion flow has been reversed.
4.
A channel spending more time in the open configuration.

[Figure Q12-19B]

195

12-20 For each of the following sentences, fill in the blank with the appropriate type of gating
for the ion channel described. You can use the same type of gating mechanism more than
once.
A.
The acetylcholine receptor in skeletal muscle cells is a __________________ ion
channel.
B.
__________________ ion channels are found in the hair cells of the mammalian
cochlea.
C.
__________________ ion channels in the mimosa plant propagate the leafclosing response.
D.
__________________ ion channels respond to changes in membrane potential.
E.
Many receptors for neurotransmitters are __________________ ion channels.
12-21 The high intracellular concentration of K+ in a resting animal cell is partly due to
(a)
the K+ leak channels in the plasma membrane.
(b)
the Na+K+ pump in the plasma membrane.
(c)
voltage-gated K+ channels in the plasma membrane.
(d)
intracellular stores of K+ in the endoplasmic reticulum.
(e)
the membrane potential.
12-22 State whether you agree or disagree with the following statement. Explain your answer.
The membrane potential arises from movements of charge that leave ion
concentrations practically unaffected and result in only a very slight
discrepancy in the number of positive and negative ions on the two sides
of the membrane.

Ion Channels and Signaling in Nerve Cells


12-23 Match the numbered lines with the following structures:
A.
Nerve terminal
B.
Cell body
C.
Axon
D.
Dendrite

Figure Q12-23
196

12-24 For each of the following sentences, fill in the blanks with the best word or phrase
selected from the list below. Not all words or phrases will be used; each word or phrase
should be used only once.
The action potential is a wave of __________________ that rapidly
spreads along the neuronal plasma membrane. This wave is triggered by a
local change in the membrane potential to a value that is
__________________ negative than the resting membrane potential. The
action potential is propagated by the opening of __________________gated channels. During an action potential, the membrane potential
changes from __________________ to __________________. The action
potential travels from the neurons __________________ along the
__________________ to the nerve terminals. Neurons chiefly receive
signals at their highly branched __________________.
anions
axon
cell body
cytoskeleton
dendrites

depolarization
hyperpolarization
less
ligand
more

negative
neutral
positive
pressure
voltage

12-25 On the diagram of an action potential shown below, draw a dashed line showing what
would have happened to the membrane potential after the initial depolarizing stimulus if
there had been no voltage-gated Na+ channels in the membrane.

Figure Q12-25

197

12-26 When a neuron receives a signal, an action potential can be triggered. A graph of the
traveling membrane depolarization assumes a characteristic shape as the depolarization
moves down the axon (i.e., the peak height and length of depolarization is constant).
What property of the voltage-gated Na+ channel ensures that the action potential assumes
this characteristic form? Explain your answer.
12-27 For each of the following sentences, fill in the blanks with the best word or phrase
selected from the list below. Not all words or phrases will be used; each word or phrase
should be used only once.
Neurons communicate with each other through specialized sites called
__________________. Many neurotransmitter receptors are ligand-gated
ion channels that open transiently in the __________________ cell
membrane in response to neurotransmitters released by the
__________________ cell. Ligand-gated ion channels in nerve cell
membranes convert __________________ signals into
__________________ ones. Neurotransmitter release is stimulated by the
opening of voltage-gated __________________ in the nerve terminal
membrane.
acetylcholine receptor
Ca2+ channels
chemical
electrical

GABA receptor
K+ channels
Na+ channels
postsynaptic

presynaptic
synapses

12-28 Inhibitory neurotransmitters such as GABA often open Cl channels. Explain how the
opening of a Cl channel will affect the firing of an action potential.
12-29 K+ leak channels are important for setting the resting membrane potential. If you were to
add a drug that inhibits K+ leak channels in neurons, would this make it easier or harder
to trigger an action potential in the treated neurons? Why?

198

How We Know: Squid Reveal Secrets of Membrane Excitability


12-30 Studies on the squid giant axon were instrumental in our current understanding of how
action potentials are generated. You decide to do some experiments on the squid giant
axon yourself.
A.
You remove the cytoplasm in an axon and replace it with an artificial cytoplasm
that contains twice the normal concentration of K+ by adding KOAc, where OAc
is an anion that is impermeable to the membrane. In this way you double the
internal concentration of K+ while maintaining bulk electrical balance of the
cytoplasmic solution. Will this make the resting potential of the membrane more
or less negative?
B.
You add NaCl to the extracellular fluid and effectively double the amount of
extracellular Na+ cation. How does this affect the action potential?
C.
You replace half of the NaCl in the extracellular fluid with choline chloride.
(Choline is a monovalent cation much larger than Na+. Note that the presence of
choline will not impede the flow of Na+ through its channels.) How will this
affect the action potential?

199

Answers
12-1

(d)

12-2

The two basic properties governing the likelihood of whether a molecule will diffuse
through a lipid bilayer are the size of the molecule and the charge of the molecule. A
smaller molecule will be more likely to diffuse through the lipid bilayer than a larger
molecule. A nonpolar (hydrophobic) molecule will be more likely to diffuse through the
lipid bilayer than a polar molecule, which is more likely to diffuse through the lipid
bilayer than a charged molecule.
A.
benzene (small nonpolar vs. larger uncharged)
B.
ethanol (polar vs. charged)
C.
glucose (large polar vs. very large highly charged)
D.
O2 (nonpolar vs. polar)
E.
adenosine (polar vs. highly charged)

12-3

Choice (d) is the correct answer. Because the lipid bilayer is permeable to carbon dioxide
and ethanol, destroying membrane proteins is unlikely to affect their exit (choices (a) and
(b)). The lipid bilayer is also permeable to water (choice (e)). On the other hand glucose
requires a membrane transport protein to be imported into the cell. ATP, which is a
highly charged molecule, also requires a transport protein to cross a membrane and thus
could not be lost from the cell by simple diffusion (choice (c)).

12-4

A molecule moves down its concentration gradient by passive transport, but requires
active transport to move up its concentration gradient. Carrier proteins and ion channels
function in membrane transport by providing a hydrophilic pathway through the
membrane for specific polar solutes or inorganic ions. Carrier proteins are highly
selective in the solutes they transport, binding the solute at a specific site and changing its
conformation in order to transport the solute across the membrane. On the other hand,
ion channels discriminate among solutes mainly on the basis of size and electrical
charge.

200

12-5

A.

B.

Molecule B is more likely to utilize a carrier protein. From the graph, the yeast
cells uptake of molecule B levels off when the concentration of carbon source is
high; this indicates a saturation point for the uptake of molecule B. Carrier
proteins work by specifically binding their solutes and transferring the solute
molecules across the membrane one at a time, down the concentration gradient of
the solute. Thus, the rate at which a solute can be transported by a carrier protein
is limited by the number of carrier proteins in the membrane and will reach a
maximum when the solute concentration is high enough for solute molecules to
saturate the carrier proteins. Since the graph indicates that the uptake of molecule
B reaches a saturation point at high concentrations, molecule B is more likely to
utilize a carrier protein.
Molecule A is more likely to be ethanol. Uptake of molecule A is proportional to
its concentration, while the uptake of molecule B reaches a saturation point at
high concentration. The linear dependence of molecule A upon its concentration
suggests that molecule A can diffuse into the cell. Since ethanol (a small polar
molecule) is more likely to diffuse into the cell compared to acetate (a charged
molecule), ethanol is likely to be molecule A. Since acetate is charged, it is more
likely to require a carrier protein in order to be transported across the membrane.

12-6

ATP-driven transport, coupled transport, light-driven transport

12-7

(a), (b), and (e)

12-8

(d)

12-9

Na+ is commonly used to drive coupled transport in animal cells because a steep
concentration gradient of Na+ (high outside and low inside) is maintained by the Na+K+
pump. Na+ readily flows back into the cell down this gradient because of the negative
membrane potential. The energy provided by the flow of Na+ down this steep
electrochemical gradient can be harnessed by coupled transporters.

12-10 (d)

If the pump is mechanistically similar to the Na+K+ pump, then the transport of
ions is driven by ATP hydrolysis, the pump is transiently phosphorylated;
phosphorylation is stimulated by one ion and dephosphorylation is stimulated by
the other ion. Since all of the protein is in the phosphorylated form in the absence
of Zn2+ (lane F), Zn2+ is probably required for dephosphorylation. K+, then,
probably binds to the dephosphorylated form and stimulates the
ATPase/autophosphorylation. So, if Zn2+ is added to the phosphorylated pump,
Zn2+ will stimulate dephosphorylation, trigger a conformational change, and be
injected into the vesicle. K+ will stimulate the kinase activity of the pump, but
since there is no ATP to be hydrolyzed in the interior of the vesicle, no
phosphorylation and hence no movement of K+ will occur.

Ouabain inhibits the Na+K+ pump and, therefore, leads to an increase in the
intracellular concentration of Na+, which is continually leaking into the cell.
Uptake of glucose into epithelial cells occurs via a Na+-glucose symport, which
uses the Na+ gradient to drive movement of glucose into the cell.

201

12-11 Choice (e) is the correct answer. The Na+K+ pump keeps Na+ out directly by pumping it
out and keeps Cl out indirectly by helping to maintain the negative membrane potential.
Cells do not have pumps for moving water molecules across the membrane (choice (a)),
since the lipid bilayer is permeable to water. Bacteria do not have Na+K+ pumps in their
plasma membranes (choice (b)). The Na+K+ pump cannot directly remove water
molecules from the cell; it helps maintain osmotic balance by pumping out the Na+ that
leaks in, which would not help if the cell is in a solution of very low ionic strength
(choice (c)). The plant cell wall is permeable to water and therefore cannot prevent
osmosis (choice (d)).
12-12 (a)
12-13 Choice (b) is the correct answer. The major purpose of the Ca2+ pumps is to keep the
cytosolic concentration of Ca2+ low. When Ca2+ does move into the cytosol, it alters the
behavior of many proteins; hence Ca2+ is a powerful signaling molecule. It is not
involved in the catalytic activity of ER enzymes (choice (c)). Since the levels of Ca2+ are
very low relative to the levels of K+ and Na+, the Ca2+ gradient does not have a
significant effect on the osmotic balance of the cell (choice (a)) or the membrane
potential (choice (d)). They are not involved in K+ import (choice (e)).
12-14 Disagree. A symporter functions by transporting two solutes in the same direction while
an antiporter transports two different solutes to opposite sides of the membrane.
Therefore, flipping the symporter around would not change its characteristics into that of
an antiporter, as it should still transport both solutes in the same direction.
12-15 A.
B.

C.

Without any ATP to provide energy for the Na+K+ pumps, no ions will be
pumped.
The pumps will utilize the energy from ATP hydrolysis to transport Na+ out of the
vesicles and K+ into the vesicles. (The pumps will stop working when either the
amount of ATP inside the vesicle is depleted or, when the K+ outside of the
vesicles is depleted.)
The pump will bind a molecule of Na+, causing the ATPase activity to hydrolyze
ATP and transfer the phosphate group onto the pump. A conformational change
will occur, leading to release of Na+ from the vesicle. However, since there is no
K+ outside of the vesicle, the pump will get stuck at that step and subsequent steps
of the cycle will not occur.

202

12-16 For an uncharged molecule, the direction of passive transport across a membrane is
determined solely by its concentration gradient. On the other hand, for a charged
molecule, an additional force called the membrane potential must also be considered.
The net driving force for a charged molecule across a membrane therefore has two
components and is referred to as the electrochemical gradient. Active transport allows
the movement of solutes against this gradient. The carrier proteins called coupled
transporters utilize the movement of one solute down its gradient to provide the energy to
drive the uphill transport of a second gradient. When this transporter moves both ions in
the same direction across the membrane, it is considered a(n) symport; if the ions move
in opposite directions, the transporter is considered a(n) antiport.
12-17 Choice (e) is the correct answer. Ions can pass either way through a channel; the direction
in which flow takes place depends on the concentration gradient and the membrane
potential. Some ion channels require a specific stimulus to open them; others, such as K+
leak channels, do not (choice (a)). Ion channels are passive transporters and therefore
require no energy source in order to function (choice (b)). Ion channels are very fast
relative to carrier proteins but are limited by the rate at which ions can move through the
channel (choice (c)). An ion channel allows specific positive or negative ions to pass, but
not both (choice (d)).
12-18 (b)
12-19 1c; 2d; 3b; 4a
12-20 A.
B.
C.
D.
E.

The acetylcholine receptor in skeletal muscle cells is a ligand-gated ion channel.


Stress-activated ion channels are found in the hair cells of the mammalian
cochlea.
Voltage-gated ion channels in the mimosa plant propagate the leaf-closing
response.
Voltage-gated ion channels respond to changes in membrane potential.
Many receptors for neurotransmitters are ligand-gated ion channels.

12-21 (b), (e) The Na+K+ pump continually transports K+ into the cell. The negative membrane
potential also helps to retain K+ in the cell. The K+ leak channels allow K+ to
move both into and out of the cell and so do not contribute to the accumulation of
K+ in the cell.
12-22 Agree. The membrane potential arises from a thin layer of ions that are close to the
membrane. Only a small number of ions (compared to the number of ions present) must
move across the membrane to set up the membrane potential.
12-23 A4; B1; C3; D2

203

12-24 The action potential is a wave of depolarization that rapidly spreads along the neuronal
plasma membrane. This wave is triggered by a local change in the membrane potential to
a value that is less negative than the resting membrane potential. The action potential is
propagated by the opening of voltage-gated channels. During an action potential, the
membrane potential changes from negative to positive. The action potential travels from
the neurons cell body along the axon to the nerve terminals. Neurons chiefly receive
signals at their highly branched dendrites.
12-25 See figure A12-25.

Figure A12-25
12-26 The ability of the voltage-gated Na+ channel to adopt an inactivated conformation allows
for the action potential to move along the membrane in a wave-like fashion. During the
firing of an action potential, the voltage-gated Na+ channel will open and then, after a
delay, adopt the inactive form. When the voltage-gated Na+ channel adopts an inactive
form, the membrane potential begins to return to its resting potential and cannot support
the production of another action potential (i.e., further membrane depolarization) until the
channel changes from an inactive to a closed state.
12-27 Neurons communicate with each other through specialized sites called synapses. Many
neurotransmitter receptors are ligand-gated ion channels that open transiently in the
postsynaptic cell membrane in response to neurotransmitters released by the
presynaptic cell. Ligand-gated ion channels in nerve cell membranes convert chemical
signals into electrical ones. Neurotransmitter release is stimulated by the opening of
voltage-gated Ca2+ channels in the nerve terminal membrane.

204

12-28 To trigger an action potential, a threshold level of membrane depolarization must be


achieved to allow for the opening of voltage-gated Na+ channels. As an action potential
is initiated, the initial opening of voltage-gated Na+ channels causes Na+ to enter the cell.
This makes the membrane potential less negative and opens even more voltage-gated Na+
channels, creating a positive feedback loop that triggers the action potential. The opening
of Cl channels by binding of inhibitory neurotransmitters permits an influx of Cl, which
makes the membrane potential more negative. The entry of Cl into the cell counteracts
the effect of the incoming Na+ on the membrane potential, making it more difficult to
depolarize the membrane and therefore, more difficult to reach the threshold potential
needed to generate an action potential.
12-29 The K+ leak channel inhibitor would make it easier to trigger an action potential in the
neuron. Inhibition of the K+ leak channel changes the permeability of the membrane with
respect to K+. The membrane potential is affected by the relative permeability of Na+ and
K+. In order for an action potential to fire, a threshold membrane potential must be
achieved. By changing the permeability of the membrane to K+ with the addition of the
inhibitor, the membrane is now closer to the threshold needed to fire an action potential
because the cell is now partially depolarized (i.e., addition of the inhibitor leads to a less
negative membrane potential). Therefore, less Na+ needs to flow into the neuron for the
threshold potential to be achieved, making it easier to trigger an action potential.
12-30 A.

B.

C.

Increasing the concentration of K+ in the squid axon cytoplasm will make the
membrane potential more negative. Doubling the amount of K+ increases the
driving force for K+ to move out of the cell, leaving the inside of the cell more
negative and thus decreasing the membrane potential. (Remember, from the
Nernst equation, the driving force for an ion across a membrane is proportional to
the ratio of the concentration of the ion on the outside to the concentration of the
ion on the inside.)
Doubling the amount of Na+ in the extracellular fluid will increase the height of
the peak of the action potential. Again, this is because now the driving force for
Na+ to enter the cell is greater than it was before. Thus, when Na+ channels open,
the flux of Na+ ions is now greater. (Remember that flux is the number of ions
entering per second.)
The action potential in this case will reach a height that is less than that normally
achieved. (Choline is added in this case to maintain bulk electrical neutrality.
Since Na+ channels are not permeable to choline, they do not contribute to the
electrochemical gradient.) You have now reduced the concentration of Na+ by
one half and thus decreased the driving force for Na+ to enter the cell.

205

CHAPTER 13
HOW CELLS OBTAIN ENERGY FROM FOOD
2009 Garland Science Publishing
3rd Edition

The Breakdown of Sugars and Fats


13-1 Which of the following stages in the breakdown of the piece of toast you had for
breakfast generates the most ATP? (a) Digestion of starch to glucose (b) Glycolysis
(c) The citric acid cycle (d) Oxidative phosphorylation (e) Conversion of pyruvate to
acetyl CoA

13-2 The advantage to the cell of the gradual oxidation of glucose during cellular respiration
compared with its combustion to CO2 and H2O in a single step is
(a) more free energy is released for a given amount of glucose oxidized.
(b) no energy is lost as heat.
(c) energy can be extracted in usable amounts.
(d) more CO2 is produced for a given amount of glucose oxidized.
(e) less O2 is required for a given amount of glucose oxidized
13-3 The final metabolite produced by glycolysis is
(a) acetyl CoA.
(b) pyruvate.
(c) 3-phosphoglycerate.
(d) glyceraldehyde 3-phosphate.
(e) fatty acids.
13-4 Which of the following steps or processes in aerobic respiration include the production of
carbon dioxide?
(a) Breakdown of glycogen
(b) Glycolysis
(c) Conversion of pyruvate to acetyl CoA
(d) Oxidative phosphorylation
(e) The citric acid cycle

207

13-5

On a diet consisting of nothing but protein, which of the following is the most likely
outcome?
(a)
Loss of weight because amino acids cannot be used for the synthesis of fat.
(b)
Muscle gain because the amino acids will go directly into building muscle.
(c)
Tiredness because amino acids cannot be used to generate energy.
(d)
Excretion of more nitrogenous (ammonia-derived) wastes than with a more
balanced diet.
(e)
Production of more carbon dioxide than with a more balanced diet.

13-6

Figure Q13-6 represents a cell lining the gut. Draw numbered labeled lines to indicate
exactly where inside a cell the following processes take place.

Figure Q13-6
1.
2.
3.
4.
5.
6.
7.

Glycolysis
Citric acid cycle
Conversion of pyruvate to activated acetyl groups
Oxidation of fatty acids to acetyl CoA
Glycogen breakdown
Release of fatty acids from triacylglycerols
Oxidative phosphorylation

13-7

Each of the ten steps of glycolysis is catalyzed by a different enzyme. Steps 1 and 3,
catalyzed by hexokinase and phosphofructokinase, involve the hydrolysis of one ATP
molecule. Steps 7 and 10, catalyzed by phosphoglycerate kinase and pyruvate kinase,
each generates a single ATP molecule. At first glance, it seems that the final ATP yield
is zero because there are two ATP-hydrolysis steps and two ATP-formation steps. How
can the net yield of glycolysis be two ATP molecules per glucose molecule?

13-8

The oxidation of sugars by glycolysis


(a)
occurs only in aerobic organisms.
(b)
generates carbon dioxide.
(c)
produces a net gain of ATP.
(d)
occurs in mitochondria.
(e)
uses NADH as a source of energy.

208

13-9

Phosphorylation of glucose (GLC) to produce glucose 6-phosphate (G6P) is the first step
in glucose metabolism after entry into cells. Thermodynamically, it is perfectly valid to
consider the cellular phosphorylation of glucose as the sum of two reactions.
(1)
(2)
NET:

GLC + Pi
ATP + H2O
GLC + ATP

G6P + H2O
ADP + Pi
G6P + ADP

G = 3.3 kcal/mole
G = 7.3 kcal/mole

But biologically it makes no sense at all. Hydrolysis of ATP (reaction 2) in one part of
the cell can have no effect on phosphorylation of glucose (reaction 1) elsewhere in the
cell.
A.
How does the cell manage to link these two reactions?
B.
What is the G for the net reaction?
13-10 What purpose is served by the phosphorylation of glucose to glucose 6-phosphate by the
enzyme hexokinase as the first step in glycolysis?
(a)
It helps drive the uptake of glucose from outside the cell.
(b)
It generates a high-energy phosphate bond.
(c)
It converts ATP to a more useful form.
(d)
It enables the glucose 6-phosphate to be recognized by phosphofructokinase, the
next enzyme in the glycolytic pathway.
(e)
It oxidizes one of the carbon atoms to yield usable energy.
13-11 Which reaction does the enzyme phosphoglucose isomerase catalyze?
(a)
glucose glucose 6-phosphate
(b)
fructose 6-phosphate fructose 1,6-bisphosphate
(c)
glucose 6-phosphate fructose 6-phosphate
(d)
glucose glucose 1-phosphate
(e)
glucose fructose
13-12 Give the full names of the reactants indicated by question marks in Figure Q13-12.

Figure Q13-12

209

13-13 For each statement below, indicate whether it is TRUE or FALSE. Explain why.
A.
Only aerobic organisms can do glycolysis, suggesting that glycolysis evolved
rather recently.
B.
Oxidation of a molecule requires the removal of electrons and can occur even if
there is no oxygen involved in the reaction.
C.
For a cell to capture energy from oxidation of food molecules, it is better to
release the energy in small packets so it can be stored in activated carrier
molecules.
D.
Fermentation produces more ATP than glycolysis.
E.
One turn of the citric acid cycle generates two molecules of CO2.
F.
The breakdown of one molecule of glucose during glycolysis results in one
molecule of pyruvate.
G.
NADH is more reduced than NAD+.
H.
The reactions of the citric acid cycle do not directly require the presence of
oxygen.
13-14 Which of the following cells rely exclusively on glycolysis to supply them with ATP?
(a)
Anaerobically growing yeast.
(b)
Aerobic bacteria.
(c)
Skeletal muscle cells.
(d)
Plant cells.
(e)
Protozoa.
13-15 In anaerobic conditions, skeletal muscle produces
(a)
lactate and CO2.
(b)
ethanol and CO2.
(c)
lactate only.
(d)
ethanol only.
(e)
lactate, ethanol, and CO2.
13-16 In mammals, liver cells are able to convert lactate to pyruvate. What purpose does this
serve for the organism?
(a)
It is an important way of generating more NADH for the organism.
(b)
It is an important way of generating NAD+.
(c)
It allows the organism to grow in anaerobic conditions.
(d)
It allows the lactate to be productively utilized.
(e)
It is an important way for the body to generate heat.
13-17 Anaerobically growing yeast further metabolizes the pyruvate produced by glycolysis to
CO2 and ethanol as part of a series of fermentation reactions.
A.
What other important reaction occurs during this fermentation step?
B.
Why is this reaction (i.e., the answer to part A) essential for the anaerobically
growing cell?

210

13-18 In the absence of oxygen, cells consume glucose at a high, steady rate. When oxygen is
added, glucose consumption drops precipitously and is then maintained at the lower rate.
Why is glucose consumed at a high rate in the absence of oxygen and at a low rate in its
presence?
13-19 Glycolysis and the citric acid cycle are comprised of several types of reactions that occur
sequentially and serve to harvest energy from the oxidation of carbon atoms. From the
two lists below, match the general class of enzyme from list 1 with the type of reaction
catalyzed from list 2.
List 1
A. Kinase

B. Isomerase
C. Dehydrogenase
D. Synthase or synthetase

List 2
1. Generation of product with the same chemical
formula as the substrate but different connections
between atoms.
2. Transfer of phosphate group from one molecule to
another.
3. Formation of additional carbon-carbon bonds.
4. Oxidation of a substrate.

13-20 The first energy-generating steps in glycolysis begin when glyceraldehyde 3-phosphate
undergoes an energetically favorable reaction in which it is simultaneously oxidized and
phosphorylated by the enzyme glyceraldehyde 3-phosphate dehydrogenase to form 1,3bisphosphoglycerate, with the accompanying conversion of NAD+ to NADH. In a second
energetically favorable reaction catalyzed by a second enzyme, the 1,3bisphosphoglycerate is then converted to 3-phosphoglycerate, with the accompanying
conversion of ADP to ATP. Which of the following statements is TRUE?
(a)
The reaction glyceraldehyde 3-phosphate 1,3-bisphosphoglycerate should be
inhibited when levels of NADH fall.
(b)
The G for the oxidation of the aldehyde group on glyceraldehyde 3-phosphate
to form a carboxylic acid is more negative than the G for ATP hydrolysis.
(c)
The high-energy bond to the phosphate group in glyceraldehyde 3-phosphate
contributes to driving the reaction forward.
(d)
The cysteine side chain on the enzyme is oxidized by NAD+.
(e)
The overall reaction glyceraldehyde 3-phosphate 3-phosphoglycerate has a
positive G.

211

13-21 The simultaneous oxidation and phosphorylation of glyceraldehyde 3-phosphate


described in Question 13-20 involves the formation of a highly reactive covalent thioester
bond between a cysteine side chain (reactive group SH) on the enzyme (glyceraldehyde
3-phosphate dehydrogenase) and the oxidized intermediate (see arrow in Figure Q1321A). If the enzyme had a serine (reactive group OH) instead of a cysteine at this
position, which could form only a much lower-energy bond to the oxidized substrate (see
arrow in Figure Q13-21B), how might this new enzyme act?

Figure Q13-21
(a)
(b)
(c)
(d)
(e)

It would oxidize the substrate and phosphorylate it without releasing it.


It would oxidize the substrate but not release it.
It would phosphorylate the substrate on the 2 position instead of the 1 position.
It would behave just like the normal enzyme.
It would use ATP instead of Pi to phosphorylate the substrate.

13-22 Acetyl CoA is


(a)
synthesized from pyruvate in the mitochondrial intermembrane space.
(b)
the intermediate through which food molecules are completely metabolized to
carbon dioxide in animal cells.
(c)
synthesized from pyruvate and CoA in a reaction that also generates NADH, CO2,
and ATP.
(d)
synthesized by the breakdown of fatty acids in the cytosol.
(e)
an intermediate in the oxidation of glucose in anaerobic skeletal muscle.

212

13-23 Assuming complete oxidation, which of the fatty acids shown in Figure Q13-23 will
generate the most ATP through cellular respiration? Why?

Figure Q13-23
13-24 During a single turn of the citric acid cycle
(a)
the two carbon atoms from acetyl CoA that enter the cycle are completely
oxidized to CO2.
(b)
three molecules of ATP are generated.
(c)
three molecules of NADH are generated.
(d)
an acetyl group is added to citric acid.
(e)
three molecules of CO2 are generated.
13-25 Explain why the following statement is FALSE.
One mole of oxaloacetate is required for every mole of acetyl CoA that is
metabolized via the citric acid cycle.
13-26 Cells oxidizing acetyl groups via the citric acid cycle require molecular oxygen in order
to
(a)
oxidize the acetyl groups to CO2.
(b)
regenerate NAD+.
(c)
regenerate FADH2.
(d)
regenerate CoA.
(e)
oxidize fatty acids to acetyl groups.
13-27 Given a mixture of all the enzymes of the citric acid cycle plus acetyl CoA, which of the
following sets of additions could support conversion of acetyl CoA to carbon dioxide?
Explain why.
(a)
Water, NAD+, GDP, phosphate, FAD+
(b)
Water, NAD+, GDP, phosphate, FAD+, oxaloacetate
(c)
Water, NAD+, GDP, phosphate, FAD+, citrate
(d)
Water, NAD+, GDP, phosphate, FAD+, citrate, coenzyme A
13-28 The last reaction of the citric acid cycle, which regenerates oxaloacetate (OAA) from
malate (MAL), has a very positive G = 7.1 kcal/mole. Despite its unfavorable
equilibrium position, material must flow through this reaction quite readily in
mitochondriaotherwise the cycle could not turn. How is flow through the cycle
accomplished in the face of such an overwhelmingly positive G?
213

13-29 Consider the schematic of the citric acid cycle shown in Figure Q13-29. The cycle
begins with the formation of a 6-carbon (6C) molecule from joining the 4-carbon (4C)
oxaloacetate and the 2-carbon (2C) acetyl group. By about halfway through the cycle,
two carbons have been lost as carbon dioxide and a 4C molecule labeled D is
regenerated. List two reasons why the cycle does not stop after production of D and
instead must continue through a series of different 4C intermediates.

Figure Q13-29
13-30 Which of the following statements regarding electron transport is TRUE?
(a)
Only high-energy electrons from NADH can be used to drive the electron
transport chain.
(b)
The proteins involved in electron transport couple oxidation to phosphorylation in
much the same way that glyceraldehyde 3-phosphate dehydrogenase couples
oxidation to phosphorylation in glycolysis.
(c)
Electron transport occurs only in eucaryotes.
(d)
Molecular oxygen is required as a donor of electrons to the electron transport
chain.
(e)
Electrons passing along the electron transport chain move to successively lower
energy states.
13-31 In the final stage of the oxidation of food molecules, a gradient of protons is formed
across the inner mitochondrial membrane, which is normally impermeable to protons. If
cells were exposed to an agent that causes the membrane to become freely permeable to
protons, which of the following effects would you expect to observe?
(a)
Cells would be completely unable to synthesize ATP.
(b)
NADH would build up.
(c)
Carbon dioxide production would cease.
(d)
Consumption of oxygen would fall.
(e)
The ratio of ATP to ADP in the cytoplasm would fall.

214

13-32 For each of the following sentences, fill in the blanks with the best word or phrase
selected from the list below. Not all words or phrases will be used; each word or phrase
may be used more than once.
Oxidative phosphorylation is a process that occurs in the
__________________ of mitochondria. It requires an electron-transport
chain that operates on the high-energy electrons taken from the activated
carrier molecules __________________ and __________________ that
are produced by glycolysis and the citric acid cycle. These electrons are
transferred through a series of molecules, and the energy released during
these transfers is used to generate a gradient of __________________, or
__________________. Since their concentration is much
__________________ outside than inside the mitochondria, the flow of
__________________, or __________________, down the concentration
gradient is energetically very __________________ and can thus be
coupled to the production of ATP from ADP. Thus, oxidative
phosphorylation refers to the oxidation of __________________ and
__________________ molecules and the phosphorylation of
__________________. Without this process, the yield of ATP from each
glucose molecule would be __________________ decreased.
ADP
ATP
cytosol
electrons
FADH2
favorable
glucose

GTP
H+
higher
inner membrane
lower
matrix
moderately

NAD+
NADH
Pi
protons
severely
slightly
unfavorable

How We Know: Unraveling the Citric Acid Cycle


13-33 Consider the following statement: Oxaloacetate acts catalytically to aid in the oxidation
of pyruvate found in suspensions of minced pigeon muscles.
A.
What was the original evidence for this statement?
B.
How did these sorts of experiments aid in the identification of intermediates in the
citric acid cycle?

215

Storing and Utilizing Food


13-34 Which of the following statements is TRUE?
(a)
Plant cells store all their food reserves as starch, whereas animals store all their
food reserves as glycogen.
(b)
Glycogen stores more energy than starch because glycogen molecules have many
more branch points that can be hydrolyzed.
(c)
Animal cells can convert fatty acids to sugars.
(d)
Plants synthesize starch for the same reason that animals synthesize glycogen.
(e)
Protein is an important form of energy storage in animal cells under normal
conditions.
13-35 In humans, glycogen is a more useful food storage molecule than fat because
(a)
a gram of glycogen produces more energy than a gram of fat.
(b)
it can be utilized to produce ATP under anaerobic conditions whereas fat cannot.
(c)
it binds water and therefore is useful in keeping the body hydrated.
(d)
for the same amount of energy storage, glycogen occupies less space in a cell than
does fat.
(e)
glycogen can be carried to cells via the bloodstream whereas fats cannot.
13-36 The intermediates of the citric acid cycle are constantly being depleted because they are
used to produce many of the amino acids needed to make proteins. These intermediates
must therefore be replenished by the conversion of pyruvate to oxaloacetate by the
enzyme pyruvate carboxylase. Bacteria, but not animal cells, have additional enzymes
that can carry out the reaction acetyl CoA + isocitrate oxaloacetate + succinate. Which
of the following compounds will not support the growth of animal cells when used as the
major source of carbon in food, but will support the growth of nonphotosynthetic
bacteria?
(a)
Pyruvate
(b)
Glucose
(c)
Fatty acids
(d)
Carbon dioxide
(e)
Fructose
13-37 Pyruvate can be converted into many other molecules by various biosynthetic and
metabolic pathways, which makes it a central hub in the regulation of cellular
metabolism. Which of the following molecules is not made from pyruvate?
(a)
Oxaloacetate
(b)
Ethanol
(c)
NADH
(d)
Lactate
(e)
Acetyl CoA

216

13-38 For each of the following sentences, fill in the blanks with the best word or phrase
selected from the list below. Not all words or phrases will be used; each word or phrase
should be used only once.
A carbon atom in a CO2 molecule in the atmosphere eventually becomes a
part of one of the enzymes that catalyzes glycolysis in one of your cells.
The CO2 first enters a cell in a corn leaf where photosynthesis fixes the
carbon to make it part of a sugar molecule, which travels from the leaf to
an ear of corn where it is stored as part of a polysaccharide
__________________ molecule in the corn seed. You then eat a corn
chip made from the corn seed. You digest the corn seed, and the free
__________________ travels in your bloodstream, eventually being taken
up by a liver cell and stored as __________________. When required,
this storage molecule breaks down into glucose-1-phosphate, which enters
the glycolytic pathway. Glycolysis produces __________________,
which is converted into acetyl CoA, which enters the
__________________. Several intermediates in this process can provide
the carbon skeleton for production of __________________, which are
then incorporated into the enzymes that catalyze steps in glycolysis.
amino acids
carbon fixation
citric acid cycle
fatty acid
fermentation
galactose
glucose
glycogen

insulin
lactate
nucleotides
oxidative phosphorylation
pyruvate
starch
triacylglycerol

217

Answers
13-1

Choice (d) is the correct answer.Oxidative phosphorylation produces about 28 ATP


molecules. Choice (a) produces no ATP; choice (b) nets 2 ATP; choice (c) produces 1
GTP; and choice (e) produces no ATP.

13-2

Choice(c) is the correct answer. Choice (a) is untrue as the same overall amount of free
energy is released by glucose oxidation, whatever the route. Choice (b) is untrue as a
proportion of the energy released is still lost as heat. Choices (d) and (e) are untrue as the
same amount of CO2 will be released and O2 consumed by the oxidation of glucose to
CO2 and H2O, whatever the route.

13-3

(b)

13-4

Choices (c) and (e) are the correct answers. To obtain the maximal energy from
pyruvate, all three carbons can be fully oxidized to carbon dioxide. First, one carbon is
oxidized and released as carbon dioxide when pyruvate is converted to acetyl CoA.
Second, the two remaining carbons are fully oxidized and released as carbon dioxide by
virtue of the citric acid cycle.

13-5

Choice (d) is the correct answer. Because ammonia is given off when amino acids are
metabolized to yield energy but is not given off when sugars and fats are metabolized,
you would expect more nitrogenous waste to be excreted. Choice (c) is incorrect since
amino acids can be converted into pyruvate and acetyl CoA and used to generate energy.
If more amino acids are consumed than are used, the body will not store them as protein
in muscle tissue but rather will store them as fat, so choices (a) and (b) are incorrect.
Amino acid metabolism does not produce more carbon dioxide than carbohydrate or fat
metabolism, so choice (e) is incorrect.

13-6

See Figure A13-6.

Insert Figure A13-16

218

13-7

The net yield of glycolysis is two ATP molecules per glucose molecule because the steps
that require ATP occur once per glucose molecule whereas those that generate ATP occur
twice per glucose molecule. This is because Step 4, catalyzed by aldolase, breaks down a
6-carbon intermediate into two 3-carbon intermediates. Each 3-carbon intermediate is
processed by the remaining steps and thus Steps 7 and 10 occur twice per original 6carbon glucose molecule.

13-8

Choice (c) is the correct answer. Glycolysis, the step-wise oxidation of glucose to
pyruvate, produces a net gain of two ATP molecules per starting glucose molecule.
Glycolysis occurs in both anaerobic and aerobic organisms; the citric acid cycle, but not
glycolysis, generates carbon dioxide and occurs in mitochondria (thus choices (a), (b),
and (d) are incorrect). Glycolysis does not use NADH and instead produces it (thus
choice (e) is incorrect).

13-9

A.

B.

The cell links these two reactions by using the same enzyme to catalyze both of
them. In essence, the enzyme hexokinase binds both GLC and ATP substrates
and catalyzes the direct transfer of a phosphate group from ATP onto the GLC.
Reactions 1 and 2 do not actually occur because free Pi is never produced.
The G for the net reaction is 4.0 kcal/mole because the free energy is the sum
of component reactions and is independent of the pathway used to convert
substrates into products. G = 3.3 kcal/mole 7.3 kcal/mole = 4.0 kcal/mole.

13-10 Choice (a) is the correct answer. It helps drive the uptake of glucose from outside the cell.
Choice (b) is incorrect since the phosphate transferred to the glucose is not held by a
high-energy covalent bond. Choice (c) is incorrect since the reaction converts ATP to
ADP, which is not useful as an energy source for most cellular reactions, even though it
still has one high-energy bond. Choice (d) is incorrect since the next enzyme in the
pathway is phosphoglucose isomerase, not phosphofructokinase. Choice (e) is incorrect
since the reaction does not involve oxidation of carbon nor does it yield usable energy.
13-11 Choice (c) is the correct answer. The isomerase part of the enzyme name indicates that it
catalyzes an isomerization reaction, and the phosphoglucose part of the name indicates
the type of substrate used. The enzyme that catalyzed reaction (e) would be called
glucose isomerase. The enzymes that catalyzed reactions (a), (b), and (d) would be called
kinases, because they transfer phosphate groups from one molecule to another.
13-12 phosphoenolpyruvate, adenosine diphosphate, pyruvate, adenosine triphosphate

219

13-13 A.
B.
C.
D.
E.
F.
G.
H.

13-14 (a)

False. All organisms can do glycolysis, suggesting that it evolved rather early.
True. Oxidation is the loss of electrons. The mechanism of oxidation often
involves reaction with oxygen atoms, but instead can occur in other ways.
True. If all the energy of oxidation were released at once, as in combustion, most
of the energy would be lost as heat and not captured in usable forms.
False. Fermentation by itself produces no ATP, but only regenerates NAD+ so
that glycolysis can continue to operate and to produce ATP.
True. Each turn of the citric acid cycle essentially exchanges the two carbons
from the input acetyl group into two carbons completely oxidized as CO2.
False. Glycolysis converts each molecule of glucose into two molecules of
pyruvate.
True. NADH is more reduced than NAD+, because it contains more electrons.
True. Oxygen is not one of the substrates in the reactions of the citric acid cycle.
Instead oxygen is needed for the citric acid cycle only indirectly, to regenerate the
oxidized forms of NAD+ and FAD, which are substrates of the citric acid cycle.
All the other cells can carry out oxidative phosphorylation to generate additional
ATP.

13-15 (c)
13-16 Choice (d) is the correct answer. Lactate is a metabolic dead end and it cannot be utilized
any further by cell. Pyruvate, on the other hand, can either be converted to acetyl CoA
and metabolized in the citric acid cycle or converted to glucose by a process called
gluconeogenesis. Conversion of lactate to pyruvate uses up NAD+ (making choice (b)
incorrect) and generates NADH, but the reason animal cells make lactate in the first place
is that they are suffering from an excess of NADH (and thus choice (a) is incorrect).
Choice (c) is incorrect since no mammal can survive without oxygen. The amount of heat
generated by converting lactate to pyruvate is negligible, so choice (e) is incorrect.
13-17 A.
B.

NADH NAD+.
Under anaerobic conditions, it is the only means of regenerating the NAD+
required for glycolysis, the main energy-generating pathway of an anaerobically
growing yeast cell.

13-18 Glucose is consumed at a much higher rate in the absence of oxygen because less usable
energy can be harvested from glucose in the absence of oxygen. Regardless of the
presence of oxygen in the environment, cells need about the same amount of energy in
the form of ATP. In the absence of oxygen, a glucose molecule yields only 2 ATP
molecules and thus many glucose molecules must be consumed to satisfy the energetic
needs of the cell. In the presence of oxygen, a glucose molecule yields about 30 ATP
molecules.
13-19 A2; B1; C4; D3

220

13-20 Choice (b) is the correct answer. This is another way of stating that the energetically
favorable oxidation of glyceraldehyde 3-phosphate provides sufficient energy to
ultimately drive the energy-requiring step of ATP synthesis from ADP. Choice (a) is
untrue: NADH is an end product of the reaction G-3-P to 1,3-PG and therefore high (not
low) levels of it would inhibit the reaction. Choice (c) is untrue: the reactions do not
involve the 3-phosphate group on glyceraldehyde 3-phosphate at all. Choice (d) is untrue,
since the cysteine on the enzyme is important in making a covalent intermediate with the
substrate and is not oxidized by NAD+. Choice (e) is untrue, since if the reaction had an
overall positive G, it could not be used to power the energetically unfavorable reactions
of ATP and NADH synthesis.
13-21 Choice (b) is the correct answer. The phosphorylation and release of the product from the
normal enzyme is possible because the high-energy thioester bond formed between the
oxidized substrate and enzyme can be attacked by a phosphate molecule. If the bond
between oxidized substrate and enzyme is a much lower-energy bond, the enzyme will
not be able to transfer the oxidized substrate to a phosphate group, and substrate and
enzyme will remain covalently bound. Choices (a), (c), and (d) could not happen, as none
of the bonds in the substrate molecule is reactive enough to be broken by a phosphate
group. Choice (e) would not happen because the enzyme does not bind ATP.
13-22 Choice (b) is the correct answer. Some food molecules can enter the citric acid cycle at
points other than acetyl CoA, but only molecules that enter as acetyl CoA are completely
oxidized to carbon dioxide. Acetyl CoA is made in the mitochondrial matrix, not the
intermembrane space, so choice (a) is incorrect. Choice (c) is incorrect since no ATP is
directly generated by the conversion of pyruvate to acetyl CoA; carbon dioxide and
NADH are the only other products of the reaction. Choice (d) is incorrect as the
breakdown of fatty acids to produce acetyl CoA occurs in the mitochondrial matrix.
Choice (e) is incorrect since acetyl CoA is not an intermediate in the anaerobic
fermentation reaction that coverts pyruvate to lactate.
13-23 (B)

This will produce 2 NADH, 2 FADH2, and 3 acetyl CoA on complete oxidation.
Because of the double bond in A, this fatty acid will produce 2 NADH and 3
acetyl CoA, but only 1 FADH2, since the initial oxidation step using FAD+ that
reduces the two-carbon unit CH2CH2 to CH=CH is not needed for twocarbon units already containing a double bond. So although the amount of acetyl
CoA entering the citric acid cycle will be the same for A and B, fewer reducing
equivalents will eventually enter the electron transport chain from the oxidation of
A, thus less ATP will be produced.

13-24 (c)
13-25 Only small amounts of oxaloacetate are required relative to the amount of acetyl CoA
metabolized because oxaloacetate is regenerated after every round of the citric acid cycle.

221

13-26 Choice (b) is the correct answer. The citric acid cycle generates high-energy electrons
that are passed to NAD+ to form NADH. NADH then donates these electrons to the
electron transport chain that drives oxidative phosphorylation, regenerating the NAD+
needed to keep the citric acid cycle going. The electrons from NADH are passed via the
electron transport chain to oxygen.
13-27

Choices (b) and (d) are correct. To get the cycle turning you need water, NAD+, GDP,
phosphate, FAD, coenzyme A, and at least one intermediate of the citric acid cycle,
which are all provided in choice (d). In addition, choice (b) will be sufficient because
oxaloacetate reacts with acetyl CoA to release a molecule of coenzyme A, which can then
be reused. Choice (c) would produce a small amount of CO2 initially from the added
citrate, but the cycle could not continue since citrate has to go through a step requiring
coenzyme A to complete the cycle. Choice (a) will not work, as this set does not include
any citric acid cycle intermediate.

13-28 The mitochondrion obtains a flow through this reaction by maintaining high
concentrations of substrates and low concentrations of products, so that the G is
negative despite a positive G. The concentration ratio of products to substrates,
[OAA][NADH]/[MAL][NAD+], must be substantially smaller than the equilibrium
constant K to overcome a positive G value.
13-29 The two reasons are to keep the cycle going and to harvest more energy. (1) The cycle
must regenerate oxaloacetate, which acts catalytically in the citric acid cycle to aid in the
oxidation of many acetyl groups. (2) Additional reactions are required to more fully
oxidize the carbons and harvest energy by producing several activated carrier molecules,
including GTP, FADH2, and NADH.
13-30 Choice (e) is the correct answer. Electrons passing along the electron transport chain
move to successively lower energy states. Choice (a) is untrue as electrons from FADH2
can be used as well. Choice (b) is untrue as the two mechanisms of coupling oxidation to
phosphorylation are quite different. Oxidative phosphorylation involves the oxidation of
NADH to NAD+ by proteins of the electron transport chain. Electron transport then
causes the formation of a proton gradient across a membrane, which drives ATP
synthesis. In contrast, glyceraldehyde 3-phosphate dehydrogenase action involves the
reduction of NAD+ to NADH and uses the G of glyceraldehyde oxidation to form a
high-energy bond that can be attacked directly by a phosphate group. Choice (c) is
untrue, as electron transport occurs in the plasma membrane of procaryotes. Choice (d) is
untrue, as molecular oxygen acts as an acceptor, not a donor, for electrons.

222

13-31 Choice (e) is the correct answer. If the inner mitochondrial membrane became permeable
to protons, the electron transport chain would continue to oxidize NADH to NAD+,
transport electrons and pump protons, so the consumption of oxygen would not fall
(choice (d)). However, the energy stored by the protons would be immediately dissipated
as heat when they flowed back across the membrane and thus could not drive the
synthesis of ATP. But, NADH would not build up (choice (b)), and the citric acid cycle
and glycolysis would continue (and thus CO2 would still be produced, contrary to choice
(c)). Since glycolysis and the citric acid cycle produce 2 molecules of ATP and one
molecule of GTP (which can be converted to ATP), respectively, ATP production would
not completely cease (choice (a)), but it would be very much less than normal.
13-32 Oxidative phosphorylation is a process that occurs in the inner membrane of
mitochondria. It requires an electron-transport chain that operates on the high-energy
electrons taken from the activated carrier molecules NADH and FADH2 that are
produced by glycolysis and the citric acid cycle. These electrons are transferred through
a series of molecules, and the energy released during these transfers is used to generate a
gradient of protons, or H+. Since their concentration is much higher outside than inside
the mitochondria, the flow of protons, or H+, down the concentration gradient is
energetically very favorable and can thus be coupled to the production of ATP from
ADP. Thus, oxidative phosphorylation refers to the oxidation of NADH and FADH2
molecules and the phosphorylation of ADP. Without this process, the yield of ATP from
each glucose molecule would be severely decreased.
13-33 A.

B.

Experiments showed that minced pigeon muscles contained large amounts of


pyruvate, but oxidized this compound rather slowly, so that little oxygen was
consumed and little carbon dioxide was produced. When a tiny amount of
oxaloacetate was added to such muscle preparations, large amounts of oxygen and
carbon dioxide were consumed and produced, respectively. If the added
oxaloacetate was simply being oxidized fully, the oxygen consumption and
carbon dioxide production would be expected to increase only slightly, but instead
the large amount of oxygen consumed suggested that each molecule of added
oxaloacetate aided in the oxidation of many molecules of some other substance.
Analogous experiments showed that the addition of several other compounds, like
succinate and fumarate, had the same consequences as adding oxaloacetate. This
was interpreted as evidence that these compounds are intermediates in the same
pathway and can be converted into, or are derived from, oxaloacetate, which was
later demonstrated more directly.

13-34 Choice (d) is the correct answer. Both starch and glycogen are storage polymers of
glucose. Choice (a) is false, since both plants and animals can also store food as fats and
oils. Choice (b) is false, as although glycogen synthesis requires ATP, no ATP is
generated by its hydrolysis to monomers (so the number of branch points is irrelevant to
energy storage). Choice (c) is false, as animal cells cannot do this. Choice (e) is false, as
the use of protein for energy occurs only under starvation conditions.

223

13-35 Choice (b) is the answer. The breakdown of glycogen to glucose does not require oxygen;
the glucose can then enter glycolysis and generate ATP by a fermentation process that
produces lactic acid. In contrast, fats are broken down to acetyl CoA that must enter the
citric acid cycle, which requires oxygen to keep turning. Choice (a) is incorrect, as a
gram of glycogen (wet or dry) produces less energy than a gram of fat. Choice (c) is
incorrect, as the water bound by glycogen is not useful in keeping the body hydrated and
merely contributes to making the glycogen weigh a lot. Choice (d) is incorrect, as the
actual mass of glycogen required to store the same amount of energy is six-fold greater
than the amount of fat. Choice (e) is incorrect, as fats can be carried in the bloodstream. If
the energy stored in glycogen is required by other cells, glycogen is broken down to
glucose, and the glucose is then released into the bloodstream.
13-36 Choice (c) is the correct answer. In oxidative metabolism, fatty acids can only be
converted to acetyl CoA, which is completely oxidized to carbon dioxide through the
citric acid cycle. In addition, bacteria can use some of this acetyl CoA as a source of
carbon atoms to replenish the citric acid cycle, whereas animals cannot. Choices (a), (b),
and (e) are incorrect, since glucose and fructose can be converted to pyruvate, and hence
to citric acid cycle intermediates, in both animal and bacterial cells, while choice (d) is
incorrect, since carbon dioxide cannot be used as a main carbon source by either
nonphotosynthetic bacteria or animal cells.
13-37 (c)

Pyruvate cannot be converted into NADH, but it can be converted into the other
metabolites in one or two steps.

13-38 A carbon atom in a CO2 molecule in the atmosphere eventually becomes a part of one of
the enzymes that catalyzes glycolysis in one of your cells. The CO2 first enters a cell in a
corn leaf where photosynthesis fixes the carbon to make it part of a sugar molecule,
which travels from the leaf to an ear of corn where it is stored as part of a polysaccharide
starch molecule in the corn seed. You then eat a corn chip made from the corn seed.
You digest the corn seed, and the free glucose travels in your bloodstream, eventually
being taken up by a liver cell and stored as glycogen. When required, this storage
molecule breaks down into glucose-1-phosphate, which enters the glycolytic pathway.
Glycolysis produces pyruvate, which is converted into acetyl CoA, which enters the
citric acid cycle. Several intermediates in this process can provide the carbon skeleton
for production of amino acids, which are then incorporated into the enzymes that
catalyze steps in glycolysis.

224

CHAPTER 14
ENERGY GENERATION IN MITOCHONDRIA
AND CHLOROPLASTS
2009 Garland Science Publishing
3rd Edition

Mitochondria and Oxidative Phosphorylation


14-1 Which of the following statements are likely to be TRUE? Note that more than one
statement may be selected.
(a) Organisms that could carry out fermentation evolved before those that could carry out
aerobic respiration.
(b) Organisms that could carry out oxygen-producing photosynthesis evolved before those that
could carry out fermentation reactions.
(c) Eucaryotic organisms were the first to evolve mechanisms of chemiosmotic coupling.
(d) Both photosynthesis and aerobic respiration first evolved in procaryotes.
(e) Aerobic respiration arose as an adaptation to increasing levels of oxygen in the atmosphere
that had been produced by photosynthesis.
14-2 Is the following statement TRUE or FALSE? Explain your answer.
The most important contribution of the citric acid cycle to energy metabolism is the
extraction of high-energy electrons during the oxidation of acetyl CoA to CO2

14-3 Is the following statement TRUE or FALSE? Explain your answer.


Each respiratory enzyme complex in the electron-transport chain has a greater
affinity for electrons than its predecessors, so that electrons pass sequentially
from one complex to another until they are finally transferred to oxygen, which
has the greatest electron affinity of all.
14-4 The citric acid cycle generates NADH and FADH2, which are then used in the process
of oxidative phosphorylation to make ATP. If the citric acid cycle (which does not use
oxygen) and oxidative phosphorylation are separate processes, as they are, then why is it
that the citric acid cycle stops almost immediately upon removal of O2?
14-5 What three essential items are missing from the following list of cellular components
required to make ATP by chemiosmotic coupling?
ADP; ATP synthase; protons; electron-transport chain; proton pump; membrane.

225

14-6

Electron transport is coupled to ATP synthesis in mitochondria, chloroplasts, and the


thermophilic bacterium Methanococcus. Which of the following are likely to affect the
coupling of electron transport to ATP synthesis in ALL of these systems? Note that more
than one statement may be selected.
(a)
A potent inhibitor of cytochrome oxidase.
(b)
The removal of oxygen.
(c)
The absence of light.
(d)
An ADP analogue that inhibits ATP synthase.
(e)
Dinitrophenol (permeabilizes membranes to protons).

14-7

In which of the four compartments of a mitochondrion are each of the following located?
A.
Porin
B.
The mitochondrial genome
C.
Citric acid cycle enzymes
D.
Proteins of the electron transport chain
E.
ATP synthase
F.
Membrane transport protein for pyruvate

14-8

Which of the following statements about mitochondria is FALSE?


(a)
Protons are pumped from the intermembrane space into the matrix.
(b)
ATP is synthesized in the matrix.
(c)
Mitochondria can change shape.
(d)
The outer membrane is permeable to protons.
(e)
The inner membrane is folded into cristae.

14-9

For each of the following sentences, fill in the blanks with the best word or phrase
selected from the list below. Not all words or phrases will be used; each word or phrase
should be used only once.
Mitochondria can use both __________________ and
__________________ as fuel. __________________ produced in the
citric acid cycle donates electrons to the electron-transport chain. The
citric acid cycle oxidizes __________________ and produces
__________________ as a waste product. __________________ acts as
the final electron acceptor in the electron-transport chain. The synthesis of
ATP in mitochondria is also known as __________________.
acetyl groups
carbon dioxide
chemiosmosis
fatty acids
glucose
NAD+

NADH
NADP+
NADPH
oxidative phosphorylation
oxygen
pyruvate

226

14-10 For each of the following sentences, fill in the blanks with the best word or phrase
selected from the list below. Not all words or phrases will be used; each word or phrase
should be used only once.
NADH donates electrons to the __________________ of the three
respiratory enzyme complexes in the mitochondrial electron-transport
chain. __________________ is a small protein that acts as a mobile
electron carrier in the respiratory chain. __________________ transfers
electrons to oxygen. Electron transfer in the chain occurs in a series of
__________________ reactions. The first mobile electron carrier in the
respiratory chain is __________________.
cytochrome c
cytochrome oxidase
first
NADH dehydrogenase
oxidation
oxidation-reduction
phosphorylation

plastoquinone
reduction
second
the cytochrome b-c1 complex
third
ubiquinone

14-11 Which of the following statements is TRUE?


(a)
Because the electrons in NADH are at a higher energy than the electrons in
reduced ubiquinone, the NADH dehydrogenase complex can pump more protons
than can the cytochrome b-c1 complex.
(b)
The pH in the mitochondrial matrix is higher than the pH in the intermembrane
space.
(c)
The proton concentration gradient and the membrane potential across the inner
mitochondrial membrane tend to work against each other in driving protons from
the intermembrane space into the matrix.
(d)
The difference in proton concentration across the inner mitochondrial membrane
has a much larger effect on the total proton-motive force than does the membrane
potential.
(e)
All of the free energy released by the net transfer of electrons from NADH to O2
is captured in the form of the proton gradient.
14-12 Some bacteria can live both aerobically and anaerobically. How does the ATP synthase in
the plasma membrane of the bacterium enable such bacteria to keep functioning in the
absence of oxygen?

227

14-13 Bongkrekic acid is an antibiotic that inhibits the ATP/ADP transport protein in the inner
mitochondrial membrane. Which of the following will allow electron transport to occur
in mitochondria treated with bongkrekic acid?
(a)
Placing the mitochondria in anaerobic conditions
(b)
Adding FADH2
(c)
Permeabilizing the inner membrane to protons
(d)
Inhibiting the ATP synthase
(e)
Increasing the concentration of ATP in the matrix
14-14 Which of the following types of ion movement might be expected to require cotransport
of protons from the intermembrane space to the matrix, inasmuch as it could not be
driven by the membrane potential across the inner membrane? (Assume that each ion
being moved is moving against its concentration gradient.)
(a)
Import of Ca2+ into the matrix from the intermembrane space
(b)
Import of acetate ion into the matrix from the intermembrane space
(c)
Exchange of Fe2+ in the matrix for Fe3+ in the intermembrane space
(d)
Exchange of ATP from matrix for ADP in the intermembrane space
(e)
Exchange of Ca2+ in the matrix for Na+ in the intermembrane space
14-15 The relationship of free-energy change (G) to the concentrations of reactants and
products is important because it predicts the direction of spontaneous chemical reactions.
Consider, for example, the hydrolysis of ATP to ADP and inorganic phosphate (Pi). The
standard free-energy change (G) for this reaction is 7.3 kcal/mole. The free-energy
change depends on concentrations according to the following equation:
G = G + 1.42 log10 ([ADP] [Pi]/[ATP])
A.

B.

In a resting muscle, the concentrations of ATP, ADP, and Pi are approximately


0.005 M, 0.001 M, and 0.010 M, respectively. What is G for ATP hydrolysis in
resting muscle? What is the G for ATP synthesis in resting muscle?
What will G equal when the hydrolysis reaction reaches equilibrium? At [Pi] =
0.010 M, what will be the ratio of [ATP] to [ADP] at equilibrium?

14-16 How many molecules of ATP are produced by the electron-transport chain and the ATP
synthase from the oxidation of the 18-carbon fatty acid derivative stearyl CoA? The first
steps in the oxidation of stearyl CoA occur in the mitochondrion and generate 9
molecules of acetyl CoA, 8 molecules of NADH, and 8 molecules of FADH2.

228

How We Know: How Chemiosmotic Coupling Drives ATP Synthesis


14-17 The respiratory chain is relatively inaccessible to experimental manipulation of intact
mitochondria. After disrupting mitochondria with ultrasound, however, it is possible to
isolate functional submitochondrial particles, which consist of broken cristae that have
resealed inside-out into small closed vesicles. In these vesicles the components that
originally faced the matrix are now exposed to the surrounding medium.
A.
How might such an arrangement aid in the study of electron transport and ATP
synthesis?
B.
Consider an anaerobic preparation of such submitochondrial particles. If the flow
of protons through ATP synthase is blocked by an inhibitor and a small amount of
oxygen is added, do you predict that the preparation will consume oxygen in
respiration reactions? Will the medium outside the particles become more acidic
or basic? Explain.
14-18 In classic experiments that helped convince investigators of the veracity of the
chemiosmotic hypothesis, artificial membrane vesicles were reconstituted with purified
bacteriorhodopsin (which is a light-driven H+ pump from a photosynthetic bacterium)
and purified ATP synthase from ox heart mitochondria. In these vesicles the membrane
complexes are oriented so that protons are pumped into the vesicle and ATP synthesis
occurs on the outer surface. When ADP and Pi were added to the medium surrounding
the vesicles and the vesicles were illuminated with appropriate light, the interior of the
vesicles became acidic and ATP was produced.
A.
If, instead, the ATP synthase molecules were randomly oriented so that about half
faced the outside of the vesicle and half faced the inside, would you expect ATP
to be synthesized? Explain your answer.
B.
If the bacteriorhodopsin molecules were randomly oriented, would you expect
ATP to be synthesized? Explain your answer.

Electron-Transport Chains and Proton Pumping


14-19 Distinguish between a proton, a hydrogen atom, a hydride ion, and a hydrogen molecule.
14-20 Which of the following statements is TRUE?
(a)
Only compounds with negative redox potentials can donate electrons to other
compounds under standard conditions.
(b)
Compounds that donate one electron have higher redox potentials than those of
compounds that donate two electrons.
(c)
The E0 of a redox pair does not depend on the concentration of each member of
the pair.
(d)
The free energy change, G, for an electron transfer reaction does not depend on
the concentration of each member of a redox pair.
(e)
If the E0 of the reaction AH2 A + 2 H+ + 2e is 600 mV and the E0 of the
reaction H2O 1/2 O2 + 2 H+ + 2e is 820 mV, then the transfer of electrons
from water to AH2 must be favorable under standard conditions.
229

14-21 Consider a redox reaction between molecules A and B. Molecule A has a redox potential
of 100 mV and molecule B has a redox potential of +100 mV. For the transfer of
electrons from A to B, is the G positive or negative or zero? Under what conditions
will the reverse reaction, transfer of electrons from B to A, occur?
14-22 For each of the following sentences, choose one of the options enclosed in square
brackets to make a correct statement.
An electron bound to a molecule with low affinity for electrons is a
[high/low] energy electron. Transfer of an electron from a molecule with
low affinity to one with higher affinity has a [positive/negative] G and
is thus [favorable/unfavorable] under standard conditions. If the reduced
form of a redox pair is a strong electron donor with a [high/low] affinity
for electrons, it is easily oxidized; the oxidized member of such a redox
pair is a [weak/strong] electron acceptor.
14-23 Which of the following reactions have a large enough free energy change to enable it to
be used, in principle, to provide the energy needed to synthesize one molecule of ATP
from ADP and Pi under standard conditions? See Table 14-23. Recall that G = n
(0.023) E0 and E0 = E0 (acceptor) E0 (donor).
(a)
The reduction of a molecule of pyruvate by NADH
(b)
The reduction of a molecule of cytochrome b by NADH
(c)
The reduction of a molecule of cytochrome b by reduced ubiquinone
(d)
The oxidation of a molecule of reduced ubiquinone by cytochrome c
(e)
The oxidation of cytochrome c by oxygen
Table 14-23
Reaction
NADH NAD+ + H+ + 2e
Lactate pyruvate + 2H+ + 2e
Reduced ubiquinone ubiquinone + 2H+ + 2e
Cytochrome b (Fe2+) cytochrome b (Fe3+) + e
Cytochrome c (Fe2+) cytochrome c (Fe3+) + e
H2O 1/2O2 + 2H+ + 2e

E0
320 mV
190 mV
30 mV
70 mV
230 mV
820 mV

14-24 Is the following statement TRUE or FALSE? Explain your answer.


Most cytochromes have a higher redox potential (higher affinity for
electrons) than iron-sulfur centers, which is why the cytochromes tend to
serve as electron carriers near the O2 end of the respiratory chain.

230

14-25 Which of the following statements is TRUE?


(a)
Ubiquinone is a small hydrophobic protein containing a metal group that acts as
an electron carrier.
(b)
A 2Fe2S iron-sulfur center carries one electron while a 4Fe4S center carries two.
(c)
Iron-sulfur centers generally have a higher redox potential than do cytochromes.
(d)
Mutation of hydrophobic amino acids near the heme group of cytochrome c to
acidic amino acids is likely to increase the redox potential of cytochrome c.
(e)
Mitochondrial electron carriers with the highest redox potential generally contain
copper ions and/or heme groups.
14-26 Which of the following is not an electron carrier that participates in the electron transport
chain?
(a)
Cytochrome
(b)
Quinone
(c)
Rhodopsin
(d)
Copper ion
(e)
Iron-sulfur center

231

14-27 In 1925, David Keilin used a simple spectroscope to observe the characteristic absorption
bands of the cytochromes that participate in the electron-transport chain in mitochondria.
A spectroscope passes a very bright light through the sample of interest and then through
a prism to display the spectrum from red to blue. If molecules in the sample absorb light
of particular wavelengths, dark bands will interrupt the colors of the rainbow. His key
discovery was that the absorption bands disappeared when oxygen was introduced and
then reappeared when the samples became anoxic. Subsequent findings demonstrated
that different cytochromes absorb different frequencies of light. When light of a
characteristic wavelength shines on a mitochondrial sample, the amount of light absorbed
is proportional to the amount of a particular cytochrome present in its reduced form.
Thus, spectrophotometric methods can be used to measure how the amounts of reduced
cytochromes change over time in response to various treatments. If isolated
mitochondria are incubated with a source of electrons such as succinate, but without
oxygen, electrons enter the respiratory chain, reducing each of the electron carriers
almost completely. When oxygen is then introduced, the carriers oxidize at different
rates, as can be seen from the decline in the amount of reduced cytochrome (see Figure
Q14-27). Note that cytochromes a and a3 cannot be distinguished and thus are listed as
cytochrome (a + a3). How does this result allow you to order the electron carriers in the
respiratory chain? What is their order?

Figure Q14-17
14-28 When molecular oxygen (O2) picks up one electron it becomes converted to the
superoxide radical O2. This radical is potentially damaging to cells as it will avidly pick
up another three electrons from a wide variety of cellular molecules. How do cells avoid
this happening during cellular respiration?
14-29 The antibiotic antimycin A blocks the electron-transport chain in bacterial cells.
Specifically, antimycin A inhibits the flow of electrons between cytochrome b and
cytochrome c1.
A.
After treatment with this antibiotic, which electron carriers in the chain will
accumulate in the reduced form?
B.
Which compounds will be likely to be oxidized more than before treatment?
C.
Will the ATP:ADP ratio in the cell rise or fall?

232

Chloroplasts and Photosynthesis


14-30 Both mitochondria and chloroplasts use electron transport to pump protons, creating an
electrochemical proton gradient, which drives ATP synthesis. Are protons pumped
across the same (analogous) membranes in the two organelles? Is ATP synthesized in the
analogous compartments? Explain your answers.
14-31 Ions are able to diffuse freely from
(a)
one thylakoid space to another thylakoid space in the same chloroplast.
(b)
the cytosol to the intermembrane space.
(c)
the intermembrane space to the stroma.
(d)
the stroma to the thylakoid space.
(e)
the interior of one granum to another in the same chloroplast.
14-32 Indicate if each of the following features is found in chloroplasts (C), mitochondria (M),
or both (B).
A.
Inner membrane
B.
Thylakoid
C.
Ribosomes
D.
Stroma
E.
ATP synthase
F.
Grana
G.
Citric acid cycle
H.
Matrix
I.
Electron-transfer reactions
J.
Carbon fixation
K.
Cristae
14-33 Write out the list of substrates and products for the overall net reaction that occurs in the
light stage of photosynthesis. Make the same list for the net overall reaction in the
dark stage of photosynthesis.

233

14-34 If you shine light on chloroplasts and measure the rate of photosynthesis as a function of
light intensity, you get a curve that plateaus at a fixed rate of photosynthesis, x, as shown
in Figure Q14-34.

Figure Q14-34
Which of the following conditions will increase the value of x?
(a)
Increasing the number of chlorophyll molecules in the antennae complexes
(b)
Increasing the number of reaction centers
(c)
Adding a powerful oxidizing agent
(d)
Decreasing the wavelength of light used
(e)
Increasing the rate at which chlorophyll molecules are able to transfer electrons to
one another
14-35 If you add a compound to illuminated chloroplasts that inhibits the NADP+ reductase,
NADPH generation ceases, as expected. Ferredoxin, however, does not accumulate in the
reduced form because it is able to donate its electrons not only to NADP+ (via NADP+
reductase), but also back to the cytochrome b6-f complex. Thus, in the presence of the
above compound, a cyclic form of photosynthesis occurs in which electrons flow in a
circle from ferredoxin, to the cytochrome b6-f complex, to plastocyanin, to photosystem I,
to ferredoxin. What will happen if you now also inhibit photosystem II?
(a)
Less ATP will be generated per photon absorbed.
(b)
ATP synthesis will cease.
(c)
Plastoquinone will accumulate in the oxidized form.
(d)
Plastocyanin will accumulate in the oxidized form.
(e)
Ferredoxin will accumulate in the reduced form.

234

14-36 For each of the following sentences, fill in the blanks with the best word or phrase
selected from the list below. Not all words or phrases will be used; each word or phrase
should be used only once.
In the carbon fixation process in chloroplasts, carbon dioxide is initially
added to the sugar __________________. The final product of carbon
fixation in chloroplasts is the three-carbon compound
__________________. This is converted into __________________
(which can be directly used by the mitochondria), into
__________________ (which is exported to other cells), and into
__________________ (which is stored in the stroma). The carbon
fixation cycle requires energy in the form of __________________ and
reducing power in the form of __________________.
3-phosphoglycerate
ATP
glyceraldehyde 3-phosphate
NADH
NADPH

pyruvate
ribose 1,5-bisphosphate
ribulose 1,5-bisphosphate
starch
sucrose

14-37 The enzyme ribulose bisphosphate carboxylase (rubisco) normally adds carbon dioxide to
ribulose 1,5-bisphosphate. However, it will also catalyze a competing reaction in which
O2 is added to ribulose 1,5-bisphosphate to form 3-phosphoglycerate and
phosphoglycolate. Assume that phosphoglycolate is a compound that cannot be used in
any further reactions. If O2 and CO2 have the same affinity for rubisco, which of the
following is the lowest ratio of CO2 to O2 at which a net synthesis of sugar can occur?
(a)
1:3
(b)
1:2
(c)
1:1
(d)
2:1
(e)
3:1
14-38 A suspension of the cyanobacterium Chlamydomonas is actively carrying out
photosynthesis in the presence of light and CO2. If you turned off the light, how would
you expect the amounts of ribulose 1,5-bisphosphate and 3-phosphoglycerate to change
over the next minute? How about if you left the light on but removed the CO2?

235

The Origins of Chloroplasts and Mitochondria


14-39 Which of the phylogenetic trees in Figure Q14-39 is the most accurate? (The
mitochondria and chloroplasts are from maize, but are treated as independent
organisms for the purposes of this question.)

Figure Q14-39

236

14-40 The biology of Methanococcus jannaschii is of interest to evolutionary biologists because


(more than one answer may be correct)
(a)
phylogenetically it is thought to be more closely related to the proposed ancestor
cell than are organisms such as cyanobacteria and yeast.
(b)
its small genome is approximately the same size as the genome of the proposed
ancestor cell.
(c)
it lives under conditions that resemble the environment in which the first living
cells may have dwelt.
(d)
its method of carbon fixation is thought to have given rise to the dark reactions
used in modern cyanobacteria and chloroplasts.
(e)
the proteins in the electron-transport chain that Methanococcus uses to generate
energy are thought to have given rise to those in the electron-transport chain used
in modern mitochondria and aerobic bacteria.
14-41 Describe what is meant by nitrogen fixation.

237

Answers
14-1

Choices (a), (d) and (e) are correct. Fermentation probably evolved before aerobic
respiration because the early earth had a very low concentration of atmospheric oxygen
(choice (a)). Fermentation probably evolved before photosynthesis because it is far
simpler and more fundamental than photosynthesis or aerobic respiration (thus b is
unlikely). Once photosynthetic reactions caused an increase in the concentration of free
oxygen, organisms that were able to catalyze reactions of aerobic respiration came to
have an enormous selective advantage (choice (e)). Chemiosmotic coupling, which
forms the basis of both aerobic respiration and photosynthesis, first evolved in
prokaryotes (thus choice (c) is incorrect). It is thought that the ancestor of eucaryotic
cells enveloped a bacterial cell capable of aerobic respiration, which then became the
mitochondria; a similar mechanism of envelopment and symbiosis that occurred later
probably originated the chloroplasts (choice (d)).

14-2

True. When the citric acid cycle is operating as a cycle of reactions, the primary products
from a single molecule of pyruvate are 3 CO2, 1 GTP, 3 reduced NADH, and 1 reduced
FADH2. The electrons in NADH and FADH2 are passed into the electron transport chain
to generate ATP via oxidative phosphorylation. Although a single GTP is made directly
by the citric acid cycle, many molecules of ATP are made by oxidative phosphorylation
of the 3 NADH molecules and 1 FADH2.

14-3

True. An electron carrier with low electron affinity carries the electron in a high-energy
state. Transfer of an electron from a low-affinity carrier to a high-affinity carrier is
energetically favorable. Thus, transfer of an electron to compounds with sequentially
higher electron affinities is thermodynamically favorable and releases energy that can be
coupled to the energetically unfavorable pumping of protons against an electrochemical
gradient. Oxygen has very high affinity for electrons, and thus serves as an efficient
electron sink for the entire chain.

14-4

The citric acid cycle stops almost immediately upon removal of oxygen because several
steps in the cycle require the oxidized forms of NAD+ and FAD. In the absence of
oxygen, these electron carriers can be reduced by the citric acid cycle reactions but
cannot be reoxidized by the electron-transport chain that participates in oxidative
phosphorylation.

14-5

(1) inorganic phosphate (Pi ); (2) an electron donor like NADH or FADH2 that provides
high-energy electrons (or high-energy electrons); (3) an electron acceptor.

14-6

Choices (d) and (e) are the correct answers. All chemiosmotic coupling systems involve
a proton gradient that is utilized by an ATP synthase that binds ADP and phosphorylates
it. Hence all chemiosmotic systems will be affected by agents that prevent ADP from
binding the synthase or that dissipate the proton gradient. Cytochrome oxidase and
oxygen are required only for mitochondria and aerobic bacteria (not Methanococcus);
light is required only for chloroplasts and photosynthetic bacteria (not Methanococcus).

238

14-7

A.
B.
C.
D.
E.
F.

Porin = outer membrane


mitochondrial genome = matrix
citric acid cycle enzymes = matrix
proteins in the electron-transport chain = inner membrane
ATP synthase = inner membrane
transport protein for pyruvate = inner membrane.

14-8

Choice (a) is the correct answer. Protons are NOT pumped from the intermembrane
space into the matrix, but are instead pumped in the reverse direction. All the other
statements are true.

14-9

Mitochondria can use both pyruvate and fatty acids directly as fuel. NADH produced
in the citric acid cycle donates electrons to the electron-transport chain. The citric acid
cycle oxidizes acetyl groups and produces carbon dioxide as a waste product. Oxygen
acts as the final electron acceptor in the electron-transport chain. The synthesis of ATP
in mitochondria is also known as oxidative phosphorylation.

14-10 NADH donates electrons to the first of the three respiratory enzyme complexes in the
mitochondrial electron-transport chain. Cytochrome c is a small protein that acts as a
mobile electron carrier in the respiratory chain. Cytochrome oxidase transfers electrons
to oxygen. Electron transfer in the chain occurs in a series of oxidation-reduction
reactions. The first mobile electron carrier in the respiratory chain is ubiquinone.
14-11 Choice (b) is the correct answer. The pumping of protons out of the matrix into the
intermembrane space creates a difference in proton concentration between the two sides
of the membrane, with the matrix at a higher pH (i.e., more alkaline) than the
intermembrane space, which tends to equilibrate with the cytosol that has a neutral pH.
The electrons in NADH are at a higher energy than the electrons in reduced ubiquinone
but proton pumping is not determined simply by the energy of the electron donors (choice
(a)). Instead, the number of protons that can be pumped by each complex is determined
by the difference in energy between the electrons in each substrate/product pair (i.e., the
difference between the electrons in NADH and reduced ubiquinone, compared to that
between reduced ubiquinone and reduced cytochrome c). The proton concentration
gradient and the membrane potential generated by the electron-transport chain work in
the same direction (choice (c)), creating a steep electrochemical gradient for protons
across the membrane. Choice (d): The difference in proton concentration has a smaller
effect on the total proton-motive force than does the membrane potential. Choice (e): All
of the free energy released by the net transfer of electrons from NADH to O2 cannot be
captured in the form of the proton gradient; some of the free energy released by the
reactions is lost as heat.
14-12 In the absence of oxygen, the respiratory chain no longer pumps protons and thus no
proton electrochemical gradient is generated across the bacterial membrane. In these
conditions the ATP synthase uses some of the ATP generated by glycolysis in the cytosol
to pump protons out of the bacterium, thus forming the proton gradient across the
membrane that the bacterium requires to import vital nutrients by coupled transport.
239

14-13 (c)

Inhibition of the ATP/ADP translocase prevents export of ATP generated by


oxidative phosphorylation in exchange for an import of ADP into the matrix. The
ensuing buildup of ATP at the expense of ADP inhibits the ATP synthase. Since
protons are no longer being used to power the ATP synthase, the proton gradient
is not dissipated; the increasingly steep proton gradient makes it increasingly
difficult for the electron-transport proteins to pump protons out of the matrix and
electron transport quickly stops. Hence, permeabilizing the inner membrane to
protons will allow electron transport to resume (although no ATP will be
synthesized).

14-14 Choices (b) and (e) are the correct answers. Since the inside of the membrane (the
mitochondrial matrix) is more negative than the outside, in principle, any traffic resulting
in an increase in the positive charge in the matrix can be driven by the membrane
potential. Hence, import of Ca2+ into the matrix, and exchange of Fe2+ (or ATP) in the
matrix for Fe3+ (or ADP) in the intermembrane space can be driven by the membrane
potential and need not require the cotransport of protons down the pH gradient. Import of
acetate ion into the matrix and exchange of Ca2+ in the matrix for Na+ in the
intermembrane space, in contrast, result in an increase in the amount of negative charge
in the matrix and cannot be driven by the charge difference between the two
mitochondrial compartments.
14-15 A.

B.

The G for hydrolysis is 11.1 kcal/mole. This result is calculated by plugging


values into equation given: G = 7.3 kcal/mole + 1.42 log10 ([0.001 M] [0.010
M]/[0.005 M]) = 7.3 kcal/mole + 1.42 log10 (0.002) = 11.1 kcal/mole. The G
for synthesis is +11.1 kcal/mole because the forward and reverse reactions always
have the same numerical value for G, but with the sign reversed.
At equilibrium, the G is equal to zero by definition. The ratio of [ATP] to
[ADP] at equilibrium is less than one to 10 million. This result is calculated by
setting G = 0, so that 1.42 log10 ([ADP] [Pi]/[ATP]) = G = 7.3 kcal/mole.
Solving for [ADP]/[ATP], the equation becomes log10 ([ADP] [0.010]/[ATP]) =
7.3/1.42 = 5.14; then [ADP]/[ATP] = (105.14)/(0.010) = 13.8 106. Thus, the
reciprocal [ATP]/[ADP] is 7.2 108.

14-16 113 molecules. Since each molecule of acetyl CoA produces 3 molecules of NADH and
1 molecule of FADH2, the oxidation of stearyl CoA generates a total of 35 molecules of
NADH and 17 molecules of FADH2. From each molecule of NADH and FADH2, 2.5
and 1.5 molecules of ATP can be formed, respectively. Not counted in this calculation
are the 9 molecules of GTP produced by the citric acid cycle, prior to electron transport.

240

14-17 A.

B.

14-18 A.

B.

This arrangement of components within the vesicles allows experimental


manipulation of the medium surrounding the vesicles in order to examine the
consequences of different conditions in the mitochondrial matrix. The medium
can be altered by changing pH, adding electron carriers and oxygen, and
providing ADP and Pi, for example. Oxidation of electron carriers, consumption
of oxygen, and production of ATP can be measured in the medium. By changing
the composition of the medium, it should be possible, for example, to identify the
electron carriers that can donate electrons from the matrix to the transport chain,
to assess the redox potentials of various components of the transport chain, and to
determine the dependence of ATP synthesis on the pH gradient across the
membrane and on the ATP:ADP ratio.
Respiration reactions will rapidly consume at least some of the added oxygen.
During the anaerobic conditions, the electron carriers in the electron transport
chain were reduced; upon addition of oxygen, electrons will be transferred to
oxygen, thereby reducing the oxygen and oxidizing the carriers. Concomitant
with the electron flow, protons will be pumped from the medium into the vesicles,
thereby making the medium slightly more basic and the inside of the vesicles
acidic. Inhibition of the ATP synthase will not have an immediate effect on
oxygen consumption or proton pumping. However, the proton concentration
inside the vesicles will quickly become too high to continue the activity of the
electron transport-coupled proton pumping and thus electron transport and oxygen
consumption will cease.
If the ATP synthase molecules were randomly oriented, you would still expect
ATP to be synthesized, although at about half the rate. The molecules that were
oriented correctly would make ATP; the oppositely oriented ATP synthase
molecules would be inert.
If bacteriorhodopsin was randomly oriented, you would expect much less ATP to
be synthesized. In vesicles with equal numbers of oppositely oriented
bacteriorhodopsin molecules, no pH difference would be generated upon exposure
to light because the proton pumping in both directions would be equal. In
vesicles with an excess of outwardly directed proton pumps, the pH difference
would be in the wrong direction to be utilized by ATP synthase and thus, no ATP
would be made. In vesicles with an excess of inwardly directed proton pumps, a
pH difference in the right orientation would be generated; thus, those vesicles
would be capable of synthesizing some ATP.

14-19 A proton is a hydrogen atom that has lost its single electron and, thus, is positively
charged. A hydride ion is a hydrogen atom that has gained an extra electron and thus is
negatively charged. A hydrogen atom is a proton plus one electron; it is neutral. A
hydrogen molecule is a pair of hydrogen atoms that share their two electrons in a
covalent bond; it is neutral.

241

14-20 Choice (c) is the correct answer. By definition, E0 refers to the standard state of equal
concentrations of each member of the redox pair. Therefore E0 does not vary with the
actual concentrations. Compounds with positive redox potentials can donate electrons to
other compounds under standard conditions, so long as the electron acceptor has a higher
(more positive) redox potential, thus option (a) is incorrect. Compounds that are able to
donate only one electron do not necessarily have higher redox potentials than those of
compounds that are able to donate two electrons, thus option (b) is incorrect. (Water, for
example, has a very high redox potential.) Although the E0 of a reaction is directly
proportional to the G of a reaction and both are independent of the concentrations of
substrates and products, the G depends on these concentrations, thus option (d) is
incorrect. In option (e), the transfer of electrons from water to AH2 is unfavorable. This
redox reaction gives a E0 = E0 (acceptor) E0 (donor) = 600 mV 820 mV = 220
mV. Since a negative E0 corresponds to a positive standard free-energy, this electron
transfer will not occur under standard conditions.
14-21 The G is negative. The sign of G is opposite of that of E0 = E0 (acceptor) E0
(donor). The acceptance of electrons by B from A has a E0 = 100 + 100 = 200 . The
reverse reaction, the donation of electrons from B to A, has a positive G and thus is
unfavorable under standard conditions. Remember that, by definition, the concentrations
of A and its redox pair A are equal under standard conditions; likewise, the concentration
of B is equal to the concentration of its redox pair B. B will be able to donate electrons
to A only when [B]>[B] and/or [A]<[A] to such an extent that the G for electron
transfer becomes negative.
14-22 An electron bound to a molecule with low affinity for electrons is a high energy electron.
Transfer of an electron from a molecule with low affinity to one with higher affinity has a
negative G and is thus favorable under standard conditions. If the reduced form of a
redox pair is a strong electron donor with a low affinity for electrons, it is easily oxidized;
the oxidized member of such a redox pair is a weak electron acceptor.
14-23 (b), (e) In order for a reaction to drive ATP synthesis under standard conditions, the G
of the reaction must be less than 7.3 kcal/mol. Since G = n (0.023) E0, the
value of E0 must be greater than 317 mV/n, where n is the number of electrons
transferred. E0 is 130 mV for the reduction of a molecule of pyruvate by
NADH, 390 mV for the reduction of a molecule of cytochrome b by NADH, 40
mV for the reduction of a molecule of cytochrome b by ubiquinone, 200 mV for
the oxidation of a molecule of ubiquinone by cytochrome c, and 590 mV for the
oxidation of cytochrome c by oxygen. The number of electrons transferred in each
of the above reactions is 2, 1, 1, 1, and 1, respectively. Thus only reactions (b) and
(e) are sufficient to drive ATP synthesis.
14-24 True. The flow of electrons down the respiratory chain moves from carriers with lower
affinity for electrons to ones with higher affinity and finally to oxygen, which has the
highest affinity of all. Thus, the relative positions of cytochromes and iron-sulfur centers
make sense in terms of the natural flow of electrons to oxygen.
242

14-25 Choice (e) is the correct answer. Cytochrome oxidase, which is the last carrier in the
mitochondrial electron-transport chain and thus has the highest redox potential, contains
copper ions and a heme group. Ubiquinone is not a protein and does not contain a metal
group (choice (a)). Both 2Fe2S and 4Fe4S centers carry one electron (choice (b)). Ironsulfur centers generally have a lower redox potential than do cytochromes (choice (c)).
The heme group in cytochrome c contains a charged iron ion. The interiors of proteins are
often hydrophobic, favoring a relatively high redox potential, since reduction of the iron
ion decreases its charge, and charges are energetically unfavorable in a hydrophobic
environment. Mutation of hydrophobic residues near the heme group of cytochrome c to
acidic (negatively charged) amino acids will neutralize the charge on the iron atom and
most likely decrease the redox potential, by making the uptake of an electron less
energetically favorable (choice (d)).
14-26 (c)
14-27 This result allows you to order the electron carriers in the respiratory chain because when
oxygen is added, the last carrier in the chain will be oxidized first. This is because
oxygen is the final sink for the electrons that flow through the chain, and it participates
directly in a redox reaction with the last electron carrier. The wave of oxidation will then
proceed backwards through the chain toward the first electron carrier in the chain; this is
because the oxidation of each carrier will convert it to a form that can accept electrons
from the upstream carrier in the chain, thereby oxidizing sequentially each upstream
carrier. The order of cytochromes in the respiratory chain is the reverse of the order in
which they are oxidized (i.e., the order in which the reduced form is lost). Listed from
first to last, the cytochromes in the chain are b, c1, c, (a + a3).
14-28 Oxygen is held tightly by the cytochrome oxidase of the respiratory chain until it has
accepted the four electrons (plus the 4 H+) required to convert it to 2 molecules of water.
Superoxide is therefore never released into the cellular milieu.
14-29 A.
B.

C.

Cytochrome b, quinone, the NADH dehydrogenase complex, and NADH will


accumulate in the reduced form.
Cytochrome c1, cytochrome c, and the cytochrome oxidase complex are more
likely to be oxidized because they will still be able to donate electrons to O2, but
will not receive any more electrons from the early part of the pathway.
The ATP:ADP ratio will fall because electron transport, the source of most
cellular ATP, will be stopped.

14-30 No, protons are pumped across different membranes in the two organelles. Protons are
pumped across the inner membrane into the intermembrane space in mitochondria. By
contrast, they are pumped across the thylakoid membrane into the thylakoid space in
chloroplasts. Yes, ATP is synthesized in the corresponding compartments in the two
organelles: in the matrix in mitochondria and in the stroma in chloroplasts.

243

14-31 (a), (b), and (e) The thylakoid spaces are continuous with one another, and grana are
merely stacks of thylakoids. Ions can flow from one thylakoid space to another
and therefore from the interior of one granum to another. Since the outer
membrane is porous, ions are also able to freely diffuse from the cytosol to the
intermembrane space.
14-32 AB; BC; CB; DC; EB; FC; GM; HM; IB; JC; KM
14-33 Light reaction: H2O + NADP+ + H+ + ADP + Pi O2 + NADPH + ATP. Dark reaction:
CO2 + NADPH + ATP (CH2O) + NADP+ + H+ + ADP + Pi. (CH2O) here indicates a
one-carbon unit of a carbohydrate; other ways of representing the sugar produced are also
valid.
14-34 (b)

The rate of photosynthesis will increase with increasing light intensity until all of
the reaction centers are being hit directly by photons. At saturating levels of light,
the rate of photosynthesis is limited by the number of reaction centers that are still
capable of being excited and hence can be increased only by increasing the
number of reaction centers or by increasing the rate at which the reaction centers
are restored to their low energy state. Increasing the number of chlorophyll
molecules in the antennae complexes, the energy per photon of light, or the rate at
which chlorophyll molecules are able to transfer energy electrons to one another
will have no effect on either of these parameters. Adding a powerful oxidizing
agent might, if anything, interfere with the reduction of the reaction center back to
its resting state.

14-35 (c)

If you now inhibit photosystem II, you will deprive plastoquinone, which can still
donate its electrons to the b6-f complex, of an electron source. Hence,
plastoquinone will accumulate in its oxidized form. In contrast, all of the other
components downstream of plastoquinone will be able to cycle between their
oxidized and reduced states. ATP synthesis will continue, since electrons are still
being fed through the b6-f complex, and in fact the same amount of ATP will be
generated.

14-36 In the carbon fixation process in chloroplasts, carbon dioxide is initially added to the
sugar ribulose 1,5-bisphosphate. The final product of carbon fixation in chloroplasts is
the three-carbon compound glyceraldehyde 3-phosphate. This is converted into
pyruvate (which can be directly used by the mitochondria), into sucrose (which is
exported to other cells), and into starch (which is stored in the stroma). The carbon
fixation cycle requires energy in the form of ATP and reducing power in the form of
NADPH.

244

14-37 (e)

Three molecules of O2 are required to form 3 molecules of 3-phosphoglycerate


and 3 molecules of phosphoglycolate. In order to break even (i.e., simply to keep
the Calvin cycle going with no net sugar produced), you need to have enough 3phosphoglycerate to synthesize ribulose 1,5-bisphosphate again. Therefore, for
every 3 molecules of O2 that react with ribulose 1,5-bisphosphate, you need to
generate 2 additional molecules of 3-phosphoglycerate. For every 3 molecules of
CO2 that go into the Calvin cycle, one molecule of 3-phosphoglycerate is formed.
So if you have at least 6 molecules of CO2 per 3 molecules of O2 going through
the Calvin cycle, you will break even. Only if you have a ratio of CO2 to O2
higher than 6:3 (2:1) can you have a net synthesis of carbohydrate.

14-38 In the absence of light and presence of CO2, ribulose 1,5-bisphosphate will decrease and
3-phosphoglycerate will increase. The presence of CO2 allows ribulose 1,5-bisphosphate
to be converted to 3-phosphoglycerate, but in the absence of light (and therefore in the
presence of reduced amounts of NADPH and ATP), 3-phosphoglycerate will accumulate
because subsequent reactions require NADPH and ATP. In the absence of CO2, ribulose
1,5-bisphosphate will accumulate (and 3-phosphoglycerate will decrease) because its
conversion to 3-phosphoglycerate is dependent on CO2.
14-39 C

Mitochondria are most closely related to Bacillus, and chloroplasts to


cyanobacteria. Maize (a eucaryote) is more closely related to Giardia (a simple
eucaryote) than it is to bacteria (procaryotes).

14-40 (a), (c) Methanococcus jannaschii is an archaebacterium that lives in very hot, anaerobic
thermal vents that are thought to resemble the conditions under which the first
cells lived. Like many organisms that live under these unpleasant conditions,
Methanococcus jannaschii seems to be more closely phylogenetically related to
the proposed ancestor cell than it is to most other organisms. However, the
genome of Methanococcus is perhaps 18 times larger than the genome of the
suspected ancestor cell, which may have had only 100 or so genes at most. The
reactions that Methanococcus uses in carbon fixation and energy generation use
many proteins that are different from those used by chloroplasts, cyanobacteria,
mitochondria, and aerobic bacteria.
14-41 Nitrogen fixation is the conversion of gaseous nitrogen (N2) from the environment into a
form (like ammonia, NH3) that cells can use for biosynthesis of nitrogenous
biomolecules, for example, the production of the bases in nucleotides. It is analogous to
carbon fixation, which is the conversion of carbon atoms in gaseous CO2 into a form (like
sugar) that can likewise be used for biosynthesis.

245

CHAPTER 15
INTRACELLULAR COMPARTMENTS
AND TRANSPORT
2009 Garland Science Publishing
3rd Edition

Membrane-Enclosed Organelles
15-1 Name the membrane-bounded compartments in a eucaryotic cell where each of the
functions listed below takes place.
A. Photosynthesis
B. Transcription
C. Oxidative phosphorylation
D. Modification of secreted proteins
E. Steroid hormone synthesis
F. Degradation of worn-out organelles
G. New membrane synthesis
H. Breakdown of lipids and toxic molecules

247

15-2

Label the structures of the cell indicated by the lines on the figure below:

Figure Q15-2
A.
B.
C.
D.
E.
F.
G.
H.
I.
J.
15-3

nucleus
free ribosomes
rough endoplasmic reticulum
Golgi apparatus
cytosol
endosome
plasma membrane
lysosome
mitochondrion
peroxisome

You discover a fungus that contains a strange star-shaped organelle not found in any
other eucaryotic cell you have seen. On further investigation you find the following
1.
the organelle possesses a small genome in its interior.
2.
the organelle is surrounded by two membranes.
3.
vesicles do not pinch off the organelle membrane.
4.
the interior of the organelle contains proteins similar to those of many bacteria.
5.
the interior of the organelle contains ribosomes.
How might this organelle have arisen?

248

Protein Sorting
15-4

For each of the following sentences, fill in the blanks with the best word or phrase
selected from the list below. Not all words or phrases will be used; each word or phrase
should be used only once.
Plasma membrane proteins are inserted into the membrane in the
__________________. The address information for protein sorting in a
eucaryotic cell is contained in the __________________ of the proteins.
Proteins enter the nucleus in their __________________ form. Proteins
that remain in the cytosol do not contain a __________________.
Proteins are transported into the Golgi apparatus via
__________________. The proteins transported into the endoplasmic
reticulum by __________________ are in their __________________
form.
amino acid sequence
endoplasmic reticulum
folded

15-5

Golgi apparatus
plasma membrane
protein translocators

sorting signal
transport vesicles
unfolded

What would happen in each of the following cases? Assume in each case that the protein
involved is a soluble protein, not a membrane protein.
A.
You add a signal sequence (for the ER) to the amino-terminal end of a normally
cytosolic protein.
B.
You change the hydrophobic amino acids in an ER signal sequence into charged
amino acids.
C.
You change the hydrophobic amino acids in an ER signal sequence into other,
hydrophobic, amino acids.
D.
You move the amino-terminal ER signal sequence to the carboxyl-terminal end of
the protein.

249

15-6

You are trying to identify the peroxisome-targeting sequence in the thiolase enzyme from
yeast. The thiolase enzyme normally resides in the peroxisome and therefore must
contain amino acid sequences that are used to target the enzyme for import into the
peroxisome. To identify the targeting sequences, you create a set of hybrid genes that
encode fusion proteins containing part of the thiolase protein fused to another protein,
histidinol dehydrogenase (HDH). HDH is a cytosolic enzyme required for the synthesis
of the amino acid histidine and cannot function if it is localized in the peroxisome. You
genetically engineer a series of yeast cells to express these fusion proteins instead of their
own versions of these enzymes. If the fusion proteins are imported into the peroxisome,
the HDH portion of the protein cannot function and the yeast cells cannot grow on media
lacking histidine. You obtain the following results:

Figure Q15-6
What region of the thiolase protein contains the peroxisomal targeting sequence? Explain
your answer.

250

15-7

What is the role of the nuclear localization sequence in a nuclear protein?


(a)
It is bound by cytoplasmic proteins that direct the nuclear protein to the nuclear
pore.
(b)
It is a hydrophobic sequence that enables the protein to enter the nuclear
membranes.
(c)
It aids protein unfolding in order for the protein to thread through nuclear pores.
(d)
It prevents the protein diffusing out of the nucleus via nuclear pores.

15-8

A gene regulatory protein, A, contains a typical nuclear localization signal but


surprisingly is usually found in the cytosol of cells. When the cell is exposed to
hormones, protein A moves from the cytosol into the nucleus where it turns on genes
involved in cell division. When you purify protein A from cells that have not been
treated with hormones, you find that protein B is always complexed with it. To
determine the function of protein B, you engineer cells lacking the gene for protein B.
You compare normal and defective cells by using differential centrifugation to separate
the nuclear fraction from the cytoplasmic fraction and then separate the proteins in these
fractions by gel electrophoresis. You identify the presence of protein A and protein B by
looking for their characteristic bands on the gel. The gel you run is shown below:

Figure Q15-8
On the basis of these results, what is the function of protein B? Explain your conclusion
and propose a mechanism for how protein B works.
15-9

Which of the following statements about import of proteins into mitochondria are TRUE?
(a)
The signal sequences on mitochondrial proteins are usually carboxyl terminal.
(b)
The first stage of import of a mitochondrial protein is across the outer membrane
into the intermembrane space.
(c)
Most mitochondrial proteins are not imported from the cytosol but are synthesized
inside the mitochondria.
(d)
Mitochondrial proteins are translocated across the inner and outer membranes
simultaneously.
(e)
Mitochondrial proteins cross the membrane in their native, folded state.

251

15-10 Proteins destined to enter the endoplasmic reticulum


(a)
are transported across the membrane after their synthesis is complete.
(b)
are synthesized on free ribosomes in the cytosol.
(c)
begin to cross the membrane while still being synthesized.
(d)
cross the membrane in a folded state.
(e)
all remain within the endoplasmic reticulum.
15-11 After isolating the rough endoplasmic reticulum from the rest of the cytoplasm, you
purify the RNAs attached to it. Which of the following proteins do you expect the RNA
from the rough endoplasmic reticulum to encode?
(a)
Soluble secreted proteins
(b)
ER membrane proteins
(c)
Mitochondrial membrane proteins
(d)
Plasma membrane proteins
(e)
Ribosomal proteins
15-12 Briefly describe the mechanism by which the presence of an internal stop-transfer
sequence in a protein causes the protein to become embedded in the lipid bilayer as a
transmembrane protein with a single membrane-spanning region. Assume that the protein
has an amino terminal signal sequence and just one internal hydrophobic stop-transfer
sequence.
15-13 Using genetic engineering techniques, you have created a set of proteins that contain two
(and only two) conflicting signal sequences that specify different compartments. Predict
which signal would win out for the following combinations. Explain your answers.
A.
Signals for import into the nucleus and import into the ER.
B.
Signals for export from the nucleus and import into the mitochondria.
C.
Signals for import into mitochondria and retention in the ER.
15-14 A protein traverses the plasma membrane three times in the orientation shown below
(N = amino terminus, C = carboxyl terminus; the hydrophobic membrane-spanning
regions are shown as open boxes). This protein is known to have a signal sequence that
is cleaved by signal peptidase in the ER.

Figure Q15-14
Sketch the ER membrane and the arrangement of the newly synthesized protein chain
after it has completed its entry into the ER membrane but before any action of signal
peptidase. Be sure to label the cytosol, the ER lumen, the signal sequence, and the amino
and carboxyl termini of the protein in your diagram.
252

15-15 The figure below shows the orientation of a multipass transmembrane protein after it has
completed its entry into the ER membrane (part A) and after it gets delivered to the
plasma membrane (part B). This protein has an amino-terminal signal sequence
(depicted as the dark grey membrane spanning box), which is cleaved off in the
endoplasmic reticulum by signal peptidase. The other membrane-spanning domains in
the protein are depicted as open boxes. Given that any hydrophobic membrane-spanning
domain can act as either a start-transfer or a stop-transfer region, draw the final
consequences of the actions described below on the orientation of the protein in the
plasma membrane. Be sure to indicate on your drawing the extracellular space, the
cytosolic face, and the plasma membrane, as well as the amino- and carboxyl-termini of
the protein.

Figure Q15-15
A.
B.

Deleting the first signal sequence.


Changing the hydrophobic amino acids in the first, cleaved, sequence to charged
amino acids.
C.
Changing the hydrophobic residues in every other transmembrane sequence to
charged residues, starting with the first, cleaved, signal sequence.
15-16 Examine the multipass transmembrane protein shown in Figure Q15-16. What would
you predict would be the effect of converting the first hydrophobic transmembrane
segment to a hydrophilic segment? Sketch the arrangement of the modified protein in the
ER membrane.

Figure Q15-16

253

Vesicular Transport
15-17 For each of the following sentences, fill in the blanks with the best word or phrase
selected from the list below. Not all words or phrases will be used; each word or phrase
should be used only once.
Proteins are transported out of a cell via the __________________ or
__________________ pathway. Fluids and macromolecules are
transported into the cell via the __________________ pathway. All
proteins being transported out of the cell pass through the
__________________ and the __________________. Transport vesicles
link organelles of the __________________ system. The formation of
__________________ in the endoplasmic reticulum stabilizes protein
structure.
carbohydrate
disulfide bonds
endocytic
endomembrane
endoplasmic reticulum
endosome
exocytic

Golgi apparatus
hydrogen bonds
ionic bonds
lysosome
protein
secretory

15-18 Name two functions of the protein coat of vesicles that bud from membranous organelles
used in vesicular transport.
15-19 An individual transport vesicle
(a)
contains only one type of protein in its lumen.
(b)
will fuse with only one type of membrane.
(c)
is endocytic if it is traveling toward the plasma membrane.
(d)
is enclosed by a membrane with the same lipid and protein composition as the
membrane of the donor organelle.

254

15-20 In class we have discussed how v-SNAREs and t-SNARES mediate the recognition of a
vesicle with its target membrane so that a vesicle displaying a particular type of vSNARE will only fuse with a target membrane containing a complementary type of tSNARE. It is also known that in some cases, v-SNAREs and t-SNAREs may also
mediate fusion of identical membranes. In yeast cells, right before the formation of a
new cell, vesicles derived from the vacuole will come together and fuse to form a new
vacuole destined for the new cell. Unlike the situation weve discussed in class, the
vacuolar vesicles contain both v-SNAREs and t-SNAREs. Your friend is trying to
understand the role of these SNAREs in the formation of the new vacuole and wants to
consult with you regarding the interpretation of his data.
Your friend has designed an ingenious assay for the fusion of vacuolar vesicles utilizing
alkaline phosphatase. The protein alkaline phosphatase is made in a pro form that
must be cleaved in order for the protein to be active. Your friend has designed two
different strains of yeast: strain A produces the pro form of alkaline phosphatase (proPase), while strain B produces the protease that can cleave pro-Pase into the active form
(Pase). Neither strain has the active form of the alkaline phosphatase, but when vacuolar
vesicles from the strains A and B are mixed, fusion of vesicles generates active alkaline
phosphates, whose activity can be measured and quantified.

Figure Q15-20A
Your friend has taken each of these yeast strains and further engineered them so that they
express only the v-SNAREs, the t-SNAREs, both (the normal situation), or neither
SNARE. He then isolates vacuolar vesicles from all strains and tests the ability of each
variant form of strain A to fuse with each variant form of strain B, using the alkaline
phosphatase assay. The data are shown in the graph depicted in Figure Q15-20B. On
this graph, the SNARE present on the vesicle of the particular yeast strain is indicated as
v (for the presence of the v-SNARE) and t (for the presence of the t-SNARE).

255

Figure Q15-20 B
What does his data say about the requirements for v-SNAREs and t-SNAREs in the
vacuolar vesicles? Be sure to comment on whether it is important to have a specific type
of SNARE (that is, v- or t-SNARE) on each vesicle.

Secretory Pathway
15-21 N-linked oligosaccharides on secreted glycoproteins are attached to
(a)
nitrogen atoms in the polypeptide backbone.
(b)
the serine or threonine in the sequence Asn-X-Ser/Thr.
(c)
the amino terminus of the protein.
(d)
the asparagine in the sequence Asn-X-Ser/Thr.
(e)
the aspartic acid in the sequence Asp-X-Ser/Thr.
15-22 Name two types of protein modification that can occur in the ER but not in the cytosol.
15-23 If you were to remove the ER-retention signal from a protein that normally resides in the
ER lumen, where do you expect the protein will ultimately end up? Be sure to explain
your reasoning, for full credit.

256

15-24 Match the set of labels below with the numbered label lines on Figure 15-24.

Figure Q15-24
A.
B.
C.
D.
E.

Cisterna
Golgi stack
Secretory vesicle
trans Golgi network
cis Golgi network

257

15-25 A plasma membrane protein carries an oligosaccharide containing mannose (Man),


galactose (Gal), sialic acid (SA), and N-acetylglucosamine (GlcNAc). These sugars are
added to the protein as it proceeds through the secretory pathway. First, a core
oligosaccharide containing Man and GlcNAc is added, followed by Gal, Man, SA, and
GlcNAc in a particular order. Each addition is catalyzed by a different transferase acting
at a different stage as the protein proceeds through the secretory pathway. You have
isolated mutants defective for each of the transferases, purified the membrane protein
from each of the mutants, and identified which sugars are present in each mutant protein.
The results are summarized in Table Q15-25.
Table Q15-25
Cell lacking:
A. Oligosaccharide
protein transferase
B. Galactose
transferase
C. SA transferase
D. GlcNAc
transferase

Sugars present in the purified protein


Man
Gal
SA
GlcNAc

+
+

+
less than in
normal cells

From these results, match each of the transferases (A, B, C, D) to its subcellular location
selected from the list below. (Assume that each location contains only one enzyme.)
1.
2.
3.
4.

Central Golgi cisternae


cis Golgi network
ER
trans Golgi network

15-26 For each of the following sentences, choose one of the options enclosed in square
brackets to make a correct statement.
New plasma membrane reaches the plasma membrane by the
[regulated/constitutive] exocytosis pathway. New plasma membrane
proteins reach the plasma membrane by the [regulated/constitutive]
exocytosis pathway. Insulin is secreted from pancreatic cells by the
[regulated/constitutive] exocytosis pathway. The interior of the trans
Golgi network is [acidic/alkaline]. Proteins that are constitutively secreted
[aggregate/do not aggregate] in the trans Golgi network.
15-27 In a cell capable of regulated secretion, what are the three main classes of proteins that
must be separated before they leave the trans Golgi network?

258

Endocytic Pathways
15-28 Name three possible fates for an endocytosed molecule that has reached the endosome.
15-29 For each of the following sentences, fill in the blanks with the best word or phrase
selected from the list below. Not all words or phrases will be used; each word or phrase
should be used only once.
Eucaryotic cells are continually taking up materials from the extracellular
space by the process of endocytosis. One type of endocytosis is
__________________, which involves utilizing __________________
proteins to form small vesicles containing fluids and molecules. After
these vesicles pinch off from the plasma membrane, they will fuse with
the __________________, where the uptaken materials are sorted. A
second type of endocytosis is __________________, which is used to take
up large vesicles that can contain microorganisms and cellular debris.
Macrophages are especially suited for this process, as they extend
__________________ (sheetlike projections of their plasma membrane) to
surround the invading microorganisms.
chaperone
cholesterol
clathrin
endosome

Golgi apparatus
mycobacterium
phagocytosis
pinocytosis

259

pseudopods
rough ER
SNARE
transcytosis

15-30 Fibroblast cells from patients W, X, Y, and Z, who each have a different inherited defect,
all contain inclusion bodies, which are lysosomes filled with undigested material. You
wish to identify the cellular basis of these defects. The possibilities are:
1.
2.
3.

a defect in one of the lysosomal hydrolases.


a defect in the phosphotransferase that is required for mannose-6-phosphate
tagging of the lysosomal hydrolases.
a defect in the mannose-6-phosphate receptor, which binds mannose-6-phosphate
tagged lysosomal proteins in the trans Golgi network and delivers them to
lysosomes.

You find that when some of these mutant fibroblasts are incubated in media in which
normal cells have been grown, the inclusion bodies disappear. This leads you to suspect
that lysosomal hydrolases are being secreted by the constitutive exocytic pathway in
normal cells and are being taken up by the mutant cells. (It is known that some mannose6-phosphate receptor molecules are found in the plasma membrane and can take up and
deliver lysosomal proteins via the endocytic pathway.) You incubate cells from each
patient with media from normal cells and media from each of the other mutant cell
cultures, and get the following results.

Cell Line
Normal
W
X
Y
Z

From
normal
cells

From
cultures
of W cells

Media
From
cultures
of X cells

+
+
+

+
+
+

From
cultures
of Y cells

From
cultures
of Z cells

+ indicates that the cells appear normal; indicates that the cells still have inclusion
bodies.
For each patient (W, X, Y, Z) indicate which of the defects (1, 2, 3) they are most likely
to have.
15-31 How is it that the low pH of lysosomes protects the rest of the cell from lysosomal
enzymes in case the lysosome breaks?

260

How We Know: Tracking Protein and Vesicle Transport


15-32 You have created a GFP fusion to a protein that is normally secreted from yeast cells.
Since you have learned about the use of temperature-sensitive mutations in yeast to study
protein and vesicle transport, you obtain a collection of three mutant yeast strains, each
one defective in some aspect of the protein secretory process. Being a good scientist, you
of course, also obtain a wild-type control strain. You decide to examine the fate of your
GFP fusion protein in these various yeast strains and engineer the mutant strains to
express your GFP fusion protein. However, in your excitement to do the experiment, you
realize that you did not label any of the mutant yeast strains and no longer know which
strain is defective in what process. You end up numbering your strains with the numbers
1 through 4, and then you carry out the experiment anyway, obtaining the following
results (note that the black dots represent your GFP fusion protein):

Figure Q15-32
Name the process defective in each of these strains. Remember that one of these strains
is your wild-type control.

261

Answers
15-1

A
B.
C.
D.
E.
F.
G.
H.

15-2

Photosynthesis = chloroplast
Transcription = nucleus
Oxidative phosphorylation = mitochondrion
Modification of secreted proteins = Golgi apparatus and rough endoplasmic
reticulum (ER)
Steroid hormone synthesis = smooth ER
Degradation of worn-out organelles = lysosome
New membrane synthesis = ER
Breakdown of lipids and toxic molecules = peroxisome

See Figure A15-2.

Figure A15-2
15-3

A genome, a double membrane, ribosomes, and proteins similar to those found in bacteria
are evidence for an organelle having evolved from an engulfed bacterium.

15-4

Plasma membrane proteins are inserted into the membrane in the endoplasmic
reticulum. The address information for protein sorting in a eucaryotic cell is contained
in the amino acid sequence of the proteins. Proteins enter the nucleus in their folded
form. Proteins that remain in the cytosol do not contain a sorting signal. Proteins are
transported into the Golgi apparatus via transport vesicles. The proteins transported into
the endoplasmic reticulum by protein translocators are in their unfolded form.

262

15-5

A.
B.
C.
D.

15-6

The protein will now be transported into the ER lumen.


The altered signal sequence will not be recognized and the protein will remain in
the cytosol.
The protein will still be delivered into the ER. It is the distribution of hydrophobic
amino acids that is important, not the actual sequence.
The protein will not enter the ER. Because the carboxyl terminus of the protein is
the last part to be made, the ribosomes synthesizing this protein will not be
recognized by the SRP and carried to the ER.

The peroxisomal targeting sequence lies between amino acids number 100 and number
125. Any fusion protein containing this sequence can be targeted for import into the
peroxisome (because the yeast cannot grow on media lacking histidine) while the fusion
proteins lacking this region do not target the fusion protein for import into the
peroxisome (because the yeast do grow on media lacking histidine). The most important
pieces of data are the fusion proteins containing amino acids 100200 of the thiolase
protein fused to HDH and the fusion protein containing amino acids 1125 of the thiolase
protein fused to HDH. Both of these fusion proteins do not allow growth on media
lacking histidine and can be used to define the minimal region necessary for targeting
thiolase for import into the peroxisome.
(Note that although these experiments show that amino acids 100125 are necessary,
these experiments do not show that this region is sufficient for peroxisomal targeting. It
is possible that amino acids 100125 is sufficient, or, it could be that this region
collaborates with redundant signals between amino acids 1100 or 125200.)

15-7

(a)

15-8

The data on the gel shows that protein A is always found in the nucleus in the absence of
protein B. Therefore, any mechanism that is proposed must explain this result.
On possible answer is that protein B binds protein A and masks the nuclear localization
signal. In the presence of hormone, protein B interacts with the hormone, which changes
its conformation so that it can no longer bind protein A. When protein B no longer binds
to protein A, the nuclear localization signal on protein A is now exposed and protein A
can enter the nucleus. Therefore, in the absence of protein B, the nuclear localization
signal on protein A is always exposed and protein A resides in the nucleus.
Another possible answer is that protein B binds protein A and sequesters it by keeping
protein A in some subcellular compartment, away from the nucleus. In the presence of
hormone, protein B interacts with the hormone, changing its conformation so that it can
no longer bind to protein A. When protein B is not present, protein A can enter the
nucleus in the presence or absence of hormone.

15-9

(d)

15-10 (c)
263

15-11 (a), (b), and (d) The rough ER consists of ER membranes and polyribosomes that are in
the process of translating and translocating proteins into the ER membrane and
lumen. Thus all proteins that end up in the lysosome, Golgi apparatus, or plasma
membrane, or are secreted, will be encoded by the RNAs associated with the
rough ER. Mitochondrial and ribosomal proteins are translated on free cytosolic
ribosomes.
15-12 The amino-terminal signal sequence initiates translocation and the protein chain starts to
thread through the translocation channel. When the stop-transfer sequence enters the
translocation channel, the channel discharges both the signal sequence and the stoptransfer sequence sideways into the lipid bilayer. The signal sequence is then cleaved, so
that the protein remains held in the membrane by the hydrophobic stop-transfer sequence.
15-13 A.

B.

C.

The protein would enter the ER. The signal for a protein to enter the ER is
recognized as the protein is being synthesized and will end up either in the ER or
on the ER membrane. Proteins destined for the nucleus get recognized by
cytosolic nuclear transport proteins once they are fully synthesized and fully
folded.
The protein would enter in the mitochondria. In order for a nuclear export signal
to work, the protein would have to end up in the nucleus first and thus would need
a nuclear import signal for the nuclear export signal to get utilized.
The protein would enter the mitochondria. In order to be retained in the ER, the
protein needs to enter the ER. Since there is no signal for ER import, the ER
retention signal would not function.

15-14 The N-terminal signal sequence initiates translocation of the protein across the ER
membrane. The signal sequence will be cleaved off by signal peptidase, leaving the
amino-terminus of the protein in the luminal side of the ER membrane. Upon fusion to
the plasma membrane, the amino terminus of the protein will reside in the extracellular
space.

Figure A15-14

264

15-15 A.

Deleting the first signal sequence completely would convert the next membranespanning domain into an internal start-transfer signal and would invert the
orientation of the protein (see Figure A15-15A).

B.

Changing the hydrophobic amino acids to charged amino acids destroys the
ability of the sequence both to act as a signal sequence and to become a
membrane-spanning sequence. Therefore, the adjacent membrane spanning
domain will now become an internal start-transfer sequence and the protein will
be inverted, as seen above in part A. The mutated signal sequence would not get
cleaved off, since it would remain on the cytoplasmic side of the membrane and
signal peptidase is found only inside the ER (see Figure A15-15B).

C.

Mutating every other membrane spanning region so that they are now charged
(and thus cannot span the membrane) would decrease the number of
transmembrane regions and increase the size of the internal loops between
membrane-spanning regions (see Figure 15-15C).

Figure A15-15
15-16 As shown in Figure A15-16, elimination of the first transmembrane segment (by making
it hydrophilic) would be expected to reverse the orientation of the protein in the
membrane. What originally was the second transmembrane segment (a stop-transfer
signal), would now be read as a start-transfer signal and would have the opposite
orientation in the membraneas would all the remaining transmembrane segments.
Although the N-terminus would still be in the ER lumen, all the rest of the external parts
of the protein would swap positions so that what was in the cytosol would now be in the
ER lumen, and vice versa.

Figure A15-16
265

15-17 Proteins are transported out of a cell via the secretory or exocytic pathway. Fluid and
macromolecules are transported into the cell via the endocytic pathway. All proteins
being transported out of the cell pass through the endoplasmic reticulum and the Golgi
apparatus. Transport vesicles link organelles of the endomembrane system. The
formation of disulfide bonds in the endoplasmic reticulum stabilizes protein structure.
15-18 1.
2.

The proteins in the coat help shape the membrane into a bud.
The proteins in the coat can also select cargoes for transport.

15-19 Choice (b) is the correct answer. An individual vesicle may contain more than one type of
protein in its lumen (choice (a)), all of which will contain the same sorting signal (or will
lack specific sorting signals). Endocytic vesicles (choice (c)) generally move away from
the plasma membrane. The vesicle membrane will not necessarily contain the same lipid
and protein composition as the donor organelle, since the vesicle is formed from a
selected subset of the organelle membrane from which it budded (choice (d)).
15-20 In order to get maximal levels of vacuolar vesicle fusion, vesicles from each strain must
carry both v-SNAREs and t-SNARES. Experiment 1, which represents the normal
scenario, is the only experiment where 100% alkaline phosphatase activity is measured.
However, as long as complementary SNAREs are present on the vesicles, some vesicle
fusion does occur (see experiments 3, 4, 6, 7, 8, 9). If both vesicles are missing either vSNAREs (experiment 2) or t-SNAREs (experiment 5) or both SNAREs (experiment 10
and 11), the level of fusion is very low. It does not matter whether a t- or v-SNARE is on
the vesicle of a particular strain, as long as the vesicle from the other strain contains a
complementary SNARE (compare experiments 3 and 4, 6 and 7, and 8 and 9).
15-21 (d)
15-22 1.
2.

Proteins in the ER can undergo disulfide bond formation. (This does not occur in
the cytosol because of its reducing environment.)
Proteins in the ER can undergo glycosylation. (Glycosylating enzymes are not
found in the cytosol.)
(Signal-sequence cleavage is also an acceptable answer, although not really what
this question is referring to.)

15-23 The protein would end up in the extracellular space. Normally, the protein would go
from the ER to the Golgi apparatus, get captured because of its ER-retrieval signal, and
return to the ER. However, without the ER-retrieval signal, the protein would evade
capture, ultimately leave the Golgi via the default pathway, and become secreted into the
extracellular space. The protein would not be retained anywhere else along the secretory
pathway, as it presumably has no signals to promote such localization since it normally
resides in the ER lumen.
15-24 A3; B1; C5; D4; E2

266

15-25 A3 (oligosaccharide protein transferase = ER)


B1 (galactose transferase = central Golgi cisternae)
C4 (SA transferase = trans Golgi network)
D2 (GlcNAc transferase = cis Golgi network
Proteins are modified in a stepwise fashion in the Golgi apparatus, with early steps taking
place in the cis Golgi, intermediate steps taking place in the central Golgi cisternae, and
late steps occurring in the trans Golgi network. If each enzyme produces the substrate for
the next step, then a mutant lacking the enzyme that catalyzes the addition of the first
sugar will be missing all of the sugars, a mutant lacking the enzyme that catalyzes the
addition of the second sugar will contain the first sugar but will lack the other three, and
so on. By this logic, mannose and GlcNAc must be the first sugars added, additional
GlcNAc the second, galactose the third, and SA the last. Hence, the oligosaccharide
protein transferase must be in the ER, the GlcNAc transferase in the cis Golgi, the
galactose transferase in the central Golgi, and the SA transferase in the trans Golgi.
15-26 New plasma membrane reaches the plasma membrane by the constitutive exocytosis
pathway. New plasma membrane proteins reach the plasma membrane by the
constitutive exocytosis pathway. Insulin is secreted from pancreatic cells by the
regulated exocytosis pathway. The interior of the trans Golgi network is acidic.
Proteins that are constitutively secreted do not aggregate in the trans Golgi network.
15-27 The three main classes of proteins that must be sorted before they leave the trans Golgi
network in a cell capable of regulated secretion are (1) those destined for lysosomes, (2)
those destined for secretory vesicles, and (3) those destined for immediate delivery to the
cell surface.
15-28 1.
2.
3.

recycled to the original membrane


destroyed in the lysosome
transcytosed across the cell to a different membrane.

15-29 Eucaryotic cells are continually taking up materials from the extracellular space by the
process of endocytosis. One type of endocytosis is pinocytosis, which involves utilizing
clathrin proteins to form small vesicles containing fluids and molecules. After these
vesicles pinch off from the plasma membrane, they will fuse with the endosome, where
the uptaken materials are sorted. A second type of endocytosis is phagocytosis, which is
used to take up large vesicles that can contain microorganisms and cellular debris.
Macrophages are especially suited for this process, as they extend pseudopods (sheetlike
projections of their plasma membrane) to surround the invading microorganisms.

267

15-30 W3 (defect in mannose-6-phosphate receptor)


X2 (defect in phosphotransferase)
Y1; Z1 (defect in lysosomal hydrolases); these will be defects in two different
lysosomal acid hydrolases.
A cell that has no mannose-6-phosphate receptor will be able to make all the lysosomal
hydrolases properly, but will not be able to send them to the lysosome and will also not
be able to scavenge hydrolases from the external media. Hence, this cell line cannot be
rescued by culture media that has had lysosomal hydrolases secreted into it and thus will
not be rescued by any of the media tested here. A cell line that has no phosphotransferase
will be able to scavenge hydrolases from the external medium, but since all of the cells
own hydrolases will lack the mannose-6-phosphate tag, it will be rescued only by media
from a cell line that is able to make all of the hydrolases. Cell lines missing one hydrolase
will be rescued by media from any cell line that is able to secrete that hydrolase in a
mannose-6-phosphate tagged form; in addition, media from cultures of cells missing a
hydrolase will rescue any cell line with another type of defect.
15-31 The lysosomal enzymes are all acid hydrolases, which have optimal activity at the low
pH (about 5.0) found in the interior of lysosomes. If a lysosome were to break, the acid
hydrolases would find themselves at pH 7.2, the pH of the cytosol, and would therefore
do little damage to cellular constituents.
15-32 Strain A has protein accumulating in the ER, which means that this cell has a mutation
that blocks transport from the ER to the Golgi apparatus. Strain B has secreted protein,
and therefore is your wild-type control. Strain C has protein accumulating in the Golgi
apparatus, and thus has a mutation that blocks exit of proteins from the Golgi apparatus.
Strain D has protein accumulating in the cis-Golgi network, and thus has a mutation that
blocks the travel of proteins through the Golgi cisternae.

268

CHAPTER 16
CELL COMMUNICATION
2009 Garland Science Publishing
3rd Edition

General Principles of Cell Signaling


16-1 Cell lines A and B both survive in tissue culture containing serum but do not proliferate.
Addition of Factor F stimulates proliferation of cell line A but not cell line B. Cell line A
produces a normal receptor protein (R), and, in order to see what the role of R might be, you
introduce this receptor protein into cell line B, using recombinant DNA techniques. You then
test all of your various cell lines in the presence of serum for their response to factor F, with the
results summarized in Table Q16-1.

Which of the following can be concluded from your results? Please explain your choices.
(a) Binding of Factor F to its receptor is required for proliferation of cell line A but not cell line
B.
(b) Binding of receptor R to its ligand is required for proliferation of cell line B but not cell line
A.
(c) Receptor R probably does not bind to Factor F.
(d) Cell line B does not normally have a receptor for Factor F.

269

16-2

For each of the following sentences, fill in the blanks with the best word or phrase
selected from the list below. Not all words or phrases will be used; each word or phrase
should be used only once.
Cells can signal to each other in various ways. A signal that must be
relayed to the entire body is most efficiently sent by
__________________ cells, which produce hormones that are carried
throughout the body through the bloodstream. On the other hand,
__________________ methods of cell signaling do not require the release
of a secreted molecule and are used for very localized signaling events.
During __________________ signaling, the signal remains in the
neighborhood of the secreting cell and thus acts as a local mediator on
nearby cells. Finally, __________________ signaling involves the
conversion of electrical impulses into a chemical signal. Cells receive
signals through a __________________, which can be an integral
membrane protein or can reside inside the cell.
amplification
contact-dependent
endocrine
epithelial

G-protein
K+ channel
neuronal
paracrine

phosphorylation
receptor
target

16-3

All members of the steroid hormone receptor family


(a)
are cell-surface receptors.
(b)
do not undergo conformational change.
(c)
are found only in the cytoplasm.
(d)
interact with signal molecules that diffuse through the plasma membrane.
(e)
regulate sexual development.

16-4

Can signaling through the steroid hormone/steroid hormone receptor pathway lead to
amplification of the original signal? If so, how?

16-5

When the neurotransmitter acetylcholine is applied to skeletal muscle cells, it binds the
acetylcholine receptor and causes the muscle cells to contract. Succinylcholine, which
has been demonstrated to bind the acetylcholine receptor on skeletal muscle cells, is a
muscle relaxant. Succinylcholine is a chemical analog of acetylcholine and is sometimes
used by surgeons as a muscle relaxant. Propose a model for why succinylcholine causes
muscle relaxation instead of contraction, as acetylcholine does.

16-6

The local mediator nitric oxide stimulates the intracellular enzyme guanylyl cyclase by
(a)
way of a G-protein-mediated mechanism.
(b)
way of a receptor tyrosine kinase.
(c)
diffusing into cells and stimulating the cyclase directly.
(d)
way of a cell-surface receptor linked to an intracellular signaling pathway.
(e)
activation of intracellular protein kinases.

270

16-7

Name the three main classes of cell-surface receptors.

16-8

Which of the following statements are TRUE?


(a)
All hydrophilic signaling molecules bind to cell-surface receptors that contain at
least one membrane-spanning domain.
(b)
All extracellular signaling molecules are transported across the plasma membrane
into the cytosol by their receptor.
(c)
A cell-surface receptor capable of binding only one natural ligand can mediate
only one kind of response.
(d)
Cells having the same set of receptors can respond in different ways to the same
ligand molecules.
(e)
Foreign substances that occupy receptor sites normally bound by signal molecules
always induce the same response that is produced by the natural ligand.

16-9

Intracellular steroid hormone receptors have binding sites for a signal molecule and a
DNA sequence. Give one reason to explain how different cells that have identical
intracellular steroid hormone receptors can activate different genes when the receptors
bind the same signal molecule.

16-10 Your friend is studying a segment of a newly discovered virus that carries an enhancer of
gene expression that confers glucocorticoid (a hormone) responsiveness on genes that are
linked to it. He constructs two versions of a reporter gene, one which has only a minimal
promoter linked to it (which contains sites for RNA polymerase binding), while the other
reporter gene has both a minimal promoter plus the viral enhancer attached to it. The
reporter gene allows him to measure the amount of transcription that occurs from each
construct. Your friend puts each of these constructs into two different cell lines and
examines the activity of the reporter gene in each cell line, as shown in the figure below.
He is puzzled by his data and asks for your help in interpreting the findings.

Figure Q16-10
A.
B.

From this data, can you tell if both cell lines contain glucocorticoid receptors?
Why?
Propose a possible explanation for the difference in the transcription of the
reporter gene in cell lines 1 and 2 after introduction of the construct containing the
viral enhancer in the presence of glucocorticoid.

271

G-Protein-Linked Receptors
16-11 Indicate by writing yes or no whether amplification of a signal could occur at the
particular steps described below. Explain your answers.
A.
An extracellular signal molecule binds and activates a cell surface receptor.
B.
The activated G-protein-linked receptors cause G to separate from G and G.
C.
Adenylyl cyclase produces cyclic AMP.
D.
cAMP activates protein kinase A.
E.
Protein kinase A phosphorylates target proteins.
16-12 When a trimeric G protein is activated by a cell-surface receptor
(a)
the subunit exchanges its bound GDP for GTP.
(b)
the GDP bound to the subunit is phosphorylated to form bound GTP.
(c)
it dissociates into a free subunit and an subunit.
(d)
the subunit exchanges its bound GDP for GTP.
(e)
it dissociates into an active subunit and an inactive subunit.

272

16-13 Acetylcholine acts at a G-protein-linked receptor on heart muscle to make the heart beat
+
more slowly by the effect of the G protein on a K channel, as shown in Figure Q16-13.
Which one or more of the following would enhance this effect of acetylcholine?

Figure Q16-13
(a)
(b)
(c)
(d)
(e)

A high concentration of a non-hydrolyzable analog of GTP.


Modification of the acetylcholine-receptor-linked G protein subunit by cholera
toxin.
Mutations in the acetylcholine receptor that weaken the interaction between the
receptor and acetylcholine.
Mutations in the acetylcholine receptor that weaken the interaction between the
receptor and the G protein.
Mutations in the G protein subunit that weaken the interaction between and
+
the K channel.

273

16-14 During the mating process, yeast cells respond to pheromones that are secreted from
other cells. These pheromones bind to G-protein-linked receptors on the surface of the
responding yeast cell and lead to activation of G proteins inside the cell. When a wildtype yeast cell senses the pheromone, its cellular physiology changes in preparation for
mating: the cell stops growing until a mating partner is found. If yeast cells do not
undergo the appropriate cellular responses upon sensation of pheromone, they are
considered sterile. Yeast cells that are defective in one or more components of the G
protein have characteristic phenotypes in the absence and presence of the pheromone,
which are listed in Table 16-14.
Table 16-14 Mating phenotypes of various mutant and nonmutant strains of yeast
Phenotype
MUTATION
MINUS PHEROMONE
PLUS PHEROMONE
None (wild type)
subunit deleted
subunit deleted
subunit deleted
and deleted
and deleted
and deleted

normal growth
arrested growth
normal growth
normal growth
normal growth
normal growth
normal growth

arrested growth, mating response


arrested growth, sterile
normal growth, sterile
normal growth, sterile
normal growth, sterile
normal growth, sterile
normal growth, sterile

Which of the following models is consistent with the data from the genetic analysis of
these mutants? Explain your answer.
(a)
G activates the mating response but is inhibited when bound by G.
(b)
G activates the mating response but is inhibited when bound by G.
(c)
The G trimer is inactive; either free G or free G is capable of activating
the mating response.
(d)
The G trimer is active; both free G and free G are required to inhibit the
mating response
(e)
The G trimer inhibits the mating response; free G and free G have no
activity.
16-15 Receipt of extracellular signals can cause changes in cellular physiology. These signals
cause changes by altering the cells cytoplasmic machinery. Some changes occur on a
very fast time scale (i.e., in less than a second) whereas other changes occur long after the
receipt of a signal (i.e., hours).
A.
Name an intracellular molecular change that could result in a quick change in cell
physiology.
B.
Name an intracellular molecular change that could lead to a slow change in
cellular physiology.
C.
Explain why the response you named in A results in a quick change while the
response you named in B results in a slow change.

274

16-16 A calmodulin-regulated kinase (CaM kinase) is involved in learning and memory. This
kinase is able to phosphorylate itself, and the kinase activity of the phosphorylated form
2+
is independent of the intracellular concentration of Ca . Thus the kinase stays active
2+
after Ca levels have dropped. Mice completely lacking this kinase have severe learning
defects but are otherwise normal.
A.
Each of the following mutations also leads to defective memory. For each case
explain the reason why.
(1)
A mutation that prevents the kinase from binding to ATP.
(2)
A deletion of the calmodulin-binding part of the kinase.
(3)
A mutation that destroys the site of autophosphorylation.
B.
What would be the effect on the kinase activity of CaM kinase if there were a
mutation that reduced the strength of binding of the kinase to the phosphatase
responsible for inactivating the kinase?
16-17 Activated G-protein-linked receptors activate G proteins by reducing the strength of GDP
binding to the subunit of the G protein, allowing GDP to dissociate and GTP, which is
present at much higher concentrations, to bind. How do you suppose the activity of a G
protein would be affected by a mutation that causes the affinity of the subunit for GDP
to be reduced without significantly changing its affinity for GTP?
16-18 When adrenaline binds to adrenergic receptors on the surface of a muscle cell, it activates
a G-protein, initiating a signaling pathway where the activated G subunit activates
adenylyl cyclase, increasing cAMP levels in the cell; the cAMP molecules then activate a
cAMP-dependent kinase that, in turn, activates enzymes that result in the breakdown of
muscle glycogen, thus lowering glycogen levels. You obtain muscle cells that are
defective in various components of the G-protein signaling pathway. Indicate how
glycogen levels would be affected in the presence of adrenaline in the following cells.
Would they be higher, lower, or the same as normal cells that are treated with adrenaline?
A.
In cells that lack adenylyl cyclase?
B.
In cells that lack the receptor?
C.
In cells that lack cAMP phosphodiesterase?
D.
In cells that have a G subunit that cannot hydrolyze GTP but can interact
properly with G and G.
16-19 The rod photoreceptors in the eye are extremely sensitive to light. Rod cells sense light
through a signal transduction cascade involving light-activation of a G-protein coupled
receptor that activates a G-protein, which activates cyclic GMP phosphodiesterase.
Describe how you would expect the addition of the following drugs to affect the lightsensing ability of the rod cells. Explain your answers.
A.
A drug that is an inhibitor of cyclic GMP phosphodiesterase?
B.
A drug that is a nonhydrolyzable analog of GTP?

275

Enzyme-Linked Receptors
16-20 The growth factor Superchick stimulates proliferation of cultured chicken cells. The
receptor that binds Superchick is a receptor tyrosine kinase, and many tumor cell lines
have mutations in the gene that encodes this receptor. Which of the following types of
mutations would be expected to induce uncontrolled cell proliferation?
(a)
A mutation that prevents localization of the receptor to the plasma membrane.
(b)
A mutation that prevents dimerization of the receptor.
(c)
A mutation that destroys the kinase activity of the receptor.
(d)
A mutation that prevents recognition of the receptor by phosphatases.
(e)
A mutation that prevents ligand binding.
16-21 Male cockroaches with mutations that strongly reduce the function of a receptor tyrosine
kinase X, are oblivious to the charms of their female comrades. This particular receptor
tyrosine kinase binds to a small molecule secreted by sexually mature females. Most
males carrying loss-of-function mutations in the gene for Ras protein are also unable to
respond to females. You have just read a paper in which the authors describe how they
have screened cockroaches that are mutant in receptor tyrosine kinase X for additional
mutations that partially restore the ability of males to respond to females. These
mutations decrease the function of a protein that the authors call Z. Which of the
following types of proteins could Z be? Explain your answer.
(a)
A protein that stimulates exchange of GDP for GTP by the Ras protein.
(b)
A protein that stimulates hydrolysis of GTP by the Ras protein.
(c)
A protein kinase activated by the Ras protein.
(d)
An adaptor protein that mediates the binding of the receptor tyrosine kinase X to
the Ras protein.
(e)
A transcription factor required for the expression of the Ras gene.
16-22 A kinase will act as an integrating device in signaling if it
(a)
phosphorylates more than one substrate.
(b)
catalyzes its own phosphorylation.
(c)
is activated or inhibited to more than one protein.
(d)
has at least two sites at which the kinase itself can be phosphorylated by different
kinases.
(e)
initiates phosphorylation cascades.
16-23 Antibodies are Y-shaped molecules that have two identical binding sites. Imagine that
you have obtained an antibody that is specific for the extracellular domain of a receptor
tyrosine kinase. When the antibody binds the receptor, it brings together two receptor
tyrosine kinase molecules. If cells containing the receptor tyrosine kinase were exposed
to the antibody, would you expect the kinase to be activated, inactivated, or unaffected?
Explain your reasoning.

276

16-24 For each of the following sentences, fill in the blanks with the best word or phrase
selected from the list below. Not all words or phrases will be used; each word or phrase
should be used only once.
There are many different signaling pathways that cells can use for signal
transduction. Some of them may involve signals that are gases, such as
__________________, which can diffuse easily into cells. Other
signaling pathways use cytokines as the signal, which bind to cytokine
receptors. Cytokine receptors have no intrinsic enzyme activity but are
associated with cytoplasmic tyrosine kinases called
__________________s, which become activated upon cytokine binding to
its receptor and go on to phosphorylate and activate cytoplasmic gene
regulatory proteins called __________________s. Some signal
transduction pathways involve chains of proteins that phosphorylate each
other, as seen in the __________________ cascade. Lipids can also
partake in relaying signals in the cell, as seen when phospholipase C
cleaves the sugar-phosphate head off a lipid molecule to generate the two
small messenger molecules __________________ (which remains
imbedded in the plasma membrane) and __________________ (which
diffuses into the cytosol).
cyclic GMP
DAG
IP3
JAK

MAP-kinase
NO
Ras
SMAD

277

STAT
TGF-

16-25 When activated by the signal, the platelet-derived growth factor (PDGF) receptor
phosphorylates itself on multiple tyrosines (as indicated below by the circled Ps; the
numbers next to these Ps indicate the amino acid number of the tyrosine). These
phosphorylated tyrosines serve as docking sites for proteins that interact with the
activated PDGF-receptor. These proteins are indicated in the figure below, and include
the proteins A, B, C, and D. The PDGF-receptor is activated when it binds to the signal,
PDGF. One of the cellular responses to PDGF is an increase in DNA synthesis, which
can be measured by incorporation of radioactive thymidine into the DNA.
To determine whether protein A, B, C, and/or D are responsible for activation of DNA
synthesis, you construct mutant versions of the PDGF-receptor that retain one or more
tyrosine phosphorylation sites. You express these mutant versions in cells that do not
make a PDGF-receptor. In these cells, the various versions of the PDGF-receptor are
expressed normally and in response to PDGF binding, become phosphorylated on
whichever tyrosines remain. You measure the level of DNA synthesis in cells that
express the various mutant receptors and obtain the data shown below.

Figure Q16-25
A.
B.
C.

From these data, which, if any, of these proteins A, B, C, and D are involved in
the stimulation of DNA synthesis by PDGF? Why?
Which, if any, of these proteins inhibit DNA synthesis? Why?
Which, if any, of these proteins appear to play no detectable role in DNA
synthesis? Why?

278

16-26 Two protein kinases, PK1 and PK2, work sequentially in a signal transduction pathway.
You create cells that contain inactivating mutations in either PK1 or PK2 and find that
these cells no longer respond to an extracellular signal. You also create cells containing a
version of PK1 that is permanently active and find that the cells behave as if they are
receiving the signal even when the signal is not present. When you introduce the
permanently active version of PK1 into cells that have an inactivating mutation in PK2,
you find that these cells also behave as though they are receiving the signal even when no
signal is present.
A.
From these results, does PK1 activate PK2 or does PK2 activate PK1? Explain
your answer.
B.
You now create a permanently active version of PK2 and find that cells
containing this version behave as if they are receiving the signal even when the
signal is not present. What do you predict will happen if you introduce the
permanently active version of PK2 into cells that have an inactivating mutation in
PK1?
16-27 The last common ancestor to plants and animals was a unicellular eucaryote. Thus, it is
thought that multicellularity and the attendant demands for cell communication arose
independently in these two lineages. This evolutionary viewpoint accounts nicely for the
vastly different mechanisms that plants and animals use for cell communication. Fungi
use signaling mechanisms and components that are very similar to those used in animals.
Which of the phylogenetic trees shown in Figure 16-27 does this observation support?

Figure Q16-27

279

How We Know: Untangling Cell Signaling Pathways


16-28 Your friend is studying mouse fur color and has isolated the G-protein-linked receptor
responsible for determining fur color as well as the signal that activates the receptor. She
finds that, upon addition of the signal to pigment cells (cells that produce the pigment
determining fur color), cAMP levels rise in the cell. She starts a biotech company and
the company isolates more components of the signaling pathway responsible for fur
color. Using transgenic mouse technology, the company genetically engineers mice that
are defective in various proteins involved in determining fur color. The company obtains
the following results:
normal mice have beige (very light brown) fur color
mice lacking the signal have white fur
mice lacking the G-protein coupled receptor have white fur
mice lacking cAMP phosphodiesterase have dark brown fur
Your friend has also made mice that are defective in the G subunit of the G protein in
this signaling pathway. This defective G works normally except that, once it binds
GTP, it cannot hydrolyze GTP to GDP. What color do you predict the fur of these mice
will be? Why?
16-29 Bacteria undergo chemotaxis toward amino acids, which usually indicates the presence of
a food source. Chemotaxis receptors bind a particular amino acid and cause changes in
the bacterial cell that induce the cell to move toward the source of the amino acid. Four
types of chemotaxis receptors that mediate responses to different amino acids have been
identified in a bacterium. The receptors are called ChrA, ChrB, ChrC, and ChrD. Each
one of these receptors specifically senses serine, aspartate, glutamate, or glycine,
although you do not know which receptor senses which amino acid. You have been
given a wild-type bacterial strain that contains all four receptors, as well as various
mutant bacterial strains that are lacking one or more of the receptors.
To figure out which receptor senses which amino acid, you conduct experiments where
you fill a capillary tube with an amino acid to attract the bacteria, dip the capillary tube
into a solution containing bacteria, remove the capillary tube after 5 minutes, and count
the number of bacteria in the capillary tube. Your results are shown in Table 16-29.
Table 16-29 Chemotaxis in wild-type and mutant strains of bacteria
NUMBER OF CELLS IN CAPILLARY TUBE
Strain Intact Receptors
Serine
Aspartate Glutamate Glycine None
1
ChrA, ChrB, ChrC, ChrD 590
1050
950
66
5
2
ChrA, ChrB, ChrC
7
840
770
130
8
3
ChrC, ChrD
340
7
590
6
6
4
ChrA, ChrC, ChrD
550
6
650
41
5
5
ChrB, ChrC, ChrD
700
590
850
9
8
From these results, indicate which receptor is used for which amino acid.
280

Answers
16-1

(a), (c) Since cell line A proliferates only in the presence of F and since cell line B can
proliferate in the absence of F once R is expressed, binding of factor F must be
required for proliferation of cell line A but not cell line B. Because expression of
R is sufficient to induce proliferation of cell line B in the absence of F, there is
some other factor, X, presumably in the serum, that is capable of binding to R and
stimulating B. Since R binds to X, it probably does not bind to F, since most
receptors do not bind more than one ligand. We do not know the effect of
eliminating receptor R from cell line A; therefore, we cannot say if binding of
receptor R to its ligand is required for proliferation cell line A. Since R is not the
receptor for F, we cannot say that cell line B does not contain a receptor for F.
Cell line B could have a receptor for F and respond to F in ways that have nothing
to do with cell proliferation.

16-2

Cells can signal to each other in various ways. A signal that must be relayed to the entire
body is most efficiently sent by endocrine cells, which produce hormones that are carried
throughout the body through the bloodstream. On the other hand, contact-dependent
methods of cell signaling do not require the release of a secreted molecule and are used
for very localized signaling events. During paracrine signaling, the signal remains in the
neighborhood of the secreting cell and thus acts as a local mediator on nearby cells.
Finally, neuronal signaling involves the conversion of electrical impulses into a chemical
signal. Cells receive signals through a receptor, which can be an integral membrane
protein or can reside inside the cell.

16-3

Choice (d) is the correct answer. All members of the steroid hormone receptor family are
intracellular proteins (thus choice (a) is not correct) that interact with molecules that can
diffuse through the plasma membrane unaided. Steroid hormone receptors shuttle
between the cytoplasm and the nucleus, which is why choice (c) is incorrect. They are
activated by either steroids or other hydrophobic signaling molecules and typically
undergo a large conformational change upon binding to the signaling molecule (choice
(b)). This conformational change activates the steroid hormone receptors, allowing them
to promote or inhibit the transcription of the appropriate genes. Steroid hormone
receptors are important for many aspects of human physiology, including, but not limited
to sexual development (choice (e)).

16-4

Since the interactions of the signal molecule with its receptor and of the activated
receptor with its gene are both one-to-one, there is no amplification in this part of the
signaling pathway. The signal can, however, be amplified when the target gene is
transcribed, since multiple copies of mRNA are usually produced from a gene once it has
been switched on, and multiple copies of protein can be made from each mRNA
molecule.

281

16-5

Although succinylcholine can bind the acetylcholine receptor, this association does not
cause the acetylcholine receptor to alter its conformation and stimulate the intracellular
signaling necessary for muscle contraction (which involves, in this case, the opening of
ion channels). Succinylcholine prevents normal muscle contraction by competing with
acetylcholine for binding to the acetylcholine receptor, thus blocking its action.

16-6

(c)

16-7

Ion-channel-linked receptors; G-protein-linked receptors; enzyme-linked receptors.

16-8

Choices (a) and (d) are correct answers. A hydrophilic molecule cannot diffuse across the
membrane and can therefore only affect a cell if it binds to a cell-surface receptor that has
at least one component that spans the bilayer (choice (a)). Cells having the same set of
receptors can respond in different ways to the same ligand molecules, since the response
to a receptor also depends on the types of intracellular relay pathways present in the cell
(choice (d)). Most signal molecules remain bound to the extracellular domain of the
receptor, while the intracellular domain mediates signal transduction. Although many
signal molecules are endocytosed with their receptor, they remain inside membranebounded compartments and are therefore not transported into the cytosol (choice (b)). A
cell-surface receptor capable of binding only one natural ligand can mediate more than
one kind of response, depending on a variety of conditions, such as the types of
intracellular signaling molecules present in the cell type on which the receptor is being
expressed (choice (c)). Foreign substances that occupy receptor sites normally bound by
extracellular signal molecules can sometimes induce the same response that is produced
by the natural ligand, but they often block the binding of the natural ligand without
stimulating an intracellular signal (choice (e)).

16-9

Cells can respond differently to activated intracellular steroid hormone receptors for
several reasons.
1.
The cells may differ in the gene regulatory proteins they express for modulating
the activity of the activated receptor.
2.
Different cells could have their DNA modified in different ways, either through
differences in chromatin configuration or differences in the methylation-state of
the DNA, which may affect the interaction of the activated receptor with its DNA.

282

16-10 A.

B.

16-11 A.
B.
C.
D.

E.

16-12 (d)

Both cell lines appear to contain the glucocorticoid receptor, because both cell
lines demonstrate increases in reporter gene activity in response to addition of
glucocorticoid. With the introduction of the construct containing the
glucocorticoid-responsive viral enhancer, cell line 1 showed a 1000-fold increase
and cell line 2 showed an 80-fold increase.
Four reasonable explanations for this difference are (any one would be correct):
1.
Cell line 1 contains a protein that does not exist in cell line 2. This protein
cooperates with the glucocorticoid receptor to activate transcription of the
reporter gene, leading to higher levels of the reporter gene in cell line 1
than cell line 2.
2.
Cell line 2 contains a slightly defective glucocorticoid receptor that does
not bind DNA as well as the normal glucocorticoid receptor in cell line 1,
thus leading to lower levels of reporter gene activity in cell line 2 than cell
line 1.
3.
Cell line 1 contains a mutant glucocorticoid receptor that binds DNA
much more strongly than the normal glucocorticoid receptor found in cell
line 2. Therefore, higher levels of reporter gene activity are seen in cell
line 1 than cell line 2.
4.
Cell line 2 contains a protein that does not exist in cell line 1. This protein
acts as an antagonist to the glucocorticoid receptor to lower the level of
transcription from the reporter gene, leading to lower levels of reporter
gene activity in cell line 2 than cell line 1.
No. Each signal molecule activates only one receptor.
No. Each G-protein-linked receptor only activates one heterotrimeric G protein.
Yes. Each activated adenylyl cyclase molecule can generate many molecules of
cAMP.
No. In unstimulated cells, protein kinase A is held inactive in a protein complex.
cAMP binding to the complex induces a conformational change, releasing the
active protein kinase A. Therefore, 1 cAMP cannot activate more than 1
molecule of active protein kinase A.
Yes. Each activated protein kinase A molecule can phosphorylate many
molecules of target proteins.
The subunit exchanges its bound GDP for GTP. (e) is incorrect since the
subunit may have an effect instead of or as well as the subunit. The other
statements are simply untrue.

283

16-13 (a), (b) The heart is induced to beat more slowly by binding of acetylcholine to a G+
protein-linked receptor, whose activated subunit binds to and opens K
channels. High concentrations of a non-hydrolyzable analogue of GTP or
modification of the acetylcholine-receptor-linked G protein subunit by cholera
toxin (which also prevents GTP hydrolysis) will increase the length of time that
+
the G protein subunit remains free of and able to activate the K channel;
they will therefore enhance the effect of acetylcholine. Receptor mutations that
weaken its interaction with either acetylcholine or the G protein, or, mutations
+
that weaken the interaction between and the K channel, all make it more
+
difficult for the signal to proceed from the receptor to the K channel.
16-14 (b)

Single mutations in the G and G subunits of the G-protein allow for growth in
the absence of pheromone and display a sterile phenotype in the presence of
pheromone, while loss of the G subunit causes growth arrest in either the
presence or absence of pheromone. Since arrested growth is a normal response
that occurs when cells sense and bind pheromone, it must be the G subunits that
normally activate the cellular mating response when released from the G
subunit. In cells lacking G, G inappropriately causes growth arrest because
G normally inhibits the action of G until G is released from G upon
receptor binding to pheromone. This interpretation is consistent with the analysis
of the double mutants where G is deleted together with either G or G. In
these double mutants, normal growth is seen in the absence of pheromone because
G and G act together as a complex to activate targets; lack of either G or G
leads to no response to pheromone, even when G is not around to inhibit the
response.

16-15 A

Any answer that involves modification of existing cellular components is correct.


For example, protein phosphorylation, protein dephosphorylation, protein
ubiquitination, lipid phosphorylation, and lipid cleavage are all examples of
correct answers.
Alterations in gene expression.
Modification of an existing cellular component can happen quickly, as it involves
components (both the modifying protein as well as the substance being modified)
that already exist in the cell. Changes in cellular physiology due to changes in
gene expression can take much longer, as the gene will need to be transcribed, the
mRNA translated, and proteins accumulated to high enough levels to instigate
change.

B.
C.

284

16-16 A.

B.

Since a complete lack of the CaM kinase causes a loss of memory, we can assume
that mutations that lead to inactivation of the kinase would also have a deleterious
effect on memory.
(1) Protein kinases have a binding site for ATP, which is the source of the
phosphate used for phosphorylation; if the kinase cannot bind ATP it will be
inactive.
(2) Since CaM kinases are activated by binding to calmodulin in the presence of
Ca2+, deletion of the calmodulin-binding portion would inactivate the
kinase.
(3) A mutation that destroys the site of autophosphorylation will also impair the
normal function of the kinase, since the kinase will become inactive as soon
as Ca2+ levels drop.
Mutations that make the kinase less likely to bind the phosphatase will increase
the time that the kinase remains active following a transient increase in Ca2+
levels.

16-17 The mutant G protein would be constantly active. Each time the subunit hydrolyzed
GTP to GDP, the GDP would spontaneously dissociate, allowing GTP to bind and
reactivate the subunit, especially because the intracellular concentration of GTP is
higher than the intracellular concentration of GDP. Normally, GDP is tightly bound by
the subunit, which keeps the G protein in its inactive state until release of GDP is
stimulated by interaction with an appropriate activated G-protein-linked receptor.
16-18 A.
B.
C.
D.

higher
higher
lower
lower

16-19 A.

A drug that inhibits cyclic GMP phosphodiesterase would decrease any visual
responses. Normally, cyclic GMP is continually being produced in the eye. The
perception of light by a rod cell normally leads to activation of cyclic GMP
phosphodiesterase, which then hydrolyzes cyclic GMP molecules; this causes Na+
channels to close, which changes the membrane potential and alters the signal
sent to the brain. If cyclic GMP phosphodiesterase is blocked, levels of cyclic
GMP would remain high and there would be no cellular response to light.
A drug that is a nonhydrolyzable analog of GTP would lead to an extended visual
response. This is because a nonhydrolyzable analog of GTP would cause
prolonged activation of the G protein in response to light. Continued activation of
the G protein would keep cyclic GMP phosphodiesterase levels higher than
normal, leading to a prolonged period of lowered levels of cyclic GMP; this in
turn would cause Na+ channels to be closed longer than normal, leading to a
prolonged change in the membrane potential and an extended visual response.

B.

285

16-20 (d)

Tyrosine receptor kinases are usually activated by ligand-induced dimerization,


which allows the receptors to phosphorylate themselves and activate intracellular
signaling proteins that are stimulated by the phosphorylated receptor. After it is
activated, the receptor is dephosphorylated, and thereby inactivated, by a
phosphatase. Therefore, a mutation in the phosphatase will inappropriately
increase the activity of the receptor and lead to uncontrolled cell proliferation.
Mutations that prevent localization of the receptor to the plasma membrane,
dimerization of the receptor (including mutations that prevent ligand binding), or
autophosphorylation will inactivate the receptor.

16-21 (b)

Mutations that increase the activity of Ras should mimic the effect of stimulating
the receptor tyrosine kinase X in a receptor-independent fashion. Since the
intracellular concentration of GTP is higher than that of GDP, some proportion of
the Ras molecules is expected to be GTP-bound and active; ridding the cells of a
protein, which stimulates GTP hydrolysis will increase this pool of active Ras.
Mutants that cannot stimulate exchange of GDP for GTP by Ras will have the
same phenotype as mutants lacking Ras, as will mutants lacking a transcription
factor required for expression of the Ras gene. Mutants lacking the kinase
activated by Ras will be unable to transmit any signal from Ras onward. Defects
in an adaptor protein that mediates the binding of receptor X to Ras will have no
further effect on a mutant already lacking the receptor.

16-22 Choices (c) and (d) are the correct answers. Integrating devices are able to relay signals
from more than one signaling pathway. Being activated or inhibited by more than one
protein or having sites that can be phosphorylated by different kinases allows a kinase (or
any other signaling molecule) to be affected by more than one upstream signal. Choices
(a), (b), and (e) affect the output signal that a kinase is able to produce, not its ability to
integrate incoming signals.
16-23 The receptor tyrosine kinase will probably become activated upon binding of the
antibody molecule. This is because receptor tyrosine kinases are usually activated by
signal-induced dimerization. When receptor tyrosine kinase molecules are brought
together, their cytoplasmic kinase domains become activated and each receptor
phosphorylates the other.

286

16-24 There are many different signaling pathways that cells can use for signal transduction.
Some of them may involve signals that are gases, such as NO, which can diffuse easily
into cells. Other signaling pathways use cytokines as the signal, which bind to cytokine
receptors. Cytokine receptors have no intrinsic enzyme activity but are associated with
cytoplasmic tyrosine kinases called JAKs, which become activated upon cytokine
binding to its receptor and go on to phosphorylate and activate cytoplasmic gene
regulatory proteins called STATs. Some signal transduction pathways involve chains of
proteins that phosphorylate each other, as seen in the MAP-kinase cascade. Lipids can
also partake in relaying signals in the cell, as seen when phospholipase C cleaves the
sugar-phosphate head off a lipid molecule to generate the two small messenger molecules
DAG (which remains imbedded in the plasma membrane) and IP3 (which diffuses into
the cytosol).
16-25 A.

B.

C.

16-26 A.

B.

Proteins A and D are involved in stimulating DNA synthesis. PDGF-receptors


that can bind to only A or D (see experiments 2 and 5) can stimulate DNA
synthesis to about 50% of normal amounts (which is represented by experiment
1). A and D are both needed and are used in an additive fashion; this is evident
from experiment 6: when a PDGF-receptor can bind both A and D, DNA
synthesis levels are close to that obtained with the normal receptor.
Protein B is an inhibitor of DNA synthesis. Consequently, receptors with binding
sites for B and D (see experiment 7) stimulate a lower DNA synthesis rate than do
receptors that only bind D (experiment 5).
Protein C plays no detectable role in DNA synthesis. Receptors that can only
bind C (experiment 4) activate DNA synthesis about as much as receptors that
dont bind any proteins (experiment 9; our negative control). Furthermore, the
binding of protein C does not affect the response mediated by protein D when the
receptor can bind both C and D (experiment 8).
Normally, PK2 activates PK1. We are told that PK1 and PK2 normally work
sequentially in a signal transduction pathway. If PK1 is permanently activated, a
response is seen independent of whether PK2 is present. If PK1 activated PK2,
no response should be seen if PK1 were activated in the absence of PK2.
You would predict that no response to signal would be observed. This is because
PK2 normally needs to activate PK1 in order for the cells to respond to the signal.
When PK2 is permanently activated in the absence of PK1, PK1 is not there to
relay the signal to affect cellular changes.

16-27 Choice (b) is the correct answer (see Figure Q16-27). The similarities in signaling
mechanisms between animals and fungi support the phylogenetic tree in which fungi
branched from the animal lineage after plants and animals separated. This branching
order is supported by a wide variety of other data, including genomic sequence
comparisons.

287

16-28 These mice will have dark brown fur. The inability to hydrolyze GTP to GDP will lead
to inappropriate activation of the signaling pathway that makes pigment. Too much
pigment will be produced, as seen in the mice lacking cAMP phosphodiesterase (which
lack the ability to dampen the signal), and the mice will end up with dark brown fur.
16-29 ChrA senses glycine, ChrB senses aspartate, ChrC senses glutamate, and ChrD senses
serine. To figure this out, you must match the pattern of intact receptors with the pattern
of responses to the various amino acids. For example, ChrD is missing in strain #2,
which does not sense serine. Therefore, ChrD is the receptor used to sense serine. Since
we know that one of the receptors senses glutamate, but all strains respond to glutamate,
ChrC must be the sensor for glutamate as it is present in all strains.

288

CHAPTER 17
CYTOSKELETON
2009 Garland Science Publishing
3rd Edition
17-1 Indicate which of the three major classes of cytoskeletal elements each statement below
refers to.
A. monomer that binds ATP
B. includes keratin and neurofilaments
C. important for formation of the contractile ring during cytokinesis
D. supports and strengthens the nuclear envelope
E. their stability involves a GTP cap
F. used in the eucaryotic flagellum
G. a component of the mitotic spindle
H. can be connected through desmosomes
I. directly involved in muscle contraction
J. abundant in filopodia
17-2 Identify the cytoskeletal structures in the epithelial cells shown in Figure Q17-2.

Intermediate Filaments

17-3 Phosphorylation of nuclear lamins regulates their assembly and disassembly during
mitosis. You add a drug to cells that are undergoing mitosis that inhibits the activity of an
enzyme that dephosphorylates nuclear lamins. What do you predict will happen to these
cells? Why?

289

17-4

For each of the following sentences, fill in the blanks with the best word or phrase
selected from the list below. Not all words or phrases will be used; each word or phrase
should be used only once.
Intermediate filaments are mainly found in cells that are subject to
mechanical stress. Some rare human genetic diseases are caused by
mutations in genes that disrupt intermediate filaments. For example, the
skin of people with epidermolysis bullosa simplex is very susceptible to
mechanical injury; people with this disorder have mutations in their
__________________ genes, the intermediate filament found in epithelial
cells. These filaments are usually connected from cell to cell through
junctions called __________________s. The main filaments found in
muscle cells belong to the __________________ family; people with
disruptions in these intermediate filaments can have muscular dystrophy.
In the nervous system, __________________s help strengthen the
extremely long extensions often present in nerve cell axons; disruptions in
these intermediate filaments can lead to neurodegeneration. People who
carry mutations in the gene for __________________, an important
protein for crosslinking intermediate filaments, have a disease that
combines symptoms of epidermolysis bullosa simplex, muscular
dystrophy, and neurodegeneration.
desmosome
keratin
kinase

lamin
neurofilament
plectin

synapse
vimentin

Microtubules
17-5

Place the following in order of size, from the smallest to the largest.
A.
Protofilament
B.
Microtubule
C.
-tubulin
D.
Tubulin dimer
E.
Mitotic spindle

290

17-6

In the three cell outlines in Figure Q17-6 indicate how the microtubules will be arranged,
showing clearly their free and attached ends. On each figure indicate the plus end and the
minus end for one of the microtubules.

Figure Q17-6
17-7

The graph in Figure Q17-7 shows the time-course of the polymerization of pure tubulin
in vitro. You can assume that the starting concentration of free tubulin is much higher
than it is in cells.

Figure Q17-7
A.
B.
17-8

Explain the reason for the initial lag in the rate of microtubule formation.
Why does the curve level out after point C?

The hydrolysis of GTP to GDP carried out by tubulin molecules


(a)
provides the energy needed for tubulin to polymerize.
(b)
occurs because the pool of free GDP has run out.
(c)
tips the balance in favor of microtubule assembly.
(d)
allows the behavior of microtubules called dynamic instability.
291

17-9

The microtubules in a cell form a structural framework that can have all the following
functions EXCEPT
(a)
holding internal organelles such as the Golgi apparatus in particular positions in
the cell.
(b)
creating long thin cytoplasmic extensions that protrude from one side of the cell.
(c)
strengthening the plasma membrane.
(d)
moving materials from one place to another inside a cell.

17-10 Do you agree or disagree with this statement? Explain your answer.
Minus end-directed microtubule motors (like dyneins) deliver their cargo
to the periphery of the cell, whereas plus end-directed microtubule motors
(like kinesins) deliver their cargo to the interior of the cell.
17-11 Kinesins and dyneins
(a)
are inhibited by colchicine and taxol.
(b)
move along both microtubules and actin filaments.
(c)
often move in opposite directions to each other.
(d)
derive their energy from GTP hydrolysis.
(e)
have tails that bind to the filaments.
17-12 Match the following labels to the numbered lines on Figure Q17-12.

Figure Q17-12
A.
B.
C.
D.

Minus end of microtubule.


Tail of motor protein.
Cargo of motor protein.
Head of motor protein.

Which one of the two motors shown above is most likely a kinesin? Explain your
answer.

292

17-13 Some lower vertebrates such as fish and amphibians can control their color by regulating
specialized pigment cells called melanophores. These cells contain small, pigmented
organelles, termed melanosomes, that can be dispersed throughout the cell, making the
cell darker, or aggregated in the center of the cell to make the cell lighter. You purify the
melanosomes from melanophores that have either aggregated or dispersed melanosomes
and find that:
1.
2.

aggregated melanosomes co-purify with dynein


dispersed melanosomes co-purify with kinesin

Given this set of data, propose a mechanism for how aggregation and dispersal of
melanosomes occur.
17-14 Consider Figure Q17-14, which shows two isolated outer doublet microtubules from a
eucaryotic flagellum with their associated dynein molecules.

Figure Q17-14
A.
B.
C.

Sketch what will happen to this structure if it is supplied with ATP.


Sketch what will happen to this structure if the linking proteins are removed and it
is supplied with ATP.
In a complete flagellum, what would happen if all the dynein molecules were
active at the same time?

293

17-15 You are curious about the dynamic instability of microtubules and decide to join a lab
that works on microtubule polymerization. The people in the lab help you grow some
microtubules in culture using conditions that allow you to watch individual microtubules
under a microscope. You can see the microtubules growing and shrinking, as you expect.
The professor who runs the lab gets in a new piece of equipment, a laser beam that is
very fine and can be used to sever microtubules. She is very excited and wants to sever
growing microtubules at their middle, using the laser beam.
A.
Do you predict that the newly exposed microtubule plus ends will grow or shrink?
Explain your answer.
B.
What do you expect would happen to the newly exposed plus ends if you were to
grow the microtubules in the presence of an analog of GTP that cannot be
hydrolyzed and then severed the microtubules in the middle using a laser beam?

Actin Filaments
17-16 Do you agree or disagree with the following statement? Explain your answer.
Nucleotide hydrolysis plays a similar role in actin polymerization and
tubulin polymerization.
17-17 Actin binding proteins bind to actin and can modify its properties. You purify a protein,
Cap1, that appears to bind and cap one end of an actin filament, although you do not
know whether it binds the plus or the minus end. To determine which end of the actin
filament your protein binds to, you decide to examine the effect Cap1 has on actin
polymerization by measuring the kinetics of actin filament formation in the presence and
the absence of Cap1 protein. You obtain the following results (see Figure Q17-17). Do
you think Cap1 binds the plus or the minus ends of actin? Explain your reasoning.

Figure 17-17
17-18 In the cell, the concentration of actin monomer is higher than the concentration required
for purified actin monomers to polymerize in vitro. Thymosin is a protein that can bind
actin monomers. If you were to add a drug that inhibits the ability of thymosin to bind
actin monomers, what effect would this have on actin polymerization? Explain your
answer.

294

17-19 In the budding yeast, activation of the GTP-binding protein Cdc42 occurs upon binding
of an external signal (pheromone) to a G-protein linked receptor. Activation of Cdc42
promotes actin polymerization. What would you predict would happen to actin
polymerization, compared to pheromone treated cells, if you were to add:
A.
pheromone + an inhibitor of G-protein linked receptors?
B.
pheromone + a nonhydrolyzable analogue of GTP?
17-20 Which of the following statement(s) are FALSE?
(a)
Some myosin tails can bind the plasma membrane.
(b)
Actin filaments are usually excluded from the cell cortex.
(c)
Integrins are transmembrane proteins that can bind to the extracellular matrix.
(d)
ARPs can promote the formation of branched actin filaments.
(e)
Cytochalasins prevent actin polymerization.

Muscle Contraction
17-21 Do you agree or disagree with the following statement? Explain your answer.
When skeletal muscle receives a signal from the nervous system to
contract, the signal from the motor neuron triggers the opening of a
voltage-sensitive Ca++ channel in the muscle cell plasma membrane,
allowing Ca++ to flow into the cell.
17-22 Which of the following structures shortens during muscle contraction?
(a)
Myosin filaments
(b)
Flagella
(c)
Sarcomeres
(d)
Actin filaments

295

17-23 You isolate some muscle fibers to examine what regulates muscle contraction. When you
bathe the muscle fibers in a solution containing ATP and Ca++, you see muscle
contraction (experiment 3 in Table Q17-23). Ca++ is necessary, as solutions containing
ATP alone or nothing do not stimulate contraction and thus the muscle remains in a
relaxed state (experiments 1 and 2 in Table Q17-23). From what you know about the
mechanism of muscle contraction, fill in your predictions of whether the muscle will be
contracted or relaxed for experiments 4, 5, and 6. Explain your answers in the space
below the table.
Table Q17-23.
Experiment number
1
2
3
4
5
6

added to muscle fibers

muscle state

nothing
ATP
ATP and Ca++
ATP, Ca++, and a drug that inhibits troponin from
binding Ca++
ATP and a drug that inhibits binding of
tropomyosin to actin
a nonhydrolyzable analogue of ATP

relaxed
relaxed
contracted

Extra credit: In what state would the muscle be if you added Ca++ but no ATP?

How We Know: Pursuing Motor Proteins


17-24 You are interested in studying kinesin movements. You therefore prepare silica beads and
coat them with kinesin molecules so that each bead, on average, contains only one
kinesin molecule attached to it. You add these kinesin-coated beads to a preparation of
microtubules you have polymerized. Using video microscopy, you watch the kinesin
move down the microtubules.
A.
Kinesin-GFP has been measured to move along microtubules at a rate of 0.3
m/second, and single-molecule studies have revealed that kinesin moves along
microtubules progressively, with each step being 8 nm. How many steps can the
kinesin molecule take in 4 seconds, assuming that the kinesin stays attached to the
microtubule the entire 4 seconds?
B.
Since each kinesin molecule is thought to take approximately 100 steps before
falling off the microtubule, will you see your silica beads detach from the
microtubule during your 4 seconds of observation?
C.
What would you predict would happen to the kinesin-coated silica beads if you
were to add AMP-PNP (a nonhydrolyzable ATP analogue)?

296

Answers
17-1

A.
B.
C.
D.
E.
F.
G.
H.
I.
J.

actin
intermediate filaments
actin
intermediate filaments
microtubules
microtubules
microtubules
intermediate filaments
actin
actin

17-2

Amicrotubules; Bintermediate filaments; Cactin filaments (microfilaments).

17-3

Cells should become arrested in mitosis. Normally, the lamins are phosphorylated during
mitosis, causing disassembly of the nuclear envelope. At the end of mitosis, the nuclear
lamins are dephosphorylated, causing the lamins to reassemble. Inhibition of this last
step should therefore prevent the nuclear lamins from reassembling after mitosis.

17-4

Intermediate filaments are mainly found in cells that are subject to mechanical stress.
Some rare human genetic diseases are caused by mutations in genes that disrupt
intermediate filaments. For example, the skin of people with epidermolysis bullosa
simplex is very susceptible to mechanical injury; people with this disorder have
mutations in their keratin genes, the intermediate filament found in epithelial cells.
These filaments are usually connected from cell to cell through junctions called
desmosomes. The main filaments found in muscle cells belong to the vimentin family;
people with disruptions in these intermediate filaments can have muscular dystrophy. In
the nervous system, neurofilaments help strengthen the extremely long extensions often
present in nerve cell axons; disruptions in these intermediate filaments can lead to
neurodegeneration. People who carry mutations in the gene for plectin, an important
protein for crosslinking intermediate filaments, have a disease that combines symptoms
of epidermolysis bullosa simplex, muscular dystrophy, and neurodegeneration.

17-5

C, D, A, B, E

297

17-6

See Figure A17-6.

Figure A17-6
17-7

A.

B.

Before they can polymerize to form microtubules, tubulin molecules have to form
small aggregates that act as nucleation centers. This aggregation step is slow, as
the molecules have to come together in the right configuration. This is why there
is a lag phase before microtubules start to be formed.
After point C an equilibrium point has been reached where the rates of
polymerization and depolymerization are exactly balanced.

17-8

(d)

The hydrolysis of GTP to GDP occurs after a GTP-bound tubulin molecule is


incorporated into a microtubule, and it makes the microtubule more susceptible to
disassembly. It is the resulting switch in microtubule stability that gives rise to the
phenomenon known as dynamic instability.

17-9

(c)

One function of actin filaments, but not microtubules, is to provide a meshwork


beneath the plasma membrane that helps to form and strengthen this membrane.
Microtubules have all of the other functions that are described.

17-10 Disagree. The plus ends of microtubules usually point toward the cell periphery while
the minus ends point toward the cell center. This is because the -tubulin in the
centrosome serves to nucleate microtubule growth. Since the centrosomes are near the
center of the cell, the minus ends of microtubules are located there. Therefore, a minusend directed microtubule motor would direct its cargo toward the center of the cell while
a plus-end directed microtubule motor would direct its cargo toward the cell periphery.
17-11 (c)
17-12 A4; B2; C1; D3
The top motor is more likely to be kinesin, as kinesins usually move toward the plus end
of the microtubules.
298

17-13 The melanosomes are transported in the cell on microtubules. When it is advantageous
for the animal to become lighter, a signal is sent to the pigment cell that causes the
melanosomes to associate with dynein. Since dynein is a minus-end directed motor, it
will transport the melanosomes toward the center of the cell, causing the melanosomes to
aggregate in the center and the cell to take on a lighter appearance. When the animal
wants to become darker, a signal is sent to the pigment cell that causes the melanosomes
to associate with kinesin. Kinesin is usually a plus-end directed motor and will move the
melanosomes away from the center of the cell so that they are more dispersed, making
the cell look darker.
17-14 A.
B.

See Figure A17-14A.


See Figure A17-14B. (Note to instructor: this question should be marked as
correct only if the microtubule is shown bending in the correct direction (in A)
and the correct microtubule is shown pushed forward (in B)).

Figure A17-14
C.

The flagellum will not bend because there is no significant relative motion of one
microtubule doublet to another, since each is trying to push its neighbor forward
at the same time. For the flagellum to bend, sets of dynein molecules on one side
of the flagellum must be selectively activated.

299

17-15 A.

B.

The newly exposed microtubule plus ends will most likely shrink if the
microtubules are severed in the middle. This is because a microtubule grows by
adding GTP-carrying subunits to the plus end. The GTP is hydrolyzed over time,
leaving only a cap of GTP-carrying subunits at the plus end with the remainder of
the tubulin protofilament containing GDP-carrying subunits. Therefore, if you
sever a growing microtubule at its middle, you will most likely create a plus end
that contains GDP-carrying subunits. The GDP-carrying subunits are less tightly
bound than the GTP-carrying subunits and will peel away from each other,
causing depolymerization of the microtubule and shrinkage.
If you were to polymerize the microtubules in the presence of a nonhydrolyzable
analogue of GTP and then severed the microtubules with a laser, the newly
exposed plus end would contain a GTP cap and so would likely continue to grow.

17-16 Agree. ATP-bound actin monomers are added onto actin filaments; GTP-bound tubulin
subunits are added onto the growing end of a microtubule. Nucleotide hydrolysis occurs
after addition of the subunit onto the filament in both actin and microtubules. ATP
hydrolysis in actin polymerization reduces the strength of binding between monomers in
the actin filaments; GTP hydrolysis during tubulin polymerization reduces the strength of
binding between the tubulin subunits in the microtubule. Because the nucleotide
hydrolysis decreases the strength of binding between subunits, depolymerization is
enhanced in both actin filaments and microtubules.
17-17 You would predict that Cap1 binds the plus ends of actin, as it appears to inhibit actin
polymerization. Actin filaments grow through the addition of monomers to the plus end
of the actin filament. A capping protein that binds the plus ends of actin can block
monomer addition to the actin filament. Thus, less actin polymerization will be seen in
the presence of the Cap1 protein.
17-18 Addition of a drug that keeps thymosin from binding actin monomer will increase the rate
of actin polymerization in the cell. Thymosin normally binds actin monomers in the cell
to sequester monomers until they are needed for actin polymerization. The reason actin
monomers do not spontaneously form filaments in the cell, despite the high concentration
of actin monomer, is because the monomers are normally bound by actin monomer
binding proteins (such as thymosin) and thereby prevented from adding onto the end of
an actin filament.
17-19 A.
B.

17-20 (b)

Less actin polymerization. Cdc42 will not be able to be activated by the Gprotein linked receptor.
More actin polymerization. Cdc42 will be more active, as it will bind the nonhydrolyzable form of GTP and not be able to be turned off.
Much of the actin in the cell is concentrated in the cell cortex, the region of the
cell just beneath the plasma membrane.

300

17-21 Disagree. The rise in intracellular Ca++ during muscle contraction comes from an
intracellular source. The Ca++ is released from the lumen of the sarcoplasmic reticulum,
which is a specialized region of endoplasmic reticulum inside a muscle cell. The signal
from the nerve terminal triggers an action potential in the muscle cell plasma membrane,
which causes a voltage sensitive transmembrane protein in the membranous transverse
tubules to open a Ca++ release channel in the membrane of the sarcoplasmic reticulum.
17-22 (c)

Sarcomeres contain actin filaments and myosin filaments that slide past each
other during muscle contraction, leading to shortening of the sarcomere; the actin
filaments and myosin filaments do not change in length. Flagella are
microtubule-based structures that are not present on muscle cells.

17-23 See table below.


Table A17-23
Experiment number
1
2
3
4
5
6

added to muscle fibers

muscle state

nothing
ATP
ATP and Ca++
ATP, Ca++, and a drug that inhibits troponin from
binding Ca++
ATP and a drug that inhibits binding of
tropomyosin to actin
a nonhydrolyzable analogue of ATP

relaxed
relaxed
contracted
relaxed
contracted
relaxed

In experiment 4, the muscle will be relaxed because troponin will not be able to bind
Ca++. By preventing troponin from binding to Ca++, troponin will not be able to undergo
the conformational change that causes tropomyosin to alter its association with actin.
This altered association is normally required for myosin to bind actin. In the absence of
troponin regulation by Ca++, myosin cannot bind actin and the muscle cannot contract.
In experiment 5, the muscle will contract because tropomyosin cannot bind to actin. If
tropomyosin cannot bind actin, myosin can. The presence of ATP means that it will be
available for myosin to hydrolyze, causing muscle contraction.
In experiment 6, the muscle will remain relaxed. The presence of Ca++ will induce a
conformational change in troponin that causes tropomyosin to shift, exposing actin for
myosin to bind. However, when myosin binds, the myosin molecule will attach and then
release because the myosin will bind the nonhydrolyzable analogue of ATP. Because no
ATP hydrolysis can occur, the muscle will remain in the relaxed state.
Extra credit: The muscle will be in a rigor state. If Ca++ is added without ATP,
troponin can bind Ca++, undergo a conformational change, which causes tropomyosin to
shift and exposes the actin for the myosin to bind. However, myosin will bind actin and
remain in the attached state, because a myosin head lacking a bound nucleotide is locked
onto the actin until nucleotide binding occurs.
301

17-24 A.

B.

C.

You would expect the kinesin molecule to travel 150 steps. The calculation is as
follows: 0.3 m = 300 nm. Therefore, in 4 seconds, the kinesin molecule could
travel 1200 nm if it were to move at a rate of 0.3 m/second. Since the step size
is 8 nm, 1200 nm/8 nm/step = 150 steps.
Yes, you should see silica beads detach sometime during the four seconds of
observation, as each bead will take more than 100 steps in that 4-second time
frame (see A above).
If you were to add AMP-PNP, you would no longer see the silica beads moving
down the microtubule. It is thought that one molecule of ATP is hydrolyzed per
step that kinesin takes; without ATP hydrolysis, translocation of the beads will be
inhibited. However, you may still see the beads associated with the microtubules,
as AMP-PNP does not inhibit the association of kinesin with the microtubule.

302

CHAPTER 18
CELL DIVISION
2009 Garland Science Publishing
3rd Edition

An Overview of M Phase
18-1 For each of the following sentences, fill in the blanks with the best word or phrase
selected from the list below. Not all words or phrases will be used; each word or phrase should
be used only once.
The cell division cycle encompasses an alternation between __________________,
which appears as a period of dramatic activity under the microscope, and a preparative
period called __________________, which consists of three phases called
__________________, __________________, and __________________. During M
phase, the nucleus is divided in a process called __________________, and the
cytoplasm is split in two in a process called __________________. The cell-cycle
control system relies on sharp increases in the activities of regulatory proteins called
__________________ or __________________ to trigger S and M phases. Inactivation
of __________________ is required to terminate M phase after chromosome
segregation.
APC
Cdks
condensation
cyclin-dependent kinases
cytokinesis
G1 phase
G1-Cdk

G2 phase
interphas
intraphase
kinesins
M phase
M-Cdk
meiosis

metaphase
microtubules
mitosis
myosins
S phase
S-Cdk

18-2 Which of the following does not occur during M phase in animal cells?
(a) Condensation of chromosomes
(b) Break down of nuclear envelope
(c) Loosening of attachments to other cells
(d) Growth of the cell
(e) Fragmentation of the endoplasmic reticulum

317

19-3

Before chromosome segregation in M phase, the chromosomes and the segregation


machinery must be appropriately prepared. Indicate whether the following statements are
TRUE or FALSE. If FALSE, change a single noun to make the statement TRUE.
A.
Sister chromatids are held together by condensins from the time they arise by
DNA replication until the time they separate near the end of mitosis.
B.
Cohesins are required to make the chromosomes more compact and thus to
prevent tangling between different chromosomes.
C.
The mitotic spindle is comprised of actin filaments and myosin filaments.
D.
Microtubule-dependent motor proteins and microtubule polymerization and
depolymerization are responsible for the organized movements of chromosomes
during mitosis.
E.
The centromere nucleates a radial array of microtubules called an aster, and its
duplication is triggered by S-Cdk.
F.
Each microtubule-organizing center contains a pair of centrioles and hundreds of
-tubulin rings that nucleate the growth of microtubules.

19-4

Describe the names, primary molecular components, and basic mechanisms of the two
distinct cytoskeletal machines that are assembled to carry out the mechanical processes of
mitosis and cytokinesis.

19-5

Examine the schematic representation of centrosome duplication in Figure Q19-5. By


analogy with DNA replication, would you classify centrosome duplication as
conservative or semi-conservative? Explain your answer.

Figure Q19-5

318

Mitosis
19-6

Which stage of mitosis in an animal cell does each part of Figure Q19-6 represent?

Figure Q19-6
19-7

A living cell from the lung epithilium of a newt is shown at different stages in M phase in
Figure Q19-7. Order these light micrographs into the correct sequence and identify the
stage in M phase that each represents.

Figure Q19-7

319

19-8

Name the stage of M phase in which the following events occur. Place the numbers 1-8
next to the letter headings to indicate the normal order of events.
A.
Alignment of the chromosomes at the spindle equator
B.
Attachment of microtubules to chromosomes
C.
Breakdown of nuclear envelope
D.
Pinching of cell in two
E.
Separation of two centrosomes to form mitotic spindle
F.
Reformation of the nuclear envelope
G.
Condensation of the chromosomes
H.
Separation of sister chromatids

19-9

The cytoskeleton of an animal cell changes dramatically between G1 and early M phase
(prophase) of the cell cycle. For each of the following sentences, choose one of the
options enclosed in square brackets that best describes the changes to the cytoskeleton
and its components.
Before mitosis, the number of centrosomes must [increase/decrease]. At
the beginning of [anaphase/prophase] in animal cells, the centrosomes
separate in a process driven partly by interactions between the
[plus/minus] ends of microtubules arising from different centrosomes.
This separation forms the bipolar mitotic spindle and is associated with a
sudden [increase/decrease] in the dynamic instability of microtubules.
Compared to an interphase microtubule array, a mitotic aster contains a
[smaller/larger] number of [longer/shorter] microtubules. Extracts from M
phase cells exhibit [increased/decreased/unchanged] rates of microtubule
polymerization and increased frequencies of [catastrophes/rescues]. The
changes in microtubule dynamics are largely due to [enhanced/reduced]
activity of MAPs and [increased/decreased] activity of catastrophins. The
new balance between polymerization and depolymerization of tubulin is
necessary for the microtubules to bind the [sister chromatids/daughter
chromosomes] and align them at the metaphase plate.

19-10 Match each of the main classes of spindle microtubules from List 1 with their functions
and features from List 2.
List 1
A. Interpolar microtubules
B. Aster microtubules
C. Kinetochore microtubules

List 2
1. Stabilized by interactions with each other via
motor proteins
2. Interact with the cell cortex
3. Link chromosomes to a spindle pole
4. Required for separation of duplicated
centrosomes
5. Depolymerize to promote anaphase A
6. Depolymerize to promote anaphase B

320

19-11 Disassembly of the nuclear envelope


(a)
occurs during prophase.
(b)
involves the separation of the inner nuclear membrane from the outer nuclear
membrane.
(c)
results in the conversion of the nuclear envelope into protein-free membrane
vesicles.
(d)
is triggered by the phosphorylation of integrins.
(e)
must occur in order for kinetochore microtubules to form in animal cells.
19-12 Consider an animal cell that has eight chromosomes (four pairs of homologous
chromosomes) in G1 phase. Indicate how many of each of the following features the cell
will have at mitotic prophase.
A.
Sister chromatids
B.
Centromeres
C.
Kinetochores
D.
Centrosomes
E.
Centrioles
19-13 Which of the following statements about kinetochores is TRUE?
(a)
A kinetochore often binds to more than one microtubule.
(b)
They contain DNA-binding proteins that recognize sequences at the telomere of
the chromosome.
(c)
Kinetochore proteins bind to the tubulin molecules at the very tip of the plus end
of microtubules.
(d)
Kinetochores assemble on chromosomes that lack centromeres.
(e)
Kinetochores on sister chromatids face in the same direction.
19-14 Is the following statement TRUE or FALSE? Explain your answer.
After the nuclear envelope breaks down, microtubules gain access to the
chromosomes and, every so often, a randomly probing microtubule
connects with a kinetochore and captures the chromosome.
19-15 A friend declares that chromosomes are held at the metaphase plate by microtubules that
push on each chromosome from opposite sides. Which of the following observations best
supports your belief that microtubules are pulling on the chromosomes, not pushing
them?
(a)
The jiggling movement of chromosomes at the metaphase plate.
(b)
The way that chromosomes behave when the attachment between sister
chromatids is severed.
(c)
The way that chromosomes behave when the attachment to one kinetochore is
severed.
(d)
The shape of chromosomes as they move toward the spindle poles at anaphase.
(e)
The behavior of the spindle when colchicine is added to cells.

321

19-16 It is estimated that as many as 25% of kinetochore microtubules and 75% of interpolar
microtubules are not anchored to the centrosome. In spite of that, all the microtubules are
focused tightly at the spindle pole. Why do you suppose the microtubule ends that are
not attached to centrosomes do not splay out away from the poles? Hint: Think about
how minus- and plus-end directed motor proteins can move and what the motors might
bind.
19-17 Many anticancer drugs act on microtubules. It was, perhaps, initially surprising that high
concentrations of some successful anticancer drugs (like colchicines) inhibit microtubule
polymerization and others (like taxol) stabilize microtubules. At lower concentrations,
both classes of drugs appear to block mitosis and induce apoptotic death of proliferating
cells. Why might these drugs with opposite effects on microtubules both prevent
mitosis? At what stage of mitosis do you expect treated cells will be blocked? What will
happen if cells treated with these drugs do not arrest at a particular stage of mitosis?
19-18 Which of the following statements is TRUE?
(a)
Anaphase is triggered by the phosphorylation of proteins that hold the sister
chromatids together.
(b)
Anaphase A must be completed before anaphase B can take place.
(c)
In cells where anaphase B predominates, the spindle will elongate much less than
in cells where anaphase A dominates.
(d)
In anaphase A, both kinetochore and polar microtubules shorten.
(e)
In anaphase B, microtubules associated with the cell cortex shorten.
19-19 Which of the following is expected to impair anaphase B and not anaphase A? Explain
your answer.
(a)
An antibody against myosin
(b)
ATPS, a nonhydrolyzable ATP analog that binds to and inhibits ATPases
(c)
An antibody against motor proteins that move from the plus end of microtubules
to the minus end
(d)
An antibody against motor proteins that move from the minus end of microtubules
towards the plus end
(e)
Colchicine
19-20 Which of the following precede re-formation of the nuclear envelope during M phase in
animal cells?
(a)
Formation of the contractile ring
(b)
Decondensation of chromosomes
(c)
Reassembly of the nuclear lamina
(d)
Activation of the anaphase promoting comples (APC)
(e)
Transcription of genes on chromosomes

322

19-21 A serine to alanine mutation in the phosphorylation site of a lamin protein will
(a)
cause cells to arrest at telophase.
(b)
stabilize the nuclear lamina and thereby prevent its disassembly during mitosis.
(c)
prevent nuclear pore assembly.
(d)
prevent mitotic spindle formation.
(e)
prevent chromosome condensation.
19-22 Which of the following structures break into fragments prior to cell division?
(a)
Mitochondria
(b)
Nuclear membrane
(c)
Centrosomes
(d)
Lysosomes
(e)
Ribosomes
(f)
Chloroplasts
(g)
Endoplasmic reticulum

How We Know: Building the Mitotic Spindle


19-23 The lengths of microtubules in various stages of mitosis depend on the balance between
the activities of catastrophins, which destabilize microtubules, and microtubuleassociated proteins (MAPs), which stabilize them. If you created cells with an increased
number of catastrophin molecules, do you predict the length of the mitotic spindle will be
longer, shorter, or unchanged relative to the corresponding stage of mitosis in wild-type
cells? What do you predict for a cell with increased numbers of MAPs? Explain your
reasoning.

323

19-24 Last summer, while working in the laboratory of Professor Mike Rotubulis, you
discovered a new protein that regulates microtubule dynamics. First, you isolated
proteins from a cellular extract that bound to a tubulin affinity column. You then
separated the proteins from each other by loading the mixture of proteins on an ionexchange column, eluting the column with increasing salt concentration and collecting
small fractions of protein as they dripped from the column. To test if each fraction
contained microtubule regulators, you mixed it with fluorescent tubulin and purified
centrosomes and analyzed the reaction microscopically to measure the size of the aster
formed. You found that fractions 8, 9, and 10 promoted formation of an unusually large
aster, consisting of long microtubules. Because electrophoretic separation of the
fractions on a gel revealed a plentiful protein with an apparent molecular weight of
98 kD, you named the protein p98.
A.
Does p98 behave like a MAP or a catastrophin? Hint: Does p98 promote or
inhibit microtubule growth?
B.
Propose two ways that p98 might change the dynamic behavior of microtubules to
account for the observed change in microtubule length. Hint: There are four
simple possible mechanisms.
C.
Video microscopy of fluorescent tubulin in reactions with purified centrosomes
allowed you to follow the behavior of individual microtubules over time. You
graphed the changes in microtubule length in the absence (Figure Q19-24A) and
presence (Figure Q19-24B) of recombinant p98. Five representative microtubules
are shown for each condition. Does p98 alter the rate of microtubule growth or
shrinkage? Does p98 alter the frequency of catastrophe or rescue? Explain your
answers.
D.
After demonstrating the consequences of p98 on microtubule dynamics in vitro
using purified components, you want to determine if the protein has the same
effects in a complex cellular extract that naturally contains p98. Briefly describe
an experiment that will allow you to determine what p98 normally does in mitotic
extracts.

Figure Q19-24

324

Cytokinesis
19-25 If a cell just entering mitosis is treated with nocodazole, which destabilizes microtubules,
the nuclear envelope breaks down and chromosomes condense, but no spindle forms and
mitosis arrests. Why do cells arrest in mitosis?
19-26 The cleavage furrow
(a)
is a puckering of the plasma membrane caused by constriction of a ring of
filaments attached to the membrane.
(b)
begins to form at the end of telophase.
(c)
will not begin to form in the absence of a mitotic spindle.
(d)
always forms perpendicular to the interpolar microtubules.
(e)
always forms in the middle of the cell.
19-27 Cytokinesis in animal cells
(a)
requires ATP.
(b)
leaves a small circular scar of actin filaments on the inner surface of the plasma
membrane.
(c)
is often followed by phosphorylation of integrins in the plasma membrane.
(d)
is generally accompanied by rearrangement of filaments making up the cell
cortex.
(e)
is assisted by motor proteins that pull on microtubules attached to the cell cortex.

325

Answers
19-1

The cell division cycle encompasses an alternation between M phase, which appears as a
period of dramatic activity under the microscope, and a preparative period called
interphase, which consists of three phases called G1 phase, S phase, and G2 phase.
During M phase, the nucleus is divided in a process called mitosis, and the rest of the cell
is split in two in a process called cytokinesis. The cell-cycle control system relies on
sharp increases in the activities of regulatory proteins called cyclin-dependent kinases or
Cdks to trigger S and M phases. Inactivation of M-Cdk is required to terminate M phase
after chromosome segregation.

19-2

(d)

Cell size increases throughout interphase and not during M phase. All of the
other phenomena are observed in M phase.

19-3

A.

False. Sister chromatids are held together by cohesins from the time they arise by
DNA replication until the time they separate near the end of mitosis.
False. Condensins are required to make the chromosomes more compact and
thus to prevent tangling between different chromosomes.
False. The contractile ring is comprised of actin filaments and myosin filaments.
True.
False. The centrosome nucleates a radial array of microtubules called an aster,
and its duplication is triggered by S-Cdk.
True.

B.
C.
D.
E.
F.
19-4

Mitosis, or the separation of chromosomes and their distribution to daughter cells, is


accomplished by the bipolar mitotic spindle. The spindle is composed of microtubules
and a variety of microtubule-dependent motor proteins. It is bipolar because in most
animal cells two centrosomes, or microtubule organizing centers, are located at opposite
poles of the cell and nucleate asters of microtubules that can interact with each other.
Microtubules and microtubule motors also interact with the chromosomes and control
their movement. Cytokinesis, or the division of an animal cell into two daughter cells, is
accomplished by the contractile ring, which is composed of actin and myosin filaments
and is located just under the plasma membrane. As the ring constricts, it pulls the
membrane inward, ultimately dividing the cell in two.

19-5

Centrosome duplication is semi-conservative. The paired centrioles in the centrosome


separate, and each serves to nucleate synthesis of a new centriole. As a consequence,
each new centrosome consists of one old and one new centriole. Thus, centrosome
duplication is analogous to DNA replication in which a new duplex consists of one old
strand and one newly replicated strand.

19-6

Atelophase; Bprophase; Canaphase; Dprometaphase

19-7

The correct order (and stage) are listed as follows: E (prophase), D (prometaphase), C
(metaphase), A (anaphase), F (telophase), and B (cytokinesis).

326

19-8

A.
B.
C.
D.
E.
F.
G.
H.

5, metaphase
4, prometaphase
3, prometaphase
8, cytokinesis
2, prophase
7, telophase
1, prophase
6, anaphase

19-9

Before mitosis, the number of centrosomes must increase. At the beginning of prophase
in animal cells, the centrosomes separate in a process driven partly by interactions
between the plus ends of microtubules arising from different centrosomes. This
separation forms the bipolar mitotic spindle and is associated with a sudden increase in
the dynamic instability of microtubules. Compared to an interphase microtubule array, a
mitotic aster contains a larger number of shorter microtubules. Extracts from M phase
cells exhibit unchanged rates of microtubule polymerization and increased frequencies
of catastrophes. The changes in microtubule dynamics are largely due to reduced
activity of MAPs and increased activity of catastrophins. The new balance between
polymerization and depolymerization of tubulin is necessary for the microtubules to bind
the sister chromatids and align them at the metaphase plate.

19-10 A1, 4; B2, 6; C3, 5


19-11 Choice (e) is the correct answer. In animal cells, kinetochore microtubules cannot form if
the chromosomes in the nucleus are separated from the microtubules in the cytoplasm by
the nuclear envelope. (But in some other cells, like the yeast S. cerevisiae, the nuclear
envelope never breaks down and yet chromosomes can attach to microtubules emanating
from the spindle poles within the nucleus.) The nuclear envelope disassembles during
prometaphase (so choice (a) is false) by breaking up into vesicles containing lipids from
both the outer and inner envelope (so choice (b) is false). Integral membrane proteins of
the nuclear envelope and some of the nuclear lamins remain associated with the vesicles
(so choice (c) is false). Phosphorylation of lamins (not integrins) triggers breakdown of
the nuclear lamina (so choice (d) is false).
19-12 A16; B16; C16; D2; E4
19-13 Choice (a) is the correct answer. The DNA-binding proteins of the kinetochore recognize
the centromere, not the telomere (choice (b)), and thus cannot bind to chromosomes
lacking centromeres (choice (d)). If kinetochore proteins bound to the very tips of
microtubules, they would soon fall off, since the microtubules are dynamic and are
constantly losing the monomers at the ends of the filament (choice (c)). Kinetochores on
sister chromatids must face in opposite directions to facilitate their attachment to
microtubules from opposite spindle poles (choice (e)).

327

19-14 True. Microtubules nucleated by the centrosomes grow outward toward the
chromosomes in a highly dynamic process, alternately growing and shrinking. When
they eventually attach to the kinetochore of a chromosome, they become stabilized and
are referred to as kinetochore microtubules.
19-15 Choice (c) is the correct answer. When the attachment to one kinetochore is severed, the
whole chromosome moves to the opposite pole, showing that the kinetochore
microtubules are pulling on their attached chromatid, not pushing it. The jiggling
movement (choice (a)) is simply a sign that the chromosomes are subject to forces from
both sides. When the attachment between sister chromatids is severed (choice (b)), both
daughter chromosomes move toward their respective poles, but they could either be
pushed there or pulled there. Similarly, the shape of the chromosomes as they move
toward the pole (choice (d)) indicates that the chromosomes are being moved via an
attachment at the centromere, but does not indicate whether they are being pushed or
pulled. The disappearance of the spindle when colchicine is added to the cells simply
shows that the spindle is composed of polymerizing and depolymerizing microtubules
(choice (e)).
19-16 Microtubules that are not directly connected to the centrosomes are held in place by
minus-end directed motors, which link them to the other microtubules and tend to walk
them toward the spindle pole.
19-17 Mitosis requires that the microtubules be dynamically growing and shrinking to probe the
cell cortex and seek attachments to kinetochores. Progression through mitosis requires
that sister kinetochores be attached to opposite spindle poles and that dynamic
microtubules exert opposing tension on the sister kinetochores. Static microtubules will
be unable to execute these requirements, thus cells are likely to arrest prior to anaphase
by virtue of the spindle assembly checkpoint. If cells do not arrest, cytokinesis will occur
despite the absence of the microtubule-dependent movements of anaphase A and
anaphase B that serve to segregate chromosomes to opposite poles of the cell. The failure
of chromosome segregation will produce multinucleate cells or cells with an aberrant
number of chromosomes.
19-18 Choice (e) is the correct answer. Anaphase is triggered by proteases that cleave the
proteins that hold the sister chromatids together, thus (a) is false. Anaphase A and
anaphase B generally occur at the same time, thus (b) is false. In cells where anaphase B
predominates, the spindle will elongate more than in cells where anaphase A
predominates, thus (c) is false. In anaphase A, only the kinetochore microtubules shorten,
thus (d) is false.

328

19-19 Choice (d) is the correct answer. The motor proteins used in anaphase B to push the
interpolar microtubules apart move toward the plus end of microtubules, whereas the
motor proteins used in anaphase A move toward the minus end. Myosin (choice (a)) is
not involved in either anaphase A or anaphase B. Motor proteins require ATP hydrolysis
(they are ATPases) and are used in both anaphase A and anaphase B, so both types of
anaphase will be affected by ATPS (choice (b)). The motor proteins used in anaphase A
move toward the minus end of microtubules as do the motor proteins attached to the cell
cortex used in anaphase B (choice (c)). Microtubules are also used in both anaphase A
and anaphase B, so colchicine will affect both (choice (e)).
19-20 The correct answer is choices (a) and (d). Activation of the APC triggers anaphase,
which must occur prior to the re-formation of the nuclear envelope in telophase. The
contractile ring in animals begins to assemble in anaphase. The chromosomes do not
decondense and the lamina cannot be assembled until the formation of the nuclear
envelope is complete and nuclear proteins (including most of the lamins) have been
imported through the pores. Transcription does not begin until the chromosomes
decondense.
19-21 Choice (b) is the correct answer. A Ser to Ala mutation in the phosphorylation site of a
lamin will prevent the lamin from being phosphorylated. Since the nonphosphorylated
form is the form that polymerizes into a lattice, such a mutation will stabilize the nuclear
lamina and prevent its dissasembly and nuclear envelope breakdown at prometaphase.
Telophase involves repolymerization of lamins and therefore will not be blocked by the
mutation (choice (a)). Lamins are not part of the nuclear pore complex (choice (c)). The
mitotic spindle begins to form before the nucleus has broken down, forming a sort of
cage around the nucleus (choice (d)). Lamins are not involved in chromosome
condensation (choice (e)).
19-22 The correct answer is choices (b) and (g). The ER and the nuclear membrane (which is
continuous with the ER) both fragment at mitosis. Mitochondria, lysosomes, ribosomes,
and chloroplasts are all small and numerous organelles and do not need to be broken into
small pieces to ensure roughly equal segregation to both daughter cells. Centrosomes do
not fragment: they duplicate in an orderly and elaborate way that is poorly understood.

329

19-23 In a cell with excessive catastrophin molecules, the balance between catastrophins and
MAPs will be disrupted. Increased catastrophin activity relative to the MAP activity will
lead to an increased frequency of microtubule catastrophes, the sudden shift from growth
to shrinkage. Thus microtubules will be shorter on average because they spend less time
growing slowly and more time shrinking rapidly. Shorter microtubules may be
assembled into a shorter mitotic spindle. Conversely, increased MAP activity relative to
catastrophin activity will stabilize microtubules by enhancing polymerization or
inhibiting depolymerization, thereby promoting formation of longer microtubules and
perhaps a longer mitotic spindle. However, in some cases the normal balance between
catastrophins and MAPs has been shown to be necessary for formation of a bipolar
spindle. It is possible that increasing the amount of MAP or catastrophin regulatory
proteins will cause formation of microtubules that are too long or too short to be
assembled into a spindle at all.
19-24 A.
B.

C.

D.

p98 behaves like a MAP, or microtubule associated protein, because it promotes


microtubule growth.
The four possible mechanisms by which a protein can promote microtubule
growth are (1) increasing the rate of polymerization, (2) decreasing the rate of
depolymerization, (3) inhibiting the occurrence of catastrophes (when a
microtubule abruptly shifts from growing to shrinking), and (4) stimulating the
occurrence of rescues (when a microtubule switches from shrinking to growing).
p98 increases the rate of microtubule growth, as seen by the steeper slopes of the
lines when p98 was added. The data do not allow a conclusion about the rate of
microtubule shrinkage. The protein decreases the rate of catastrophes: two of the
five microtubules in the absence of p98 underwent catastrophe (a sudden and
rapid decline in microtubule length), whereas none did so in the presence of p98.
No rescues were observed, so it is uncertain if p98 alters the rescue rate.
The p98 protein can be removed from a Xenopus egg mitotic extract using
antibodies that specifically recognize p98. The depleted extract with little to no
p98 protein can then be mixed with sperm nuclei, centrosomes, and fluorescent
tubulin. Based on the previous experiments, the p98-depleted extract would be
expected to have shorter and more dynamic microtubules. Ideally, this kind of
experiment would be supplemented with the production and examination of intact
cells in which p98 function has been abolished by mutation or another means.

19-25 Cells arrest in mitosis due to a checkpoint. Nocodozole treatment disassembles


microtubules, preventing formation of a spindle. Because the nucleus breaks down and
chromosomes condense, these events must be independent of aster formation by
centrosomes, for example, and of any other microtubule-dependent process. Mitosis
finally arrests because the unattached chromosomes trigger a signal that engages the
spindle-attachment checkpoint, which halts the cycle.

330

19-26 The correct answer is choices (a), (c), and (d). The cleavage furrow begins to form
before the end of telophase, thus (b) is incorrect. Although the furrow eventually
becomes independent of the mitotic spindle, the furrow requires the mitotic spindle for its
establishment. The furrow always forms perpendicular to the interpolar microtubules
about midway between the poles, but since the spindle is normally displaced in some
cells, cell division does not always occur in the middle of the cell, thus (e) is incorrect.
19-27 The correct answer is choices (a) and (d). All cell movement requires ATP, and in
cytokinesis actin and myosin molecules are moving relative to one another to cause
contraction of the contractile ring. The myosin is an ATPase that hydrolyzes ATP to
power this movement. The assembly of the contractile ring requires a general
rearrangement of the filaments in the cell cortex. The contractile ring completely
disassembles after mitosis (choice (b)). Phosphorylation of integrins, which weakens their
hold on the extracellular matrix and allows cells to round up, generally precedes
cytokinesis and is part of the general rearrangement of cell structure (including the cell
cortex) that accompanies cell division (choice (c)). Microtubules do not play an important
role in animal cytokinesis (choice (e)).

331

CHAPTER 19
GENETICS, MEIOSIS, AND THE
MOLECULAR BASIS OF HEREDITY
2009 Garland Science Publishing
3rd Edition

The Benefits of Sex


19-1 Why is sexual reproduction more beneficial to a species living in an unpredictable
environment than to one living in a constant environment?
19-2 Which of the following statements is TRUE?
(a) Diploid organisms reproduce only sexually.
(b) All sexually reproducing organisms have at least two copies of each gene at some stage in
their life cycle.
(c) Gametes have only one chromosome.
(d) Another name for the unfertilized egg cell is the zygote.
(e) Mutations in somatic cells are passed on to individuals of the next generation.
19-3 Imagine a diploid sexually reproducing organism, Diploidus sexualis, that contains three
pairs of chromosomes. This organism is unusual in that no recombination between
homologous chromosomes occurs during meiosis.
A. Assuming that the chromosomes are distributed independently during meiosis, how many
different types of sperm or egg cells can a single individual of this species produce?
B. What is the likelihood that two siblings of this species will be genetically identical?

19-4 A certain type of worm has two genders: males that produce sperm and hermaphrodites
that produce both sperm and eggs. The diploid adult has four homologous pairs of
chromosomes that undergo very little recombination. Given a choice, the hermaphrodites prefer
to mate with males, but just to annoy the worm, you pluck a hermaphrodite out of the wild and
fertilize its eggs with its own sperm. Assuming all the resulting offspring are viable, what
fraction do you expect to be genetically identical to the parent worm? Assume that each
chromosome in the original hermaphrodite is genetically distinct from its homologue.
(a)
(b)
(c)
(d)
(e)

All
None
Half
1/16
1/256
333

20-5

For each of the following sentences, fill in the blanks with the best word or phrase
selected from the list below. Not all words or phrases will be used; each words or phrase
may be used more than once.
To reproduce sexually, an organism must create haploid
__________________ cells, or __________________, from diploid cells
via a specialized cell division called __________________. During
mating, the fathers haploid cells, called __________________ in animals,
fuse with the mothers haploid cells, called __________________. Cell
fusion produces a diploid cell called a __________________, which
undergoes many rounds of cell division to create the entire body of the
new individual. The cells produced from the initial fusion event include
__________________ cells that form most of the tissues of the body as
well as the __________________-line cells that give rise to the next
generation of progeny.
allele
bivalent
eggs
gametes

germ
meiosis
mitosis
pedigree

pollen
somatic
sperm
zygote

20-6

Which of the following statements is FALSE?


(a)
Asexual reproduction typically gives rise to offspring that are genetically
identical.
(b)
Germ-line cells have half the number of chromosomes as somatic cells.
(c)
Sexual reproduction allows for a wide variety of gene combinations.
(d)
A zygote can only be produced through asexual reproduction.
(e)
Gametes are specialized sex cells.

20-7

Is the following statement TRUE or FALSE? Explain.


Somatic cells leave no progeny and thus, in an evolutionary sense, exist
only to help create, sustain, and propagate the germ cells.

20-8

Sexual reproduction is a large drain on the limited resources of an individual.


Nonetheless, sexual reproduction is common. In fact, to allow sexual reproduction, many
elaborate anatomical structures, cellular processes, and chemical signals have evolved.
For example, flowers exist entirely to further the goal of sexual reproduction and many
plants have enlisted the help of bees and birds to aid in the dissemination of their germ
cells. Describe one reason why the majority of multicellular organisms have evolved to
reproduce sexually instead of relying solely on asexual reproduction.

334

Meiosis
20-9

Indicate whether the following is TRUE for meiosis, mitosis, both, or neither.
A.
Formation of a bivalent
B.
Products are genetically identical
C.
Chromosomes condense
D.
All paternal chromosomes segregate to one cell
E.
Involves DNA replication

20-10 Meiosis is a highly specialized cell division in which several events occur in a precisely
defined order. Please order the meiotic events listed below.
1.
Loss of cohesins near centromeres
2.
Chromatid pairing
3.
Chromosome condensation
4.
Chromosome replication
5.
Degradation of cohesins bound to chromosome arms
6.
Formation of chiasmata (chiasmata = plural of chiasma)
7.
Homolog pairing
8.
Alignment of chromosomes at the metaphase plate
20-11 You have received exactly half of your genetic material from your mother, who received
exactly half of her genetic material from her mother (your grandmother).
A.
Explain why it is unlikely that you share exactly one quarter of your genetic
material with your grandmother, and instead it is more accurate to say that in
general people receive an average of one quarter of their genetic endowment from
each grandparent.
B.
Consider a gene on chromosome 3 that you received from your grandmother. Is it
likely you received an entire chromosome 3 from your grandmother? Why or
why not?
C.
What portion of your genetic material do you share with your sibling? Your aunt?
Your cousin?

335

20-12 Figure Q20-12 is a diagram of chromosomes during meiosis.

Figure Q20-12
A.
B.

On the diagram, indicate which label lines correspond to the following items:
(1) sister chromatids, (2) homologous chromosomes, (3) bivalent, (4) chiasma.
On the figure, draw as small circles (oooo) the cohesin glue that is released in
meiotic division I, and draw as small crosses (xxxx) the cohesin glue that is
released in meiotic division II.

20-13 For each of the following sentences, choose one of the options enclosed in square
brackets to make a correct statement.
Starting with a single diploid cell, mitosis produces [two/four]
[identical/different] [haploid/diploid] cells, whereas meiosis yields
[two/four] [identical/different] [haploid/diploid] cells. This is
accomplished in meiosis because a single round of chromosome
[replication/segregation] is followed by two sequential rounds of
[replication/segregation]. Mitosis is more like meiosis [I/II] than meiosis
[I/II]. In meiosis I, the kinetochores on sister chromatids behave
[independently/coordinately] and thus attach to microtubules from the
[same/opposite] spindle. The cohesin-mediated glue between
[chromatids/homologs] is regulated differently near the centromeres than
along the chromosome arms. Cohesion is lost first at the
[centromeres/arms] to allow segregation of [chromatids/homologs] and is
lost later at the [centromeres/arms] to trigger segregation of
[chromatids/homologs].

336

20-14 For each of the following sentences, fill in the blanks with the best word or phrase
selected from the list below. Not all words or phrases will be used; each word or phrase
should be used only once.
An organism that has five homologous pairs of chromosomes can produce,
in the absence of recombination, __________________ genetically
different types of gametes. Recombination will __________________ the
number of genetically different gametes possible.
5
8
10

15
25
32

decrease
increase

20-15 Meiosis includes a recombination checkpoint that is analogous to the checkpoints in cell
cycle progression. Recombination in meiosis is initiated by double-stranded breaks in the
DNA. The broken end of a DNA molecule finds the corresponding sequence on a
homologous chromosome and exchanges a chromosomal segment with its homolog,
thereby repairing the break. Ongoing recombination sends a negative regulatory signal
that prevents cells from entering meiotic division I.
A.
Mutations in several genes inactivate the recombination checkpoint. What do you
predict will happen if a cell proceeds through meiotic division I before completing
recombination?
B.
What will happen if a cell fails to initiate recombination and proceeds through
meiotic division I? Meiotic division II?

337

20-16 In some fungi, cell division during meiosis gives rise to an ordered spore sac containing a
row of four haploid spores, as shown in Figure Q20-16A. The position of each spore
within the sac reflects its relation to its neighbors, i.e., spores that result from the same
meiosis II division are positioned next to each other. You notice that a strain of the
fungus produced by crossing a dark-colored strain with a light-colored strain gives rise
mostly to spore sacs as shown in Figure Q20-16B, with a few spore sacs like those in
Figure Q20-16C. Which of the following can be concluded from the above observations?
Explain your reasoning.

Figure Q20-16
(a)
(b)
(c)
(d)
(e)

Meiosis I and meiosis II in your fungus occur in the reverse order from that which
occurs in humans.
Recombination in the fungus can occur during prophase I.
Recombination in the fungus cannot occur during prophase II.
The spore sacs in panel C result from recombination between the centromere and
the gene responsible for spore color.
Recombination has occurred between the gene responsible for spore color and the
end of the chromosome arm.

338

20-17 In mammals, there are two sex chromosomes, X and Y, which behave like homologous
chromosomes during meiosis. Normal males have one X chromosome and one Y
chromosome, and normal females have two X chromosomes. Males with an extra Y
chromosome (XYY) occasionally are found. Which of the following could give rise to
such an XYY male? Explain your answer.
(a)
Nondisjunction in the first meiotic division of spermatogenesis; normal meiosis in
the mother.
(b)
Nondisjunction in the second meiotic division of spermatogenesis; normal
meiosis in the mother.
(c)
Nondisjunction in the first meiotic division of oogenesis; normal meiosis in the
father.
(d)
Nondisjunction in the second meiotic division of oogenesis; normal meiosis in the
father.
(e)
Nondisjunction in the first meiotic division of both the mother and fathers
gametes.

Mendel and the Laws of Inheritance


20-18 Is the following statement TRUE or FALSE? Explain.
The phenotype of an organism reflects all of the alleles carried by that
individual.
20-19 Do you agree or disagree with the following statement? Explain your answer.
If a diploid organism has 16 chromosomes (and thus 8 pairs of
homologous chromosomes), that organism can produce only 28 genetically
different gametes.
20-20 With respect to gene E on the chromosome drawn in Figure Q20-20, which gene is least
likely to behave according to Mendels law of independent assortment? Explain your
answer.

Figure 20.20

339

20-21 Cystic fibrosis is due to mutations in a single gene that lies on chromosome 7. Only
homozygous mutant (ff) individuals are sick; homozygous wild-type (FF) and
heterozygous(Ff) individuals are healthy. A healthy married couple has one child with
cystic fibrosis and the wife is pregnant with a second child.
A.
What is the genotype of the mother? The father?
B.
What is the chance that the second child will have cystic fibrosis?
20-22 Sickle cell anemia is caused by a mutation in the hemoglobin allele. Individuals with two
mutant alleles have sickle cell anemia. Individuals homozygous and heterozygous for the
mutant gene are more resistant to malaria than those with two wild-type alleles. Is this
mutation dominant, recessive, or co-dominant?

340

20-23 Your friend has obtained some pea seeds from the Abbey of St. Thomas in Brno, where
Gregor Mendel worked. He is very excited because not only did he obtain some yellow
and green pea seeds from true-breeding plants (like the ones used in the famous
experiment described in your book), he was also able to obtain some purple pea seeds
from a true-breeding plant. First, your friend takes the true-breeding yellow and green
pea seeds, repeats the cross that Gregor Mendel did, and obtains the same results where
he sees 100% yellow-seeded pea plants in the F1 generation and 75% yellow-seeded pea
plants and 25% green-seeded pea plants in the F2 generation. His results are illustrated in
Figure Q20-23A below. Your friend decides to set up two more crosses. First, for cross
#2, he crosses the true-breeding purple-seeded pea plants to the true-breeding yellowseeded pea plants. The results from this cross are shown in Figure Q20-23B below.
Next, for cross #3, he crosses the true-breeding purple-seeded pea plants to the truebreeding green-seeded pea plants. These results are shown in Figure Q20-23C.

Figure Q20-23
341

Given these results, if you were to take the purple-seeded pea plants produced in the F1
generation in cross #2 and cross them to the purple-seeded pea plants produced in the F1
generation of cross #3, what do you expect the phenotype of the progeny would look
like? Explain your answer.
20-24 You are given two true-breeding strains of hamster. One strain has white fur color while
the other strain has a dark brown fur color. When you cross the white fur strain to the
dark brown fur strain, you obtain F1 progeny that have a light brown fur color. When you
cross the F1 progeny with each other, you obtain 25% of the hamsters having white fur,
25% of the hamsters having dark brown fur, and 50% of the hamsters with light brown
fur. How many genes crucial for fur coloration differ between the two starting strains?
Explain your answer.
Extra credit: Propose a molecular mechanism for how fur color is determined in this
species of hamster.

Genetics as an Experimental Tool


20-25 Conditional alleles are mutant gene versions that encode proteins that can function
normally at the permissive condition but are defective at the restrictive condition; one
commonly used condition is temperature. Conditional alleles are especially useful to
geneticists because they permit the study of essential genes. At the permissive
temperature, the organism lives normally. When the organism is shifted to the nonpermissive temperature, the effect of inactivating the gene can be studied. Which of the
three types of mutations shown in Figure Q20-25 is most likely to lead to a conditional
allele? Explain your answer.

Figure Q20-25

342

20-26 You performed a genetic screen in C. elegans and found two mutations (#1 and #2) that
cause antisocial behavior. You want to learn if these are mutations in the same gene or
different genes, so you cross mutant #1 and mutant #2 together to perform a
complementation test. Imagine that the mutations are in different genes: mutation #1 is
in gene M and mutation #2 in N.

Figure Q20-26
A.

B.

C.
D.

What is the outcome of a complementation test if #1 and #2 are both recessive


mutations (m and n, respectively)? Fill in the empty oval in Figure Q20-28A and
indicate if the offspring exhibits social or antisocial behavior.
What is the outcome if #1 is dominant and #2 is recessive (M* and n)? Fill in the
empty oval in Figure Q20-28B and indicate if the offspring exhibits social or
antisocial behavior.
For the case in A., would the phenotype of the offspring be different if the two
mutations were in the same gene?
For the case in B., would the phenotype of the offspring be different if the two
mutations were in the same gene? Are complementation tests useful for analyzing
dominant mutations?

20-27 Do you agree or disagree with the following statement? Explain your answer.
A trait that is found at a low frequency in the population has to be a
recessive trait.

343

20-28 You are studying a diploid yeast strain that normally utilizes glucose as an energy source
but can use maltose when no glucose is present. You are interested in understanding how
this yeast strain utilizes maltose as an alternative energy source. To begin to understand
maltose metabolism, you undertake a genetic screen to isolate genes involved in maltose
metabolism by screening for yeast that cannot grow when maltose is the sole energy
source. You isolate 6 different mutants, all of which are recessive, and name these alleles
mal1, mal2, mal3, mal4, mal5, and mal6. Next, you isolate gametes from the
homozygous diploid mutant yeast strains and perform crosses between the different
strains to do complementation analysis, because you wish to determine whether the
mutations are likely to affect the same or different genes. Your results are shown in the
table below:
Table Q20-28. Complementation analysis of mal genes.
mutation
mal1
mal2
mal3
mal4
mal5
mal6
mal1

+
+
+

mal2

+
+
+

mal3
+
+

+
mal4
+
+
+

+
+
mal5
+
+

+
mal6

+
+
+

+ = growth on maltose-containing media; = no growth on maltose-containing


media.
In how many genes are you likely to have isolated mutations? Which alleles appear to
affect the same genes? Explain your answer.
20-29 Gene A is located near gene B on chromosome 13 in humans. A mutation in the germ line
of an individual with the haplotype AB generates gametes with the genotype Ab. Many
descendents of this founder individual carry the b mutation, which predisposes carriers to
high blood pressure. Initially, all descendents who inherit the b mutation also inherit the
neighboring A allele. Through the generations, fewer and fewer descendents with the b
mutation carry the A allele, and instead they have the a allele. (Individuals with A and a
are equally healthy and fit.) Explain how the b and A alleles are separated.

344

20-30 Shown in Figure Q20-30 is a genetic pedigree of a family with several members affected
by a heritable disease. Affected individuals are shown in black and healthy individuals
are shown in white. Males are shown as boxes and females as circles. Can a single
mutation explain the pattern of inheritance? Is the mutation responsible for the disease
dominant or recessive? Is the mutation carried on the X chromosome, the Y
chromosome, or an autosome?

Figure Q20-30

How We Know: Reading Genetic Linkage Maps


20-31 You are trying to map a human gene thought to be involved in cat allergies. Because you
know this gene is on chromosome 20, you decide to examine the linkage of several SNPs
located on chromosome 20 with respect to the gene involved in cat allergies. You have
obtained DNA from 10 individuals and know whether they are allergic to cats. Your
SNP results are shown in Table Q20-31.
Table Q20-31. SNP tests for chromosome 20. (+ = presence of SNP)
allergic to
cats?
yes
no
no
yes
yes
yes
no
yes
yes
no
A.
B.

SNP1

SNP2

SNP3

SNP4

SNP5

+
+

+
+

+
+

+
+

+
+
+

+
+

+
+

+
+

+
+

Which SNP is most likely to be tightly linked to the gene involved in cat
allergies? Explain your answer.
Of the SNPs tested above, which is likely to be the next closest to the gene
responsible for the allergic state? Why?

20-32 Any two human beings typically have an estimated 0.1% difference in their nucleotide
sequences, which is equivalent to about 3 million nucleotide differences. These
differences are the basis of the SNPs used to construct genetic linkage maps. Some of
these SNPs actually lie in the region of the DNA that codes for the protein yet have no
effect on the phenotype of individuals carrying the SNP on both homologous
chromosomes. Explain how some SNPs can lie within the portion of the DNA that codes
for the protein and yet have no discernable effect on the proteins activity.
345

Answers
20-1

The real benefit in sexual reproduction seems to be that parents produce children that are
genetically unlike either parent and that are not genetically identical to each other. This
provides more variation in the population than asexual reproduction could provide, and is
an advantage if the environment is variable, since any one combination of the parents
characteristics, however well adapted to the prevailing conditions, may or may not be the
best in a new situation.

20-2

Choice (b) is the correct answer. The sexual life cycle involves an alternation of diploid
and haploid phases; and these haploid cells combine by the process of mating or
fertilization to produce a new diploid. Some diploid organisms (e.g., many plants) are
capable of asexual reproduction (choice (a)). Gametes have only one member of each pair
of homologous chromosomes, but since most organisms have more than one pair of
homologous chromosomes, most gametes have more than one chromosome (choice (c)).
Choice (d) is untrue because the zygote is formed by fusion of the egg and sperm. Choice
(e) is untrue because only mutations in germ cells are passed on to progeny individuals.

20-3

A.

B.

Eight different types. With respect to each of the three chromosomes, an


individual can produce two kinds of gametes. The gamete can receive the copy
that the individual received from mom or the copy from dad. Thus, with
three chromosomes there are 2 2 2 = 8 possible gametes.
1/64. The mother and father together can produce 8 8 = 64 different genetic
combinations.

20-4

(d)

The parent is most likely heterozygous for each chromosome, and so can produce
24 = 16 types of eggs and 24 = 16 types of sperm. Any of the eggs produced will
be able to give rise to an adult that is identical to the parent, but in order to do so,
it must be fertilized by the right type of sperm. For each type of egg, only one of
the 16 possible sperm will produce a diploid that is identical to the parent.
Therefore, 1 out of 16 of the offspring should be identical to the parent. In other
words, a sexually reproducing organism with several chromosomes has a
relatively high probability of producing genetically distinct offspring even when
the parent mates to itself.

20-5

To reproduce sexually, an organism must create haploid germ cells, or gametes, from
diploid cells via a specialized cell division called meiosis. During mating, the fathers
haploid cells, called sperm in animals, fuse with the mothers haploid cells, called eggs.
Cell fusion produces a diploid cell called a zygote, which undergoes many rounds of cell
division to create the entire body of the new individual. The cells produced from the
initial fusion event include somatic cells that form most of the tissues of the body as well
as the germ-line cells that give rise to the next generation of progeny.

20-6

(d)

The fusion of two haploid gametes produces a zygote during sexual reproduction.

346

20-7

True. Only germ cells contribute genetic material to the next generation of organisms,
and thus only germ cells leave an evolutionary legacy in the gene pool of the species.
The only contribution of somatic cells to subsequent generations arises through the
assistance they provide to dissemination of the genetic material in the germ cells. In
accordance with this, mutations that arise in somatic cells are not passed along to
offspring.

20-8

A definitive explanation of the evolutionary advantages of sexual reproduction over


asexual reproduction is elusive, but several benefits seem clear. (1) The reshuffling of
genes that occurs during sexual reproduction generates most of the diversity among
organisms within a species. Having a large variety of different genetic combinations in a
population may help guarantee that at least a few individuals will survive after a sudden
and unpredictable change in the environment. This is why some ecologists are concerned
about the global trend toward agricultural monoculture, i.e., limiting planting variety to
only a small number of genetically identical subspecies of plants. (2) The competition
among individuals in a population for mating partners may increase the frequency of
advantageous or more highly functional genes in the gene pool. This is because
individuals will choose mating partners who are stronger, healthier, and more successful
than their peers. (3) Sexual reproduction may help to eliminate deleterious recessive
genes from a population. A recessive deleterious mutation in a gene may be unmasked
in a haploid germ cell because, unlike the situation in a heterozygous diploid cell, a
normal allele would not be present to provide the gene function. Because there is
competition among germ cells for fertilizing partner cells, gametes with recessive
deleterious mutations are less likely to form zygotes.

20-9

A.
B.
C.
D.
E.

meiosis
mitosis
both
neither
both

20-10 4, 2, 7, 6, 3, 8, 5, 1. Chromosome replication (4) occurs during meiotic S phase, and


immediately after replication the resultant sister chromatids are paired tightly by cohesins
(2). During meiotic G2, or prophase, the two homologous chromosomes pair (7) and
undergo recombination to produce reciprocal exchanges of genetic material that are
visible as chiasmata (6). Meiotic division I begins with chromosome condensation (3)
and proceeds to metaphase, when chromosomes align at a central plane (8). Anaphase of
meiotic division I is triggered by loss of the cohesin glue on the chromosome arms (5),
which allows the homologs to be segregated. Anaphase of meiotic division II is triggered
by degradation of cohesins near the centromere (1), which allows sister chromatids to be
segregated.

347

20-11 A.

B.

C.

20-12 A.
B.

It is unlikely that you share exactly one quarter of your genetic material with your
grandmother, but it is true that organisms receive an average of one quarter of
their genes from each grandparent. This is because when cells in your mothers
germ line were undergoing meiosis, the chromosomes that she received from your
grandmother and your grandfather were shuffled by recombination and then
randomly assorted into the gametes. Each of these gametes probably received
slightly less or slightly more than half of its genetic material from your
grandmother. Thus, fusion of your mothers gamete with your fathers created a
zygote that shared approximately, but not exactly, one quarter of its genes with
your grandmother; this zygote divided repeatedly to form all the cells in your
body.
It is unlikely that you received the entire chromosome 3 from your grandmother,
because each chromosome undergoes at least one recombinational crossover with
its homolog to ensure proper chromosome segregation in meiotic division I.
You share an average of one half of your genetic material with your sibling, one
quarter with your aunt, and one-eighth with your cousin. Since you and your
sibling each get 1/2 of each parents DNA, you have approximately 1/2 of your
genetic material in common with your sibling. As your mother and her sister
share approximately 1/2 of their genetic material, you and your aunt share 1/2
1/2 = 1/4. As your cousin has 1/2 of your aunts genetic material, you share 1/2
1/4 = 1/8.
See Figure A20-12. (1) a, (2) c, (3) b, (4) d.
See Figure A20-12.

Figure A20-12

348

20-13 Starting with a single diploid cell, mitosis produces two identical diploid cells, whereas
meiosis yields four different haploid cells. This is accomplished in meiosis because a
single round of chromosome replication is followed by two sequential rounds of
segregation. Mitosis is more like meiosis II than meiosis I. In meiosis I, the
kinetochores on sister chromatids behave coordinately and thus attach to microtubules
from the same spindle. The cohesin-mediated glue between chromatids is regulated
differently near the centromeres than along the chromosome arms. Cohesion is lost first
at the arms to allow segregation of homologs and is lost later at the centromeres to
trigger segregation of chromatids.
20-14 32; increase. Since homologous chromosomes assort randomly at meiosis and a gamete
has two choices for each chromosome (because sexual organisms are diploid), there are
5
2 , or 32, possible genetically different gametes. Recombination effectively increases the
number of types of chromosomes and therefore increases the number of genetically
different gametes.
20-15 A.

B.

If a cell proceeds through meiotic division I even though its chromosomes are
broken and incompletely repaired, segregation will be disastrous. Some
recombination intermediates will be pulled to opposite spindle poles, thus
breaking the DNA. Other chromosome fragments lacking centromeres will not be
attached to microtubules and thus will segregate randomly, causing some meiotic
products to have too little DNA and others to have too much.
In the absence of recombination, the homologs will not segregate from each other
properly in meiotic division I but the sister chromatids will segregate normally in
meiotic division II. In meiotic division I, the unrecombined homologous
chromosomes will not be held together by chiasmata. Thus, the homologous
chromosomes will line up independently on the metaphase plate and segregate
randomly, causing chromosome nondisjunction. Some products will have both
homologs of a given chromosome and others will have none. In meiotic division
II, the events will proceed normally and sister chromatids will be properly
segregated to opposite poles. Nonetheless, because the chromosome sets at the
start of meiotic division II were unevenly distributed, the gametes produced after
meiotic division II will be aneuploid (i.e., will have an incorrect number of
chromosomes).

349

20-16 (b) and (d) In normal meiosis, the chromosomes are distributed as shown in Figure A2016A (the dark-colored chromosome carries the allele for dark pigment). The
homologs are separated from each other during meiotic division I and the sister
chromatids are separated during meiotic division II. This pattern of chromosome
segregation, in the absence of recombination, always gives the pattern of spores
that was shown in the question (Figure A20-16B). To yield the rare oddball spore
sacs, there must be recombination between the two homologous chromosomes.
Such recombination always occurs when both homologs are in the same cell (i.e.,
in prophase I of meiosis).

Figure A20-16
(a)

(b)

(c)

Incorrect. If meiosis I and meiosis II were reversed, we would expect a


completely random pattern of spores in spore sacs, since the two homologs could
go to either cell in the second division, as shown in Figure Q20-16C. This cannot
explain the observation that most of the spore sacs appear like Figure Q20-16B.
Correct. The oddball spore sacs must result from recombination between the
homologs carrying different alleles of the color gene. Recombination between the
homologs occurs during prophase I.
Incorrect. The results shown cannot distinguish whether recombination has
occurred during prophase II. During prophase II, there is only one homolog in
each cell; if recombination took place between the two sister chromatids of this
homolog we would not be able to detect it, as the sister chromatids are identical
(except for recombinational exchanges).

350

(d)

(e)

Correct. Segregation of chromosomes is mediated by microtubules that pull on


the centromeres of chromosomes. Thus, oddball segregation patterns arise from
recombination (and thus loss of a covalent connection) between the centromere
and the gene of interest.
Incorrect. If recombination had taken place only between the gene for spore color
and the end of that chromosome arm, it would look as if no recombination had
taken place (Figure Q20-16B) because the centromere would still be covalently
connected to the gene of interest. See also the answer to option (d).

20-17 Choice (b) is the correct answer. Nondisjunction in one or the other meiotic
division will give rise to gametes with two sex chromosomes instead of one. Since
the only source of Y chromosomes is the father, the gametes that produce an
XYY male must be X (a normal egg) and YY (an abnormal sperm).
Nondisjunction in the second meiotic division of spermatogenesis could give rise
to a YY gamete that fertilizes a normal egg and thereby creates an individual with
the karyotype XYY (see Figure A20-17). Choice (a) is incorrect because
nondisjunction in the first meiotic division of spermatogenesis would result in two
XY sperm and two sperm with no sex chromosome. Choices (c), (d), and (e) are
incorrect because nondisjunction during oogenesis cannot yield an individual with
two Y chromosomes.

Figure A20-17
20-18 False. The phenotype, or the observable traits, of an organism often does not fully reflect
its genotype, or the catalog of all alleles in the chromosomes. This is because some
alleles are dominant (call these A) and other alleles are recessive (a). An individual who
is heterozygous (Aa) for a dominant allele will have the same phenotype as one who is
homozygous (AA).
20-19 Disagree. There are two distinct mechanisms to get genetic variation in gametes. One
method involves the reassortment of chromosomes during meiosis. This method would
result in a diploid organism with 16 chromosomes producing 28 genetically different
gametes. However, a much greater number of genetically different gametes can be
produced due to the recombination between chromosomes that occur during every
meiosis.

351

20-20 Gene F is least likely to behave according to Mendels law of independent assortment
because it lies the closest to gene E and is least likely to be separated from gene E by
recombination.
20-21 A.
B.

The genotypes of the mother and father are the same: Ff. The only way that a
child can have the disease is if both parents are carriers of the cystic fibrosis gene.
The chance that the second child will have cystic fibrosis is one quarter. The
chance that the mother will transmit her mutant f allele to the offspring is one half,
multiplied by an equal chance that the father will transmit his mutant f allele to
the offspring: 1/2 1/2 = 1/4. (The existence of a first child with cystic fibrosis
does not change the probability of another child having the disease.)

20-22 The classification of all mutations depends on the phenotype under consideration. With
regard to the sickle cell anemia phenotype, the mutant allele is recessive because a
heterozygous individual has the same healthy phenotype as a homozygous wild-type
individual. With regard to the malaria phenotype, the mutant allele is dominant. This is
because a heterozygous individual has the same phenotype as a homozygous mutant
individual, namely resistance to malaria.
20-23 You would expect the progeny to be 75% purple-seeded plants and 25% yellow seeded
plants. The crosses are diagramed below, where we designate the purple-seeded causing
allele as P, the yellow-seeded causing allele as Y, and the green-seeded causing allele as
G. Note that P, Y, and G are all alleles of the same gene.
Table A20-23A: Cross #1 F2 results (F1 = YG YG)
allele
Y
G

Y
YY (yellow)
YG (yellow)

G
YG (yellow)
GG (green)

Table A20-23B: Cross #2 F2 results (F1 = YP YP)


allele
Y
P

Y
YY (yellow)
YP (purple)

P
YP (purple)
PP (purple)

Table A20-23C: Cross #3 F2 results (F1 = GP GP)


allele
G
P

G
GG (green)
GP (purple)

P
GP (purple)
PP (purple)

Table A20-23D: Your cross (or Cross #2 F1 Cross #3 F1 = YP GP)


allele
G
P

Y
GY (yellow)
YP (purple)
352

P
GP (purple)
PP (purple)

20-24 One gene involved in determining fur color differs between these two strains. The white
allele is not fully recessive to the dark brown allele. Therefore, any animal that is
heterozygous (and has one white allele and one dark brown allele) will be light brown in
fur color. Thus the white allele and the dark brown allele are co-dominant. When two
heterozygous animals are crossed together, 25% will be homozygous for one allele, 25%
will be homozygous for the other allele, and 50% will heterozygous, exactly what is
observed in the cross you performed.
Extra credit answer: Any reasonable answer is fine. There are many possible answers,
including the two below:
1.
The dark brown and white alleles are alleles of a gene involved in synthesizing
pigment. Hamsters with the white allele do not produce any pigment; hamsters
with the dark brown allele produce pigment. When hamsters are heterozygous,
they have one chromosome that carries the white allele and does not produce any
pigment. Therefore, they produce half the amount of pigment needed normally
and thus are light brown instead of dark brown.
2.
The dark brown and white alleles are alleles of a gene involved in a microtubule
motor important for transporting the pigment to the correct place in the cell.
Hamsters with the white allele produce a defective version of the microtubule
motor and thus no pigment is properly transported, leading to white fur. The
hamsters with the dark brown allele produce a properly functioning microtubule
motor. When hamsters are heterozygous, they have one chromosome that carries
the white allele and one chromosome that carries the dark brown allele; therefore,
only half the amount of functioning microtubule motor is being produced in the
heterozygote. Because the amount of microtubule motors is limiting in the cell,
half the amount of properly functioning microtubule motor can only transport half
the amount of pigment, leading to a light brown fur.
20-25 A single nucleotide substitution is most likely to cause a conditional allele. Nucleotide
addition and nucleotide deletion usually will cause a shift in the reading frame of the
protein, leading to a string of many amino acid substitutions (and an altogether different
protein) and possibly even the formation of a stop codon, which would result in the
premature truncation of the protein. In contrast, a single nucleotide substitution would be
more likely to result in an amino acid substitution that would disrupt protein function in a
conditional manner. For example, this amino acid substitution may affect protein folding
at high temperature due to a greater charge; at the permissive temperature, this protein
may still fold properly and function normally.

353

20-26 A.
B.

Social. See Figure A20-26A.


Antisocial. See Figure A20-26B.

Figure A20-26
C.

D.

Yes, different. If the two mutations were in the same gene, it is probable that the
offspring would have an antisocial phenotype, unlike the case in panel (A). Such
a cross is overwhelmingly likely to produce an antisocial offspring, because two
mutations in the same gene are unlikely to complement one another. However,
there are rare cases in which two different mutations in the same gene can
complement one another. This can be observed if, for example, a gene encodes a
protein with two functions and each mutation impairs a different function.
No, not different. If the dominant and recessive mutations were in the same gene,
the phenotype of the offspring would still be antisocial. Thus, complementation
tests do not work for dominant mutations: the phenotype of the offspring when
the two mutations are in the same gene is the same as the phenotype of the
offspring when the mutations are in different genes. The phenotype of the
offspring will always be mutant, because a single M* allele is sufficient for the
mutant phenotype.

20-27 Disagree. The frequency of a trait in a population has nothing to do with whether this
trait is dominant or recessive. In order to test dominance or recessiveness, the
segregation of this trait must be observed. For example, the defective version of the gene
involved in Huntingtons disease occurs at a relatively low frequency in the population,
but behaves in a dominant fashion to cause disease.

354

20-28 Three genes: mal1, mal2, and mal6 are mutations in one gene; mal3 and mal5 are
mutations in a second gene; mal4 is the only mutation you identified in a third gene.
These results can be deduced from the complementation analysis. For example, the
diploid yeast containing the mal1 and mal6 alleles cannot grow on maltose-containing
media, and therefore these alleles are likely to be alleles of the same gene. However, the
diploid yeast containing the mal1 and the mal4 alleles do grow on maltose-containing
media, and these alleles are, therefore, likely to be alleles of different genes.
20-29 Eventually, the b mutation will be separated from the A allele by meiotic recombination.
Meiotic recombination exchanges portions of homologous chromosomes, and thereby
generates great diversity among the gametes of each individual. The locations of the 15
exchanges per chromosome during meiosis in humans are more or less random. Thus,
each passing generation increases the cumulative likelihood of a recombinational
crossover between any two neighboring genes (A and b). Such a recombinational
crossover in an individual heterozygous for both genes (Ab and aB) will separate two
alleles that were originally linked (yielding AB and ab).
20-30 Yes, a single mutation can explain the pattern. The mutation responsible for the disease
is recessive, since two unaffected parents can produce affected offspring. The mutation
is likely carried on the X chromosome, since males (having only one X) are more likely
to have the disease than their sisters (having two X chromosomes). If the mutation were
on an autosome, male and female offspring would have equal probability of having the
disease. If the mutation were located on the Y chromosome, all males in the family
would be affected.
20-31 A.

B.

SNP3 is most likely to be tightly linked to the gene involved in cat allergies
because it is found in all people who are allergic to cats and none of the people
who arent allergic to cats. It is possible that this SNP is responsible for the
allergic response, but the proof of this would require additional studies.
SNP4 is probably the next closest to the gene involved in cat allergies because it
is found in all but one person who is allergic to cats and none of the people who
arent allergic to cats. SNP1, SNP2, and SNP5 seem to segregate randomly in
both people allergic and not allergic to cats and therefore are probably more
distant from the gene involved in cat allergies.

20-32 Because of the redundancy of the genetic code, in which more than one codon can code
for the same amino acid, some single nucleotide changes may not cause changes in the
amino acid coded for by that codon. Furthermore, some amino acid substitutions are
neutral (e.g., substitution of a small uncharged amino acid for another amino acid with
the same properties) and thus do not change the function of the protein even though the
amino acid has been changed.

355

CHAPTER 20
TISSUES AND CANCER
2009 Garland Science Publishing
3rd Edition

Extracellular Matrix and Connective Tissues


20-1 Both multicellular plants and animals have (a) cells capable of locomotion. (b)
cells with cell walls. (c) vascular systems containing circulating cells. (d) a
cytoskeleton composed of actin filaments, microtubules and intermediate filaments. (e)
tissues composed of multiple different cell types.

20-2 For each of the following sentences, fill in the blanks with the best word or phrase selected
from the list below. Not all words or phrases will be used; each word or phrase should be used
only once.
Plants are sedentary and thus their cells have different needs than cells found in
motile animals. For example, in plant cells, __________________ generates the turgor pressure
that drives cell growth. Plants have cell walls, but cell growth is possible in the developing tissue
because the __________________ cell walls are expandable. The plant vascular system is
composed of two types of complex tissues. The __________________ is important for the
transport of organic solutes in the plant. The __________________ can include cell walls that
have been strengthened by the deposition of a __________________ network, which serves to
mechanically reinforce the matrix and makes the tissue waterproof. The outer protective covering
of the plant, also known as the dermal tissue, contains __________________, which regulate gas
exchange. cellulose lignin secondary collagen membranous stomata epidermis osmosis
translocation glycine phloem xylem ground tissue primary

357

20-3 A. Indicate the direction in which the plant cell shown in Figure Q20-3 is most likely to
grow.

Figure Q 20-3
B. Explain your answer.

20-4 Indicate whether the following molecules are found in plants, animals, or both.
A. intermediate filaments
B. cell walls
C. microtubules
D. cellulose
E. collagen
20-5 A major distinction between the connective tissues in an animal and other main tissue
types such as epithelia, nervous tissue, or muscle is (a) the ability of connective tissue cells
such as fibroblasts to change shape. (b) the abundant extracellular matrix in connective
tissues. (c) that connective tissues can withstand mechanical stresses. (d) the exceptional
amounts of polysaccharides in the extracellular matrix of connective tissues. (e) the
numerous connections connective tissue cells make with each other.

20-6 What are the main structures providing tensile strength in


A. animal connective tissue? B. animal epithelium? C. plant
cell walls?

20-7 Do you agree or disagree with the following statement? Explain your answer.
Like many other extracellular proteins, newly synthesized collagen molecules undergo
post-translational processing inside the cell to convert them into their mature form, and
are then secreted and self-assemble into fibrils in the extracellular space.

20-8 Fibroblasts organize the collagen of the extracellular matrix by


(a) cutting and rejoining the fibrils.
(b) crawling along existing fibrils and adjusting the packing of collagen molecules
within them.
(c) twisting fibrils together to make ropelike fibers.
(d) pulling the collagen into sheets or cables after it has been secreted.
(e) controlling the local Ca2+ concentration so as to regulate collagen assembly.

358

20-9 Proteoglycans in the extracellular matrix of animal tissues (a)


provide tensile strength. (b) form a dense compact matrix. (c) are linked
to microtubules through the plasma membrane. (d) are polysaccharides
composed of glucose subunits. (e) provide cartilage with the ability to
resist compression.

20-10 Which of the following statement(s) are FALSE? (a) Proteoglycans can act as filters to
regulate which molecules pass through the extracellular medium. (b) The negative charge
associated with proteoglycans attracts cations, which cause water to be sucked into the
extracellular matrix. (c) Proteoglycans are a major component of compact connective tissues,
but relatively unimportant in watery tissues such as the jellylike substance in the interior of the
eye. (d) Proteoglycans help tissues resist mechanical compression. (e) Glycosaminoglycans
are components of proteoglycan.

Epithelial Sheets and Cell-Cell Junctions

20-11 For each of the following sentences, fill in the blanks with the best word or phrase selected
from the list below. Not all words or phrases will be used; each word or phrase should be used only
once. __________________ join the intermediate filaments in one cell to those in the neighboring
cell. __________________ anchor intermediate filaments in a cell to the extracellular matrix.
__________________ involve cadherin connections between neighboring cells and are anchorage
sites for actin filaments. __________________ allow for the exchange of small molecules from
one cell to its adjacent cell. __________________ prevent the leakage of molecules between
adjacent cells. adherens junctions gap junctions highway junctions desmosomes
hemidesmosomes tight junctions

20-12 A basal lamina (a) is a thin layer of connective tissue cells and matrix underlying an
epithelium. (b) is a thin layer of extracellular matrix underlying an epithelium. (c) is
attached to the apical surface of an epithelium. (d) is impermeable to small organic
molecules. (e) separates epithelial cells from each other.

359

20-13 Label the five different types of cell-cell junctions shown in Figure Q20-13, and identify
the apical and basal surfaces of the epithelium.

Figure Q20-13

20-14 Match the molecules (List 1) with the cell structures in which they play a part (List 2). A
cell structure may be listed more or less than once. List 1 List 2
List 1
A. Cadherin
B. Cellulose
C. Collagen
D. Connexons E.Occludins
F. Claudins
G. Keratins

Tissue Maintenance and


Renewal

List 2
1. Desmosomes
2. Cell wall
3. Gap junctions
4. Nucleus
5. Basal lamina
6. Plasmodesmata 7.
Tight junctions

20-15 Name the three key mechanisms important for maintaining the organization of cells into
tissues.
20-16 Place the following in order of their replacement times from shortest to longest.
A. Epidermal cell B. Nerve cell C. Bone matrix D. Red blood cell E. Cell lining the
gut

360

A stem cell divides into two daughter cells and one of the daughter cells goes on to
20-17
become a terminally differentiated cell. What is the typical fate of the other daughter cell?

20-18 Your friend is a pioneer in ES cell research. In her research, she uses an ES cell line that
originated from an inbred strain of laboratory mice called FG426. She has just figured out
methods that allow her to grow an entire liver from an ES cell and has successfully grown 10
livers. She demonstrates that the newly grown livers are functional by successfully transplanting
one of the new livers into a FG426 laboratory mouse. You are particularly excited about this,
because you have a sick pet mouse, Squeaky. You are very attached to Squeaky, as you found
him when you were out camping in New Hampshire. Unfortunately, Squeaky has developed liver
disease and will not live much longer without a liver transplant. After you see your friend on TV
talking about her new method for growing mouse livers, you immediately grab your cell phone to
ask her whether Squeaky could have one of the newly grown livers. Just as you are about to dial
your friend, you remember something you learned in cell biology and realize that instead, you
should ask your friend about possibly using therapeutic cloning for Squeakys benefit. A. Why
do you think that one of the newly grown livers may not work in Squeaky? B. Explain why
therapeutic cloning would solve this problem.

Cancer
20-19 A malignant tumor is more dangerous than a benign tumor because (a)
its cells are proliferating faster. (b) it metastasizes by traveling through the
digestive tract. (c) it causes neighboring cells to mutate. (d) its cells attack and
phagocytose neighboring normal tissue cells. (e) its cells invade other tissues.

20-20 A certain mutation in the receptor for epidermal growth factor (EGF) causes the mutated
receptor protein to send a positive signal along associated intracellular signaling pathway even
when EGF is not bound to it. This leads to abnormal cell proliferation in the absence of growth
factor. On the basis of this information, would you class the gene for the EGF receptor as a
tumor-suppressor gene or a potential oncogene? Explain your answer.

361


20-21
Ras is a GTP-binding protein that is often defective in cancer cells. Normal cells are
often stimulated to divide by receiving a signal from a growth factor through a receptor tyrosine
kinase. When the receptor tyrosine kinase binds the growth factor, Ras is stimulated to bind
GTP. Ras in turn activates proteins that promote cell proliferation. A common mutation in
cancerous cells causes Ras to behave as though it were bound to GTP all the time.

A. Why is this mutation advantageous to cancerous cells?


B. Your friend decides that the signaling pathway involving the Ras protein is a good target for
drug design, since the Ras protein is often defective in cancer cells. Your friend designs a drug
that will turn off the receptor tyrosine kinase by preventing its ability to dimerize. Do you think
that this will affect cells that have a defective Ras protein that acts as if it were always bound to
GTP? Why or why not?

20-22 People who inherit one copy of the Rb (retinoblastoma) gene that is normal and one copy
that is mutatedthat is, people who are heterozygous for Rbhave a greatly increased risk of
cancer.
Given this information, do you agree or disagree with the following statement? Explain your
answer.
The Rb mutation must have a dominant effect, which means that it must result in an
increase in Rb function. Thus, Rb in its mutant form must be an oncogene.

362

20-23 Figure 20-23 shows a sequence of mutations that might underlie the development of
colorectal cancer. Given what you have learned about the p53 protein in Chapter 18,
explain why the loss of p53 is advantageous to cancerous cells.

Figure Q20-23

20-24 Drugs that block the function of oncogenic proteins hold great promise in the fight
against cancer. Should cancer researchers also be attempting to design drugs that will
interfere with the products of tumor suppressor genes like p53? Why or why not?

363

How We Know: Making Sense of the Genes that Are Critical for Cancer
20-25 Your studies in cell biology have revealed how APC, a protein encoded by a tumor
suppressor gene frequently inactivated in people with colorectal cancer, functions in the Wnt
signaling pathway (see Figure Q20-25), and this has inspired you to do further research on Wnt
signaling. You would like to design a drug to treat people with colorectal cancer. Given the
pathway above and the knowledge that a majority of human colorectal tumors harbor mutations
in the APC gene, name a protein from the pathway above that would be a good target for an
activity-blocking drug. Explain your answer.

Figure Q20-25

364


Answers

20-1 Choice (e) is the correct answer. Choice (d) is incorrect as plant cells do not have

intermediate filaments. Plant cells are not capable of locomotion (choice (a)) and also do not
have
a vascular system with circulating cells (choice (c)). Only plant cells have cell walls

(choice (b)).

20-2 Plants are sedentary and thus their cells have different needs than cells found in motile

animals.
For example, in plant cells, osmosis generates the turgor pressure that drives cell
growth. Plants have cell walls, but cell growth is possible in the developing tissue because the

primary
cell walls are expandable. The plant vascular system is composed of two types of
complex tissues. The phloem is important for the transport of organic solutes in the plant. The

xylem
can include cell walls that have been strengthened by the deposition of a lignin network,
which serves to mechanically reinforce the matrix and makes the tissue waterproof. The outer

protective covering of the plant, also known as the dermal tissue, contains stomata, which
regulate
gas exchange.

20-3
A. Verticallysee Figure A20-3. B. Cellulose fibers are highly resistant to

stretching so a plant cell tends to grow, under the stimulus of turgor pressure, in a direction
perpendicular
to the orientation of the fibers in the cell wall.

Figure A203

20-4 A. animals B. plants


C. both D. plants E.
animal

20-5 (b) Cells in many other types of tissue can change shape (e.g., muscle cells when they
contract), and can withstand mechanical stress (e.g., cells of the epidermis). Connective-tissue
cells tend to be scattered throughout extracellular matrix, and unlike, for example, nerve cells or
epithelial cells, make few or no contacts with each other.
20-6 A. Collagen fibers B. Intermediate
filaments C. Cellulose fibers

365


20-7 Disagree.
Newly synthesized molecules of collagen are secreted from the cell in an immature
form as procollagen, and the peptides at the ends of the procollagen molecules then have to be
cleaved off in the extracellular space before assembly of fibrils can occur. This ensures that
collagen fibrils will not assemble prematurely inside of the cell.

20-8 (d)
20-9 (e)
20-10 (c) Proteoglycans are MORE important in the interior of the eye, where collagen is very
scarce, than in compact connective tissues such as bone and tendon, whose structure is
dominated by collagen.
20-11 Desmosomes join the intermediate filaments in one cell to those in the neighboring cell.
Hemidesmosomes anchor intermediate filaments in a cell to the extracellular matrix.
Adherens junctions involve cadherin connections between neighboring cells and are
anchorage sites for actin filaments. Gap junctions allow for the exchange of small molecules
from one cell to its adjacent cell. Tight junctions prevent the leakage of molecules between
adjacent cells.
20-12 (b)
20-13 1, apical surface; 2, tight junction; 3, adherens junction; 4, desmosome junction; 5, gap
junction; 6, hemidesmosome junction; 7, basal surface.
20-14 A1; B2; C5; D3; E7; F7; G1
20-15 1. Cell communication 2. Selective
cell-cell adhesion 3. Cell memory

20-16 Cell lining the gut (few days) < epidermal cell (1 or 2 months) < red blood cell
(4 months) < bone matrix (10 years) < nerve cell (lifetime).
20-17 The other daughter cell typically remains a stem cell.
20-18 A. For organ transplantation to be successful, the donor and the recipient should be as
close a genetic match as possible to minimize the risk of immunological rejection. Because you
found Squeaky in the fields of New Hampshire, Squeaky is likely to have genetic differences with
the FG426 inbred laboratory mice. B. Therapeutic cloning may help save Squeaky because it is a
method for generating ES cells from Squeaky himselfcells that will be genetically identical to
Squeaky. Hopefully, your friend will be successful when applying her methods to the ES cell line
derived from Squeaky and can grow a functional liver for him.

20-19 (e)
366


20-20
The mutation described leads toward cancerous cell behavior (excessive proliferation) by
making the gene product hyperactive. The mutant gene is therefore, by definition, an oncogene.
This effect is seen even if only one copy of the gene is affected; in other words, the mutation is
dominant, as is typical for an oncogene. Mutations that delete an EGF receptor gene would be
expected to have either no effect or an inhibiting effect on cell division. Thus the normal EGF
receptor is classed as a potential oncogene (a proto- oncogene).

20-21 A. A Ras mutation that causes Ras to behave as though it is bound to GTP all the time is
advantageous to cancer cells because Ras is then activated and turns on the activities of proteins
required for cell proliferation. If the cell proliferation proteins are always turned on, the cancer
cell will be able to proliferate at an unregulated rate, outgrowing its normal neighbors. The ability
to proliferate in a signal-independent fashion is one of the hallmarks of a cancer cell. B.
Unfortunately, a drug that blocks activation of the receptor that activates Ras will be unlikely to
have a useful effect on a cell containing mutant Ras protein that acts as though it is always bound
to GTP. A mutant Ras protein of this type behaves as though it is constantly activated. Because
Ras acts downstream of the receptor, the activating mutation makes its effect felt, regardless of the
state of the receptor on which Ras activation would normally depend: mutant Ras that is always
active is no longer dependent on the receptor for activation. Therefore, blocking the ability of the
receptor to dimerize and activate Ras will probably not have an effect on cells containing the
mutant Ras protein.

20-22 Disagree. It is true that the Rb mutation is dominant, in the sense that a person who is
heterozygous (inherits one normal copy and one mutant copy of the gene) is likely to show the
mutant trait (that is, will be cancer-prone). However, this does not mean that the mutation in Rb
has to be causing an increase in Rb gene function; in fact, the opposite is truethe propensity for
cancer arises from a loss of Rb gene function. Therefore, Rb should be classified as a tumor
suppressor gene and not as an oncogene. Most people have two functional Rb genes in each of
their cells. Thus, for one of their cells to turn cancerous by losing Rb function, both copies of the
gene in that cell must be inactivated or lost, a two-step process. However, in a person born lacking
one copy of the Rb gene, each cell is only one step away from complete loss of Rb function.
Consequently, such a person has a high risk that at least one of the cells in the body will undergo a
mutation that precipitates cancer. In this way, at the level of the whole person, the Rb loss-offunction mutation is dominant, even though at the level of the individual cell it is recessive.

367

p53 is a protein that is activated and stabilized when DNA is damaged. When p53 is
20-23
active, it stops the cell cycle in order to give the cell time to repair its damaged DNA, or, if that
is not feasible, it causes the cell to commit suicide by apoptosis. Cells lacking p53 will continue
to replicate their DNA, will avoid suicide, and will go through cell division even when the DNA
has been damaged, producing mutant daughter cells. Repeated rounds of cell division under
these circumstances perpetuate the genetic damage and allow still more mutations to occur.
Some of these newly accumulated mutations will give the cell an increased ability to survive,
proliferate, and metastasize, resulting in invasive cancer.

20-24 Oncogenic proteins lead toward cancer, because they have an excessive or unregulated
activity as compared with the corresponding normal proteins. By blocking this activity with a
drug molecule that simply clogs the active site of the oncogenic protein, one can remove the
danger. For a tumor suppressor gene, the danger lies in a loss of function, and there is generally
no simple way for a drug molecule to restore a protein function that has been lost. It is hard to
see, therefore, how one could achieve any useful effect on cancer by means of drugs that
interfere with tumor suppressor gene products. A drug that simply inhibited their function
would be expected to promote, not cure, cancer.
20-25 There are two possible answers; -catenin and TCF. Since most colorectal cancers
consist of cells that lack APC, drugs that interfere directly with the Wnt signal protein, or with
its receptor in the cell membrane, or with the intracellular signaling protein that acts
upstream of APC will not be very useful: if APC is missing, large quantities of active
- catenin will be produced even in the presence of such drugs. However, drugs that
-catenin or TCF activity would be good candidates, provided that they do not interfere with
vital functions of these proteins elsewhere in the body.

368

Potrebbero piacerti anche